Você está na página 1de 267

Conjuntos A ssunto

1
Matemática I

1. Conceitos Teorema 1 (número de subconjuntos):


Um conjunto A de n elementos possui 2n subconjuntos.
1.1 Elemento e conjunto
Dem.: Seja A = {a1, a2,...,an}. Para formar um subconjunto X de A,
São conceitos primitivos, isto é, não são definidos. devemos decidir, para cada elemento de A, se ele pertencerá ou não a X.
Se um elemento x pertence ao conjunto A, diz-se que x ∈ A. Caso Como temos n elementos e para cada elemento, temos duas possibilidades
contrário, diz-se que x ∉ A. (ou ele está no subconjunto ou não está), o número de subconjuntos de
Usam-se geralmente letras maiúsculas para representar conjuntos e A é 2 ⋅ 2 ⋅ ... ⋅ 2 = 2n ⋅

letras minúsculas para representar elementos. n vezes

1.7 Conjunto universo


1.2 Conjunto vazio É um conjunto que contém todos os elementos do contexto
O conjunto vazio é aquele que não possui elementos. Representa-se envolvido e também todos os conjuntos desse contexto. Por exemplo, se
por ∅ ou { }. estivermos em um problema envolvendo conjuntos de números inteiros,
um possível conjunto universo é Z. Poderíamos escolher também para
Ex.: {x ∈ N|1 < x < 2} conjunto universo Q. Em geral, usa-se a letra U para representar o
conjunto universo.

1.3 Conjunto unitário


1.8 Diagramas de Venn
O conjunto unitário é aquele que possui apenas um elemento.
Os diagramas de Venn são diagramas que mostram todas as possíveis
Ex.: A = {2012}, B = {x ∈ N|1 < x < 3} relações lógicas entre uma coleção finita de conjuntos. É mais frequente o
uso de diagramas de Venn para representar dois ou três conjuntos. Nesse
caso, usamos círculos para representá-los.
1.4 Subconjunto
Um conjunto A é um subconjunto de um conjunto B quando todos os Obs.: os diagramas de Venn são muito úteis para resolver problemas
elementos de A pertencem a B. Representamos a inclusão de conjuntos de conjuntos, pois ajudam a organizar os dados do problema de forma
por A ⊂ B (lê-se A está contido em B) ou B ⊃ A (lê-se B contém A). Se bastante clara, como veremos nos exercícios resolvidos.
não ocorre a inclusão, usamos ⊄ (não está contido) ou ⊃
/ (não contém).

Ex.: {1, 2} ⊂ {1, 2, 3, 4} ; {1, 3} ⊄ {2, 3, 4} 2. Operações envolvendo conjuntos


Obs.: ∅ ⊂ A (o conjunto vazio é subconjunto de qualquer conjunto) 2.1 União
A união de dois conjuntos A e B é o conjunto formado por todos os
De fato, dizemos que A ⊄ B se existe a ∈ A tal que a ∉ B. Dizer, elementos que pertençam a A ou B (para que um elemento esteja na união,
portanto, que ∅ ⊄ A significa dizer que existe elemento x ∈ ∅ tal que basta que ele pertença a pelo menos um dos conjuntos).
x ∉ A. Isso não é possível, já que o conjunto vazio não possui elemento.
A ∪ B = {x | x ∈ A ou x ∈ B}
1.5 Igualdade de conjuntos Ex.:
Dois conjuntos A e B são ditos iguais quando todos os elementos de {1, 2} ∪ {3, 4} = {1, 2, 3, 4} ; {1, 2} ∪ {2, 3} = {1, 2, 3} ;
A pertencem a B e vice-versa, isto é, A ⊂ B e B ⊂ A. {1} ∪ ∅ = {1}
2.2 Interseção
1.6 Conjunto das partes (conjunto potência) A interseção de dois conjuntos A e B é o conjunto formado pelos
É o conjunto formado por todos os subconjuntos de um certo conjunto. elementos que pertençam a A e a B (para que um elemento esteja na
O conjunto das partes é representado por P(A) ou 2A (esta última não interseção, ele deve pertencer aos dois conjuntos).
é tão usual.)
A ∩ B = {x|x ∈ A e x ∈ B}
Ex.: Se S é o conjunto de três elementos {1, 2, 3}, a lista de subconjuntos Ex.:
de S é: {1, 2, 3} ∩ {3, 4, 5} = {3} ; {1, 2, 3, 4} ∩ {1, 4} = {1, 4} ;
{1, 2, 3} ∩ {4, 5, 6} = ∅
∅, {1}, {2}, {3}, {1, 2}, {1, 3}, {2, 3}, {1, 2, 3}
Obs.: Dois conjuntos A e B são ditos disjuntos quando A ∩ B = ∅
Assim, o conjunto das partes de S é P(S) = {∅, {1}, {2}, {3},
{1, 2}, {1, 3}, {2, 3}, {1, 2, 3}}. Repare que P(S) tem 8 = 23 elementos
e que S tem 3 elementos.

IME-ITA 157
Matemática I – Assunto 1

2.3 Diferença de conjuntos 3.3 1a Lei de De Morgan


Dados dois conjuntos A e B, definimos A – B = {x tal que x ∈ A e x ∉ B}
(elementos que pertencem a A, mas não pertencem a B). (A ∪ B)C = AC ∩ BC

Ex.: A = {1, 2, 3, 4, 5, 6} e B = {1, 2, 3, 6}. Temos que A – B = {4 ,5}. Dem.:


Parte 1: (A ∪ B)C ⊂ AC ∩ BC
Obs.: Também representamos a diferença entre os conjuntos A e B Seja x ∈ (A ∪ B)C. Queremos provar que x ∈ AC ∩ BC. Como x ∉
por A \ B. (A ∪ B), então x ∉ A e x ∉ B. Com isso, segue que x ∈ AC e x ∈ BC, o que
nos dá x ∈ AC ∩ BC.
Complementar
Parte 2: (A ∪ B)C ⊃ AC ∩ BC:
Dados dois conjuntos A e B com A ⊂ B, o complementar de A em
Análogo à parte 1.
relação a B é o conjunto cujos elementos estão em B e não estão em
A. O complementar de A em relação a B é denotado por CBA = B – A.
O complementar de um conjunto A em relação ao conjunto universo 3.4 2a Lei de De Morgan
U, representado por AC (ou A), é o conjunto formado pelos elementos
do universo U que não pertencem ao conjunto A. (A ∩ B)C = AC ∪ BC

Ex.: A = {1, 2, 3, 6} e B = {1, 2, 3, 4, 5, 6} ; CBA = B – A = {4, 5} 3.5 A – B = A ∩ BC


U = * e A = {x ∈ *|x > 4}. Temos que AC = {1, 2, 3}.
4. Princípio da inclusão-exclusão:
Para calcular o número de elementos de uma união, usamos as
2.4 Diferença simétrica seguintes fórmulas (válidas apenas para conjuntos finitos):
A diferença simétrica de dois conjuntos A e B é o conjunto dos Obs.: n(X) representará a quantidade de elementos de X.
elementos que pertencem à união dos dois conjuntos, mas não pertencem
à interseção dos dois conjuntos. 4.1 Para dois conjuntos

A Δ B = (A – B) ∪ (B – A) = (A ∪ B) – (A ∩ B) n(A ∪ B) = n(A) + n(B) – n(A ∩ B)

Ex.: A = {1, 2, 3} e B = {2, 3, 4, 5}. Temos que A Δ B = {1, 4, 5}. Essa fórmula expressa que para calcular o número de elementos
de uma união, não basta somar as quantidades de elementos dos dois
3. Propriedades conjuntos, pois alguns elementos podem ser contados duas vezes. Esses
elementos que são contados duas vezes são justamente os elementos da
3.1 Distributiva (da união em relação à interseção) interseção e, por isso, devemos retirar n(A ∩ B). Essa ideia se estende
de maneira análoga para mais conjuntos.
A ∪ (B ∩ C) = (A ∪ B) ∩ (A ∪ C)
4.2 Para três conjuntos
Dem.:
A demonstração consiste em duas partes (este passo é padrão em n(A ∪ B ∪ C) = n(A) + n(B) + n(C) – n(A ∩ B) – n(B ∩ C) –
demonstrações de igualdades de conjuntos): n (C ∩ A) + n(A ∩ B ∩ C)
4.3 Para n conjuntos
Parte 1: A ∪ (B ∩ C) ⊂ (A ∪ B) ∩ (A ∪ C)
Seja x ∈ A ∪ (B ∩ C). Queremos provar que x ∈ (A ∪ B) ∩ (A ∪ C).
n  ∪ Ai  = ∑ n ( Ai ) − ∑ n( Ai ∩ Aj ) + ∑ n( Ai ∩ Aj ∩ Ak ) − ...
Temos que, x ∈ A ou x ∈ (B ∩ C). Dividiremos então em dois casos: i  i i< j i< j<k

Caso 1: x ∈ A + ( −1))n +1 ⋅ n  ∩ Ai 
i 
Aqui, como x ∈ A, segue que x ∈ A ∪ B e x ∈ A ∪ C e então Teorema 2 (estimativa do número de elementos da união):
x ∈ (A ∪ B) ∩ (A ∪ C) .
Dados m conjuntos A1, A2, ... Am, temos que n(A1 ∪ A2 ∪ ... ∪ Am)
Caso 2: x ∈ (B ∩ C) ≤ n(A1) + n(A2) + ... + n(Am), com igualdade se, e somente se, os
Aqui, x ∈ B e x ∈ C e então x ∈ A ∪ B e x ∈ A ∪ C, o que nos dá conjuntos são disjuntos dois a dois.
x ∈ (A ∪ B) ∩ (A ∪ C).
Dem.:
Parte 2: A ∪ (B ∩ C) ⊃ (A ∪ B) ∩ (A ∪ C)
Cada elemento de A1 ∪ A2 ∪ ... ∪ Am é contado exatamente uma única
vez no lado esquerdo, enquanto no lado direito cada elemento é contado
Análogo à parte 1 (mas em uma prova, você deve escrever).
pelo menos uma vez. Caso um elemento esteja em mais de um conjunto,
será contado mais de uma vez no lado direito e então a desigualdade será
3.2 Distributiva(da interseção em relação à união)
estrita. Assim, o caso de igualdade ocorre se cada elemento é contado
exatamente uma vez do lado direito, ou seja, quando os conjuntos não
A ∩ (B ∪ C) = (A ∩ B) ∪ (A ∩ C) possuem elementos em comum e, portanto, são disjuntos dois a dois.

158 Vol. 1
Conjuntos

EXERCÍCIOS RESOLVIDOS
01 (EN) Sendo A = {{1}, {2}, {1, 2}} pode-se afirmar que: Usando a informação (I) e o total de pacientes, temos que

(A) {1} ∉ A  x + y + 16 = 41
(B) {1} ⊂ A 
3 x + 2 y + 24 = 75
(C) {1} ∩ {2} ⊄ A
(D) 2 ∈ A
(E) {1} ∪ {2} ∈ A Resolvendo o sistema, temos que x = 1 (e y = 24).

Solução 03 (UFC) Sejam M e N conjuntos que possuem um único elemento


(A) F, pois {1} é elemento de A (é verdade que {1} ∈ A) em comum. Se o número de subconjuntos de M é igual ao dobro do
(B) F, pois 1 não é elemento de A número de subconjuntos de N, o número de elementos do conjunto M
(C) F, pois {1} ∩ {2} = ∅ e sabemos que ∅ ⊂ A para qualquer  N é:
conjunto A
(D) F, pois 2 não é elemento de A (A) o triplo do número de elementos de M.
(E) V, pois {1} ∪ {2} = {1, 2} e {1, 2} é elemento de A. (B) o triplo do número de elementos de N.
(C) o quádruplo do número de elementos de M.
Obs.: Usa-se que a ∈ A se existe uma cópia de a dentro do (D) o dobro do número de elementos de M.
conjunto A. Além disso, usa-se {a} ⊂ A se a ∈ A . Lembre, também, (E) o dobro do número de elementos de N.
que não há nenhum problema em um conjunto ser elemento de outro
conjunto (e neste caso, usamos o símbolo de pertence). Solução: Letra E.
Sejam m e n as quantidades de elementos dos conjuntos M e N. O
02 (UNESP) Considere os pacientes da AIDS classificados em três número de subconjuntos de M é 2m e o número de subconjuntos de N
grupos de risco: hemofílicos, homossexuais e toxicômanos. Em um é 2n. Do enunciado, temos que 2m = 2 · 2n  m = n + 1. O número
certo país, de 75 pacientes, verificou-se que: de elementos da união M  N é dado por n(M) + n(N) – n(M ∩ N) =
n + 1 + n – 1 = 2n, que é o dobro do número de elementos de N.
I. 41 são homossexuais;
II. 9 são homossexuais e hemofílicos, e não são toxicômanos; 04 Prove que (A ∩ B)c = Ac ∪ Bc. (2a Lei de de Morgan)
III. 7 são homossexuais e toxicômanos, e não são hemofílicos; Sejam X = (A ∩ B)c e Y = Ac ∪ Bc .
IV. 2 são hemofílicos e toxicômanos, e não são homossexuais;
V. 6 pertencem apenas ao grupo de risco dos toxicômanos; 1a parte: X ⊂ Y
VI. o número de pacientes que são apenas hemofílicos é igual ao
número de pacientes que são apenas homossexuais; Seja u ∈ X. Como X = (A ∩ B)c, temos que u ∉ A ∩ B, o que nos
VII. o número de pacientes que pertencem simultaneamente aos três dá que u não pode ser elemento de A e B simultaneamente. Em outras
grupos de risco é a metade do número de pacientes que não palavras, temos que u ∉ A ou u ∉ B e, por isso, temos que u ∈ Ac ou
pertencem a nenhum dos grupos de risco. u ∈ Bc. Portanto, u ∈ Ac ∪ Bc = Y. Como provamos que um elemento
genérico de X = (A ∩ B)c é elemento de Y = Ac ∪ Bc, segue que X ⊂ Y .
Quantos pacientes pertencem simultaneamente aos três grupos de
risco? 2a parte:

Solução Seja v ∈ Y. Como Y = AC ∪ BC, temos que v ∈ AC ou v ∈ BC ,


Esse é um problema clássico de conjuntos e a principal ideia é montar o que nos dá que v ∉ A ou v ∉ B. Em outras palavras, temos que v
o diagrama de Venn com os três conjuntos: A é o conjunto dos não pode estar simultaneamente em A e B, logo, v ∉ A ∩ B, então
homossexuais, B o dos hemofílicos e C o dos toxicômanos. Sejam x o v ∈ (A ∩ B)C = X. Com o mesmo argumento da 1a parte, chegamos a Y ⊂ X.
número de pacientes que estão nos 3 grupos e y o número de pacientes ∴X=Y
que são apenas homossexuais. Utilizando as informações de (II) a (VII),
temos o seguinte diagrama: 05 (Princípio da inclusão-exclusão para três conjuntos) Sendo A, B
e C conjuntos, prove que:

A n(A ∪ B ∪ C) = n(A) + n(B) +n(C) – n(A ∩ B) – n(B ∩ C) –


n(C ∩ A) + n(A ∩ B ∩ C)
9 B
y y Solução
Vamos usar o mesmo princípio para dois conjuntos:
x
2
7 n(X ∪ Y) = n(X) +n(Y) – n(X ∩ Y)

6 n ( A ∪ B ∪ C) = n ( A ∪ B) + n ( C) − n (( A ∪ B) ∩ C)
Temos que:  (*)
 n ( A ∪ B ) = n ( A) + n ( B ) − n ( A ∩ B )
2x
C

IME-ITA 159
Matemática I – Assunto 1

Usando a distributiva da interseção em relação à união, temos Obs. 1: O princípio é bastante intuitivo e pode ser entendido de maneira
que (A ∪ B) ∩ C = (A ∩ C) ∪ (B ∩ C). Usando mais uma vez informal através de um diagrama de Venn.
o princípio da inclusão-exclusão com dois conjuntos, temos que
n (( A ∩ C) ∪ ( B ∩ C)) = n ( A ∩ C) + ( B ∩ C) − n ( A ∩ B ∩ C) ( * * ) Obs. 2: Para demonstrar o caso geral (com n conjuntos), podemos usar
(aqui, usamos que (A ∩ C) ∩ (B ∩ C) = A ∩ B ∩ C. o princípio da indução finita ou um argumento de combinatória.

Somando as equações de (*) e substituindo (**), temos o resultado.

EXERCÍCIOS NÍVEL 1

01 Dado um conjunto E = {1, 2, 3, 4, 5} e três subconjuntos de E, a 04 (CN) Em um grupo de 142 pessoas foi feita uma pesquisa sobre três
saber A, B e C, tais que: programas de televisão A, B e C e constatou-se que:

A ∩ B = {2, 4}; A ∪ B = {2, 3, 4, 5}; A ∩ C = {2, 3}; A ∪ C = I. 40 não assistem a nenhum dos três programas;
{1, 2, 3, 4}, determine C ∩ (B ∪ A) e A ∩ (B ∩ C). II. 103 não assistem ao programa C;
III. 25 só assistem ao programa B;
02 (CMRJ) São dados os conjuntos A, B e C, tais que n(B ∪ C) = 18, IV. 13 assistem aos programas A e B;
n(A ∩ B) = 6, n(A ∩ C) = 5, n(A ∩ B ∩ C) = 2 e n(A ∪ B ∪ C) = 21. V. O número de pessoas que assistem somente aos programas B e C é
O valor de n[A – (B ∩ C)] é: a metade dos que assistem somente a A e B;
VI. 25 só assistem a dois programas; e
(A) 6. (D) 9. VII. 72 só assistem a um dos programas.
(B) 7. (E) 10.
(C) 8. Pode-se concluir que o número de pessoas que assistem

03 (CN) Sejam U o conjunto das brasileiras, A o conjunto das cariocas, (A) ao programa A é 30.
B o conjunto das morenas e C o conjunto das mulheres de olhos azuis. (B) ao programa C é 39.
O diagrama que representa o conjunto de mulheres morenas ou de olhos (C) aos três programas é 6.
azuis, e não cariocas; ou mulheres cariocas e não morenas e nem de olhos (D) aos programas A e C é 13.
azuis é: (E) aos programas A ou B é 63.

(A) (D) 05 (EPCAR) Para uma turma de 80 alunos do CPCAr, foi aplicada uma
A B A B prova de matemática valendo 9,0 pontos distribuídos igualmente em 3
questões sobre:

I. Função;
II. Geometria;
III. Polinômios.

Sabe-se que:
C C
• apesar de 70% dos alunos terem acertado a questão sobre FUNÇÃO,
(B) (E) apenas 1/10 da turma conseguiu nota 9,0;
A B A B • 20 alunos acertaram as questões sobre FUNÇÃO e GEOMETRIA;
• 22 acertaram as questões sobre GEOMETRIA e POLINÔMIOS;
• 18 acertaram as questões sobre FUNÇÃO e POLINÔMIOS.

A turma estava completa nessa avaliação, ninguém tirou nota zero, no


critério de correção não houve questões com acertos parciais e o número
de acertos apenas em GEOMETRIA é o mesmo que o número de acertos
apenas em POLINÔMIOS.
C Nessas condições, é correto afirmar que:
C
(C) A B (A) o número de alunos que só acertaram a segunda questão é o dobro
do número de alunos que acertaram todas as questões.
(B) metade da turma só acertou uma questão.
(C) mais de 50% da turma errou a terceira questão.
(D) apenas 3/4 da turma atingiu a média maior ou igual a 5,0.

160 Vol. 1
Conjuntos

06 (PUC) Numa comunidade constituída por 1800 pessoas há três tipos 10 (UFRJ) Um clube oferece a seus associados aulas de três modalidades
favoritos de programas de TV: Esporte (E), Novela (N) e Humorismo (H). de esporte: natação, tênis e futebol. Nenhum associado pôde se inscrever
A tabela seguinte indica quantas pessoas assistem a estes programas. simultaneamente em tênis e futebol, pois, por problemas administrativos,
as aulas destes dois esportes serão dadas no mesmo horário. Encerradas
as inscrições, verificou-se que: dos 85 inscritos em natação, 50 só
Programas Número de espectadores
farão natação; o total de inscritos para as aulas de tênis foi de 17 e, para
E 400 futebol, de 38; o número de inscritos só para as aulas de futebol excede
N 1220 em 10 o número de inscritos só para as de tênis. Quantos associados se
H 1080 inscreveram simultaneamente para aulas de futebol e natação?
EeN 220
NeH 800 11 (UFRJ) Uma amostra de 100 caixas de pílulas anticoncepcionais
EeH 180 fabricadas pela Nascebem S.A. foi enviada para a fiscalização sanitária.
E, N e H 100 No teste de qualidade, 60 foram aprovadas e 40 reprovadas, por conterem
pílulas de farinha. No teste de quantidade, 74 foram aprovadas e 26
Através desses dados, calcule o número de pessoas da comunidade reprovadas, por conterem um número menor de pílulas que o especificado.
que não assiste a qualquer dos três tipos de programa. O resultado dos dois testes mostrou que 14 caixas foram reprovadas em
ambos os testes. Quantas caixas foram aprovadas em ambos os testes?
07 (FUVEST) Depois de n dias de férias, um estudante observa que:
12 (UFRJ) Os 87 alunos do 3o ano do ensino médio de uma certa escola
I. Choveu 7 vezes, de manhã ou à tarde. prestaram vestibular para três universidades: A, B e C. Todos os alunos
II. Quando chove de manhã não chove à tarde. dessa escola foram aprovados em pelo menos uma das universidades,
III. Houve 5 tardes sem chuva. mas somente um terço do total obteve aprovação em todas elas. As provas
IV. Houve 6 manhãs sem chuva. da universidade A foram mais difíceis e todos os alunos aprovados nesta
foram também aprovados em pelo menos uma das outras duas.
Então n é igual a: Os totais de alunos aprovados nas universidades A e B foram,
respectivamente, 51 e 65. Sabe-se que, dos alunos aprovados em B,
08 (UDESC) O Festival de Dança de Joinville é considerado o maior do 50 foram também aprovados em C. Sabe-se também que o número de
mundo pelo Guinness Book of Records de 2005. Desde 1998, este festival aprovados em A e em B é igual ao de aprovados em A e em C.
é realizado no Centreventos Cau Hansen, que tem capacidade para 4.200 Quantos alunos foram aprovados em apenas um dos três vestibulares
pessoas por noite. prestados? Justifique.
Suponha que no 28o Festival de Dança, realizado em julho de 2010,
houve uma noite exclusiva para cada uma das seguintes modalidades: 13 (ITA) Sejam A um conjunto com 8 elementos e B um conjunto tal
ballet, dança de rua e jazz. A noite da dança de rua teve seus ingressos que A ∪ B contenha 12 elementos. Então, o número de elementos de
esgotados; na noite do jazz restaram 5% dos ingressos; e a noite do ballet P (B \ A) ∪ P (∅) é igual a:
teve 90% dos ingressos disponíveis vendidos. Sabe-se que algumas
pessoas costumam prestigiar mais de uma noite do Festival. Neste ano, 700 (A) 8.
pessoas assistiram à dança de rua e ao jazz; 1.610 assistiram ao ballet e à (B) 16.
dança de rua; 380 assistiram ao ballet e ao jazz e 105 prestigiaram as três (C) 20.
modalidades de dança. Se todas as pessoas que adquiriram os ingressos (D) 17.
do Festival assistiram à(s) apresentação(ões), então o número total de (E) 9.
pessoas distintas que assistiu a pelo menos uma das três modalidades
anteriormente mencionadas foi: 14 (UDESC) Considere em um conjunto universo, com 7 elementos, os
subconjuntos A, B e C, com 3, 5 e 7 elementos, respectivamente. É correto
(A) 9385. afirmar que:
(B) 9070.
(C) 9595. (A) (A  B)  C tem no máximo 2 elementos.
(D) 6275. (B) (A  B)  C tem no mínimo 1 elemento.
(E) 6905. (C) C  B tem 3 elementos.
(D) A  C tem 2 elementos.
09 (UFF) Considere T o conjunto de todas as pessoas do mundo; M o (E) A  B pode ser vazio.
conjunto de todas aquelas que são muçulmanas e A o conjunto de todas
aquelas que são árabes. Sabendo que nem toda pessoa que é muçulmana 15 (AFA) Em um grupo de n cadetes da Aeronáutica, 17 nadam, 19 jogam
é árabe, pode-se representar o conjunto de pessoas do mundo que não basquetebol, 21 jogam voleibol, 5 nadam e jogam basquetebol, 2 nadam e
são muçulmanas nem árabes por: jogam voleibol, 5 jogam basquetebol e voleibol e 2 fazem os três esportes.
Qual o valor de n, sabendo-se que todos os cadetes desse grupo praticam
(A) T – (A  M). pelo menos um desses esportes?
(B) T – A.
(C) T – (A  M). (A) 31.
(D) (A – M)  (M – A). (B) 37.
(E) M – A. (C) 47.
(D) 51.

IME-ITA 161
Matemática I – Assunto 1

16 (AFA) Entrevistando 100 oficiais da AFA, descobriu-se que 20 deles 18 (EFOMM) Foi feita uma pesquisa entre 50 alunos de uma turma sobre
pilotam a aeronave TUCANO, 40 pilotam o helicóptero ESQUILO e 50 não suas preferências em relação a dois professores A e B. O resultado foi o
são pilotos. Dos oficiais entrevistados, quantos pilotam o TUCANO e o seguinte:
ESQUILO?
I. Vinte alunos preferiram o professor A.
(A) 5. (C) 15. II. Trinta e cinco alunos preferiram o professor B.
(B) 10. (D) 20. III. Cinco alunos não tiveram preferência.

17 (EFOMM) Representando graficamente o conjunto CBA∩ C, temos: Baseado nesse resultado, pode-se afirmar que o número de alunos que
preferiu os dois professores foi:
(A)
A C (A) 5.
(B) 10.
(C) 15.
(D) 20.
(E) 25.

19 (EFOMM) Dados os conjuntos:

B A = {x ∈ R | –2 < x ≤ 4}
(B) B = {x ∈ R | –1 ≤ x < 3}
A C C = {x ∈ R | –3 ≤ x < 5}
B = {x ∈ R | x ≥ 0}

O resultado de ( A ∩ CCB ) ∪( D ∩ CAB ) é :

(A) [3,4]
(B) ]–2, –1[  [3, 4]
(C) [–2, –1]  [3,5]
(D) ]–2, 4]  [5, + ∞]
B (E) ]–3, –1]
(C)
A C 20 (AFA – Adaptada) Considere um subconjunto A contido em * e
constituído por y elementos dos quais 13 são múltiplos de 4; 7 são múltiplos
de 10; 5 são múltiplos de 20 e 9 são números ímpares. É correto dizer
que y é um número.

(A) par menor que 19.


(B) De 24 a 29.
(C) ímpar entre 10 e 20.
B (D) primo maior que 21.

(D) 21 (CN) Considere os conjuntos A = {1, {1}, 2} e B = {1, 2,{2}} e as cinco


A C afirmações:

I. A – B = {1}
II. {2} ⊂ B ∩ AC
III. {1} ⊂ A
IV. A ∩ B = {1, 2, {1, 2}}
V. B – A = {{2}}

B Logo,
(E) (A) todas as afirmações estão erradas.
A C
(B) se existe apenas uma afirmação correta.
(C) as afirmações ímpares estão corretas.
(D) as afirmações III e V estão corretas.
(E) as afirmações I e IV são as únicas incorretas.

162 Vol. 1
Conjuntos

EXERCÍCIOS NÍVEL 2
01 (CMRJ) Considere o conjunto C = {1, 2, 3}. Para n ∈ C, sejam: 07 (EN) Considere os conjuntos A = {x} e B = {x,{A}} e as proposições:
An = {x ∈ R |2n – 2 < x < 2n} e Bn = {x ∈ R|2n – 1 < x < 2n + 1}.
Podemos afirmar que: I. {A} ∈ B;
II. {x} ∈ A;
(A) a interseção da união dos conjuntos An com a união dos conjuntos Bn III. A ∈ B;
é o intervalo ]0, 7]. IV. B ⊂ A
(B) a união de todos os conjuntos da forma An ∩ Bn é o intervalo ]1, 6[. V. {x , A} ⊂ B
(C) a interseção de todos os conjuntos da forma An ∪ Bn é vazia.
(D) a união da interseção dos conjuntos An com a interseção dos conjuntos As proposições falsas são:
Bn é o intervalo ]2, 4[.
(E) a interseção da interseção dos conjuntos An com a interseção dos (A) I, III e V.
conjuntos Bn é o intervalo ]1, 7[. (B) II, IV e V.
(C) II, III, IV e V.
02 Seja P o conjunto das pessoas em uma festa. Para cada pessoa x ∈ (D) I, III, IV e V.
P, vamos definir o subconjunto Ax ⊂ P como o conjunto dos amigos de x, (E) I, III e IV.
isto é, Ax = {y ∈ P; y amigo de x}. Estamos considerando aqui que, se
x é amigo de y, então y também é amigo de x e também que x ∈ Ax (x é  1
08 (EN) Se Ah é o intervalo 0, , h ∈  então A1 ∩ A2 ∩ A3 ∩ A4 ... é:
amigo de si próprio). Assinale a alternativa incorreta.
 h 
(A) Se x e y tem um amigo em comum, então Ax ∩ Ay ≠ ∅ .
(B) Se a interseção de todos os subconjuntos Ax é não vazia (  A ≠ ∅ ),
x ∈P
x
(A) {};
 1
(B) 0,  ;
então existe alguém que é amigo de todas as pessoas da festa.  h
(C) Se  Ax ≠ ∅ , então existe uma pessoa z, tal que Az = P .
x ∈P (C) é um conjunto unitário;
(D) Se x ∈ Ay e y ∈ Az então, necessariamente, x ∈ Az .  1
(D) 0,  ;
(E)  ( Ax − { x }) pode ser diferente de P, pois pode ocorrer que alguém  h
x ∈P
(E) n.r.a.
não possua amigos na festa (além de si próprio).
09 Seja ℘(A) o conjunto de todos os subconjuntos do conjunto A. Sobre
03 (ITA) Denotemos por n(X) o número de elementos de um conjunto
as afirmativas:
finito X. Sejam A, B e C conjuntos tais que n(A ∪ B) = 8, n(A ∪ C) = 9,
n(B ∪ C) = 10, n(A ∪ B ∪ C) = 11 e n(A ∩ B ∩ C) = 2.
I. A ∈ ℘(A);
Então, n(A) + n(B) + n(C) é igual a:
II. Se A ⊂ B, então A ∈℘(B);
III. Se A ∈℘(B) e B ∈℘(A), então A = B;
(A) 11. (D) 18.
IV. Se A∈℘(B) e B∈℘(C), então A ⊂ C.
(B) 14. (E) 25.
(C) 15.
Podemos concluir que o número de sentenças verdadeiras é:
04 (UERJ) Considere um grupo de 50 pessoas que foram identificadas
(A) 0.
em relação a duas categorias: quanto à cor dos cabelos, louras ou
(B) 1.
morenas; quanto à cor dos olhos, azuis ou castanhos. De acordo com
(C) 2.
essa identificação, sabe-se que 14 pessoas no grupo são louras com
(D) 3.
olhos azuis, que 31 pessoas são morenas e que 18 têm olhos castanhos.
(E) 4.
Calcule, no grupo, o número de pessoas morenas com olhos castanhos.
10 (CN) Numa cidade, 28% das pessoas têm cabelos pretos e 24%
05 (UFU) Sejam A, B e C conjuntos de números inteiros, tais que A tem
possuem olhos azuis. Sabendo que 65% da população de cabelos pretos
8 elementos, B tem 4 elementos, C tem 7 elementos e A  B  C tem
têm olhos castanhos e que a população de olhos verdes que tem cabelos
16 elementos. Então, o número máximo de elementos que o conjunto
pretos é 10% do total de pessoas de olhos castanhos e cabelos pretos,
D = (A  B)  (B  C) pode ter é igual a:
qual a porcentagem do total de pessoas de olhos azuis, que tem os cabelos
pretos?
(A) 1. (C) 3.
(B) 2. (D) 4.
Obs.: Nesta cidade só existem pessoas de olhos azuis, verdes ou
castanhos.
06 (UFSJ) Assinale a alternativa que indica quantos são os números
inteiros de 1 a 21.000, que não são divisíveis por 2, por 3 e nem por 5.
(A) 30,25%.
(B) 31,25%.
(A) 6.300.
(C) 32,25%.
(B) 5.600.
(D) 33,25%.
(C) 7.000.
(E) 34,25%.
(D) 700.

IME-ITA 163
Matemática I – Assunto 1

11 Sejam A, B e C subconjuntos de um conjunto universo U. Das é (são) verdadeira(s):


afirmações:
(A) apenas I.
I. Se A ∩ B = A ∩ C, então B = C. (B) apenas II.
II. ((A ∪ B) ∩ C) ∩ ((A ∪ BC) ∩ C) = A ∩ C (C) apenas I e II.
III. (A ∪ B ∪ C)C = (A ∪ C)C ∩ (A ∪ B)C (D) apenas II e III.
(E) todas.
é (são) verdadeira(s):
14 Sejam A, B, C e D subconjuntos de um conjunto universo U. Das
(A) apenas II. afirmações:
(B) apenas III.
(C) apenas I e II. I. Se A ∈ B e B ∈ C, então A ∈ C
(D) apenas II e III. II. Se A ∈ B e B ⊂ C , então A ∈ C
(E) todas. III. P ({A ∩ [(BC \ CC) ∪ D]} ∩ [(DC \ A) ∩ (C \ B)] é unitário
IV. P(A ∪ B) = {A1 ∪ B1 | A1 ∈ P(A) e B1 ∈ P(B)}
12 Prove que se A∆B = A∆C, então B = C .
o número de afirmações verdadeiras é:
13 Sejam A, B, C e D subconjuntos de um conjunto universo U. Das
afirmações: (A) 0.
(B) 1.
I. (A ∩ B) ∪ (C ∩ D) = (A ∪ C) ∩ (B ∪ C) ∩ (A ∪ D) ∩ (B ∪ D) (C) 2.
II. A \ (B ∩ C) = (A \ B) ∪ (A \ C) (D) 3.
III. Se (A \ B) ∪ B = A, então B = A. (E) 4.

EXERCÍCIOS NÍVEL 3

01 Dados os conjuntos A e B, seja X um conjunto com as seguintes A ∩ X = B


05 Sejam A, B, C conjuntos tais que B ⊂ A ⊂ C. Seja X tal que  .
propriedades: Prove que X = (C \ A) ∪ B. A ∪ X = C

I. A e B estão contidos em X. 06 Sejam A, B, C conjuntos tais que B ⊂ A e A e C são disjuntos. Seja X


II. Se A está contido em Y e B está contido em Y, então X está contido A\ X = B
em Y. tal que  . Prove que X = (A \ B) ∪ C.
X \ A = C
Prove que X = A ∪ B.

02 (EN) Se 70% da população gostam de samba, 75% de choro, 80% 07 (AIME) Dados conjuntos A, B e C, sejam |A|,|B|,|C| suas
de bolero e 85% de rock, quantos por cento da população, no mínimo, quantidades de elementos, respectivamente, e sejam s(A), s(B), s(C)
gostam de samba, choro, bolero e rock? suas quantidades de subconjuntos, respectivamente. Sabendo que
|A| = |B| = 100 e s(A) + s(B) + s(C) = s(A∪B∪C), determine:
03 Em Porto Alegre foi feita uma pesquisa com a população sobre suas
bebidas prediletas e o resultado foi: a. |C|
– 60% tomam refrigerante (A) 70% tomam vinho (B) b. |A ∪ B ∪ C|
– 80% tomam café (C) 90% tomam chimarrão (D) c. o valor mínimo de |A ∩ B ∩ C|

Verifica-se ainda que nenhuma pessoa consome as quatro bebidas. Qual


a percentagem das pessoas que consomem refrigerante ou vinho?

04 Em um concurso, 3 problemas A, B e C foram propostos. Dos


concorrentes, 25 resolveram pelo menos 1 problema. Dos concorrentes
que não resolveram o problema A, o número dos que resolveram B foi
o dobro do número dos que resolveram C. O número de estudantes
que resolveram apenas o problema A foi uma unidade maior do que o
número dos que resolveram A e pelo menos um outro problema. Dos que
resolveram apenas 1 problema, metade não resolveu A. Calcule o número
de estudantes que resolveram apenas o problema B.

164 Vol. 1
Lógicas e Técnicas
de demonstração
A ssunto
2
Matemática I

1. Conceitos
II. p: Todos os homens são elegantes.
~p: Nem todos os homens são elegantes.
1.1 Proposição
É toda oração declarativa que exprime uma (proposição simples) No exemplo 1, em termos de conjuntos, sendo A = {x | x é irmão de
ou mais (proposição composta) informações. Uma proposição sempre Roberto}, p significa x ∈ A. Sua negação é x ∉ A.
é verdadeira ou falsa, nunca ambas simultaneamente (Princípio da não
contradição) e também não admite uma terceira hipótese (Princípio do 2.2 Disjunção (∨) / União (∪)
terceiro excluído).
Chama-se disjunção de duas
p q p∨q
Ex.: A lua é um satélite da Terra (verdadeira); Vasco da Gama descobriu proposições p e q a proposição
o Brasil (falsa) representada por “p ou q”, cujo V V V
valor lógico é a verdade (V) quando V F V
1.2 Proposição funcional ao menos uma das proposições é
verdadeira e é a falsidade quando F V V
p(x) é uma proposição funcional num dado conjunto U quando ela
assumir valores verdadeiros ou falsos a partir dos elementos de U. O as proposições são ambas falsas. F F F
conjunto dos valores para os quais uma proposição funcional é definida Representamos a disjunção de p e q
denomina-se seu conjunto-universo e o conjunto de valores para os quais por p ∨ q. Podemos resumir as informações em uma tabela verdade:
a proposição é verdadeira denomina-se seu conjunto-verdade ou solução.
Ex.: p: João é irmão de Roberto.
Ex.: p(x): o número natural x é par. q: Maria não é mãe de João.
p ∨ q: João é irmão de Roberto ou Maria não é mãe de João.
1.3 Conectivos
Chamam-se conectivos palavras usadas para formar novas Considerando as proposições: p: x ∈ A, q: x ∈ B, veja que p ∨ q
proposições a partir de outras. Os conectivos usuais são: “e”, “ou”, “não”, significa x ∈ A ∪ B
“se ... então ...”, “ ... se e somente se ...”.
Obs.: Disjunção Exclusiva(∨)
Ex.: O triângulo ABC é equilátero se e somente se é equiângulo.

1.4 Tabela verdade O valor lógico da disjunção exclusiva p q p∨q


é a verdade somente quando p é
É o dispositivo que permite a determinação dos valores lógicos de verdadeira ou q é verdadeira, mas não V V F
uma proposição composta, isto é, que possui mais de uma informação, quando p e q são ambas verdadeiras. V F V
a partir dos valores lógicos das proposições simples que a compõem. O
número de linhas de uma tabela verdade é 2n, em que n é o número de F V V
Ex.: p: João viajou de ônibus.
proposições simples (cada proposição simples admite 2 valores). F F F
q: João viajou de avião.
p ∨ q: João viajou de ônibus ou de
Obs.: Quando uma proposição composta só admite valores verdadeiros avião.
na última coluna, dizemos que ela é uma tautologia, quando só admite
valores falsos, dizemos que é uma contradição e quando admite ambos 2.3 Conjunção (∧) / Interseção (∩)
os valores lógicos, dizemos que é uma contingência.
Chama-se conjunção de duas
p q p∧q
proposições p e q a proposição
2. Operações Lógicas representada por “p e q”, cujo valor V V V
lógico é a verdade (V) quando as V F F
2.1. Negação (~) / Complementar (AC) proposições são ambas verdadeiras
e a falsidade (F) nos demais casos. F V F
Chama-se negação de uma proposição p
p ~p Representamos a conjunção de p e q por F F F
a proposição representada por “não p”, cujo
p ∧ q. Podemos resumir as informações
valor lógico é verdade (V) quando p é falsa V F
em uma tabela verdade:
(F) e é falso (F) quando p é verdadeira. Assim, F V
“não p” tem o valor lógico oposto ao de p.
Representamos a negação de p por ~p. Podemos resumir as informações Ex.: p: Maria comprou bala.
em uma tabela verdade: q: Maria comprou chiclete.
p ∧ q: Maria comprou bala e chiclete.
Ex.: Considerando as proposições: p: x ∈ A, q: x ∈ B, veja que p ∧ q
significa x ∈ A ∩ B.
I. p: João é irmão de Roberto.
~p: João não é irmão de Roberto.

IME-ITA 165
Matemática I – Assunto 2

2.4 Condicional (→) / Inclusão (⊂) 2.5 Bicondicional (↔) / Igualdade (=)
Chama-se condicional uma p q p→q Chama-se bicondicional uma p q p↔q
proposição representada por “se p então
V V V proposição representada por “p se
q”, cujo valor lógico é a falsidade (F) no V V V
V F F e somente se q”, cujo valor lógico
caso em que p é verdadeira e q é falsa
F V V é a verdade (V) quando ambas são V F F
e é a verdade (V) nos demais casos.
verdadeiras ou ambas são falsas e F V F
Representamos a condicional por p → F F V
é a falsidade (F) nos demais casos.
q. Podemos resumir as informações em F F V
Representamos a bicondicional por
uma tabela verdade:
p ↔ q. Podemos resumir as informações
em uma tabela verdade:
Ex.: p: Roberto ingressará no IME.
q: Roberto ganhará um carro.
Ex.: p: ABCD é um paralelogramo
p → q: Se Roberto ingressar no IME, então Roberto ganhará um carro.
q: As diagonais de ABCD se cortam ao meio.
Obs. 1: Atenção! Um erro comum é achar que se Roberto ganhou um p ↔ q: ABCD é um paralelogramo se e somente se as suas diagonais
carro, ele passou no IME. Não podemos concluir isso a partir da frase: se cortam ao meio.
“Se Roberto ingressar no IME, então Roberto ganhará um carro.”
Obs.: Um teorema do tipo p ↔ q tem duas par tes a serem
Obs. 2: Para demonstrar um teorema do tipo p → q, o que se faz é supor demonstradas: p → q e q → p.
que p é verdadeira e a partir daí concluir que q também é.

3. Construção de tabelas verdade


Analisaremos as duas proposições a seguir:

3.1 P: ~((~p) ∧ (~q))

p q ~p ~q (~p) ∧ (~q) ~((~p) ∧ (~q))

V V F F F V

V F F V F V

F V V F F V

F F V V V F

3.2 P(p, q, r): (p → ((~q) ∨ r)) ∧ ~(q ∨ (p ↔ (~r)))

p q r ~q ~q ∨ r p → (~q ∨ r) ~r p↔~r q∨(p↔~r) ~(q∨ (p↔~r)) ∧

V V V F V V F F V F F

V V F F F F V V V F F

V F V V V V F F F V V

V F F V V V V V V F F

F V V F V V F V V F F

F V F F F V V F V F F

F F V V V V F V V F F

F F F V V V V F F V V

Obs.: Podemos usar tabelas verdade para demonstrar igualdades temos 4 possibilidades a testar). Veremos como este método pode ser
entre conjuntos (esta é uma saída mecânica, que é bastante eficiente empregado nos exercícios resolvidos.
principalmente para igualdades envolvendo apenas 2 conjuntos, pois só

166 Vol. 1
Lógicas e Técnicas
de demonstração

4. Quantificadores Ex.: Prove que 2 não é racional.

4.1 Quantificador universal Solução:


(∀x) (p(x)). Lê-se: Para todo x, vale a proposição p(x). Suponha, por absurdo, que 2 seja racional. Nesse caso, existiriam
a
a e b inteiros tais que 2 = e a e b não têm fatores comuns (podemos
Ex.: (∀x > 0) ; (2 x > x) (Para todo x > 0, tem-se 2x > x) b
pegar a fração irredutível). Daí, elevando ao quadrado, temos que a2 =
4.2 Quantificador existencial 2b2. Daí, temos:
(∃x) (p(x)). Lê-se: Existe x, tal que vale a proposição p(x).
a2 par ⇒ a par ⇒ a = 2 k ⇒ ( 2 k ) = 2 b2 ⇒ 2 k 2 = b2 ⇒ b2 par ⇒ b par .
2

Ex.: (∃k ∈ ) ; (8k + 1 é primo). (Existe k natural tal que 8k + 1 é primo.)


Veja então que a e b são pares, o que é uma contradição a ‘a e b não têm
fatores comuns’.
5. Negação das operações lógicas Portanto, devemos ter que 2 não é racional.
I. ~(~p) ↔ p 6.2 Contrapositiva
II. ~(p ∨ q) ↔ (~p) ∧ (~q)
Podemos mostrar, através de uma tabela verdade, que provar um
III. ~(p ∧ q) ↔ (~p) ∨ (~q)
teorema do tipo p → q é equivalente a demonstrar ~q → ~p.
IV. ~(p → q) ↔ p ∧ (~q)
V. ~(p ↔ q) ↔ (p ∧ (~q)) ∨ ((~p) ∧ q)
Ex.: Se o produto de dois números reais positivos é maior que 100, então
VI. A negação do quantificador universal é o quantificador existencial, isto
pelo menos um dos números é maior que 10.
é, (~∀) ⇔ ∃
VII. A negação do quantificador existencial é o quantificador universal, isto
é, (~∃) ⇔ ∀ Temos p: xy > 100 e q: x > 10 ou y > 10. O teorema que queremos
provar é p → q. Para isso, provaremos a contrapositiva ~q → ~p.
Ex.: Como ~p: xy ≤ 100 e ~q: x ≤ 10 e y ≤ 10, basta provarmos que se x
I. ~p: Nem todos os homens são elegantes. ≤ 10 e y ≤ 10, então xy ≤ 100, o que segue multiplicando-se as duas
Negação: Todos os homens são elegantes. desigualdades.

II. p ∨ q : 2 > 5ou Santos é a capital de São Paulo. 6.3 Princípio da indução finita
p: 2 > 5 ; q: Santos é a capital de São Paulo. O princípio da indução finita (PIF) é muito útil para se provar propriedades
~p: 2 ″ 5 ; ~q: Santos não é a capital de São Paulo. referentes aos números naturais. A ideia do princípio consiste no seguinte:
Negação de p ∨ q: 2 ″ 5 e Santos não é a capital de São Paulo. se sabemos que uma propriedade vale para determinado natural n0 e, além
disso, também sabemos que se a propriedade é válida para um natural
III. p ∧ q: Brasília é a capital do Brasil e (20 = 0 ou 30 = 1). n ≥ n0 , então ela é válida para n + 1, podemos concluir que tal propriedade
p: Brasília é a capital do Brasil; q: 20 = 0 ou 30 = 1. é válida para todos os naturais maiores ou iguais a n0 (o funcionamento
~p: Brasília não é a capital do Brasil; ~q: 20 ≠ 0 e 30 ≠ 1. da indução é como um efeito dominó, em que uma pecinha empurra a
Negação de p ∧ q: Brasília não é a capital do Brasil ou (20 ≠ 0 e 30 ≠ 1). outra, derrubando todas).

IV. p → q: Se 3 + 2 = 6, então 4 + 4 = 9. Podemos enunciar o princípio como a seguir:


p: 3 + 2 = 6; q: 4 + 4 = 9.
~q: 4 + 4 ≠ 9 Primeiro princípio da indução finita:
Negação de p → q: 3 + 2 = 6 e 4 + 4 ≠ 9
I. (Base da indução) Inicialmente, mostra-se que a proposição é válida
V. p ↔ q: tanπ = 0 se, e somente se, sen π = 0. para n = n0.
p: tan π = 0; q: sen π = 0
~p: tan π ≠ 0; q: sen π ≠ 0 II. (Hipótese de indução) Supõe-se que a proposição vale para n ≥ n0;
Negação de p ↔ q: (tan π = 0) e sen π ≠ 0 ou (tan π ≠ 0 e sen π = 0).
III. (Passo indutivo) A partir da hipótese, conclui-se que a proposição
VI. p: Todo brasileiro é magro. vale para n + 1.
~p: Existe brasileiro que não é magro.
Obs.: O PIF é um dos axiomas de Peano, que são os axiomas que dão
VII. p: (∃x) (|x| < 0) base aos números naturais.
~p: (∀x) (|x| ≥ 0)
Obs. 2: Temos uma segunda versão do PIF, que é a seguinte:
6. Técnicas de demonstração
(Segundo princípio da indução finita / Indução forte):
6.1 Redução ao absurdo
I. (Base da indução) Inicialmente, mostra-se que a proposição é válida
O argumento do absurdo é bastante simples. Consiste em negar para n = n0.
a proposição do problema (por isso a importância de saber negar II. (Hipótese de indução) Supõe-se que a proposição vale para todo
proposições!) e então chegar a uma contradição. n0 ≤ k ≤ n.

IME-ITA 167
Matemática I – Assunto 2

III. (Passo indutivo) A partir da hipótese, conclui-se que a proposição k( k + 1)( 2 k + 1)


III. Passo indutivo: Como 12 + 22 + ... + k 2 = , somando
vale para n + 1. 6
(k + 1)2 aos dois lados, temos que:
n( n + 1)( 2 n + 1)
Ex.: Prove, por indução, que 12 + 22 + ... + n2 = .
6 k( k + 1)( 2 k + 1)
12 + 22 + ... + k 2 + ( k + 1)2 = + ( k + 1)2 =
Solução: 6
Usaremos o 1o PIF: ( k + 1)[ k( 2 k + 1) + 6( k + 1)] ( k + 1)( 2 k 2 + 7 k + 6)
= = =
6 6
1 ⋅ (1 + 1) ⋅ ( 2 ⋅ 1 + 1) 1⋅ 2 ⋅ 3 ( k + 1)( k + 2)( 2 k + 3)
I. Base: n = 1 : 12 = ⇔ 1= = 1 (OK !) = .
6 6 6
k( k + 1)( 2 k + 1)
II. Hipótese de indução: suponha que12 + 22 + ... + k 2 =
6 Isso conclui a indução.

Obs.: É muito importante saber esta fórmula de antemão.

EXERCÍCIOS RESOLVIDOS
01 (OBM) Há três cartas viradas sobre uma mesa. Sabe-se que em p q r q ∨ r p ∧ q p ∧ r p ∧ (q ∨ r) (p ∧ q) ∨ (p ∧ r) ↔
cada uma delas está escrito um número inteiro positivo. São dadas a
V V V V V V V V V
Carlos, Samuel e Tomás as seguintes informações:
V V F V V F V V V
I. todos os números escritos nas cartas são diferentes; V F V V F V V V V
II. a soma dos números é 13; V F F F F F F F V
III. os números estão em ordem crescente, da esquerda para a direita. F V V V F F F F V
F V F V F F F F V
Primeiro, Carlos olha o número na carta da esquerda e diz: “Não tenho
F F V V F F F F V
informações suficientes para determinar os outros dois números.” Em
F F F F F F F F V
seguida, Tomás olha o número na carta da direita e diz: “Não tenho
informações suficientes para determinar os outros dois números.” Como obtemos uma tautologia, a igualdade segue.
Por fim, Samuel olha o número na carta do meio e diz: “Não tenho
informações suficientes para determinar os outros dois números.” 03 Para n inteiro, prove que se n2 é par, então, n é par.
Sabendo que cada um deles sabe que os outros dois são inteligentes e
escuta os comentários dos outros, qual é o número da carta do meio? Solução:
Temos uma implicação p → q e, neste caso, é mais fácil demonstrar
Solução:
a sua contrapositiva ~q → ~p que é ‘se n é ímpar, então n2 é ímpar’.
Sejam x, y e z os números das cartas da esquerda, do meio e da direita,
Veja que se n é ímpar, temos que n = 2k + 1, com k ∈ Z. Daí,
respectivamente. Temos que x < y < z e x + y + z = 13. Assim,
n2 = (2k + 1)2 = 4k(k + 1) + 1 e podemos concluir que n2 é ímpar.
x + x + x < x + y + z ⇒ x ≤ 4. Observemos que x ≠ 4 (se x = 4,
teríamos y = x). Se x = 3, Carlos concluiria que y = 4 e z = 6, portanto,
n ( n + 1)
x ≠ 3. Assim, x = 1 ou x = 2 e; portanto, y + z ≥ 11. Como 2 < y < 04 Prove, por indução, que 1 + 2 + 3 + ... + n = , para todo n
z, conclui-se que 6 ≤ z ≤ 9. Se z = 6, Tomás concluiria que y = 5 e natural. 2
x = 2, portanto z ≠ 6. Se z = 9, Tomás concluiria que x = 1 e y = 3.
Assim, z = 7 ou z = 8. Solução:
Neste momento, Samuel poderia achar todas as possíveis soluções. Se n ( n + 1)
x = 1 e z = 7, teríamos y = 5; se x = 1 e z = 8, teríamos y = 4; se Seja P(n) a afirmativa 1 + 2 + 3 + ... + n = . Inicialmente, veja
2
x = 2 e z = 7, teríamos y = 4; se x = 2 e z = 8, teríamos y = 3.
Assim, Samuel saberia que os possíveis valores de y são 3, 4 e 5. Ora, 1 ⋅2
se y = 3 ou y = 5, Samuel descobriria os números (se y = 3, Samuel que P(1) : 1 = é verdadeira. O princípio da indução finita consiste
concluiria que x = 2 e z = 8; se y = 5, Samuel concluiria que x = 1 2
e z = 7). Logo, o número da carta do meio é 4. em trocar a dificuldade de provar que P(n) é verdadeira por provar que
 P ( n ) ⇒ P ( n + 1)
02 Propriedade distributiva – Prove que A ∩ (B ∪ C) = (A ∩ B)  . Ou seja, agora que já sabemos que P(1) é verdade,
∪ (A ∩ C).  P (1) é verdade
precisamos provar que sempre que P(n) for verdadeira, teremos P(n +
Solução: 1) verdadeira.
Sejam p: x ∈ A, q: x ∈ B e r: x ∈ C. Queremos provar que p ∧ (q ∨ r) Então, suponhamos que P(n) seja verdadeira para um certo valor natural
de n. Então, temos que 1 + 2 + 3 + ... + n = (
↔ (p ∧ q) ∨ (p ∧ r). Como temos 3 proposições, devemos construir n n + 1)
. Somando n + 1
uma tabela verdade com 23 = 8 linhas: 2
nos dois lados da equação, teremos que:

168 Vol. 1
Lógicas e Técnicas
de demonstração

n ( n + 1) n  ( n + 1)( n + 2) Solução:
1 + 2 + 3 + ... + n + ( n + 1) = + ( n + 1) = ( n + 1)  + 1 =
2 2  2 Nesse caso, cada termo da sequência depende dos dois anteriores. Por
isso, não é possível aplicar o princípio da indução finita em seu formato
que é exatamente a sentença P(n + 1). Portanto, pelo princípio da
tradicional. Neste caso, precisamos usar o que chamamos de ‘indução
indução finita, temos que P(n) é verdadeira para todo n natural.
forte’ ou ‘2o princípio de indução finita’.
Inicialmente, vejamos os dois casos iniciais: F6 = 8 > 6 e F7 = 13 >
Obs. 1: É muito importante verificar o caso inicial P(1), pois, sem ele,
7. Agora, suponha que já provamos que Fn > n para n = 6, 7, 8, ..., n.
a indução ‘não começa’.
Vamos provar agora que a propriedade também vale para n + 1:
Obs. 2:  Fn > n
 ⇒ Fn +1 = Fn + Fn −1 > n + n − 1 = 2 n − 1 > n + 1
Veja a simplicidade do princípio da indução finita. Ao sabermos que  Fn −1 > n − 1
 P ( n ) ⇒ P ( n + 1) (a última passagem precisa de n > 2 e já sabemos que n ≥ 6).
 , temos que P(1) ⇒ P(2) ⇒ P(3) ⇒ ... ⇒ P(n) ⇒ ...
Portanto, pelo 2o princípio da indução finita, temos que Fn > n para
 P (1) é verdade todo n ≥ 6.

Obs.: Na verdade, nem utilizamos todos os valores menores que n para


05 Considere a sequência de Fibonacci: 1, 1, 2, 3, 5, 8, 13, 21, ... fazer o passo de indução. Sendo P(n): F n > n, provamos que
 F1 = 1
definida por  e Fn = Fn–1 + Fn–2 para todo n natural maior ou  P ( n − 1) , P ( n ) ⇒ P ( n + 1)
 F2 = 1  e isso fecha o problema. Repare a
igual a 3. Prove que, para n ≥ 6, tem-se Fn > n.  P ( 6 ) , P ( 7 ) são verdadeiras
importância de exibirmos os dois casos iniciais, já que a indução precisa
exatamente desses dois casos para ‘começar’.

EXERCÍCIOS NÍVEL 1
01 Julgue como (verdadeiros ou falsos) os itens a seguir: 07 (EN) A negação da proposição “x ≠ 3 e y < 2” é:

a. x2 – 18x + 81 = 49 ⇔ x =16 (A) “x = 3 e y ≥ 2”. (D) “x ≠ 2 e y < 3”.


b. x2 – x – 12 ≠ 0 ⇔ x ≠ –3 ou x ≠ 4 (B) “x = 3 e y > 2”. (E) “x ≠ 3 ou y < 2”.
c. x2 – x – 12 ≠ 0 ⇔ x ≠ –3 e x ≠ 4 (C) “x = 3 ou y ≥ 2”.

02 Determine a contrapositiva das proposições abaixo: 08 (EN) Considere a proposição: “Se x > 5 então y = 6”. A proposição
equivalente é:
a. Se um quadrilátero é um quadrado, então ele é um retângulo.
b. Se um número é ímpar, então seu quadrado é ímpar. (A) “Se x < 5 então y ≠ 6”. (D) “Se y ≠ 6 então x ≤ 5”.
c. x1 ≠ x2 ⇒ ƒ(x1) ≠ ƒ(x2) (aqui, ƒ(x) representa um objeto qualquer (B) “Se y ≠ 6 então x < 5”. (E) “Se x ≤ 5 então y ≠ 6”.
associado a x). (C) “Se y > 5 então x = 5”.

03 Um aluno concluiu que 1 = 0 com a seguinte sequência de argumentos 09 (OBM) O programa “Quem não quer o bode?” ficou muito famoso
nos Estados Unidos. O programa era como a seguir: o participante deve
x = 1 ⇔ x 2 = x ⇔ x 2 − 1 = x − 1 ⇔ ( x + 1)( x − 1) = x − 1 ⇔ escolher uma dentre três portas. Atrás de uma das portas, há um carro
x + 1= 1⇔ x = 0 e atrás de cada uma das outras duas, há um bode. O convidado ganhará
Determine quais conectivos foram empregados de forma errada pelo aluno. o que estiver atrás da porta escolhida. Entretanto, os organizadores do
programa perceberam, com o passar do tempo, que aproximadamente
04 Prove, usando uma tabela verdade, as leis de De Morgan: (A ∪ B)C = dois em cada três participantes ganhavam o carro e, com isso, decidiram
AC ∩ BC e (A ∩ B)C = AC ∪ BC. mudar o programa. Agora, cada uma das três portas teria um números
de 1 a 3 e haveria um porteiro identificado com o número da porta. Cada
05 (EN) A negativa da proposição (∀x) (∀y) (x + y < 2 → porteiro faz uma afirmação que pode ser verdade ou mentira. Em seguida,
(x ≥ 0 ∨ y < 0)) é: o participante escolhe a porta na qual acredita que o carro está. Em um
dos programas, foram ditas as seguintes afirmações pelos porteiros:
(A) (∃x) (∀y) (x + y ≥ 2 → (x < 0 ∨ y ≥ 0)).
(B) (∃x) (∃y) (x + y < 2 → (x < 0 ∧ y ≥ 0)). • Porteiro 1: O carro não está atrás da porta 3.
(C) (∃x) (∃y) (x + y < 2 ∧ (x < 0 ∧ y ≥ 0)). • Porteiro 2: O carro está atrás da minha porta.
(D) (∀x) (∃y) (x + y ≥ 2 → (x ≥ 0 ∧ y ≥ 0)). • Porteiro 3: O carro não está atrás da minha porta.
(E) (∃x) (∃y) (x + y ≥ 2 ∧ (x < 0 ∨ y ≥ 0)).
Sabe-se que pelo menos uma das afirmações é verdade e que pelo menos
06 (EN) Dada a proposição p ∧ (q ∨ r) ⇔ (p ∧ q) ∨ (p ∧ r), podemos uma é mentira. Atrás de qual porta está o carro?
afirmar que é:
(A) Porta 1. (D) Não é possível identificar.
(A) logicamente falsa. (D) equivalente a (p ⇔ q) ∨ r. (B) Porta 2. (E) Não é possível que esteja em nenhuma delas.
(B) uma tautologia. (E) equivalente a ( p ∨ q ) ⇔ r . (C) Porta 3.
(C) equivalente a (p ∨ q) ⇔ r.

IME-ITA 169
Matemática I – Assunto 2

10 (OBM) Qual é o produto da quantidade de vogais pela quantidade de 12 (OBM) Quatro amigos, Arnaldo, Bernaldo, Cernaldo e Dernaldo estão
consoantes na alternativa correta? (Não considere as letras A, B, C, D, E jogando cartas. São 20 cartas diferentes, cada carta tem uma entre 4
das alternativas na contagem.) cores (azul, amarelo, verde, vermelho) e um número de 1 a 5. Cada amigo
recebe cinco cartas, de modo que todas as cartas são distribuídas. Eles
(A) Vinte e quatro. fazem as seguintes afirmações:
(B) Trinta e seis.
(C) Quarenta e dois. Arnaldo: “Eu tenho quatro cartas com o mesmo número.”
(D) Quarenta e oito. Bernaldo: “Eu tenho as cinco cartas vermelhas.”
(E) Cinquenta e seis. Cernaldo: “As minhas cinco cartas são de cores que começam com
a letra V.”
11 (OBM) No Planeta Nérdia, existem três espécies de nerds: ET-nerds, Dernaldo: “Eu tenho três cartas de um número e duas cartas de outro
UFO-nerds e OVNI-nerds. A primeira mente quando chove e diz a verdade número.”
quando não chove; a segunda sempre mente; a terceira sempre diz a
verdade. Certo dia, Bruberson, um nerd muito camarada, se encontra com Sabe-se que somente uma das afirmações é falsa. Quem fez essa
quatro nerds. E eles falam: afirmação?

X: "Hoje está chovendo." (A) Arnaldo. (D) Dernaldo.


Y: "O nerd que acabou de falar está mentindo." (B) Bernaldo. (E) Não é possível definir.
Z: "Hoje não está chovendo." (C) Cernaldo.
W: "O primeiro nerd mentiu ou eu sou um ET-nerd."
13 (OBM) Sempre que Agilulfo volta para casa depois da escola com uma
Com quantos ET-nerds Bruberson falou no máximo? advertência, se sua mãe está em casa, ela o coloca de castigo. Sabendo-se
que ontem à tarde Agilulfo não foi colocado de castigo, qual das seguintes
(A) 0. afirmações é certamente verdadeira?
(B) 1.
(C) 2. (A) Agilulfo recebeu advertência ontem.
(D) 3. (B) Agilulfo não recebeu advertência ontem.
(E) 4. (C) Ontem à tarde a sua mãe estava em casa.
(D) Ontem à tarde a sua mãe não estava em casa.
(E) Nenhuma das afirmações acima é certamente verdadeira.

EXERCÍCIOS NÍVEL 2

01 (OBM) A figura a seguir foi recortada em cartolina e depois dobrada 02 (OBM) A figura representa uma barra de chocolate que tem um
para formar um icosaedro. As faces em branco foram numeradas de modo amendoim apenas num pedaço. Elias e Fábio querem repartir o chocolate,
que ao redor de cada vértice (pontas do sólido) apareçam os números de mas nenhum deles gosta de amendoim. Eles combinam de dividir o
1 a 5. Qual número está na face com a interrogação? chocolate quebrando-o ao longo das linhas verticais ou horizontais da
barra, um depois do outro e retirando o pedaço escolhido, até que alguém
tenha que ficar com o pedaço do amendoim. Por sorteio, coube a Elias
2 ? começar a divisão, sendo proibido ficar com mais da metade do chocolate
logo no começo. Qual deve ser a primeira divisão de Elias para garantir
3 1 que Fábio fique com o amendoim ao final?

(A) Escolher a primeira coluna à esquerda.


(B) Escolher as duas primeiras colunas à esquerda.
(C) Escolher a terceira linha, de cima para baixo.
Icosaedro (D) Escolher as duas últimas linhas, de cima para baixo.
(E) Qualquer uma, já que Fábio forçosamente ficará com o amendoim.
(A) 1.
(B) 2.
(C) 3.
(D) 4.
(E) 5.

170 Vol. 1
Lógicas e Técnicas
de demonstração
03 Prove, usando indução finita, as proposições abaixo: * * 3 *
× * * 3
a. 12 + 22 +  + n2 = n ⋅ ( n + 1) ⋅ ( 2 n + 1) 3 * * *
6 * * * 3 3
 n ⋅ ( n + 1) 
2
* * * *
3 3 3
b. 1 + 2 +  + n =   * * * * * * *
 2 
04 Na Inglaterra um garoto escreve ao pai a seguinte carta:
2 x n +1 − 1
n
c. 1 + x + x +  + x = ,em que x ≠ 1 SEND
x −1
MORE +
d. (1 + x ) ≥ 1 + nx , ∀x ∈ [−1, +∞[ ( desigualdade de Bernoulli )
n

MONEY
04 Demonstre, usando indução, que o número de subconjuntos de um Quanto dinheiro (money) ele pediu ao pai? (substitua cada letra por um
conjunto de n elementos é 2n. algarismo, letras diferentes por algarismos diferentes)

05 Prove que se um segmento de comprimento unitário é dado, é sempre 05 N caixas menores são colocadas em uma caixa vazia. A seguir, em
possível construir um segmento de comprimento n . cada uma destas caixas menores, ou são colocadas N caixas menores
ainda ou não é colocada caixa alguma. O processo se repete um certo
06 número K de vezes. Determine o número de caixas vazias, sabendo que
a. Mostre, usando indução, que para todo natural n, ao final há M caixas cheias.
1 1 1 1
+ + ... + ≥
n + 1 n + 2 2 n 2 1 1 1 1 n
06 Prove que: (∀n)( n ∈ Ν → 1 + + + + + n > ).
1 1 1 1 1 2 3 4 2 −1 2
b. Mostre que para todo natural n, + + + ... + ≥ (tente n
n n+1 n+2 2n 2 23 + 1
07 Prove que é inteiro para todo n natural.
fazer por indução para ver o que ocorre). 3 n +1
08 Prove que todo inteiro maior que 1 é primo ou produto de primos.
n( n + 1)( n + 2)
07 Mostre que 1 ⋅ 2 + 2 ⋅ 3 + ... + n( n + 1) = para todo (Teorema Fundamental da Aritmética)
natural n. 3
09 (Torre de Hanói) Dispõe-se de 3 pinos e n discos de vidro, com
08 Prove que 22n + 24n – 10 é múltiplo de 18 para todo n natural. um furo no meio, sendo que os discos têm pesos distintos dois a dois.
Sabe--se que se um disco de peso maior é colocado sobre um disco de
09 Considere a sequência (Sn) dada por Sn +1 = Sn2 − 2 ∀n, S0 = 4. Mostre peso menor, então este se quebra. É proposto então o seguinte jogo: “Todos
n n
que Sn = ( 2 + 3 )2 + ( 2 − 3 )2 para todo natural n. os n discos estão encaixados no primeiro pino, de maneira que olhando
de baixo para cima estão em ordem decrescente de peso. O movimento
10 Quadrado Mágico – Desejamos preencher um tabuleiro 3 x 3 permitido é passar o disco que está na posição superior de um pino para
colocando em cada casa um número entre 1 e 9 (inclusive) de modo que outro pino.” Qual é o menor número de movimentos necessários para se
seja respeitada a condição de que a soma dos números de uma linha, passar todos os discos para o terceiro pino, podendo usar o segundo pino
coluna ou diagonal qualquer, seja constante. Qual o valor obtido para a (sem quebrar nenhum disco)?
soma constante e qual o número que deve ocupar o centro do tabuleiro?
10 É comum utilizarmos um diagrama com 3 circunferências para resolver
11 (OBM) Prove que em qualquer pentágono convexo existem dois problemas envolvendo três conjuntos. Mostre que é impossível fazer um
ângulos internos consecutivos cuja soma é maior ou igual a 216°. diagrama desse tipo, com circunferências, para o caso de 4 conjuntos.

EXERCÍCIOS NÍVEL 3 11 Em uma festa, toda mulher dança com algum homem e nenhum
homem dança com todas as mulheres. Demonstre que existem homens
H0 e H1 e mulheres M0 e M1 tais que H0 dança com M0, H1 dança com M1,
01 O plano euclidiano é dividido em regiões traçando-se n retas. Mostre
H1 não dança com M0, H0 não dança com M1.
que o número de regiões desenhadas é sempre menor do que 2n.

( )
k

02 Duzentos alunos de alturas diferentes são posicionados em 10 linhas,


12 Prove que todo número da forma 2 − 1 , k natural, pode ser
cada uma com 20 alunos. De cada uma das 20 colunas assim formadas, colocado na forma N − N − 1.
o menor aluno é escolhido e então, dentre esses 20, marcamos o maior
como sendo o aluno A. Em seguida, voltando-se a configuração inicial, 13 Há uma moeda falsa entre doze moedas dadas. Sabe-se que a moeda
escolhe-se o maior aluno de cada linha, e desses 10 alunos escolhidos, falsa difere das demais apenas no peso, porém, não se sabe se a moeda
marcamos o menor aluno como sendo B. Qual dos dois alunos marcados falsa é mais leve ou mais pesada que as demais. Todas as moedas
é o maior? genuínas têm o mesmo peso. Identifique a moeda falsa e se ela é mais
leve ou mais pesada que as demais, fazendo apenas três pesagens em
03 (IME) No produto abaixo, o * substitui algarismos diferentes de 3 e numa balança de braços.
não necessariamente iguais. Determine o multiplicando e o multiplicador.
14 Considere 2n pontos no plano. Prove que o número máximo de
segmentos que podem ser traçados ligando pares desses pontos sem
que se forme um triângulo é n².

IME-ITA 171
Relações e funções A ssunto
3
Matemática I

1. Par ordenado A B

1.1 Conceito 1 1
Admitiremos o par ordenado (a, b) como conceito primitivo, levando-se
em consideração que a ordem em que os números aparecem é relevante: R1:
2 2
se a ≠ b, então (a, b) ≠ (b, a).
3
Obs. 1: Kuratowski, um matemático polonês, definiu (a, b): ={{a}, {a,
b}}, mas não há necessidade de se preocupar com esta definição.

Obs. 2: A igualdade entre dois pares ordenados (a, b) = (c, d) ocorre se, A B
e somente se, a = c e b = d.
1
2. Produto cartesiano 1
2
2.1 Definição R2: 2

Sendo A e B conjuntos, definimos A × B = {(a, b)|a ∈ A ∧ b ∈ B}


(é o conjunto dos pares ordenados em que a primeira entrada pertence 3
ao conjunto A e a segunda entrada pertence ao conjunto B).

Ex.: 3.2 Domínio


A = {1, 2, 3} O domínio (Dom) de uma relação R de A em B é o conjunto formado
B = {1, 2} pelos elementos de A que, de fato, se relacionam com alguém de B. Ou
A × B = {(1, 1), (1, 2), (2, 1), (2, 2), (3, 1), (3, 2)} seja, Dom(R) = {a ∈ A|∃b ∈ B com aRb}. Em termos de flechas, o
domínio é composto pelos elementos de A que mandam flechas para B.
Teorema 1 (Número de elementos do produto cartesiano): Se A e B
são conjuntos finitos, segue que A × B é finito e n(A × B) = n(A) · n(B). Ex.:
R: {1, 2, 3} → {1, 2, 3, 4}, R = {(1, 1), (1, 2), (3, 2)}
Obs. 1: A × B lê-se como “A cartesiano B”. Dom(R) = {1, 3}
Obs. 2: Em geral, A × B ≠ B × A (de fato, a igualdade só ocorre quando
A = B). 3.3 Contradomínio
O contradomínio (Cd) de uma relação R de A em B é o próprio B (é o
3. Relação conjunto dos elementos que podem se relacionar com elementos de A).
Em termos de flechas, o contradomínio é formado pelos elementos que
3.1 Definição podem receber flechas (no caso, todo o conjunto B).
Dados os conjuntos A e B, chama-se relação de A em B qualquer
Ex.: R: {1, 2, 3} → {1, 2, 3, 4}
subconjunto de A × B, isto é:
R = {(1, 1), (1, 2), (3, 2)}
R é relação de A em B ⇔ R ⊂ A × B. Cd(R) = {1, 2, 3, 4}

Ex.: A = {1, 2, 3} e B = {1, 2} 3.4 Imagem


R1 = {(1, 1), (2, 1)} A imagem (Im) de uma relação R de A em B é o conjunto formado
pelos elementos de B que de fato se relacionam com alguém de A. Ou
R2 = {(1, 1), (2, 1), (3, 1), (3, 2)}
seja, Im(R) = {b ∈ B|∃a ∈ A com aRb}. Em termos de flechas, a imagem
é composta pelos elementos de B que efetivamente recebem flechas.
Obs. 1: (a, b) ∈ R pode ser representado por aRb.
Ex.: R: {1, 2, 3} → {1, 2, 3, 4}
Obs. 2: Podemos pensar em uma relação como um diagrama de flechas,
no qual representamos de um lado o conjunto A e do outro o conjunto B. R = {(1, 1), (1, 2), (3, 2)}
Para representar a relação, ligamos a a b com uma flecha se aRb. Essa Im(R) = {1, 2}
intuição ajudará a entender os conceitos futuros.

172 Vol. 1
Relações e funções

3.5. Relação composta Demonstração:


Seja R uma relação de A em B e S uma relação de B em C. Definimos l: y = x
a relação composta T da seguinte forma: T = S ° R = {(a, c) ∈ A × C| P(a,b)
b ∈ B com aRb ∧ bRc} Intuitivamente, olhamos para a relação R e b
buscamos os elementos da imagem de R que estão no domínio de S (o
conjunto B funciona como uma “ponte”).
A(a,a)
a Q(b,a)
Ex.: R: {1, 2, 3} → {1, 2, 3, 4}, R = {(1, 2), (1, 3), (2, 1)}
S: {1, 2, 3, 4} → {3, 4, 5}, S = {(2, 5), (3, 4), (4, 5)}
S ° R: {1, 2, 3} → {3, 4, 5}, SoR = {(1, 5), (1, 4)}
a b
Os elementos da imagem de R que estão no domínio de S são 2 e 3
(esses elementos farão a “ponte” da relação composta). Suponha P = (a, b) ∈ G(R) (gráfico de R). Então, Q = (b, a) ∈ G(R–1) .
Vejamos um diagrama de flechas ilustrando o que ocorre: Seja A = (a, a).

A B C I. AP = AQ → ∆PAQ isósceles 


R S II. 45º = ang < l, AQ > = ang < AP,  >→  é bissetriz.
1 1 
3 De I e II, l é mediatriz de PQ, donde P e Q equidistam de .
2 Como o ponto P é um ponto qualquer de G(R), tem-se G(R) e G(R–1)
2 4 simétricos em relação à reta .
3 5
3 3.7 Relação em um conjunto
4
3.7.1 Conceito
A C Seja U um conjunto. Chama-se relação em U toda relação de U em U.

Ex.:
T
1 R: {1, 2, 3, 4} → {1, 2, 3, 4}
3
R = {(1, 2), (2, 3), (2, 4)}
2 4
3.7.2 Propriedades
3 5 Seja R uma relação de U em U:

I. Reflexiva: R é reflexiva ⇔ (∀x) (x ∈ U → xRx)


R S
1→2→5
T
1→5 Ex.:
R = {1, 2, 3} → {1, 2, 3}
R S T
1→3→4 1→4 R = {(1, 1), (2, 2), (3, 3), (2, 3), (1, 3)}

Obs.: A operação de composição é associativa, mas não é comutativa. II. Simétrica: R é simétrica ⇔ (∀x) (∀y) (x ∈ U ∧ y ∈ U ∧ xRy → yRx)

Ex.:
3.6 Relação inversa R = {1, 2, 3} → {1, 2, 3}
Seja R uma relação de A em B. Definimos a relação inversa por R –1 R = {(1, 2), (2, 1), (2, 2)}
={(y, x) ∈ B × A|(x, y) ∈ R}. Em termos de um diagrama de flechas,
basta “inverter” as flechas. III. Antissimétrica: R é antissimétrica ⇔ (∀x)(∀y)(x ∈ U ∧ y ∈ U ∧ xRy
∧ yRx → x = y)
Ex.: Ex.:
R: {1, 2, 3} → {1, 2, 3, 4}, R = {(1, 1), (1, 2), (3, 2)} R = {1, 2, 3} → {1, 2, 3}
R–1: {1, 2, 3, 4} → {1, 2, 3}, R–1 = {(1, 1), (2, 1), (2, 3)} R = {(1, 2), (2, 2), (3, 1)}

Teorema 2 (Simetria do gráfico da relação inversa): Se R é relação IV. Transitiva: R é transitiva ⇔ (∀x)(∀y)(∀z)(x ∈ U ∧ y ∈ U ∧ z ∈ U ∧
em  e R–1 é sua inversa, então os gráficos dessas relações no plano xRy ∧ yRz → xRz)
cartesiano são simétricos em relação à reta y = x.

IME-ITA 173
Matemática I – Assunto 3

Ex.: A)(∃y ∈ B)((x, y) ∈ ƒ).


R = {1, 2, 3} → {1, 2, 3} II. Todo elemento de A está relacionado com um único elemento de B
R = {(1, 2), (2, 3), (1, 3)} (∀x ∈ A)(∃ y, y’ ∈ B)((x, y) ∈ ƒ ∧ (x, y’) ∈ ƒ → y = y’).

3.7.3 Relação de equivalência Em termos de flechas, todo elemento de A manda uma e apenas
uma flecha para B.
Seja R uma relação em U:
Nesse caso, em vez de escrevermos x ƒ y, escrevemos y = ƒ(x) sem
R é relação de equivalência quando (R é reflexiva) ∧ (R é simétrica)
ambiguidade.
∧ (R é transitiva).
Ex.:
Ex.:
ƒ: {1, 2, 3} → {1, 2, 3}
U = {x|x é uma reta}
G(ƒ) = {(1, 2), (2, 2), (3, 1)}
R: U → U
(G(ƒ), denominado gráfico de ƒ, é o conjunto dos pares (x, y) tais
R = {(x, y) ∈ U2 | x // y}
que y = ƒ(x)).
I. R é reflexiva: com efeito, (∀x ∈ U) (xRx)
Obs.: O domínio de uma função ƒ: A → B é o conjunto A e o contradomínio
II. R é simétrica: com efeito, (∀x, y ∈ U) (xRy → yRx)
é o conjunto B. A imagem é definida como em relações (intuitivamente,
III. R é transitiva: com efeito, (∀x, y, z ∈ U ) (xRy ∧ yRz → xRz) de I, II
são os elementos de B que recebem efetivamente flechas).
e III: R é relação de equivalência
Obs. 2: Em concursos como a AFA, quando nada for dito sobre o domínio,
3.7.4 Relação de ordem devemos supor que o domínio é o mais amplo possível nos reais, isto é, o
Seja R uma relação de U em U: domínio é o conjunto dos valores para os quais a função está bem definida
(portanto, devemos fazer restrições como denominadores diferentes de 0,
R é relação de ordem quando (R é reflexiva) ∧ (R é antissimétrica) ∧
expressões dentro de radicais de índices pares devem ser não negativas,
(R é transitiva).
condições de existência de logaritmos).
Ex.: 4.2 Função identidade
U = {x|x é um conjunto} Seja A um conjunto não vazio. A função identidade IdA: A → A é dada
R: U → U por IdA(a) = a para todo a ∈ A.
R = {(x, y) ∈ U2|x ⊂ y}
4.3 Função constante
I. R é reflexiva: com efeito, (∀x ∈ U) (xRx) Sejam A e B conjuntos. Uma função ƒ: A → B é constante se ƒ(a) = b
II. R é antissimétrica: com efeito, (∀x, y ∈ U) (xRy ∧ yRx → x = y) para todo elemento a ∈ A (ou seja, a função assume um único valor: todas
III. R é transitiva: com efeito, (∀x, y, z ∈ U) (xRy ∧ yRz → xRz) de I, II as flechas chegam a um mesmo elemento).
e III: R é relação de ordem
4.4 Função injetora (injetiva)
3.7.5 Classe de equivalência Sejam A e B conjuntos. Uma função ƒ: A → B é injetora seƒ(x) = ƒ(y)
⇒ x = y (ou seja, a função não “repete valor”). Em termos de flechas,
Sejam U um conjunto e R uma relação de equivalência de U em U. cada elemento da imagem recebe uma única flecha.
Chama-se classe de equivalência por R um subconjunto de U constituído
por um elemento x de U e por todos os elementos y de U tais que yRx.
Obs. 1: No gráfico de uma função injetora, ao traçarmos uma reta
horizontal, esta reta corta o gráfico em no máximo um ponto (pode não
Ex.: cortar em ponto algum).
U = {0, 1, 2, 3, 4, 5, ...}
R: U → U, R = {(x, y) | x ≡ y(mod 3)} Como verificar que uma função é injetora? Um método prático e
A = {x|x ≡ 1(mod 3)}; B = {x|x é da forma 3k + 2, k natural}; ... eficiente é supormos ƒ(x) = ƒ(y) e, através de manipulações algébricas,
A, B, ... são classes de equivalência por R em U chegar a x = y.

Obs.: O conjunto das classes de equivalência por R em um conjunto U é Ex.: A função ƒ:  →  dada por ƒ(x) = x3 é injetora. De fato, ƒ(x) =
denominado conjunto-quociente U/R. No exemplo acima, U/R = {A, B, ... }. ƒ(y) ⇒ x 3 = y 3 ⇒ (x – y)(x 2 + xy + y 2) = 0. Logo, x = y ou
2
 y 3y2
x 2 + xy + y 2 ⇒  x +  + = 0 ⇒ x = y = 0 . Em ambos os
4. Funções  2 4
casos, segue que x = y e, portanto, a função é injetora.
4.1 Definição
Obs. 2: O conceito de função injetora depende fortemente do domínio. Por
Sejam A e B conjuntos. Uma função ƒ de A em B (representamos
exemplo, a função ƒ: [–1, 1] → [–1, 1] dada por ƒ(x) = x2 não é injetora,
por ƒ: A → B) é um tipo especial de relação em que duas condições
já que ƒ(–1) = ƒ(1) = 1. Entretanto, a função g: [0,1] → [0,1] dada pela
são satisfeitas:
mesma lei de formação g(x) = x2 é injetora. De fato, se g(x) = g(y), segue
que x2 = y2 ⇒ x = y ∨ x = –y. Como x, y ≥ 0, a última possibilidade nos
I. Todo elemento de A está relacionado com um elemento de B: (∀x ∈

174 Vol. 1
Relações e funções

dá x = y = 0. Em ambos os casos, então, segue que x = y e g é injetora. 4.7 Função composta


Isto ressalta a importância de que o domínio e o contradomínio de uma
Sejam ƒ: A → B e g: B → C duas funções. Definiremos a composição
função são partes cruciais de sua definição!
da função g com a função ƒ (h = g ° ƒ – lê-se “g de ƒ” ou “g bola ƒ”)
da seguinte maneira:
Obs. 3: No caso de domínio e contradomínio finitos, se ƒ: A → B é injetora,
segue que n(A) ≤ n(B).
I. O domínio de h é o conjunto A.
4.5 Função sobrejetora (sobrejetiva) II. O contradomínio de h é o conjunto C.
III. h(x) = g(ƒ(x)) (aqui funciona exatamente da mesma forma que na
Sejam A e B conjuntos. Uma função ƒ: A → B é sobrejetora se
composição de relações)
∀y ∈ B, ∃x ∈ A, tal que ƒ(x) = y (ou seja, Im ƒ = Cd ƒ = B). Em
termos de flechas, todo elemento de B recebe pelo menos uma flecha.
Ex.: Sendo g(x) = x2 e ƒ(x) = 2x + 1, temos que g(ƒ(x)) = g(2x + 1) =
(2x + 1)2 = 4x2 + 4x + 1.
Obs. 1: No gráfico de uma função sobrejetora, ao traçarmos uma reta
horizontal, esta reta corta o gráfico em pelo menos um ponto.
Obs. 1: Para existir h = g ° ƒ, o contradomínio de ƒ deve ser igual ao
domínio de g.
Como verificar se uma função é sobrejetora? Basicamente, o que
devemos fazer é considerar um elemento y do contradomínio de ƒ e tentar Obs. 2: A composição de funções não é comutativa. Além disso, pode
resolver a equação ƒ(x) = y. Se para cada y essa equação possuir ao acontecer que g ° ƒ esteja definida, mas ƒ ° g não.
menos uma solução x pertencente ao domínio de ƒ, a função é sobrejetora.
Teorema 3 (Critério para garantir injetividade e sobrejetividade):
3x − 5
Ex.: A função ƒ:  – {2} →  – {3} dada por f (x ) = é Sejam ƒ: A → B e g: B → C duas funções. Se g ° ƒ: A → C é injetora,
x −2 então ƒ é injetora. Se g ° ƒ: A → C é sobrejetora, então g é sobrejetora.
sobrejetora. Para verificarmos isso, devemos considerar um real y ≠ 3 e
tentar resolver 3 x − 5 = y ⇒ 3 x − 5 = xy − 2 y ⇒ x(y − 3) = 2y − 5 . Demonstração:
x −2
2y − 5 Parte 1: g ° ƒ: A → C é injetora
Como y ≠ 3, segue que x = . Para a demonstração ficar completa,
y −3 Suponha que ƒ(x) = ƒ(x'). Logo, g ° ƒ(x) = g ° ƒ(x') e, como g ° ƒ é
injetora, segue que x = x'. Logo, ƒ é injetora.
devemos ainda verificar que essa expressão encontrada para x nunca pode
ser 2 (pois o domínio da função exclui o número 2), o que é evidente, pois
x = 2 ⇒ 2y – 5 = 2y – 6, contradição. Parte 2: g ° ƒ: A → C é sobrejetora
Queremos provar que para qualquer z ∈ C, existe y ∈ B, tal que g(y)
Obs. 2: O conceito de função sobrejetora depende for temente do = z. Como g ° ƒ: A → C é sobrejetora, existe x ∈ A, tal que g(ƒ(x)) = z.
contradomínio. Por exemplo, a função ƒ: [–1, 1] → [–1, 1] dada por Finalmente, como ƒ(x) ∈ B, basta tomarmos y = ƒ(x) e assim g é sobrejetora.
ƒ(x) = x2 é sobrejetora. Entretanto, a função g: [–1, 1] → [–1, 1] ∪ {4}
dada pela mesma lei de formação g(x) = x2 não é sobrejetora. De fato, 4.8 Função inversa
se g(x) = 4, segue que x2 = 4 ⇒ x = ± 2. Como 2 e – 2 não estão no Dada uma função ƒ: A → B, queremos definir uma função g: B → A
domínio da função, não existe x ∈ Dom ƒ tal que ƒ(x) = 4. (a inversa de ƒ) de forma que se ƒ(x) = y, então g(y) = x. Note que para
que g seja de fato uma função, é necessário que:
Obs. 3: No caso de domínio e contradomínio finitos, se ƒ: A → B é
sobrejetora, segue que n(A) ≥ n(B). I. cada elemento de B só mande uma flecha de volta (para isso, a função
ƒ não pode repetir valores e, portanto, ƒ deve ser injetora);
4.6 Função bijetora (bijetiva) II. todo elemento de B precise mandar flechas (para isso, todo elemento
Sejam A e B conjuntos. Uma função ƒ: A → B é bijetora se é injetora de B deve receber flechas da função ƒ e, portanto, ƒ deve ser
e sobrejetora simultaneamente. sobrejetora).

Obs. 1: No gráfico de uma função bijetora, ao traçarmos uma reta Teorema 4 (Condição de existência da inversa):
horizontal, esta reta corta o gráfico em exatamente um ponto. Uma função ƒ: A → B admite inversa se, e somente se, é bijetora.
Para verificar que uma função é bijetora, basta seguir os passos de 4.4 e Denotamos a inversa de ƒ por ƒ–1 : B → A. Veja que segue da definição
4.5 para verificação da injetividade e da sobrejetividade. que ƒ ° ƒ–1 = IdB e ƒ–1 ° ƒ = IdA.

Ex.: A função ƒ: [0, π] → [–1, 1] dada por ƒ(x) = cos x é bijetora. Teorema 5 (Método prático para calcular a inversa):
Dada a função bijetora ƒ: A → B definida pela sentença y = ƒ(x), para
Obs. 2: Assim como em 4.4 e 4.5, o conceito de função bijetora depende obtermos a expressão de ƒ–1, procedemos como a seguir:
fortemente do domínio e do contradomínio!
I. Na sentença y = ƒ(x), trocamos as variáveis x ↔ y, escrevendo
Obs. 3: No caso de domínio e contradomínio finitos, se ƒ: A → B é bijetora, x = ƒ(y).
segue que n(A) = n(B). II. Transforma-se a expressão x = ƒ(y), expressando y em função de x,
chegando a y = ƒ–1(x).

IME-ITA 175
Matemática I – Assunto 3

Formalmente, usamos que ƒ(ƒ–1(x)) = x e, a partir disso, encontra-se


a expressão ƒ–1(x).
5.4 Periodicidade
Exs.: Uma função ƒ:  →  é dita periódica quando existe T > 0 (dito
I. Determine a inversa da função ƒ(x) = y = 5x + 3. um período de ƒ) tal que ƒ(x + T) = ƒ(x) para todo x real. O menor real
Trocando x ↔ y, temos x = 5y + 3. Resolvendo em y, segue que positivo T com essa propriedade é chamado de período de ƒ (às vezes
x −3 chamado de período fundamental).
y= e, portanto, f −1 (x) = x − 3 .
5 5
Ex.:
II. Determine a inversa da função ƒ: + → [1, +∞] dada por ƒ(x) = y
= x2 + 1. A função ƒ(x) = sen x é periódica de período 2π, pois ƒ(x + 2π) =
Trocando x ↔ y, temos x = y2 + 1. Resolvendo em y, segue que ƒ(x).
f −1 (x) = x − 1, x ≥ 1.
6. Gráficos
Obs.: Se o domínio da função fosse o conjunto dos reais não positivos,
seguiria que a inversa é − x − 1, pois o contradomínio da inversa seria 6.1 Deslocando o gráfico de uma função
o conjunto dos reais não positivos! Tenha atenção com isso!
 k > 0 desloca 
a funçãok unidadespracima
I. f ( x ) + k 
4.9 Operações entre funções  k < 0 
desloca a funçãok unidades prabaixo
Sejam A e B conjuntos e ƒ: A → B, g: A → B funções. Definimos as
seguintes operações:
 k > 0 desloca 
o gráfico paraesquerda
II. f ( x + k ) 
I. (ƒ ± g)(x) = ƒ(x) ± g(x)  k < 0 desloca 
o gráfico paradireita
II. (ƒ · g)(x) = ƒ(x) · g(x)
f f (x ) Ex.:
III. (x )=
g g(x )  π
Vejamos o gráfico da função y = 1 + cos  x − 
IV. ƒg(x) = ƒ(x)g(x)  3

1o Passo: Primeiro devemos desenhar o gráfico da função y = cos x.


5. Funções reais de variável real
1.0
5.1 Conceito
São funções de  em . 0.5

5.2 Paridade
–π –π π 2π
I. Dizemos que ƒ:  →  é par se ƒ(–x) = ƒ(x) para todo x ∈ .
Graficamente, isso significa que a função ƒ é simétrica com relação –0.5
ao eixo y.
–1.0
Ex.: ƒ(x) = x2, g(x) = cos x
  π
II. Dizemos que ƒ:  →  é ímpar se ƒ(–x) = –ƒ(x) para todo x ∈ . 2o Passo: Depois desenhamos o gráfico da função y = cos  x −  ,
 3
π
Graficamente, isso significa que a função ƒ é simétrica com relação deslocando a função anterior para a direita.
à origem. 3

Ex.: u(x) = x3, v(x) = sen x 1.0

5.3 Monotonismo 0.5


• Função estritamente crescente
Dizemos que ƒ:  →  é estritamente crescente se x < y ⇒ –6 –4 –2 2 4 6
ƒ(x) < ƒ(y) .
–0.5
Ex.: ƒ(x) = 2x + 1
–1.0
• Função estritamente decrescente
Dizemos que ƒ:  →  é estritamente decrescente se x < y ⇒ ƒ(x)
> ƒ(y) .

Ex.: ƒ(x) = –2x + 1

176 Vol. 1
Relações e funções

3o Passo: Finalmente desenhamos a função desejada y = 1 + cos  x − π  EXERCÍCIOS RESOLVIDOS


 
 3
, deslocando a anterior uma unidade para cima. 01 Seja n um inteiro fixo. Prove que a relação ‘aRb ⇔ a – b é múltiplo
2.0 de n’ é de equivalência.

Solução
1.5
Para provar que a relação é de equivalência, precisamos verificar
reflexividade, simetria e transitividade.
1.0
I. reflexividade: temos que aRa, pois a – a = 0 é sempre múltiplo
0.5 de n.
II. simetria: veja que se a – b é múltiplo de n, então b – a = –(a – b)
é múltiplo de n, então, aRb ⇔ bRa.
–6 –4 –2 2 4 6  aRb ⇒ a − b é múltiplo de n
III. transitividade:  . Daí, veja que a
6.2 Esticando e contraindo uma função  bRc ⇒ b − c é múltiplo de n

 k > 1 estica a função em y


III. k f ( x ) – c = (a – b) + (b – c) é uma soma de dois múltiplos de n;
0 < k < 1 contrai a funçãoem y logo, a – c é múltiplo de n; portanto, aRc.

02 Considere uma turma de 30 alunos (Abílio e Deuclécio são dois


 k > 1contrai 
o gráfico em x
IV. f ( kx ) deles). Seja A o conjunto dos alunos dessa turma e ƒ: A → N a função
0 < k < 1esticaa gráfico em x que associa cada aluno à sua quantidade de amigos dentro da turma.
Considere que a relação de amizade é recíproca (ou seja, se X é amigo
Ex.: Monte o gráfico da função y = 3sen(2x) de Y, então Y é amigo de X) e que ninguém é amigo de si mesmo. É
possível que tenhamos simultaneamente ƒ(Abílio) = 0 e ƒ(Declécio)
1o Passo: Montamos o gráfico da função y = sen x. = 29?

1.0 Solução
Se ƒ(Deuclécio) = 29, teríamos que Deuclécio é amigo de todos
0.5 da turma (pois ele não é amigo de si mesmo). Daí, veja que Abílio
deveria ter pelo menos um amigo (Deuclécio) e então não é possível
termos ƒ(Abílio) = 0.
–2π –π π 2π
–0.5 03 Sabendo que h é uma função tal que h(2x + 1) = x2 – x, para
todo x real, determine a lei de formação de h.
–1.0
1a Solução: Como h(2x + 1) = x2 – x para todo x real, podemos
2 Passo: Desenhamos o gráfico da função y = sen (2x), lembrando
o substituir x por qualquer número ou expressão. Então, é conveniente
x x2 x
que se estamos multiplicando x por dois, estamos dividindo o período trocar x → chegando a h ( x + 1) = − . Agora, basta trocar x
da função por 2. 2 4 2
( x − 1) − ( x − 1) .
2
1.0
→ x – 1, chegando a h ( x ) =
4 2
0.5
Obs.: É claro que poderíamos fazer uma única substituição
x −1
x→ . Apresentamos a solução em 2 passos para ficar um
–π π π π 2
− pouco mais natural.
2 2
–0.5 y −1
2a solução: Fazendo 2x + 1 = y, temos que x = . Substituindo
2
–1.0
2
 y − 1  y − 1
3o Passo: Finalmente desenhamos o gráfico da função y = 3sen(2x), na expressão dada, temos que h ( y ) =   −  , para
lembrando que estamos esticando a função no eixo y.  2   2 

3 todo y real. Veja que isso define h, pois, já que esta relação vale para
2
2  x − 1  x − 1
todo y, podemos dizer que h ( x ) =   − .
1  2   2 

π π π Obs.: Veja que o fato de a relação valer para todo y, nos permite
–2π − –1
2 2 substituir o y por qualquer coisa, inclusive por x.
–2
–3

IME-ITA 177
Matemática I – Assunto 3

04 Sejam ƒ e g funções reais tais que ƒ(x) = 4x –1 e ƒ ° g(x) = EXERCÍCIOS NÍVEL 1


3x2 + 7x + 1. Determine a lei de formação da função g.
01 Seja A = {1, 2, 3} e seja R a relação sobre A definida por “x + 2y =
Solução 8”, isto é, R = {(x, y) ∈ A2|x + 2y = 8}.
De ƒ(x) = 4x – 1, tiramos que ƒ(g(x)) = 4g(x) – 1, portanto, 4g(x) – 1
= 3x2 + 7x + 1. a. Determine o gráfico de R.
3x2 + 7x + 2 b. Determine o gráfico de R–1.
Então, a função g tem g ( x ) = como lei de formação.
4
02 Dado o conjunto A = {0, 1, 2, 3, 4, 5, 6}, considere os pares (x, y)
e a relação R, tais que: x ∈ A, y ∈ A, xRy ⇔ 1 ≤ x + y ≤ 3. Escreva os
3x + 1
05 Seja ƒ:  – {2} →  – {b} uma função tal que f ( x ) = . pares (x, y) que pertencem ao produto cartesiano A × A e que satisfazem
x −2 a relação R.
Determine o valor de b para que a função ƒ seja bijetora e determine
sua inversa. 03 Dado o conjunto A = {1, 2, 3}, verifique, dentre as relações abaixo,
quais são de equivalência e quais são de ordem em A:
Solução:
Antes de tudo, vamos determinar a lei de formação da inversa de ƒ. Para R1 = {(1, 1), (2, 2), (3, 3), (4, 4)};
3y + 1 R2 = {(1, 1), (2, 2), (3, 3), (1, 2), (2, 1)};
a inversa, temos que x = e, isolando o y, temos que R3 = {(1, 1), (2, 2), (3, 3), (1, 2), (2, 3), (3, 1)};
y −2
2x + 1 R4 = {(1,3), (2,4)}.
y= (*). Veja, então, que 3 não pertence ao domínio da inversa;
x −3
portanto, precisamos excluir 3 do contradomínio de ƒ (para que ƒ seja  πx πy 
04 Considere a relação R = (x , y ) ∈ Z x Z | cos = cos . Determine
sobrejetora). Então, b = 3. É fácil ver que a função dada é injetora, pois  2 2
na inversa, cada y está definido unicamente a partir de um x, pela equação a classe de equivalência do 0.
(*).
05 Seja  o conjunto dos números naturais e a, b ∈ . Mostre que a
−1 2x + 1 relação R = {(a, b) | m.d.c. [a, b] = a} é uma relação de ordem.
Então, f (x ) = e b = 3.
x −3
06 (OMERJ) Determine todas as funções ƒ: R* → R tais que 06 Seja A = {1, 2, 3}:

 1 a. Determine uma relação de equivalência R em A com cinco elementos.


2x · f (x) + f   = x 2 para todo x real não nulo. b. Determine [1]R, [2]R, [3]R para essa relação.
x
c. Determine o conjunto quociente A|R para essa relações.
Solução
Como a relação dada vale para todo x não nulo, podemos substituir 07 Sejam as relações F e G abaixo, definidas em A = {1, 2, 3, 4, 5}:
o x por qualquer número ou expressão não nula. Nesse problema,
1
a substituição vantajosa é trocar x por (em todos os lugares): F = {(1, 3), (2, 4), (3, 3), (4, 3), (5, 4)}
x G = {(1, 2), (2, 3), (1, 3), (4, 5), (3, 4)}
2  1 1
⋅ f   + f ( x ) = 2 (* ) . Juntando esta última relação com a
x x x
 1 a. Quais das relações acima são funções?
que foi dada, temos um sistema de variáveis f (x ) e f   e é fácil b. Defina, pelo conjunto de pares ordenados, a relação composta de F
x
determinar ƒ(x) . com G.

 1 08 Suponha que exista ƒ:  →  tal que ƒ(2n + ƒ(n)) = n. Prove que


Da 1a equação, vem que f   = x − 2 x ⋅ f ( x ). Substituindo em
2
ƒ é sobrejetiva.
 x
2 1
(*), temos que ⋅ ( x 2 − 2 x ⋅ f ( x ) ) + f ( x ) = 2 , e segue que
x x 09 (OBM) Seja ƒ uma função definida para todo x real, satisfazendo as
1 1 condições:
f ( x ) =  2 x − 2 .
3 x 
 f (3) = 2

Obs.: Normalmente, em problemas de ‘equações funcionais’, é  f (x + 3) = f (x ) . f (3)
necessário substituir a função encontrada na relação dada para
ver se ela realmente é solução do problema (já que após algumas Então, ƒ(–3) vale:
substituições no x não usamos todos os dados contidos numa
equação que vale para todo x). No entanto, neste problema isso não (A) –6.
é necessário, pois como resolvemos um sistema de 2 equações e (B) 0.
2 incógnitas para encontrar ƒ(x), obviamente a solução encontrada (C) 1/2.
satisfaz a relação dada (que é uma das equações do sistema). (D) 2.
(E) –1.

178 Vol. 1
Relações e funções

10 Seja S = {1, 2, 3, 4, 5} e considere uma função bijetora de S em S, 15 (AFA) Seja D = {1, 2, 3, 4, 5} e f: D →  a função definida por
tal que: f(x) = (x – 2)(x – 4). Então, pode-se afirmar que f:

I. Se x ∈ S, a imagem de x não pode ser igual a x – 1, nem igual a x, (A) é bijetora.


nem igual a x + 1. (B) é somente injetora.
II. Se x ∈ S e a imagem de x é y, então a imagem de y não pode ser nem (C) é somente sobrejetora.
x, nem x + 1. (D) possui conjunto-imagem com 3 elementos.

Nessas condições, a imagem do número 3 é igual a: 2+ x


16 (AFA) A imagem da função real ƒ definida por f (x ) = é:
2− x
(A) 1.
(B) 2. (A)  – {1}. (C)  – {–1}.
(C) 3. (B)  – {2}. (D)  – {–2}.
(D) 4.
(E) 5. 17 (EsPCEx) A função ƒ, de domínio real mais amplo possível, é tal
que f (x ) = ax + b − 5 . Sabendo que ƒ(3) não existe e ƒ(–1) = 1, o
f (x ) ax + 3 b
11 (AFA) As funções f e g são dadas por ƒ(x) = ax + bx e g(x ) = .
f (x − 2) valor de a2 + b2 é:
Então, g(3) é igual a:
(A) 50/16. (D) 50/8.
(A) a2 + b2.
(B) 25/3. (E) 50/9.
(B) (a + b)2.
(C) 25/2.
(C) (a – b)2.
(D) a2 – ab + b2.
18 (AFA) Seja f a função real cujo gráfico se apresenta a seguir:
12 (AFA) A função f satisfaz a relação: ƒ(x + 1) = x · ƒ(x), x > 0. Se
 1 3
f   = π , o valor de ff  2  é:
2
   

π
(A) .
2
(B) 2 π .


(C) .
2
(D) π . Analisando o gráfico, é incorreto afirmar que:

n (A) f(f(1)) = f(0,5).


 2 se n é par (B) f(x) + 1 > 0, ∀ x ∈ .
13 (AFA) A função ƒ:  →  definida por ƒ(x) =  é:
(C) f(0) ≤ f(x), ∀ x ∈ .
 n + 1 se n é ímpar
 2 5
(D) se g(x) = f(x) – 1, então g(–2) = f  .
(A) bijetora. 2
(B) somente injetora. 19 (AFA) Considere as funções f, g e h, todas de domínio [a, b] e
(C) somente sobrejetora. contradomínio [c, d], representadas através dos gráficos abaixo.
(D) não injetora e não sobrejetora.
f(x) g(x)
 x − 1
14 (AFA) Se ƒ for uma função real, tal que f   = x + 3, então ƒ(x)
é definida por:  x + 1 d d
e
4 − 2x c c
(A) .
1− x
x x
4x − 2 0 a b 0 a b
(B) .
1+ x h(x)

2x + 1 d
(C) .
x− 1 e
c
(D) 2x − 1 .
1− x
x
0 a b

IME-ITA 179
Matemática I – Assunto 3

Com base nos gráficos, é correto afirmar que: 25 (EN) Seja ƒ uma função e x um ponto do seu domínio. Diz-se que é
um ponto fixo de ƒ se ƒ(x) = x. Considere a função g:  →  definida
(A) f é uma sobrejeção, g não é uma injeção, h é uma sobrejeção. x
f (2x + 1) =
por x 2 + 1. É correto afirmar que:
(B) f é uma sobrejeção, g é uma injeção, h não é uma sobrejeção.
(C) f é uma injeção, g não é uma sobrejeção, h é uma bijeção.
(A) g não possui ponto fixo em [0, 1].
(D) f é uma bijeção, g não é uma injeção, h não é uma sobrejeção.
(B) g possui um ponto fixo em [0, 1].
(C) g possui dois pontos fixos em [0, 1].
20 (EN) É dada uma função tal que:
(D) g possui três pontos fixos em [0, 1].
(E) g possui quatro pontos fixos em [0, 1].
I. f(x) · f(y) = f(x + y)
II. f(1) = 2 e f( 2 ) = 4
26 (OBM) A função ƒ é dada pela tabela a seguir.
Podemos concluir, então, que f(3 + 2) é igual a:
x 1 2 3 4 5
(A) (3 + 2 )2.
(B) 16.
(C) 24. ƒ(x) 4 1 3 5 2
(D) 32.
(E) 64. f ( f (...( f ( f (4))...))?
Por exemplo, ƒ(2) = 1. Quanto vale   
2004 vezes
21 (EN) Determine o conjunto-imagem da função (fog) para:
(A) 1.
(B) 2.
1, se x < 0
0, se x < 0 x (C) 3.
  (D) 4.
f (x) = 2x, se 0 ≤ x ≤ 1 e g(x) =  , se 0 ≤ x ≤ 1
0, se x > 1 2 (E) 5.
 1, se x > 1
27 Sejam as funções reais de variável real ƒ e g, definidas por
(A) |0, 1| ∪ {2}. 5 x − 3 e g(x) = 3 . Pede-se:
f (x) =
(B) (–∞, +∞). 4x + 1 x
(C) |0, 1|. a. obter as leis que definem g ° ƒ e ƒ ° g;
(D) |0, +∞). b. calcular g ° ƒ(2) e ƒ ° g(2).
(E) {1}.
28 Dada a função real de variável real ƒ(x) = ax2 + bx + c, pede-se:
22 (EsPCEx) Seja ƒ:  →  uma função tal que –2 ≤ ƒ(x) < 5 e g:  → 
dada por g(x) = 1 – ƒ(x). Então o conjunto-imagem da função g(x) é: a. obter ƒ(x + 1);
b. obter ƒ(–x);
(A) ]–4, 3]. c. determinar a, b e c de modo que se tenha ƒ(x + 1) = ƒ(–x).
(B) [–4, 3].
(C) ]–4,3[. x
29 Se f (2x + 1) = , determine ƒ(x – 1).
(D) [–3, 4[. x2 + 1
(E) ]–3, 4].
30 Se ƒ(x) = 4x + 1 e ƒ(g(x)) = x2 + 1, determine a função g(x).
23 (ITA) Sejam f, g:  →  funções tais que: g(x) = 1 – x e ƒ(x) +
2ƒ (2 – x) = (x – 1)3 para todo x ∈ . Então ƒ [g(x)] é igual a: 1
31 (AFA) Sejam as funções reais definidas por f(x) = x2 – 1 e g(x ) = .
Então, f(g(–1)) é igual a: x
(A) (x – 1) .
3

(B) (1 – x)3.
(C) x3. (A) –1.
(D) x. (B) 0.
(E) 2 – x. (C) 1.
(D) 2.
24 (ITA) Qual das funções definidas abaixo é bijetora?
32 (AFA) Sejam A = {0,1,2,3} e f: A → A uma função definida por
Obs.: + é o conjunto dos reais não negativos. f(0)= 2, f(1) = 1, f(2) = 3 e f(3) = 0. Calculando f ° f ° f ° f ° f(1),
encontra-se:
(A) ƒ:  → + tal que ƒ(x) = x2.
(B) ƒ: + →  tal que ƒ(x) = x +1. (A) 0.
(C) ƒ: [1, 3] → [2, 4] tal que ƒ(x) = x+1. (B) 1.
(D) ƒ: [0, 2] →  tal que ƒ(x) = sen x. (C) 2.
(E) n.d.a. (D) 3.

180 Vol. 1
Relações e funções

3x − 2
33 (AFA) Se f e g são funções de  em  definidas por f (3 x + 2) = 41 ( ITA) Seja a função f:  – {2} →  – {3} definida por
e g(x – 3) = 5x – 2, então ƒ(g(x)) é: 2
2x − 3 . Sobre sua inversa podemos garantir que:
f (x) = +1
x −4 x −2
(A) . (C) 5x + 13.
5 (A) não está definida pois f é não injetora.
2x + 9 5 x + 11 (B) não está definida pois f não é sobrejetora.
(B) . (D) .
5 5 y−2
(C) está definida por f −1 (y ) = , y ≠ 3.
1/ x , se x ∈ Ζ * y−3
34 (EsPCEx) Sendo ƒ:  →  definida por f (x) =  e
−1, se x ∈ Q 2,se x ∈ ℜ - Ζ * y+5
(D) está definida por f −1 (y) = − 1 , y ≠ 3.
g:  →  definida por g(x) =  , então (ƒ ° g ° ƒ ° g) y − 3
(2 + 2 ) é igual a: 1/ 2, se x ∈ ℜ − Q
−1 2y − 5
(E) está definida por f (y ) = , y ≠ 3.
y−3
2
(A) –1. (D) 1 − .
2 42 Se ƒ(x) é periódica de período T, determine o período de g(x) = ƒ(ax
(B) 1/2. (E) –2.
(C) 2. + b), sendo a ≠ 0.

35 (AFA) Observe os gráficos abaixo, das funções f e g, definidas no 43 (AFA) Indique a alternativa correta:
intervalo [0,1].
(A) Se ƒ é uma função par, então é bijetora.
(B) Se ƒ(x) – ƒ(– x) = 0, então ƒ pode ser relação, mas não função.
(C) Se ƒ é uma função par e x ∈ *, então ƒ* é par só quando x for primo.
(D) Se ƒ :  →  é uma função real qualquer, então ƒ pode ser escrita
como soma de duas funções reais g:  →  e h:  → , em que g
é par e h é impar.

44 (AFA)
“O Brasil tem um encontro marcado com o caos. No dia 1o de junho
começa o plano de racionamento de energia”.
Com base nos gráficos, assinale a alternativa falsa:
“O modelo energético brasileiro é baseado quase que exclusivamente
(A) g(f(0,4)) ≥ g(f(x)), ∀x ∈ [0,1] em hidrelétricas, que produzem 97% da energia consumida no País. Sem
(B) g(f(0,05)) > g(f(0,1)) chuva, entra em colapso”.
(C) g(g(x)) = x, ∀x ∈ [0,3; 0,8] (Revista Veja – 16/5/2001.)
(D) g(f(0,6)) > g(f(1)) No gráfico, tem-se o nível da água armazenada em uma barragem ao longo
dos últimos anos, que foi construída para represar água a fim de mover
36 Determine a função inversa de ƒ(x) = x5 + 1. as turbinas de uma usina hidrelétrica.

3x −1 nível(m) o nível máximo


37 Sendo f (x) = 3 2 x + 3 − 1 e g(x) = , ache ƒ–1, g–1 e g ° g. 120
2x + 5
80
38 Seja a função ƒ: [2, ∞) → I, ƒ(x) = x2 – x + 1, determine qual deve
ser o intervalo I para que ƒ admita uma função inversa.

x −1 30
39 (AFA) Determine a função inversa de f (x)= .
x o nível mínimo para gerar energia
1+ x 10
1
(A) . (D) .
1− x 1− x 1989 1995 2000 temp
1 1− x
(B) . (C) .
1+ x 1+ x Analise as alternativas e marque a opção correta:

(A) O nível da água permaneceu constante num período de 8 anos.


40 (AMAN) Sejam ƒ e g funções de A em A com gráficos f* = {(1, 2), (B) O nível de 80 metros foi atingido exatamente duas vezes até o ano
(2, 1), (3, 5), (4, 4), (5, 2)} e g* = {(1, 1), (2, 3), (3, 5), (4, 3), (5, 1)}. 2000.
Logo, ƒ–1(4) · g–1(5) vale: (C) Após o ano de 2000, o nível da água da barragem foi insuficiente para
gerar energia.
(A) 0. (D) 6. (D) No período de 1995 a 2000, o nível da água só diminuiu.
(B) 2. (E) 12.
(C) 25.

IME-ITA 181
Matemática I – Assunto 3

4 x − 6 x − 1 se
2
x≥1 49 Esboce no plano cartesiano os gráficos das seguintes funções:
45 (AFA) Considere as funções reais ( f  g )( x ) = 
4 x + 3 se x <1
(A) ƒ:  → {– 1, 1}
e g(x) = 2x – 3. Com base nessas funções, classifique cada afirmativa x → y = signx, função sinal, signx = {1 se x ≥ 0 e –1 se x < 0}
abaixo como verdadeira ou falsa. (B) ƒ:  → 
x → y = x = (n ∈  / n ≤ x< n + 1), função parte inteira.
I. ƒ(x) é par; (C) ƒ:  → [0,1[
II. ƒ(x) admite inversa em todo o seu domínio; x → {x} = x – x, função parte fracionária.
III. ƒ(x) é crescente em {x ∈  / x < – 1 ou x ≥ 1};
IV. se x < – 6, então ƒ(x) > – 3. 50 A figura abaixo representa o gráfico da função definida por f(x) =
acos(bx). Os valores de a e b são, respectivamente:
A sequência correta é:
Y
(A) V – V – F – V.
(B) F – F – V – F.
(C) F – F – V – V.
(D) F – V – V – F. 1

46 (EN) Sabendo que f, g e h são funções reais de variável real e que f e 3π 4π


g não se anulam, considere as afirmações abaixo: –π 0 π 2π x

I. ƒ ° (g + h) = ƒ ° g + ƒ ° h;
–1
II. (g + h) ° ƒ = g ° ƒ + h ° ƒ;

1  1 (A) 1 e 2.
III. =   ° g;
f °g  f 
1
(B) 1 e .
1  1 2
IV. = f ° 
f °g g 1
(C) – 1 e .
2
Podemos afirmar que: (D) –1 e 1.
(E) –1 e 2.
(A) todas as afirmativas acima são verdadeiras.
(B) somente I e II são verdadeiras.
(C) somente IV é falsa. EXERCÍCIOS NÍVEL 2
(D) somente II e III são verdadeiras.
(E) somente I é falsa. 01 Determine o conjunto-imagem das funções abaixo:

47 (ITA) Consideremos as seguintes afirmações sobre uma função f:  → . | x|


(A) ƒ:  – {0} → , x → y =
x
I. Se existe x ∈  tal que ƒ(x) ≠ ƒ(– x), então f não é par.
II. Se existe x ∈  tal que f(– x) = – f(x), então f é ímpar. (B) ƒ: [4, +∞[ → , x → y = x + x −4
III. Se f é par e ímpar, então existe x ∈  tal que f(x) = 1.
IV. Se f é ímpar, então f ° f (f composta com f) é ímpar. 1
(C) ƒ:  → , x → y =
x2 + 1
Podemos afirmar que estão corretas as afirmações de números:
x
(D) ƒ:  → , x → y =
(A) I e IV. x2 + 1
(B) I, II e IV. 02 (IME) Sejam q e r funções cujo domínio são os inteiros maiores
(C) I e III. que zero. Sabe-se que q(1) = 1, r(1) = 0 e:
(D) III e IV.
(E) I, II e III.  r( n + 1) = r( n) + 1
se r(n) < 2q(n) + 1, então 
q( n + 1) = q( n)
48 (ITA) Considere a função y = f(x) definida por f(x) = x3 – 2x2 + 5x, para
cada x real. Sobre esta função, qual das afirmações abaixo é verdadeira?
 r( n + 1) = 0
se r(n) = 2q(n) + 1, então 
(A) y = f(x) é uma função par. q( n + 1) = q( n) + 1
(B) y = f(x) é uma função ímpar.
(C) ƒ(x) ≥ 0 para todo real x. Determine q(5) e r(5).
(D) ƒ(x) ≤ 0 para todo real x.
(E) f(x) tem o mesmo sinal de x, para todo real x ≠ 0.

182 Vol. 1
Relações e funções

03 Mostre que se ƒ: A → B é injetiva e #A = #B, então ƒ é uma bijeção. 11 (OBM) A função ƒ é definida para todos os pares ordenados (x; y) de
inteiros positivos e tem as seguintes propriedades:
04 Classifique a função ƒ: N × N → N, ƒ(m, n) = 2m · 3n quanto a ƒ(x; x) = x; ƒ(x; y) = ƒ(y; x); (x + y)ƒ(x; y) = (2x + y)ƒ(x; x + y). Qual
injetividade e sobrejetividade. o valor de ƒ(21; 12)?

05 Determine o valor da expressão 7


.
(A) 4
 1   2   3   1998   1999   2000 
f +f +f  + ... + f  3  + f  2  + f  1  , 4
 2000   1999   1998        (B) .
7

x2 11
em que f ( x ) = . (C)
6
.
1+ x 2
06 (OBM) Seja ƒ uma função real de variável real que satisfaz a condição 6
(D) 11.
 2002 
f ( x ) + 2f   = 3 x para x > 0. O valor de ƒ(2) é igual a: 1
 x  (E) .
2003
(A) 1.000.
(B) 2.000.
12 (OBM) Seja ƒ(x) = x2 – 3x + 4. Quantas soluções reais tem a equação
(C) 3.000.
ƒ(ƒ(ƒ(...ƒ(x)))) = 2 (em que ƒ é aplicada 2001 vezes)?
(D) 4.000.
(E) 6.000.
(A) 0.
(B) 1.
07 (OBM) A função real ƒ, definida nos inteiros, satisfaz
(C) 2.
ƒ(n) – (n + 1) ƒ(2 – n) = (n + 3)2, para todo n inteiro . Quanto vale ƒ(0)?
(D) 2001.
(E) 22001.
(A) –17. (D) 2.
(B) 0. (E) 9.
13 (OBM) Seja ƒ uma função de Z em Z definida como
(C) 1.
ƒ(x) = x/10 se x é divisível por 10 e ƒ(x) = x + 1 caso contrário.
Se a0 = 2001 e an+1 = f(an), qual o menor valor de n para o qual an = 1?
08 (OBM) Seja ƒ: Z → Z uma função tal que ƒ(0) = 0, ƒ(1) = 1,
ƒ(2) = 2 e ƒ(x + 12) = ƒ(x + 21) = ƒ(x) para todo x ∈ Z. Então ƒ(2009) é:
(A) 20.
(B) 38.
(A) 0.
(C) 93.
(B) 1.
(D) 2000.
(C) 2.
(E) an nunca é igual a 1.
(D) 3.
(E) 2009.
14 Seja ƒ:  →  uma função que: (∀x1, x2∈ )(ƒ(x1 + x2) = ƒ(x1)·ƒ(x2)):
n
09 (OBM) Para todo n natural definimos a função ƒ por: f ( n ) = (A) Prove que ƒ(x) ≥ 0, ∀x∈;
se n é par, ƒ(n) = 3n + 1 se n é ímpar. 2
(B) Calcule ƒ(0);
(C) Prove que (ƒ(x) ≡ 0 ↔ ƒ(0) = 0);
O número de soluções da equação ƒ(ƒ(ƒ(n))) = 16 é: (D) Se ƒ(1) = 2, calcule ƒ(2), ƒ(4), ƒ(– 2) e a relação entre ƒ(– x) e ƒ(x).

(A) 2. 15 Seja ƒ: +→  tal que: (∀x1, x2∈ +) (ƒ(x1 · x2) = ƒ(x1) + f(x2)
(B) 3.
(C) 4. a. Calcule ƒ(1);
(D) 5.
(E) 6. b. Determine a relação entre ƒ(x– 1) e ƒ(x);

10 (OBM) Seja ƒ uma função real que tem as seguintes propriedades: x1


c. Prove que ƒ ( ) = ƒ(x1) – ƒ(x2), ∀x1, x2 ∈ +.
x2
I. Para todos x, y reais, ƒ(x + y) = x + ƒ(y);
II. ƒ(0) = 2.
16 (EN) O conjunto-imagem da função f ( x ) = x 2 − 4 + 4 − x 2 é:
Quanto vale ƒ(2000)?
(A) {x ∈  / x > 0}.
(A) 0. (B) {x ∈  / – 2 < x < 2}.
(B) 2. (C) {0}.
(C) 1998. (D) {x ∈  / x ≤ – 2 ou x ≥ 2}.
(D) 2000. (E) +.
(E) 2002.

IME-ITA 183
Matemática I – Assunto 3

17 (EN) Considere a função real ƒ definida por:  x + 2 , se x ≤ −1



20 (ITA) Seja ƒ:  →  a função definida por f(x) =  x 2 , se − 1 < x ≤ 1
 x 2 − 1 se x < − 2
 4 , se x > 1
 3 se − 2 ≤ x < − 1

 x2 − 1 Lembrando que se A ⊂  então f–1(A) = {x ∈ : f(x) ∈ A} considere as

 3 se − 1 < x < 1 afirmações:
 x2 − 1
f (x) =  I. f não é injetora e f–1 ([3 , 5]) = {4}
 3
se 1 < x < 2 II. f não é sobrejetora e f–1 ([3 , 5]) = f–1 ([2 , 6])
 x2 − 1
 3
x se x ≥ 2 III. f é injetora e f–1 ([0 , 4]) = [–2 , +∞[

 2 se x = 1
 Então podemos garantir que:
 2 se x = − 1
(A) Apenas as afirmações II e III são falsas.
A imagem da função ƒ é o conjunto: (B) As afirmações I e III são verdadeiras.
(C) Apenas a afirmação II é verdadeira.
(A) ] –∞, –3] U [1, +∞[. (D) Apenas a afirmação III é verdadeira.
(B) ]–∞, –1[ U [2, +∞[. (E) Todas as afirmações são falsas.
(C) ]–∞, –3[ U ]–1, 1[ U ]1,+∞[.
(D) ]–∞, –2[ U ]–2, –1[ U ]–1, +∞[. 21 (OBM) A função real ƒ, definida nos inteiros, satisfaz ƒ(n) – (n + 1)
(E) R – {–1, 1} ƒ(2 – n) = (n + 3)2, para todo n inteiro. Determine ƒ(n)?

18 ( I TA ) S e j a m t r ê s f u n ç õ e s f, u , v :  →  t a i s q u e f 22 (ITA) Sejam  o conjunto dos números reais e f uma função de  em


 1 1 . Se B ⊂  e o conjunto f
−1
{ }
( B ) = x ∈ ; f ( x ) ∈ B , então:
 x +  = f( x ) + para todo x não nulo e (u(x))2 + (v(x))2 = 1
 x  f ( x)
para todo x real. Sabendo-se que x0 é um número real tal que u(x0) · v(x0) ( )
(A) f f −1 ( B ) ⊂ B
 1 1   u (x 0 ) 
≠ 0 e f
 u (x 0 )
.
v (x )
0 
= 2, o valor de f   é:
 v (x 0 ) 
( )
(B) f f −1 ( B ) = B, se ƒ é injetora.

(A) –1. ( )
(C) f f −1 ( B ) = B,

(B) 1. ( )
(D) f −1 f ( B ) = B, se f é injetora.

(C) 2. (E) n.d.a

23 (ITA) Sejam A e B subconjuntos não vazios dos números reais e


(D) 1 .
2 ƒ: A → B, g : B → A duas funções tais que f  g = Id B. Então podemos
afirmar que:
(E) –2.
(A) ƒ é sobrejetora.
19 (ITA) Dadas as sentenças:
(B) ƒ é injetora.
(C) ƒ é bijetora.
I. Sejam ƒ: X → Y e g: Y → X duas funções satisfazendo (goƒ) (x) =
(D) g é injetora e par.
x, para todo x ∈ X. Então ƒ é injetiva, mas g não é necessariamente
(E) g é bijetora e ímpar.
sobrejetiva.
II. Seja ƒ: X → Y uma função injetiva. Então, ƒ(A) ∩ ƒ(B) = ƒ(A ∩ B),
24 ( I TA ) S e j a ƒ :  →  u m a f u n ç ã o s a t i s f a z e n d o
em que A e B são dois subconjuntos de X.
ƒ(x + αy) = ƒ(x) + αƒ(y) para todos a, x, y reais. Se a1, a2, a3,..., an é uma
III. Seja ƒ: X → Y uma função injetiva. Então, para cada subconjunto A de X,
progressão de razão d, então podemos dizer que ƒ(a1), ƒ(a2), ƒ(a3),..., ƒ(an)
ƒ(AC) ⊂ (ƒ(A))C onde AC = {x∈X / x∉A} e (ƒ(A))C = {x∈Y / x∉ƒ(A)]}.
(A) é uma progressão aritmética de razão d.
Podemos afirmar que está (ao) correta(s):
(B) é uma progressão aritmética de razão f(d) cujo primeiro termo é a1.
(C) é uma progressão geométrica de razão f(d).
(A) as sentenças no I e no II;
(D) é uma progressão aritmética de razão f(d).
(B) as sentenças no II e no III;
(E) nada se pode afirmar.
(C) apenas a sentença no I;
(D) as sentenças no I e no III;
x −3
(E) todas as sentenças. 25 (EN) Seja x ∉ {–1, 0, 1}. Se f1( x ) = e ƒn+1(x)= ƒ1(ƒn(x)) para
todo n natural, então f1988(x) igual a: x +1

184 Vol. 1
Relações e funções

(A) x − 3 . (D)
3− x 1+ f ( x )
. 31 Seja a um real fixo positivo e ƒ uma função tal que f ( x + a ) =
x +1 x +1 1− f ( x )
para todo x. Prove que ƒ é periódica determinando um período.
(B) x. (E) x + 3 .
x −1  −d  a ax + b
(C) x + 3 . 32 Considere uma função f :  −   →  −   , ƒ(x) = ,
 c  c cx + d
1− x
onde ad ≠ bc e a, b, c, d∈* (funções desse tipo recebem o nome
26 (EN) Determine o conjunto-imagem da função (fog) para: de função homográfica ou função de Möbius). Prove que ƒ é bijetiva e
determine sua inversa f–1.
1 se x < 0
0 se x < 0  33 (EsPCEx) Seja a função ƒ:  – { –1, 1} → , definida por
 x x3
f ( x ) = 2x se 0 ≤ x ≤ 1 e g( x ) =  se 0 ≤ x ≤ 1 f( x ) = 2 .
0 se x > 1 2 x − 1 Podemos afirmar que essa função é:
 1 se x > 1
(A) bijetora e não par nem ímpar. (D) par e sobrejetora.
(B) par e injetora. (E) ímpar e sobrejetora.
(A) |0,1| ∪ {2}. (D) |0, +∞). (C) ímpar e injetora.
(B) (–∞, +∞). (E) {1}.
(C) |0, 1|. 34 Seja f:  →  uma função estritamente decrescente, isto é, quaisquer
x e y reais com x < y tem-se f(x) > f(y). Dadas as afirmações:
27 (ITA) Sejam f, g :  →  funções tais que: g(x) = 1 – x e
ƒ (x) + 2ƒ (2 – x ) = (x – 1)3 para todo x ∈ . Então ƒ [g(x)] é igual a: I. f é injetora;
II. f pode ser uma função par;
(A) (x – 1)3. (D) x. III. se f possui inversa então sua inversa também é estritamente
(B) (1 – x)3. (E) 2 – x. decrescente.
(C) x3.
Podemos assegurar que:
28 Determine a inversa da função u: ( –∞, –1) → (2, + ∞) definida por
u(x)=x2 + 2x + 3. (A) Apenas as afirmações I e III são verdadeiras.
(B) Apenas as afirmações II e III são falsas.
29 (ITA) Considere x = g(y) a função inversa da seguinte função: (C) Apenas as afirmações I é falsa.
1 (D) Todas as afirmações são verdadeiras
“y = f(x) = x2 – x + 1, para cada número real x ≥ ”. Nestas condições,
a função g é assim definida: 2 (E) Apenas a afirmação II é verdadeira.

1 3 3 35 (ITA) Considere as afirmações:


(A) g( y ) = + y − , para cada y ≥ .
2 4 4 I. Se f:  →  é uma função par e g:  →  uma função qualquer,
então a composição g°f é uma função par.
1 1 1 II. Se f:  →  é uma função par e g:  →  uma função ímpar, então
(B) g( y ) = + y − , para cada y ≥ . a composição f°g é uma função par.
2 4 4
III. Se f:  →  é uma função ímpar e inversível, então f–1:  →  é uma
função ímpar.
3 3
(C) g( y ) = y − , para cada y ≥ .
4 4 Então:

(A) Apenas a afirmação I é falsa.


1 1
(D) g( y ) = y − , para cada y ≥ . (B) Apenas as afirmações I e II são falsas.
4 4
(C) Apenas a afirmação III é verdadeira.
(D) Todas as afirmações são falsas.
3 1 1 (E) n.d.a.
(E) g( y ) = + y − , para cada y ≥ .
4 2 2
36 (ITA) Dadas as funções f:  →  e g:  → , ambas estritamente
30 (ITA) Seja f uma função real definida para todo x real tal que: f é ímpar; decrescentes e sobrejetoras, considere h = f°g. Então podemos afirmar que:

f ( x ) − f (1) (A) h é estritamente crescente, inversível e sua inversa é estritamente


f(x – y) = f(x) – f(y); e f(x) ≥ 0, se x ≥ 0. Definindo g( x ) = , crescente.
x
se x ≠ 0, e sendo n um número inteiro positivo, podemos afirmar que: (B) h é estritamente decrescente, inversível e sua inversa é estritamente
crescente.
(A) f é não decrescente e g é uma função ímpar. (C) h é estritamente crescente, mas não necessariamente inversível.
(B) f é não decrescente e g é uma função par. (D) h é estritamente crescente, inversível e sua inversa é estritamente
(C) g é uma função par e 0 ≤ g(n) ≤ f(1). decrescente.
(D) g é uma função ímpar e 0 ≤ g(n) ≤ f(1). (E) n.d.a.
(E) f é não decrescente e 0 ≤ g(n) ≤ f(1)

IME-ITA 185
Matemática I – Assunto 3

37 (ITA) Considere uma função f:  →  não constante e tal que 03 Mostre que toda função ƒ:  →  pode ser escrita com uma soma
f(x + y) = f(x)f(y), ∀x, y ∈ . P(x) + I(x), em que P é uma função par e I é ímpar.

Das afirmações: 04 A função ƒ é tal que, para cada número real x, vale a relação
ƒ(x) + ƒ(x – 1) = x2. Se ƒ(19) = 94, então ƒ(94) vale:
I. f(x) > 0, ∀ x ∈ .
II. f(nx) = [f(x)]n, ∀x ∈ , ∀n ∈ N*. (A) 3227.
III. f é par. (B) 3572.
(C) 3763.
é (são) verdadeira(s):
(D) 4245.
(A) apenas I e II. (E) 4561.
(B) apenas II e III.
(C) apenas I e III. 05 (OBM) A função ƒ:  →  satisfaz ƒ(x + ƒ(y)) = x + ƒ (ƒ(y)) para
(D) todas. todos os números reais x e y. Sabendo que ƒ(2) = 8, calcule ƒ(2005).
(E) nenhuma.
06 (IBERO) A cada inteiro positivo n se associa um inteiro não negativo
38 Seja ƒ(x) uma função real, definida em  e satisfazendo a equação ƒ(n) de tal maneira que se satisfazem as seguintes condições:

I. ƒ(r.s) = ƒ(r) + ƒ(s);


 x − 1
funcional f ( x ) + f   = 1 + x . A expressão de ƒ(x) é: II. ƒ(n) = 0, sempre que o algarismo da unidade de n seja 3;
 x  III. ƒ(10) = 0.

x3 − x2 − 1 x3 + x2 − 1 Determine ƒ(1985).
(A) . (D) .
2 x ( x − 1) x ( x − 1)
07 Resolva a equação a+ a+ x = x
3 2 3 2
x + x +1 x + x −1
(B) . (E) . 08 Encontre as raízes reais da equação
x ( x − 1) x ( x + 1)
1 1 1
x 2 + 2 ax + = − a + a2 + x − (0 < a < )
x3 − x2 + 1 16 16 4
(C) .
x ( x − 1)
09 (OBM) Seja ƒ: N → R uma função tal que
1+ x
39 A função real definida por f ( x ) = pode ser decomposta, de ƒ(1) = 999 e ƒ(1) + ƒ(2) +...+ ƒ(n) = n2 ƒ(n). Determine f(1998).
1− x
maneira única, como uma soma da forma P( x ) + I( x ), onde P(x) é uma
função par e I(x) é uma função ímpar. A expressão de I(x) é: 10 Seja ƒ:  →  uma função tal que ƒ(0) = 1 e para todo x e y ∈ ,
ƒ(xy + 1) = ƒ(x) · ƒ(y) – ƒ(y) – x + 2. Determine ƒ.
x 4x
(A) . (D) .  1 1− x
1− x 2 1− x 2 11 Existe função ƒ: * → , tal que xf ( x ) + x 3 f   = ?
x x
2x 5x
(B) . (E) .
1− x 2 1− x 2
 1 
3x 12 Determine a função que satisfaz f ( x ) + f   = x para todos
(C) .  1− x 
1− x 2 x ∉ {0,1}

13 (OBM) Seja ƒ uma função dos reais não nulos nos reais não nulos tal
EXERCÍCIOS NÍVEL 3 que:

01 (OBM) Se ƒ:  →  é uma função tal que, para todo x real, • (ƒ(x) + ƒ(y) + ƒ(z))2 = ƒ(x))2 + (ƒ(y))2 + (f(z))2 para todos x, y, z
ƒ(x) · (ƒ(x) – x) = 0, então: reais não nulos tais que x + y + z = 0;
• ƒ(– x) = – ƒ(x)para todo x real não nulo;
(A) ƒ é a função nula. • ƒ(2011) = 1.
(B) ƒ é a função identidade, ou seja, ƒ(x) = x para todo x real.
(C) ƒ é a função nula ou a função identidade. Encontre o inteiro mais próximo de ƒ(33).
(D) Há 4 possíveis funções ƒ.
(E) Há infinitas funções ƒ.
{ }
14 (OBM) Para cada inteiro positivo n, seja An = x ∈ R+ ; x ⋅  x  = n ,
em que R+ é o conjunto dos reais positivos e  x  é o maior inteiro
02 (OBM) Determine todas as funções ƒ:  → , ƒ função par,
menor ou igual a x. Determine a quantidade de elementos do conjunto
satisfazendo ƒ(x + y) = ƒ(x) + ƒ(y) + 8xy +115 para todos os reais x
A1 ∪ A2 ∪ A3 ∪ ... ∪ A2009.
e y.

186 Vol. 1
Álgebra básica A ssunto
1
Matemática II

Neste material, estudaremos os princípios básicos da álgebra. No Teorema 2 (Multiplicação por zero)
decorrer das seguintes semanas, ficarão claras sua importância e suas
Para todo x real, tem-se que x · 0 = 0.
aplicações em todas os outros assuntos da matemática. Podemos dizer que
o aluno que não alcançar um domínio mínimo neste assunto certamente
terá dificuldades com as outras áreas. Demonstração: A ideia é utilizar que x · (0 + 0) = x · 0. Fazendo
distributiva do lado esquerdo, temos x · 0 + x · 0 = x · 0 + 0. Pela lei do
corte, segue que x · 0 = 0.
1. Axiomas e conceitos
básicos dos números reais () Comentário: Por isso não se define a divisão por 0. Se por um
1
No conjunto dos reais, são definidas duas operações: a soma e o momento aceitássemos = x , teríamos que 1 = 0 · x, o que não é
produto. Essas operações possuem propriedades básicas que não podem possível. 0
ser demonstradas e, por isso, as chamamos de axiomas.
Teorema 3 (Produto igual a zero)
I. Comutativa Se x e y são reais tais que x · y = 0, então x = 0 ou y = 0.
x+y=y+x
x·y=y·x Demonstração: Pelo teorema 2, é fácil ver que se x = 0, a equação
x · y = 0 é satisfeita. Caso x ≠ 0, sabemos que existe o seu inverso x–1.
II. Associativa Multiplicando os dois lados de x · y = 0 por x–1, temos x–1 · (x · y) = x–1 · 0.
(x + y) + z = x + (y + z) Utilizando a associativa do lado esquerdo e o teorema 2 do lado direito, temos que
(x · y) · z = x · (y · z) y = 0. Portanto, ou x = 0 ou y = 0 .
III. Elemento Neutro
x + 0 = x Essa é uma das principais propriedades da álgebra e é uma das
x·1=x grandes motivações para se aprender a fatorar. Em algum problema em
que uma expressão é igual a zero, se conseguirmos fatorar essa expressão,
IV. Elemento Simétrico / Inverso podemos transformar o problema em dois geralmente mais simples.
x + (–x) = 0 (simétrico)
x · x–1 = 1, para x ≠ 0 (inverso)
Teorema 4 (Regra dos sinais):
I. (–x) · y = x · (–y) = –(x · y)
V. Distributiva
x · (y + z) = x · y + x · z II. (–x) · (–y) = x · y
(y + z) · x = x · y + x · z
Esse é o famoso "menos com menos dá mais e menos com mais
Assim, definimos também: dá menos".

Diferença: x – y = x + (–y) Demonstração: Para (1), a ideia é usar que (x + (– x)) · y = 0 · y =


0 Fazendo a distributiva do lado esquerdo, temos x · y + (– x) · y = 0.
x Daí, basta somar –(x · y) dos dois lados e ficamos com (– x) · y = –(x ·
Divisão: = x ⋅ y −1, para y ≠ 0
y y). A outra parte de (1) é análoga.
Axiomaticamente, podemos, em uma igualdade, somar uma mesma
quantidade dos dois lados, como também podemos multiplicar os dois Para (2), usaremos que (– x) · ((– y) + y) = (– x) · 0 = 0. Fazendo
lados por uma mesma quantidade. Com isso e as regras iniciais, já é a distributiva, temos que (– x) · (– y) + (– x) · y = 0. Usando (1), temos
possível demonstrar alguns teoremas. (– x) · (– y) + (–(x · y)) = 0. Agora, basta somar (x · y) e ficamos com
(– x) · (–y) = x · y.
Teorema 1 (Lei do corte – soma)
Teorema 5 (Lei do corte – produto)
Se x, y e z são números reais tais que x + y = x + z, então y = z.
a = 0
Demonstração: Na equação x + y = x + z, podemos somar (–x) Se ax = ay, então ou .
dos dois lados: (–x) + (x + y) = (–x) + (x + z). Utilizando a propriedade x = y
associativa, podemos somar antes o x ao seu simétrico: (–x + x) + y = 
(–x + x) + z. Daí, 0 + y = 0 + z, o que nos dá y = z.
Demonstração: Em ax = ay, somamos – ay dos dois lados, ou
seja, se "passarmos o ay para o outro lado", temos ax – ay = 0, ou seja,
a(x – y) = 0. Como já vimos, temos que a = 0 ou x – y = 0 (ou seja, x = y).

IME-ITA 187
Matemática II – Assunto 1

É importante observar que você não deverá fazer o passo a passo


de nenhum desses teoremas durante os exercícios. Fazemos essas I. Como (x + z) – (y + z) = x – y, a ordenação não se altera;
demonstrações apenas para a teoria ficar completa e para que você II. (x + z) – (y + w) = (x – y) + (z – w) é a soma de dois positivos,
aumente sua capacidade de abstrair e de utilizar conceitos já dados para então é positivo;
chegar a novos resultados. III. Basta somar as duas inequações e cancelar o y;
IV. xz – yz = (x – y)z é positivo quando x – y e z têm o mesmo sinal;
Teorema 6  x > y ⇒ xz > yz
V.  e termina pela transitividade;
(Tirando raiz quadrada em uma equação)  z > w ⇒ yz > yw
Se x2 = y2, então x = y ou x = – y. (e mque x2 = x · x)
1 1 x−y
VI. − = é o quociente entre dois positivos
y x xy
Demonstração: Aqui utilizaremos um ‘produto notável’ que será visto
mais à frente: usaremos que x2 – y2 = (x – y) · (x + y) (*). Então, em
x2 = y2, somemos – y2 dos dois lados: x2 – y2 = 0. Daí, por (*), temos
2.2 Teorema 7
(x – y) · (x + y) = 0. Pelo teorema 3, segue que x = y ou x = – y. (Quadrado maior ou igual a zero)
Para todo x real, tem-se que x2 ≥ 0.
2. Inequações
Demonstração: Caso x seja positivo, x2 = x · x é o produto de dois
O conjunto dos números reais pode ser dividido em 3 partes: positivos, portanto positivo. Caso x seja negativo, x2 = x · x é o produto
de dois negativos, portanto positivo. Caso x seja nulo, x2 = x · x é nulo.
– reais positivos (*
+
); Portanto, x2 = x · x é sempre maior ou igual a zero.
– zero (0);
– reais negativos (*–).
Muitas desigualdades famosas decorrem dessa propriedade e isso
será cobrado ao longo do material.
*– *
+

• Erros comuns em inequações


0
I. Não é permitido subtrair inequações

Axiomaticamente, temos que se x e y são reais positivos, então x +


y e x · y também são. Além disso, temos também que se x é real positivo, 8 > 6
Por exemplo,  é verdade, mas 8 – 7 > 6 – 3 (1 > 3) não é!
então – x é real negativo. Daí, pela regra de sinais, podemos ver que o 7 > 3
produto de dois negativos é um positivo e o produto de um positivo por II. Só se pode elevar ao quadrado se os 2 lados são positivos
um negativo é negativo.
Por exemplo, 1 > – 2 é verdade, mas 12 > (– 2)2 (1 > 4) não é!
2.1 Relação de Ordem
Dizemos que: III. Não se pode passar variável multiplicando para o outro lado
x > y se x – y ∈ *+ ; x < y se x – y ∈ *–
1 1 x
Considere as inequações > (* ) e 1 > (**).
Além disso, x ≥ y se x > y ou x = y; x ≤ y se x < y ou x = y. x 2 2
Veja que de (*) para (**), a inequação foi multiplicada por x. No
• Propriedades
entanto, caso x fosse negativo, o sinal da inequação deveria ser modificado
(propriedade 4).
I. (somar dos dois lados / lei do corte) x > y ⇔ x + z > y + z;
Esse é o erro mais comum neste assunto, tome muito cuidado!
x > y
II. (somar inequações)  ⇒ x + z > y + w;
z > w 3. Potências e Raízes
x > y
III. (transitividade)  ⇒ x > z; 3.1 Potência
y > z
Para a real não nulo (base) e n inteiro positivo (expoente) definimos
 x > y , se z > 0 1
IV. (lei do corte – produto) xz > yz ⇔  a = a · an–1 e a0 =1. Para expoentes negativos, definimos a− n = n .
n
 x < y , se z < 0 a
Propriedades:
 x > y> 0
V. (multiplicar inequações)  ⇒ xz > yw n
z > w > 0 an  a
I. am · an = am + n IV. n
= 
1 1 b  b
VI. (inversão) x > y > 0 ⇒ <
x y am
II. = am− n V. (am)n = amn
Demonstrações: an
III. anbn = (ab)n

188 Vol. 1
Álgebra básica

Todas as propriedades podem ser rapidamente demonstradas. 4. Produtos notáveis e


3.2 Raízes fatorações iniciais
1
Determinadas expressões aparecem muitas vezes em matemática. As
Para a real e n inteiro positivo, definimos n a = a n .
primeiras desse tipo são os chamados produtos notáveis:

Essa definição é bastante natural, já que, de forma habitual, definimos (0) (distributiva/colocar em evidência) (a + b) x = ax + bx
n
 1 (1) (distributiva/agrupamento) (a + b)(x + y) = ax + ay + bx + by
n  
como x = a um número tal que x = a e a
n n = a. (2) (produto de Stevin) (x + a)(x + b) = x2 + (a + b)x + ab
 
  (3) (quadrado da soma) (a + b)2 = a2 + 2ab + b2
(4) (quadrado da soma de 2 termos) (a + b + c)2 = a2 + b2 + c2 + 2ab
Observação: Caso n seja par, pelo teorema 6, a equação xn = a nos + 2bc + 2ac
dá que a ≥ 0. Além disso, para não haver duplo sentido, acrescentamos (5) (diferença de quadrados) (a + b) (a – b) = a2 – b2
à definição que n a ≥ 0 para n par. (6) (cubo da soma) (a + b)3 = a3 + 3a2b + 3ab2 + b3
(7) (soma de cubos) (a + b) (a2 – ab + b2) = a3 + b3
Por exemplo, 9 = 3. (8) (cubo da soma de 3 termos) (a + b + c)3 = a3 + b3 + c3 + 3(a +
b) (b + c) (c + a)
Imagine, por um momento, que aceitássemos que radicais de índice Não é difícil fazer a distributiva no lado esquerdo de cada uma das
par pudessem ter 2 valores (no exemplo anterior, +3 e –3). Neste caso, equações acima e chegar ao lado direito. Caso a necessidade fosse
sempre essa, você poderia refazer isso a cada problema. No entanto, a
que valor assumiria a expressão 4 + 9 + 16 ? Vários valores seriam
maior utilidade desse ponto é já conhecer de antemão essas expressões
possíveis: 2 + 3 + 4, 2 – 3 +4, 2 – 3 –4... Com isso, uma expressão
para que, rapidamente, se possa substituí-las pela sua forma fatorada.
simples geraria uma grande confusão! Para isso não acontecer, aceitamos
apenas o sinal de +. Como essas propriedades valem para todos a, b, c e x, podemos
substituir essas letras como quisermos. Em particular, trocando b por –b
• Propriedades em (1), (4) e (5), obtemos:
(3’) (quadrado da diferença) (a – b)2 = a2 – 2ab + b2
Para a e b positivos, temos: (6’) (cubo da diferença) (a – b)3 = a3 – 3a2b + 3ab2 – b3
(7’) (diferença de cubos) (a – b)(a2 + ab + b2) = a3 – b3
n Todas essas expressões devem ser memorizadas e, para que se tenha
p a n a mais facilidade nisso, sugere-se que muitos exercícios sejam feitos.
I. n p III. =
a = a n n
b b
II. n
a · n b = n ab IV.
m n
a = mn a Comentários: Como já vimos, não é difícil partir de cada lado
esquerdo até chegar ao lado direito correspondente. No entanto, em alguns
Dessa forma, pela propriedade (1), definimos potências para expoentes momentos isso pode parecer artificial. Um bom exemplo é (5), já que será
racionais (fracionários). muito mais comum aparecer a3 + b3. Caso não soubéssemos que a3 + b3
pode ser escrito como (a + b)(a2 – ab + b2) , como poderíamos chegar a
• Expoentes irracionais esse resultado? Em álgebra, assim como em toda matemática, a maneira
mais eficiente de se resolver um problema é associá-lo a alguma situação
já vista anteriormente. Podemos ver que a3 + b3 está no desenvolvimento
Vamos entender este conceito através de um exemplo.
de (a + b)3, que é uma expressão muito comum.

Qual seria o valor de 3 2 ?


Daí, (a + b)3 = a3 + 3a2b + 3ab2 + b3 implica a3 + b3 =
= (a + b)3 – (3a2b + 3ab2) = (a + b)3 – 3ab(a + b) =
Este é um problema computacional. Para chegar a este valor,
precisamos fazer aproximações (pelo truncamento da representação  
 (a + b)2 − 3 ab  = (a + b)(a2 − ab + b2 )
decimal do expoente) por cima e por baixo     
 a2 + 2 ab + b2 
 
31 = 3 32 = 9
31,4 = 4,65554... 31,5 = 5,19615... É possível abordar de forma similar outros produtos notáveis.
31,41 = 4,70697... 31,42 = 4,75896...
31,414 = 4,72770... 31,415 = 4,73289...
31,4142 = 4,72873... 31,4143 = 4,72925...

Assim, fazendo essas aproximações cada vez mais precisas, temos


o valor de 3 2 = 4 ,72880...
Com essa expansão da definição de potência, carregamos todas as
propriedades vistas inicialmente para expoentes inteiros.

IME-ITA 189
Matemática II – Assunto 1

EXERCÍCIOS RESOLVIDOS

a2 + ab + ac + bc Solução:
01 Simplifique a expressão A = . Primeiramente, vejamos que a expressão a2 – 1 pode ser vista como uma
b2 + ab + ac + bc
diferença de quadrados e, por isso, pode ser fatorada: a2 – 12 = (a + 1)(a – 1).
(a + 1)(a − 1) (a + 1)(a − 1)
Solução: Daí,temosque: E = − = (a + 1) − (a − 1) = 2
Antes de fazer qualquer tipo de cancelamento, precisamos fatorar o a−1 a+1
numerador e o denominador. E, para fatorar essas expressões com 4 , que não depende de a.
parcelas, normalmente usamos o que é chamado de ‘agrupamento’:
x+y
 a2 + ab + ac + bc = a(a + b) + c(a + b) = (a + c)(a + b) 03 (Desigualdade das médias para 2 termos) Prove que ≥ xy
 2 para todo x e y positivos. 2
 b + ab + ac + bc = b(a + b) + c((a + b) = (b + c)(a + b)
Solução:
(a + c)(a + b) a + c Para provar uma desigualdade, uma das ideias é olhar para a diferença
Substituindo, temos que A = = .
(b + c)(a + b) b + c entre os dois lados:
( x) ( y) = ( )
2 2 2
x+y x − 2 xy + y −2 x y + x − y
a −1 a −1 2 2 − xy = = .
02 Para a ≠ ± 1, prove que a expressão E = − não 2 2 2 2
depende de a. a−1 a+1 Como todo quadrado de número real é ≥ 0, então, segue que
x+y
− xy ≥ 0, o que finaliza o problema.
2

5. Técnicas de fatoração 5.3 Distributiva inteligente


É comum o vício de fazer a distributiva termo a termo quando duas
5.1 Completando o quadrado expressões estão sendo multiplicadas. No entanto, em alguns momentos
Em algumas situações, não é possível fatorar a expressão dada é interessante agrupar alguns termos antes disso.
apenas agrupando as parcelas. Uma das saídas pode ser se aproveitar da
semelhança entre a expressão e algum quadrado perfeito. Ex.: Fatorar E = (a2 + 3a + 3)(a2 + 3a + 5) – 15

Ex.: Fatorar a expressão a4 + 4. Solução:


Aqui, não vale a pena fazer toda a distributiva termo a termo, porque
Solução: ficaríamos com uma expressão com muitas parcelas e perderíamos a
Já que temos uma expressão com apenas 2 parcelas, não é possível repetição de parcelas que acontece na expressão dada. Por isso, fazemos
apenas colocarmos fatores em evidência. E = ((a2 + 3a) + 3)((a2 + 3a) + 5) – 15 e temos uma distributiva de
Então, percebemos que a expressão se assemelha a (a2 + 2)2 e, por isso, apenas 4 parcelas em vez de 9. Daí:
fazemos a4 + 4 = a4 + 4a2 + 4 – 4a2 = (a2 + 2)2 – (2a)2 o que nos leva
a uma diferença de quadrados. E = (a2 + 3a)2 + 3(a2 + 3a) + 5(a2 + 3a) + 15 – 15
Por isso, a4 + 4 = (a2 + 2 + 2a)(a2 + 2 – 2a). Por uma questão de E = (a2 + 3a)2 + 8(a2 + 3a)
organização, é comum colocarmos as potências em ordem decrescente
de expoente: a4 + 4 = (a2 + 2a + 2)(a2 – 2a + 2). e podemos colocar a2 + 3a em evidência, ficando com E = (a2 + 3a)
(a2 + 3a + 8). Além disso, ainda podemos fatorar o 1o fator e E = a(a
+ 3)(a2 + 3a + 8).
5.2 Teorema 8 (fórmula de Bhaskara)
5.4 Quebrando parcelas
−b ± ∆ O popular ‘agrupamento’ precisa de 4 parcelas para ser feito. Em
Para a ≠ 0, se ax2 + bx + c = 0, então x = , em que
2a algumas ocasiões, temos apenas 3 parcelas e podemos quebrar uma
∆ = b2 – 4ac.
delas em duas.

Demonstração: A ideia é completar o quadrado na equação para que Ex.: Fatorar  = 3x2 + 10 xy + 3y2.
a variável x apareça apenas um vez e não duas, como na situação inicial.
Solução:
b c Aqui, a ideia será quebrar o coeficiente 10 em duas partes. De nada seria
Primeiramente, dividimos tudo por a: x 2 + x + = 0.
a a útil escrevê-lo como 8 + 2, 7 + 3, 6 + 4 ou 5 + 5, pois 8, 7, 6 e 5 não
têm fator comum com 3 e, por isso, não conseguiríamos colocar nada em
b2
Para completar o quadrado, somando , temos: evidência em U. A boa ideia é quebrar 10 = 9 + 1. Assim:
4 a2
b b2 b2 c b2 − 4 ac U = 3x2 + 9xy + xy + 3y2
2 .
x + x+ 2 = 2 − = U = 3x(x + 3y) + y(x + 3y)
a 4a 4a a 4 a2
U = (3x + y)(x + 3y)
2
Daí,  x + b  = ∆ e x + b = ± ∆ e está finalizado.
 
 2a  4 a2 2a 2a

190 Vol. 1
Álgebra básica

5.5 Teorema 9 (soma de 3 números igual a Por outro lado, se multiplicarmos o numerador e o denominador de
1 3 1, 73205
0, soma dos cubos fatora) por 3 , teremos ≈ ≈ 0,5773.
3 3 3
Se a + b + c = 0, então a3 + b3 + c3 = 3abc. A esse processo de eliminação de radicais em denominadores, damos
o nome de ‘racionalização’.
Demonstração: Dado que a + b + c = 0, temos a + b = – c. Elevando
ao cubo, ficamos com (a + b)3 = (– c)3. Desenvolvendo: a3 + 3a2b + 3ab2 1
Ex.: Racionalizar .
+ b3 = – c3. Daí, segue que a3 + b3 + c3 = – 3a2b – 3ab2 = –3ab(a + b). 4+ 3
Como a + b = – c, temos a3 + b3 + c3 = 3abc. Solução:
Aqui, faremos uso do produto notável (a + b)(a – b) = a2 – b2 e
EXERCÍCIOS RESOLVIDOS
multiplicaremos o numerador e o denominador por 4 − 3 :
n+1 n x n +1
01 Sendo xn = 22 + 22 + 1, prove que é inteiro para todo n
natural. xn 1 4− 3 4− 3 4− 3
= = =
4+ 3 ( 4 + 3 )( 4 − 3 ) 16 − 3 13
Solução:
n n+1 n n
Inicialmente, faça 22 = a. Veja, então, que 22 = 22 · 2 = (22 )2 = a2
n+2 n n
e 22 = 2 2 · 2 = (2 2 ) 4 = a 4 . Com isso, temos que
2
Comentário: Uma outra forma de tratar o problema é encontrar uma
2 n+2 2 n+1 4 2 equação que tenha a expressão do denominador como raiz. No exemplo,
x n +1 2 +2 +1 a + a +1
= n+1 n
= 2 (* ) . fazendo 4 + 3 = x, temos x − 4 = 3 e, elevando ao quadrado, x2 – 8x +
xn 2
2 +2 +1 2 a + a+1 16 = 3 , o que nos dá a equação x2 – 8x + 13 = 0, que era o nosso objetivo.
1
Para concluir o problema, precisamos fatorar o numerador e, para Agora, podemos isolar :
isso, vamos ‘completar o quadrado’. x

1 8− x
Temos a4 + a2 + 1 = a4 + 2a2 + 1 – a2 = (a2 + 1)2 – a2 = (a2 + 1 ( x − 8) x = − 13 ⇒ =
+ a)(a2 + 1 – a). Substituindo em (*), temos x 13
Veja que, fazendo isso, eliminamos o radical do denominador.

x n +1 a4 + a2 + 1 (a2 + a + 1)(a 2 − a + 1) Substituindo o valor de x, temos


1 8− x 8− 4+ 3
= =
(=
4− 3
.
)
= 2 = = a2 − a + 1, que é x 13 13 13
xn a + a+1 a2 + a + 1
inteiro.
6.1 Teorema 10 (radicais duplos):
02 Prove que não existem a, b e c distintos tais que A+C A − C , em que
A± B = ± C= A2 − B
3 a−b + 3 b − c + 3 c − a = 0. 2 2

Solução: Demonstração: Basta comparar os quadrados dos dois lados.


Teríamos a soma de 3 números sendo igual a zero, portanto, a soma (Para uma motivação de como chegar a essa expressão, faça
de seus cubos é igual a 3 vezes o seu produto: A ± B = x ± y e escolha x e y convenientes.)
Em geral, é conveniente usar essa fórmula quando A2 – B for quadrado
(a − b) + (b − c) + (c − a) = 3 3 a − b 3 b−c3 c− a perfeito, para que transformemos um radical duplo em uma soma de
radicais simples.
Como o lado esquerdo é nulo e o lado direito é o produto de 3 fatores
não nulos (já que a, b e c são distintos), temos uma impossibilidade. EXERCÍCIOS RESOLVIDOS

01 Prove que o número x = 3 − 8 − 3 + 8 é inteiro e negativo.


6. Radicais
Uma situação muito comum é encontrarmos uma expressão que Solução
contenha radicais no denominador. Veja que efetuar uma divisão por um
É óbvio que x é negativo, pois 3 − 8 < 3 + 8 . Para provar que
número irracional pode nos levar a erros de aproximação, dependendo da
x é inteiro, elevamos ao quadrado:
precisão com que se tenha o denominador.
1
( )
2
Por exemplo, considere o cálculo do número . Caso utilizemos a x2 = 3− 8 − 3+ 8 =
3
1 1
aproximação 3 ≈ 1,7, encontraremos ≈ ≈ 0,5888. Caso refinemos
( ) ( )⇒
2 2
3 1, 7 3− 8 −2 3− 8 3+ 8 + 3+ 8
1 1
a aproximação com 3 ≈ 1,73, obteremos ≈ ≈ 0,5780.
( 8)
2
3 1, 73 x 2 = 3 − 8 − 2 32 − + 3 + 8 ⇒ x 2 = 4.
Por isso, como na prática não temos todas as casas de 3, a divisão Como x é negativo, temos que x = – 2, que é inteiro.
1
não deve ser feita dessa maneira.
3 Também é possível utilizar a fórmula do radical duplo.

IME-ITA 191
Matemática II – Assunto 1

Solução:
02 Qual é o valor de x = 6 + 6 + 6 + 6 + ... , com infinitos Pelo teste das raízes racionais, as possíveis raízes racionais de K são
radicais? 1 1 1
+ 1, − 1, + , − . Testando uma a uma, vemos que x = anula a
2 2 2
Solução 1
expressão K. Por isso, sabemos que x − é um fator de K, o que é
equivalente a 2x – 1 ser fator K. 2
2
Elevando ao quadrado, temos que x = 6 + 6 + 6 + 6 + ... .
Como há uma quantidade infinita de radicais, ficamos com x2 = 6 Agora, basta forçar o surgimento desse fator somando e subtraindo
+ x, ou seja, x2 – x – 6 = 0. As raízes dessa equação são x = 3 e x os termos corretamente:
= – 2. Como x é um radical, x é positivo.
K = 2x3 + 5x2 – x – 1 = 2x3 – x2 + 6x2 – 3x + 2x – 1
Por isso, x = 3.
x2 · (2x – 1) + 3x · (2x – 1) + 1 · (2x – 1) = (2x –1) (x2 + 3x + 1).
Observação: A rigor, antes de darmos os argumentos acima
deveríamos provar que x é um número real. Como x é o limite de uma
sequência, neste caso, poderia ser que x tendesse ao infinito. Aqui,
9. Expressões homogêneas
para formalizar, basta provar (por indução no número de raízes) que Em uma expressão algébrica, definimos o conceito de grau de uma
x é menor que 3. parcela como a soma dos expoentes nas suas variáveis. Por exemplo, o
grau de x3 y5 z2 é 10.
Dizemos que uma expressão é homogênea quando todas as suas
Agora, vereos outras técnicas de fatoração.
parcelas têm o mesmo grau. Por exemplo, x3 + 3x2y + 7xy2 – 4y3 é
homogênea pois todas as suas parcelas têm grau 3.
7. Diferença e soma de potências Caso haja uma expressão homogênea, há um artifício, muitas vezes
Dois produtos notáveis que também podem ser muito úteis são: com vantagens até mais psicológicas, que pode ser muito útil.

(1) xn – an = (x – a)(xn–1 + axn–2 + a2xn–3 +...+ an–1), para n natural Ex.: Considere a equação x3 + 3x2y + 7xy2 – 11y3 = 0. Veja que x = y = 0
3 2
(2) xn + an = (x + a)(xn–1 – axn–2 + a2xn–3 –...+ an–1), para n natural ímpar x x x
é solução. Para y ≠ 0, divida tudo por y3:   + 3   + 7   − 11 = 0
y y y
Para demonstrar as duas relações, basta fazer a distributiva nos lados
x
direitos. Fazendo = t , reduzimos a equação original de duas variáveis a
y
equação de apenas uma variável t3 + 3t2 + 7t – 11 = 0. Agora, poderíamos
8. Raízes × Fatores seguir utilizando as técnicas de fatoração já vistas.
8.1 Teorema 11 (raiz implica fator)
Se x = a anula uma expressão polinomial em x, então (x – a) é um
10. Mudança de variáveis
fator dessa expressão. Em muitos problemas, é interessante fazer uma mudança de variáveis
para simplificar a solução.
Demonstração: Considere a expressão E = bmxm + bm–1xm–1 +...+b1x 12
Ex.: Determinar as raízes reais de x 2 + 3 x + 1 = 2 .
+ b0. Se x = a anula a expressão E, então bmam + bm–1am–1 +...+ b1a + x + 3x + 2
b0 = 0. Subtraindo uma equação da outra, ficamos com E = bm(xm – am)
Solução:
+ bm–1(xm–1 – am–1) +...+ b1(x – a).
Inicialmente, repare que se multiplicarmos, chegaremos a uma
Agora, veja que pelo produto notável da diferença de raízes, cada equação do 4a grau, o que não seria bom. Ao perceber a semelhança entre
parênteses tem (x – a) como fator e, assim, E possui (x – a) como fator.
12
as expressões, façamos x2 + 3x + 2 = a. Daí, a equação é a − 1 = ,
8.2 Teorema 12 (teste das raízes racionais) a
Considere uma equação polinomial em x de coeficientes inteiros que equivale a a2 – a – 12 = 0, que tem soluções a = 4 e a = – 3.
Substituindo de volta, ficamos com as equações x2 + 3x – 2 = 0 e x2 +
E = bmxm + bm–1xm–1 +...+b1x + b0 −3 ± 17
3x + 5 = 0. Resolvendo-as, temos que x = .
p 2
Se x = anula a expressão E (fração irredutível, ou seja, p e q inteiros
q
 p é divisor de b0
11. Lema de Gauss (método dos
e primos entre si), então  .
q é divisor de bm coeficientes a determinar)
A demonstração deste teorema será vista em outro assunto mais à frente. Em geral, ao tentar fatorar uma expressão algébrica, tentamos usar
as ferramentas estudadas na seguinte ordem:
Esse teste é muito útil pois nos dá uma lista de frações que podem
anular a expressão em questão. Daí, usando o teorema 11, podemos
encontrar uma fatoração diretamente. – colocar em evidência/agrupamento;
– quebrar parcelas/completar quadrado;
– “chutar” uma raiz racional para obter um fator de grau 1;
Ex.: Fatorar K = 2x3 + 5x2 – x – 1 . – fazer alguma mudança de variáveis.

192 Vol. 1
Álgebra básica

Caso nenhuma dessas tentativas dê certo, podemos utilizar o método EXERCÍCIOS RESOLVIDOS
deste tópico.
Formalmente, o Lema de Gauss diz o seguinte: "se um polinômio de 01 Resolva a equação x4 + 3x3 – 2x2 + 3x + 1 = 0 nos reais.
coeficientes inteiros pode ser fatorado como produto de polinômios de
coeficientes racionais, então ele também pode ser fatorado como produto Solução:
de polinômios com coeficientes inteiros". Essa equação polinomial é chamada de recíproca (isso acontece
quando os coeficientes equidistantes do centro são iguais). Nesse
Na prática, isso significa que, no seu rascunho, você deve supor
caso, temos uma solução padrão. Dividindo tudo por x2 temos que
que os coeficientes dos fatores são inteiros, porque se não forem, serão
3 1
irracionais e será difícil encontrá-los. x2 + 3x − 2 + + 2 = 0 , q u e p o d e s e r e s c r i t a c o m o
x x
Ex.: Fatorar U = x4 + 3x3 + 3x2 – 2 .  2 1  1 1
. Agora, fazendo x + = t , temos
 x + 2  + 3 x + x  − 2 = 0 x
 x   
Solução  1
2
2 2 1 2
Primeiramente, veja que os candidatos a raízes racionais de U são  x + x  = t , o que nos dá x + x 2 = t − 2. Substituindo na
 
+1, –1, +2, –2. Testando, veja que nenhuma delas funciona. Com isso, equação, temos (t2 – 2) + 3t – 2 = 0, que dá t2 + 3t – 4 = 0. Essa
não temos raízes racionais e, por isso, não há fatores de grau 1 em U.
Então, devemos escrever U como um produto de dois fatores de grau 2: 1
equação tem raízes t = – 4 e t = 1. Colocando em x + = t , temos
x
U = (ax2 + bx + c)(dx2 + ex + f) as equações do 2o grau x2 + 4x + 1 = 0 e x2 – x + 1 = 0. A primeira

dá as raízes x = −2 ± 3 e a segunda não tem raízes reais.


A ideia é fazer a distributiva e igualar os coeficientes aos da expressão
original. No coeficiente de x4, temos ad = 1. Agora é que temos a
vantagem de considerar que os coeficientes são inteiros. Veja que temos
{
Então, S = −2 ± 3 . }
a = d = 1 ou a = d = –1. Podemos considerar que a = d = 1, pois no 02 Sejam x, y e z números reais tais que 1 ≠ x ≠ y ≠ 1 e
2o caso bastaria multiplicar os 2 fatores de U por –1. Então, ficamos com: yz − x 2 xz − y 2
= . Prove que essas duas frações são iguais a x +
1− x 1− y
U = (x2 + bx + c)(x2 + ex + f) y + z.

Agora, analisando o termo independente de x, temos que cf = –2. Para Solução:


c e f inteiros, temos 2 casos: c = 1, f = –2 ou c = –1, f = 2. Para facilitar, utilizaremos uma propriedade de razões e proporções,
que enunciaremos como ‘lema’:
1o caso: c = 1, f = –2 a c a−c
Lema: Se = = k e b ≠ d, então = k.
b d b−d
U = (x2 + bx + 1)(x2 + ex – 2) = x4 + (b + e)x3 + (–1 + be)x2 +
(e – 2b) x – 2 a c
Prova do lema: Como = = k, temos que a = bk e c = dk. Daí,
b d
b + e = 3 a − c bk − dk ( b − d ) k
 = = =k .
Igualando os coeficientes, temos o sistema e − 2 b = 0 . b−d b−d b−d
−1 + be = 3
 Agora, a partir do lema, as frações dadas no problema são iguais a:
As duas primeiras equações nos dão b = 1, e = 2 que não funcionam
( yz − x 2 ) − ( xz − y 2 ) = ( y 2 − x 2 ) + ( yz − xz ) = ( y − x ) ( y + x ) + z ( y − x ) = y + x + z o
(1 − x ) − (1 − y ) y−x y−x
na terceira equação.
2o caso: c = –1, f = 2 que encerra a demonstração.
U = (x2 + bx – 1)(x2 + ex + 2) = x4 + (b + e)x3 + (1 + be)x2 +
(2b – e) x – 2.
b + e = 3 EXERCÍCIOS NÍVEL 1

Igualando os coeficientes, temos o sistema 2 b − e = 0 x 3
1 + be = 3 01 Quantas soluções tem a equação = ?
 x − 3 x −3
As duas primeiras equações nos dão b = 1, e = 2, que funcionam na
terceira equação. 02 Resolva a equação (x + 1)(x – 4)(x – 2)(x2 + 3x + 2) = (x + 1)
(x – 4)(x – 2)(x2 + 8x + 3).
Portanto, temos que U = (x2 + x – 1)(x2 + 2x + 2).
3
03 Resolva = 2.
3
Veja que este processo pode ser muito trabalhoso caso o coeficiente 1+
independente de x possua muitos divisores. 3
1+
1− x

IME-ITA 193
Matemática II – Assunto 1

2 2 12 (OBM 1a fase 2008) Sendo x = 10–2008, assinale a alternativa que


04 Resolva = . apresenta o maior valor.
1 5
5+
1
4+ 1
1 (A) . (D) x.
5− x
x+2
1 x .
(B) . (E)
1 1 x( x + 1) 1
− x+
1− x 1+ x 1 x
05 Determine as soluções de = 1. (C) .
x 1 1
+ 1+
1− x 1+ x 1
1+
x
1
06 A expressão 1 − é igual a:
a 13 (OBM 1a fase 2011) Sendo a e b reais tais que 0 < a ≤ 1 e 0 < b ≤ 1,
1+
1− a o maior valor que a b pode assumir é:
(A) a, se a ≠ 0. a+ b
(B) 1, para todo a. 1
(A) 0. (D) .
(C) –a, se a ≠ 1. 2
1
(D) 1 – a, para todo a. (B) . (E) 1.
(E) a, se a ≠ 1. 4
1
(C) .
 a y   y a  3
07 ( EN) Seja  + ÷ −  = − 1, a ≠ 0.
a+ y a− y a+ y a− y 14 (OBM 1a fase 2010) Qual das seguintes frações é mais próxima de
A igualdade é válida: 7?

(A) para todos, exceto dois, valores de y. 3


(A) .
(B) só para dois valores de y. 1
(C) para todos os valores de y.
(D) só para um valor de y. 5
(B) .
(E) para nenhum valor de y. 2
8
(C) .
x2 + 1 + x2 − 1 x2 + 1 − x2 − 1 3
08 Calcule + . 13
x2 + 1 − x2 − 1 x2 + 1 + x2 − 1 (D) .
5
18
1010 + 1020 + 1030 (E) .
09 (UFF) Qual é o valor simplificado da fração ? 7
1020 + 1030 + 1040
15 Dados n números reais a1, a2, ..., an tais que a1 < a2 < ... <an, prove
10 (OCM) Qual dos números é maior: a1 + a2 + ... + an
que a1 < < an .
999 n
123456 + 10999 ou 123457 + 10 ?
16 Dados x, y ∈ , se x2 + y2 = 0, prove que x = y = 0.
123457 + 10999 123458 + 10999
17 Prove que:
11 (ITA-2002) Considere as seguintes afirmações sobre números reais
positivos: 1
A. se x é positivo, então x + ≥ 2;
x
I. Se x > 4 e y < 2, então x² – 2y > 12.
1
II. Se x > 4 ou y < 2, então x² – 2y > 12. B. se x é negativo, então x + ≤ −2 ;
x
III. Se x² < 1 e y² > 2, então x² – 2y < 0.
1
C. conclua que x + ≥ 2 para todo x real.
Então, destas é (são) verdadeira(s): x

(A) apenas I. 18 (OBM 1a fase 2011) Qual é o valor da expressão 201120112 +


(B) apenas I e II. 201120032 – 16 × 20112007?
(C) apenas II e III.
(D) apenas I e III. (A) 2 × 201120072.
(E) todas. (B) 2 × 201120032.
(C) 2 × 20112007.
(D) 2 × 20112003.
(E) 2 × 201120112.

194 Vol. 1
Álgebra básica

19 (SPIA) Fatore as expressões algébricas abaixo: x1 x 2 xn


29 Se = = = ... = k , prove que, dados a1, a2, ..., an tais que
y1 y 2 yn
A. a4 – 1; G. a4 + 2a3 – 2a – 1; a x + a2 x 2 + ... + an x n
a1y1 + a2y2 + ... + anyn ≠ 0, tem-se: 1 1 = k.
B. a6 – 1; H. 4b2c2 – (b2 + c2 – a2)2; a1y1 + a2 y 2 + ... + an y n
C. a6 + 1; I. c4 – (1 + ab)c2 + ab;
D. a4 – 18a2 + 81; J. 2a4 + a3 + 4a2 + a + 2;
30 (OBM 1a fase 2009) Se x2 = x + 3, então x3 é igual a:
E. a12 – 2a6 + 1; K. a4 + 3a3 + 4a2 – 6a – 12;
F. a5 + a3 – a2 – 1;
(A) x2 + 3.
(B) x + 4.
20 Prove que se (a + b)2 + (b + c)2 + (c + d)2 = 4(ab + bc + cd),
(C) 2x + 2.
então, a = b = c = d
(D) 4x + 3.
(E) x2 – 2.
21 (OBM 3a fase 2003 nível 2) Mostre que x2 + 4y2 – 4xy + 2x – 4y +
2 > 0 quaisquer que sejam os reais x e y.
31 (CN) Sabe-se que a3 – 3a + 1 = 93 e K = a4 – 6a + 1. Logo, K
2
também pode ser expresso por:
22 Se  x + 1  = 3, então, x 3 + 1 é igual a:
 x x3 (A) 3a2 + 86a + 1. (D) 6a2 + 84a + 1.
(A) 0. (D) 3. (B) 3a2 + 84a + 1. (E) 9a2 + 86a + 1.
(B) 1. (E) 4. (C) 6a2 + 86a + 1.
(C) 2.
2 3
32 (EFOMM – 03) Que termo se deve acrescentar ao binômio x + b x
1 1 4 3
23 Se x e y satisfazem x + = y + , mostre que, necessariamente, x
1 x y de modo que se obtenha um trinômio que seja quadrado perfeito?
= y ou x = .
y
6 b3
24 Determine x real tal que: (A) b . (D) .
3 3
A. x3 + 3x2 + 3x – 1 = 0; b4 b6 .
(B) . (E)
9 9
B. x3 – 3x2 + 3x + 1 = 0. 6
(C) b .
x+y 2
25 Dado que x2 + y2 = 6xy (x > y > 0), determine o valor de .
x−y 33 (OBM 1a fase 2006) Os dois números reais a e b são não nulos e
satisfazem ab = a – b. Assinale a alternativa que exibe um dos possíveis
a b
a3 + b3 a+b valores de + − ab.
26 Qual é a restrição em a e b reais para que = ? b a
a + ( a − b)3 a + ( a − b)
3
(A) – 2.
a+b a−b
27 Dados a e b de módulos diferentes tais que + = 6, calcule 1
3 3 3 3 a−b a+b (B) − .
a +b a −b 2
+ .
a3 − b3 a3 + b3 1
(C) .
28 (SPIA) Simplifique as expressões abaixo: 3
(D) 1 .
a a2 + a − 1 a2 − a − 1 2 a3 2
(A) + + − ; (E) 2.
a2 − 1 a3 − a2 + a − 1 a3 + a2 + a + 1 a4 − 1
1 1 34 (SPIA) Fatore as expressões algébricas a seguir:
+
a b+ c b 2 + c 2 − a2 
(B)  1 +  ; A. a4 + a2b2 + b4;
1 1  2 bc 
− B. a4 + 4a2 – 5;
a b+c
C. 4a4 + 5a2 + 1;
1 1 1 D. a5 + a4 + a3 + a2 + a +1;
(C) + + ;
( a − b)( a − c) ( b − c)( b − a) ( c − a)( c − b) E. a4 + 324;
F. a4 + a2 + 1;
G. a4 + 9;
a+b b+c c+a H. a4 + 4b4;
(D) + + ;
( b − c)( c − a) ( c − a)( a − b) ( a − b)( b − c) I. (a + b + c)3 – (a3 + b3 + c3);
J. (a – b)3 + (b – c)3 – (a – c)3;
K. (a2 + b2)3 – (b2 + c2)3 – (a2 – c2)3;
a−c a3 − c3  c 1 + c  c(1 + c) − a
(E) 1 + − ÷ ; L. (a + b)5 – (a5 + b5).
a + ac + c a b − bc2
2 2 2
 a−c c  bc

IME-ITA 195
Matemática II – Assunto 1

( a2 − b2 )3 + ( b2 − c2 )3 + ( c2 − a2 )3 45 (OBM 1a fase 2006) Quantos ternos de números reais x, y, z satisfazem


35 Simplifique . o sistema abaixo?
( a − b)3 + ( b − c)3 + ( c − a)3
x( x + y + z ) = 2005
y( x + y + z ) = 2006
36 Calcule a2 .
z( x + y + z ) = 2007
37 Calcule a4 . (A) Nenhum. (D) 3.
(B) 1. (E) 2006.
38 Calcule 2 2 − 3 (1 + 3 ). (C) 2.

46 (OBM 1a fase 2013) Determine x + y, onde x e y são reais, sabendo


39 Calcule 2 − 3 ( 6 − 2 )( 2 + 3 ). que x3 + y3 = 9 e xy2 + x2y = 6.

40 Racionalize as frações: (A) 1.


(B) 2.
1 5+ 3 (C) 3.
(A) . (C) .
2+ 3− 5 (D) 4.
5− 3
(E) 5.
1 1
(B) 4 . (D) . 47 (OBM 2a fase 2012) Sendo a, b, c reais tais que ab(a + b + c) =
5− 3 4 14 + 21 + 15 + 10
1001, bc (a + b + c) = 2002 e ca (a + b + c) = 3003, encontre abc.

41 Sendo a, b e c positivos, simplifique a2 + 4 ab + 6 ac + 4 b2 + 12 bc + 9c2 . 48 (SPIA) Simplifique as expressões a seguir:

5 a2 − a − 4 2 a4 + 7 a2 + 6
99 (A) . (E) .

1 a3 − 1 3 a4 + 3 a2 − 6
42 Calcule .
k =1
k +1+ k
a6 + a4 + a2 + 1 5 a4 + 5 a2 − 3 a2 b − 3 b .
(B) . (F)
3 2
43 (OBM 1 fase 2006) Sejam x e y números racionais. Sabendo que
a a + a + a+1 a4 + 3 a2 + 2
x − 5 2006
também é um número racional, quanto vale o produto xy? 4 2 a4 + a2 b 2 + b 4
4 − y 2006 (C) a + a − 2 . (G)
a6 + 8 a6 − b 6
(A) 20. a4 − a2 − 12
(B) Pode ser igual a 20, mas também pode assumir outros valores. (D) .
a4 + 8 a2 + 15
(C) 1.
(D) 6.
(E) Não se pode determinar. 1 1 1 1
49 Prove que se + + = , então, x2 (y + z) + y2(x + z)
x y z x+y+z
44 (SPIA) Simplifique as expressões abaixo: + z2(x + y) = – 2xyz.

−1 EXERCÍCIOS NÍVEL 2
 1  4
 1 1 2 a 3 − 2  1 01 (OBM 1 fase 2010) Os números x e y são distintos e satisfazem
a
(A)  + −  − a3 ;
1 1 1 1 2 1
 a3 − a6 + 1 a3 + a6 + 1 a3 − a 3 + 1 4 1 1
  x − = y − . Então xy é igual a:
x y
1/ 2
4 4 4 4  
−1  (A) 4
(B)  b ( a − b ) + 2 ab −  4 b + 1 + 1 ⋅ 8 ab ; (B) 1
 ( 4 b + 4 a )2  a  
    (C) –1
−1 (D) –4
  2 2
−1
 1 1 
−2
(E) São necessários mais dados.
 ( a + b)  a 3 − b 3  − ( 3 a2 b − 3 ab2 )  b 3 − a 3  
     
(C)       + 26 a ; 02 Sendo a, b e c números distintos, simplifique a expressão:


(
( 6 a + 6 b ) 3 b + 6 ab − 2 3 a ) 

  2 b−c 2 c−a 2 a−b
  + + + + +
b − c ( c − a) ( a − b) c − a ( a − b) ( b − c) a − b ( b − c) ( c − a)
−1
566 2 3
(D) b − a b +
6
a3 b2 − 6 a5  6 ab9 + 6 a10  (A) a + b + c
  + 1.
6
b+ 6
a  a − ab + b  (B) a – b
  (C) (a – b)(b – c)(c – a)
(D) 2abc.
(E) 0.

196 Vol. 1
Álgebra básica

03 Prove que 2 2 3 3 3
16 Para quais valores reais de a e b vale a + b ≥ a + b ?
x y z x ( y − z ) y( z − x ) z( x − y )
= = ⇒ + + =0
p( y + z ) q( x + z ) r( x + y ) p q r 17 ( A FA – 1 9 9 7 ) O p r o d u t o d a s r a í z e s d a e q u a ç ã o
.
( ) +( ) = 4 pertence ao conjunto dos números:
x x
2+ 3 2− 3

04 (OBM 1a fase 2012) Se x2 = 2x + 4, então (x + 1)–1 é igual a: (A) naturais e é primo.


(B) inteiros e é múltiplo de 4.
(A) x + 2. (C) complexos e é imaginário puro.
(B) x – 3. (D) racionais positivos e é uma fração imprópria.
(C) x – 1.
(D) 2x + 5.
( )
2011
18 (OBM – 1a fase) Sendo a e b inteiros tais que 1 + 2 = a + b 2,
(E) 3x + 5.
( )
2010
1− 2 é igual a:
1 1
05 Definindo x + = u, calcule u n = x n + n em função de u para (A) a + 2 b + ( a − b)
x x 2.
n = 2,3,4,5.
(B) a − 2 b + ( a − b) 2.
06 (OBM 2a fase 2005) Determine todos os pares de inteiros (x; y) tais (C) a + 2 b + ( b − a) 2.
que 9xy – x2 – 8y2 = 2005. (D) 2 b − a + ( b − a) 2.
(E) a + 2 b − ( a + b) 2.
07 (CN) Sejam A = 72011, 112011  e B = {x ∈  / x = (1 – t) · 72011 + t
19 Racionalize:
· 112011, com t ∈ [0,1]}, o conjunto A – B é igual a:
1
(A)
(A) A ∩ B. 4
2+ 4 +48+2
4

(B) B – {112011}.
1+ 3 2
(C) A – {72011}. (B)
(D) A. 1+ 3 2 + 3 4
(E) ∅. 1
(C) 3
2 −1
08 Os números reais a, b, c e d são tais que a < b < c < d e a + d =
b + c. Qual é maior: ad ou bc? 1
(D) 3
4 +36+39
09 (OBM 1a fase 2010) Os números a e b são reais não negativos tais 1
que a3 + a < b – b3. Então: (E)
2+33
(A) b < a < 1.
(B) a = b = 1.
20 (CN) O número real 3 26 − 15 3 é igual a:
(C) a < 1 < b.
(D) a < b < 1.
(A) 5 − 3 . (D)
13 − 3 3 .
(E) 1 < a < b.

a1 a2 a (B) 7 − 4 3 . (E) 2.
10 Se , ,..., n pertencem ao intervalo (α, β) e b1, b2, ..., bn são
b1 b2 bn
(C) 3 − 2 .
a + a + ... + an
positivos, prove que 1 2 também pertence a (α, β). Nas
b1 + b2 + ... + bn
(3 + 2 2 )
2008
t1a1 + t2 a2 + ... + t n an
mesmas condições, se t1, t2,...,tn ≥ 0, prove que 21 (CN) O valor de + 3 − 2 2 é um número
t1b1 + t2 b2 + ... + t n bn
(5 2 + 7)
1338

também pertence ao intervalo (α, β).


(A) múltiplo de 11. (D) múltiplo de 3.
11 Prove que, para todos x e y reais, tem-se que x2 – xy + y2 ≥ 0. (B) múltiplo de 7. (E) primo.
(C) múltiplo de 5.
x2 + y2
12 Para todos x, y e z reais, prove que ( x + y ) z ≤ + z2.
2 22 (OBM 1a fase 2005) O número ( 2 + 2 )3 (3 − 2 )4 + ( 2 − 2 )3
13 Para a, b e c reais, prove que a + b + c ≥ ab + ac + bc.
2 2 2
(3 + 2 )4 é:

 1 1 1
14 Prove que se a, b e c são positivos, então ( a + b + c )  + +  ≥ 9. (A) inteiro ímpar.
 a b c (B) inteiro par.
a+ b+c+d 4 (C) racional não inteiro.
15 Prove que ≥ abcd para a, b, c, d não negativos. (D) irracional positivo.
4
(E) irracional negativo.

IME-ITA 197
Matemática II – Assunto 1

23 (OBM 1a fase 2005) Os inteiros positivos x e y satisfazem a equação 30 (Barbeau) Prove que se (x + a + b)(x–1 + a–1 + b–1) = 1, então (xn
+ an + bn)(x–n + a–n + b–n) para todo n inteiro ímpar.
1 1
x+ y − x− y = 1. 31 (SPIA) Simplifique:
2 2
Qual das alternativas apresenta um possível valor de y? a − b b − c c − a ( a − b)( b − c)( c − a)
a. + + + ;
a + b b + c c + a ( a + b)( b + c)( c + a)
(A) 5.
(B) 6. a3 b − ab3 + b3 c − bc3 + c3 a − ca3
(C) 7. b. ;
(D) 8. a2 b − ab2 + b2c − bc2 + c2 a − ca2
(E) 9.
c.  b + a   a − b  −  a + b   b − a  .
24 (OBM 2a fase 2004) Cada um dos números x1, x2,..., x2004 pode ser  a+ b  a − b   a+ b  a − b 
igual a 2 − 1 ou a 2 + 1. Quantos valores inteiros distintos a soma
2004
32 (CN) Se a, b, c e d são números reais não nulos tais que ad2 + bc2
∑ x2 k −1x2 k = x1x2 + x3 x4 + x5 x6 + ... + x2003 x2004 pode assumir? = 0, pode-se afirmar que:
k =1

25 (SPIA) Fatore as expressões a seguir: a c a+c c b b+c


(A) + = ; b + d ≠ 0. (D) + = ; a + d ≠ 0.
b d b+d a d a+d
a. a8 + a4 + 1;
b. a5 + a4 + a3 + a2 + a + 1; a b a+ b c d c+d
(B) + = ; c + d ≠ 0. (E) + = ; a + b ≠ 0.
c. (ab + ac + bc)(a + b + c) – abc; c d c+d b a a+ b
d. 2a2b + 4ab2 – a2c + ac2 – 4b2c + 2bc2 – 4abc; a b a+ b
e. a(b – 2c)2 + b(a – 2c)2 – 2c(a + b)2 + 8abc; (C) + = ; c + d ≠ 0.
d c c+d
f. a3(a2 – 7)2 – 36a;
g. a2b2(b – a) + b2c2 (c – b) + a2c2 (a – c); EXERCÍCIOS NÍVEL 3
h. 8a3(b + c) – b3 (2a + c) – c3 (2a – b);
i. a3 + 5a2 + 3a – 9; 1 1 1
j. a(a + 1)(a + 2)(a + 3) + 1; 01 (CN) Sejam a, b e c números reais não nulos tais que + + =p
a b c a b c ab bc ac
k. (a +1)(a + 3)(a + 5)(a + 7) + 15; , + + + + + = q e ab + ac + bc = r. O valor de q + 6q é 2

l. 2(a2 + 2a – 1)2 + 5(a2 + 2a – 1)(a2 + 1) + 2(a2 + 1)2; b a a c c b


m. (a – b)c3 – (a – c)b3 + (b – c)a3; sempre igual a:
n. a2b + ab2 + a2c + ac2 + b2c + bc2 + 3abc;
o. a4 + 2a3 + 3a2 + 2a + 1; p2 r 2 + 9
p. a4 + b4 + c4 – 2a2b2 – 2a2c2 – 2b2c2; (A) .
4
q. a4 + 2a3b – 3a2b2 – 4ab3 – b4.
p2 r 2 − 9 p
(B) .
12
 x + y = 1
3 3

26 Determine as soluções reais de  2 2 3


. (C) p2r2 – 9.
 x y + 2 xy + y = 2
 3 xy p2 r 2 − 10
(D) .
x + y =2 4r

 4 xz (E) p2r2 – 12 p.
27 Resolva o sistema  = 3.
x + z 02 (OBM 2a fase 2010) Calcule:
 5 yz
y + z =6
 ( 2 + 2 + 1) ( 4
4 2 4
)( ) (
+ 4 2 + 1 64 + 62 + 1  324 + 322 + 1 )
 x − 3 xy + 2 y + x − y = 0
2 2 (1 + 1 + 1) (3
4 2 4
+3 2
+ 1) ( 5 4
+5 2
+ 1)( 31 + 31 + 1)
4 2

28 (Hungria) Prove que  2 2


⇒ xy − 12 x + 15 y = 0.
 x − 2 xy + y − 5 x + 7 y = 0
03 Sejam x, y e z reais do intervalo [0,1]. Prove que xyz + (1 – x)(1 – y)
29 (SPIA) Calcule as somas a seguir: (1 – z) ≤ 1.

1 1 2a 4 a3 8 a7 04 Este exercício tem como objetivo demonstrar a ‘desigualdade das


a. − − − − médias’ para 3 termos.
1 − a 1 + a 1 + a2 1 + a4 1 + a8
1 1 2 4 8 16 a. Fatore a3 + b3 + c3 – 3abc.
b. + + + + + x+y+z 3
1 − a 1 + a 1 + a2 1 + a4 1 + a8 1 + a16 b. (Desigualdade das médias para 3 termos) Prove que ≥ xyz .
3
1 1 1 1 1
c. + + + +
a( a + 1) ( a + 1)( a + 2) ( a + 2)( a + 3) ( a + 3)( a + 4) ( a + 4)( a + 5)

198 Vol. 1
Álgebra básica

05 ( CN) Sejam y e z números reais distintos não nulos tais que 16 (OBM 3a fase 2010 nível 2) Sejam a, b e c reais tais que a ≠ b e
4 y2 z2 a2(b + c) = b2(c + a) = 2010. Calcule c2(a + b).
+ + = 3. Qual é o valor de y + z?
yz 2 z 2 y
17 (OBM 1a fase 2007) Sejam a, b e c números tais que:
(A) –2. (D) 2. a2 – ab = 1
(B) –1. (E) 3. b2 – bc = 1
(C) 0. c2 – ac = 1
06 (OBM 1a fase 2010) Qual é o maior valor de xy2 se x e y são reais O valor de abc · (a + b + c) é igual a:
positivos cuja soma é 3?
(A) 0.
(A) 3. (D) 6. (B) 1.
(B) 4. (E) 7. (C) 2.
(C) 5. (D) –1.
(E) –3.
07 (OBM 3a fase 2012 nível 2) Considere os números reais a e b tais
que (a + b)(a + 1)(b + 1) = 2 e a3 + b3 = 1. Encontre o valor de a + 18 (OBM 2a fase 2011) Encontre todas as soluções reais (x, y, z) do
b. sistema.
1 1
08 Prove que se a + b + c = 1, então a2 + b2 + c2 ≥ . 2y = x +
3 x
1
09 Racionalize: 2z = y +
y
1
4
8− 2 +1 2x = z +
a. ; z
4 4
8+ 2 −1 − 8− 2 −1
1 19 (OBM 3a fase 2009 nível 2) Resolva, nos números reais, o sistema:
b. .
1+ 3 2 + 23 4  1 1 1
x + = y + = z +
10 (Yaglom)  y z x
a = c  xyz = 1
a. Prove que se a, b, c e d são racionais, então a + b 2 = c + d 2 ⇒  . 
b = d
b. Conclua que sob as mesmas condições, tem-se  x + y + z = 77
20 (OBM 2a fase 2010) Resolva o sistema 
a + b 2 = c + d 2 ⇒ a − b 2 = c − d 2. sendo x ≤ y ≤ z inteiros não negativos.  xy + yz + zx + xyz = 946

c. Existem x, y, z e w racionais tais que 21 (OBM 1a fase 2013) Para quantos inteiros positivos k menores que
2013, existem inteiros a, b e c, não necessariamente distintos, satisfazendo
( x + y 2) + ( z + w 2)
4 4
= 2 + 2 2? a2 + b + c = b2 + c + a = c2 + a + b = k?
1 1 1
11 Se a, b e c são racionais, prove que + + é (A) 43.
( a − b) ( b − c) ( c − a)
2 2 2
o quadrado de um racional. (B) 44.
(C) 87.
12 Se x + y + z = 0, prove que (D) 88.
(E) 89.
 y − z z − x x − y  x y z 
 + +  + +  = 9.
 x y z  y − z z − x x − y 22 Fatore a expressão a4 – 2a3b – 8a2b2 – 6ab3 – b4.

23 Fatore:
13 Dado que xyz = 1 e x, y, z > 0, determine a diferença entre o máximo
x +1 y +1 z +1 a. x4 + x2 + 4x – 3
e o mínimo da expressão + + .
xy + x + 1 yz + y + 1 zx + z + 1 b. x4 + 4x3 + 8x2 + 9x + 6
14 (Yaglom) O produto de três números positivos é igual a 1 e a soma c. x 4 + x 2 + 2 x + 2
desses números é maior que a soma dos seus inversos. Prove que, dentre
esses três números, um deles é maior do que 1 e dois deles são menores 24 Fatore:
do que 1.
a. x5 + x + 1
15 Sejam a, b, c, x, y, z reais distintos tais que ax + by + cz = 0. Prove b. x10 + x5 + 1
ax 2 + by 2 + cz 2 c. x11 + x6 + x + 1
que a expressão não depende nem
bc( y − z )2 + ca( z − x )2 + ab( x − y )2
de x, nem de y, nem de z.

IME-ITA 199
Matemática II – Assunto 1

25 (OBM 1a fase 2010) Para cada subconjunto A de {1;2;3;4;5;6;7;8;9;10}, 27 ( OBM 1 a fase 2006) Qual é o menor valor que a expressão
seja p(A) o produto de seus elementos. Por exemplo, p({1;2;4;5}) = 40 e x 2 + 1 + ( y − x )2 + 4 + ( z − y )2 + 1 + (10 − z )2 + 9 pode assumir,
p(A) = 10! = 1 · 2 · 3 · ... · 10. Por convenção, adote p(∅) = 1. A soma sendo x, y e z reais?
de todos os 210 produtos p(A) é igual a:
(A) 7.
(A) 211. (D) 211!.
(B) 11!. (E) 112!. (B) 13.
(C) 1111.
(C) 4 + 109 .
( a − b)( b − c)( c − a) 1
26 (OBM 3a fase 2005 nível 2) Dado que = , qual
( a + b)( b + c)( c + a) 11 (D) 3 + 2 + 90 .
a b c
é o valor de + + ?
a+ b b+c c+ a (E) 149 .

RASCUNHO

200 Vol. 1
Sequências A ssunto
1
Matemática III

1. Introdução 3.2 Propriedades da P.A.


O presente assunto tem por objetivo definir o que é uma sequência, Uma das principais propriedades da P.A. é a simetria em relação ao
estudar os principais tipos (progressões aritméticas e geométricas), e centro. Assim, quando temos uma P.A. com um número pequeno de termos,
determinar os seus termos, conhecendo-se os termos iniciais. podemos escrevê-la a partir do termo central para facilitar algumas contas.
– P.A.’s com um número ímpar de termos:
Além disso, estudaremos a soma de seus elementos, as propriedades
da P.A. e da P.G. e aplicações em matemática financeira, como juros
3 termos: (x – r, x, x + r)
simples e compostos.
5 termos: (x – 2r, x – r, x, x + r, x + 2r)
Finalmente veremos outros tipos de sequências, como defini-las de
forma recursiva (em função de termos anteriores), e alguns métodos para E assim sucessivamente.
obter um termo geral, como a soma telescópica e outros truques algébricos. – P.A.’s com um número par de termos:

2. Sequências 4 termos: (x – 3r, x – r, x + r, x + 3r)


De forma intuitiva, uma sequência é uma ordenação dos elementos de 6 termos: (x – 5r, x – 3r, x – r, x + r, x + 3r, x + 5r)
um conjunto, ou seja, devemos associar cada elemento a uma posição, de
modo que exista um primeiro elemento (a1), um segundo elemento (a2), Atenção: Repare que, ao escrever uma P.A. com um número par de
um terceiro (a3) e assim por diante. termos nessa forma, a razão da P.A. é 2r.
Chamaremos de ai o termo na posição i. Veja que associamos cada
elemento a um número natural i (sua posição). Escrevendo a P.A. com esse formato, conseguimos visualizar outra
propriedade importante da P.A.: a soma de termos equidistantes do centro
Ex.: (ou das pontas) é constante, ou seja:
• (1, 2, 3, 4, 5, ..., n) (sequência dos n primeiros números naturais);
• (1, 1, 2, 3, 5, 8, ...) (sequência de Fibonacci: cada termo é a soma a1 + an = a2 + an – 1 = a3 + an – 2 = … = ap + an – p + 1
de dois anteriores);
• (–3, 2, 7, 12, 17, 22) (diferença entre termos consecutivos constante);
• (5, 10, 20, 40, 80) (razão entre termos consecutivos constante). Além disso, se a P.A. possuir um termo central então esse termo é a
média aritmética das extremidades.

3. Progressão aritmética (P.A.) 3.3 Soma da P.A.


Chamamos de progressão aritmética toda sequência (a1, a2, a3, ..., an) Dada uma P.A. (a1, a2, a3, ..., an), definimos Sn como a soma dos n
cuja diferença entre termos consecutivos é constante: primeiros termos da P.A., ou seja,

ak – ak – 1 = r = cte, k ∈ {2, 3, ..., n} Sn = a1 + a2 + …+ an

Neste caso, dizemos que r é a razão da P.A. Como podemos calcular essa soma?

Ex.: (3, 7, 11, 14, ...) P.A. de razão 4. Ex.:


S = 1 + 2 + 3 + ... + 100 →
Veja que a P.A. fica bem definida se dermos um termo e sua razão, uma
S = 100 + 99 + 98 + ... + 1
vez que qualquer termo pode ser obtido através desses dois parâmetros.
Dizemos que a P.A. é crescente se r > 0 e decrescente se r < 0. No Somando ambas as equações: 2S = 101 + 101 + ... + 101 =
caso em que r = 0 dizemos que a P.A. é estacionária. 101 ∙ 100 → S = 101 ∙ 50 = 5050 100 vezes
3.1. Termo geral Repare que escrevendo a P.A. ao contrário juntamos os termos
Como dito anteriormente, todo termo de uma P.A. pode ser obtido equidistantes das pontas, e como vimos anteriormente, a soma desses
através de outro termo e da razão. Por exemplo, se tivermos o termo termos sempre é igual à soma das extremidades, assim:
inicial a1 e a razão r podemos determinar an através da seguinte relação: Sn = a1 + a2+ … + an
an = a1 + (n – 1)r Sn = an + an–1+ …+ a1
Para ver que essa relação é verdadeira, basta pensar que, cada vez Somando e utilizando a propriedade anterior: 2Sn = (a1 + an) · n →
que andamos para frente na sequência, somamos a razão uma vez. Como ( a + an ) ⋅ n
Sn = 1
queremos chegar ao termo na posição n, partindo do primeiro termo, 2
devemos “dar n – 1 passos” na sequência, somando então n – 1 vezes r. Ex.:
De forma geral, vale a seguinte relação: I. Sn = 1 + 2 + 3 + ... + n = (n(n + 1))/2
an = ap + (n – p)r II. Sn = 1 + 3 + 5 + ... + (2n – 1) = n2

IME-ITA 201
Matemática III – Assunto 1

4. Progressão geométrica 4.3 Produto dos termos da P.G.


Chamamos de progressão geométrica toda sequência (a1, a2, a3, ..., an) Sendo Pn o produto dos n primeiros termos de uma P.G., tem-se:
cuja razão entre termos consecutivos é constante:
Pn = a1a2 ... an = a1(a1q)(a1q2) … (a1qn–1) → Pn = a1n q1 + 2 +...+ (n – 1) →
ak / a k −1 = q = cte, k ∈{2, 3, ..., n}
n ( n − 1)
Neste caso, dizemos que q é a razão da P.G. Pn = a1n q 2

Ex.: (3, 6, 12, 24, ...) P.G. de razão 2


Outra maneira de calcular o produto é usando uma propriedade antes
Veja que, como na P.A. a P.G. fica bem definida se tivermos um termo e sua citada em P.A., que continua valendo para P.G: o produto dos termos
razão; assim podemos achar o termo geral, utilizando a mesma ideia da P.A.. equidistantes das pontas é igual ao produto dos extremos.
Dizemos que a P.G. é crescente se ak – ak–1 > 0.
Para que isso ocorra devemos ter: a1 > 0 e q > 1 ou a1 < 0 e 0 < q < 1.  a1an = a1an
 a a = aa
Dizemos que a P.G. é decrescente se ak – ak–1 < 0.  2 n −1 1 n
Para que isso ocorra devemos ter: a1 > 0 e 0 < q < 1 ou a1 < 0 e q > 1.  a3 an − 2 = a1an Multiplicando as equações: Pn2 = ( a1an )n
……………….
Caso q < 0, dizemos que a P.G. é alternante e, se q = 1, dizemos 
que esta é estacionária.  an a1 = a1an

4.1 Termo geral


Obs.: Deve-se tomar cuidado ao extrair a raiz quadrada na relação, pois
Em função do primeiro termo e da razão: an = a1qn – 1 o produto dos termos pode ser negativo (dependendo da quantidade de
termos negativos na sequência).
Em função de um termo qualquer e da razão: a = a qn – p
n p 4.4 Progressão geométrica infinita
4.2 Soma dos n primeiros termos da P.G. Chamamos de P.G. infinita toda P.G. com um número infinito de termos.
Considere uma P.G. (a1, a2, a3, ..., an) de razão q ≠ 1, seja: Dependendo da razão desta P.G., podemos calcular a soma de seus
elementos, ou seja, existem alguns casos em que a soma infinita converge
Sn = a1 + a2 + …+ an = a1 + a1 q + a1q2+ …+ a1qn–1 (1) (resultando em um número finito).
Multiplicando por q: qSn = a1q + a1q2 + a1q3+ … + a1qn (2) Sabemos que numa P.G. finita vale a seguinte fórmula:

Subtraindo (2) – (1): Sn(q – 1) = a1(qn – 1) →  q n − 1


Sn = a1   , q ≠ 1;
 q −1
 q n − 1
Sn = a1  , q ≠ 1
 q −1 em que n representa o número de termos a serem somados. Queremos
Mais importante que entender a demonstração da fórmula é lembrar saber o que ocorre quando esse n tende a infinito.
da ideia por trás dela. Quando queremos calcular uma soma, podemos Vamos analisar, por exemplo, o caso q = 1/2:
“perturbá-la”, ou seja, podemos pensar em algum meio de achar uma
expressão muito parecida com ela, para subtrair (ou fazer outra operação) 2 3 10
de modo que a maioria dos termos cancele.  1 1  1 1  1 1
 2  = 4 ;  2  = 8 ;  2  = 1024
Veja que essa foi a motivação principal para multiplicar pela razão, uma      
vez que ao multiplicar os termos de uma P.G. por q, apenas “andamos”
com a P.G. para frente. Veja que quanto maior o expoente menor o valor de qn; assim, se n
Outro modo de enxergar a fórmula da soma da P.G. é usar a própria tende a infinito, podemos ver que qn tende a zero.
definição de P.G.: É fácil ver que isso ocorre para todo q, com |q| < 1. Nesse caso,
iremos trocar qn na P.G. finita por zero, assim:
a2 = a1 q
a3 = a2 q
a4 = a3 q  0 − 1 a1
S∞ = a1   → S∞ = , q <1
..............  q − 1 1− q
an = an-1 q

Somando tudo: Sn – a1 = q(Sn – an ) → qSn – Sn = qan – a1 = a1 +∞;a1 > 0



 q n − 1 Obs.: Caso q ≥ 1, temos: S∞ =  −∞;a1 < 0 ,
q – a1 → Sn = a1 
n
, q ≠ 1  0;a = 0
 q −1  1

Obs.: Se a razão for igual a 1, a P.G. é constante e sua soma é igual a Sn basta pensar no que ocorre com qn na fórmula da P.G. finita. Se q ≤
= a1 + a1 + …+ a1 = na1 –1, a soma infinita não existe.

202 Vol. 1
Sequências

EXERCÍCIOS RESOLVIDOS

01 Calcule o valor da soma 2 + 5 + 8 + 11 + ... + 92. temos: a1 = 1 e a11 = 31. Usando que a11 = a1+10r, têm-se: r = 3,
logo: an = a1 + (n – 1)r = 1 + (n – 1) · 3 = 3n – 2.
Solução:
Esta é a soma de uma P.A. com a1 = 2, an = 92 e r = 3. Antes de 06 A soma de três números em P.G. é 26 e o produto é 216. Então, os
calcularmos a soma, precisamos saber quantos termos há. Usando que termos da P.G. valem:
an = a1 + (n – 1)r, temos 92 = 2 + (n – 1) · 3, que nos dá n = 31.
( a + a ) n ( 2 + 92).31 = 1457.
Portanto, segue que a soma é igual a: 1 n = Solução:
2 2 Três números em P.G. (x/q, x, xq), multiplicando: x3 = 216 ⇒ x = 6.
02 Prove que se (a, b, c) é simultaneamente uma P.A. e uma P.G., então, 1 1 10
Somando: x(q + 1 + ) = 26 ⇒ q + = ⇒ 3q2 – 10q + 3 =
a = b = c. q q 3
1
0 ⇒ q = 3 ou q = .
Solução: 3
a+c Logo: (2, 6, 18) ou (18, 6, 2).
Como é P.A., temos b = . Como é P.G., temos b2 = ac.
2
Substituindo a 1a na 2a, temos que: 07 Em um certo jogo de azar, apostando-se uma quantia X, tem-se
2
 a+c 2 2
uma das duas possibilidades seguintes:
 2  = ac ⇒ a + 2 ac + c = 4 ac ⇒
 
I. Perde-se a quantia X apostada;
⇒ a2 − 2 ac + c2 = 0 ⇒ ( a − c ) = c ⇒ a = c
2
II. Recebe-se a quantia 2X.

a+c Uma pessoa jogou 21 vezes da seguinte maneira: na primeira vez,


Como b = , segue que b = a = c.
2 apostou 1 centavo; na segunda vez, apostou 2 centavos, na terceira
vez, apostou 4 centavos e assim por diante, apostando em cada vez o
03 O produto dos 15 primeiros termos da progressão geométrica, de
dobro do que havia apostado na vez anterior.
primeiro termo 1 e razão 10, vale:
Nas 20 primeiras vezes, ela perdeu. Na 21a vez, ela ganhou. Sendo T
a quantidade total por ela desembolsada e Q a quantidade recebida na
Solução:
n ( n − 1) 15 ⋅ 14
21a jogada, determine uma relação entre T e Q:
Usando que Pn = a1n q 2
→ P15 = 10 2
= 10105
Solução:
04 Um atleta corre sempre 500 metros a mais do que no dia anterior. Veja que as apostas dele crescem como P.G. de razão 2, assim, o total
Sabendo-se que ao final de 15 dias ele correu um total de 67.500 metros, desembolsado é:
qual o número de metros percorridos no terceiro dia? T = 1 + 2 + 4 + 8 + ... + 220. Como T é uma soma de P.G.:
 q n − 1  221 − 1 21
T = a1  = = 2 − 1. Na 21a jogada ele recebe o dobro do
Solução:  q −1  2−1
Veja que, como o atleta sempre corre 500 m a mais que no dia anterior,
que investiu: Q = 2.220 = 221, logo: Q = T + 1.
a sequência formada pelas distâncias percorridas diariamente por ele
é uma P.A. de razão 500.
08 Em um paralelepípedo retângulo a soma das medidas de todas as
A distância total percorrida por ele ao término de 15 dias representa
arestas é 52 e a diagonal mede 91. Se as medidas das arestas estão
uma soma de P.A., logo:
em progressão geométrica, então o seu volume é:
( a1 + a15 ) . 15 = 67500 → a + a + 14 ⋅ 500 =
1 (1 )
2 Solução:
2 ⋅ 67500 a 
= = 9000 → a1 = 2000 Sejam os lados  1 ,a1,a1q  temos:
15 q 
Desse modo: a3 = a1 + 2r = 3000 m 1  52  1 
05 A soma dos 11 primeiros termos da progressão aritmética (a1, a2, a1  + 1 + q  = = 13 e a12  2 + 1 + q 2  = 91 .
 q  4  q 
..., an, ...) é 176. Se a11 = a1 + 30, então, para qualquer n ∈ * temos:
1 1 1
(A) an = 3n – 2. (D) an = 2n + 3. Fazendo t = q + , têm-se: t 2 = q 2 + 2 + 2 → q 2 + 2 = t 2 − 2 .
substituindo: q q q
(B) an = 2n – 3. (E) an = 3n + 2.
(C) an = n + 3. ( )
 a12 t 2 − 1 = 91
. Dividindo a1(t – 1) = 7 → a1t = a1 + 7, a1t = 13 – a1

 a1 ( t + 1) = 13 
Solução: Letra A.

S11 =
( a1 + a11 ).11 = 176 ⇒ a + a11 = 32 , porém, como a11 = a1 + 30,
= = a1 + 7 → a1 = 3, logo: V 3
= a=
1 27 u.v.
1
2

IME-ITA 203
Matemática III – Assunto 1

Solução: S S
n

= a1an , então,   = ( a1an ) .


n
⇒ Portanto, temos que
É fácil ver que a relação é válida caso a razão seja igual a 1. Supondo I I
 q n − 1
que a razão não seja igual a 1, temos que S = a1  . Agora, veja n
S
 q −1  Como é sabido que Pn2 = ( a1an ) , segue que P 2 =   .
n

I
1 1
que I é uma soma de P.G. de 1o termo igual a e razão igual a : 10 Seja Q um quadrado de lado 4. A partir dos pontos médios de Q,
a1 q
construímos o quadrado Q1. Prosseguindo da mesma forma, a partir
 1  dos pontos médios dos lados do quadrado Qi, construímos o quadrado
− 1
1 1 1 1 1  qn Qi+1. Determine a soma das áreas de todos os quadrados citados no
I= + + + ... + n −1 =  ⇒
enunciado.
a1 a1q a1q 2 a1q a1  1 − 1 
 q 
 
Solução:
É fácil ver que a área de cada quadrado é a metade da área do
1
1  q n − 1 1  q n − 1 quadrado anterior. Portanto, temos uma P.G. de razão igual a . Como
⇒I=  =   2
a1q n −1  q − 1  an  q − 1  16
o 1o quadrado tem área 16, a soma das áreas é = 32.
1
1−
2

5. Matemática financeira o dinheiro”. A taxa seria uma compensação paga pelo tomador do
empréstimo para ter o direito de usar o dinheiro até o dia do pagamento.
5.1. Porcentagem 5.2.1. Juros simples
No regime de juros simples, a taxa de juros é sempre aplicada sobre
As frações (ou razões) que possuem denominadores iguais a 100 o valor inicial, ou seja, não leva em consideração o capital acumulado.
são conhecidas por razões centesimais e podem ser representadas pelo Como a taxa de juros ocorre sempre sobre o mesmo valor, estamos
símbolo “%”. sempre somando uma constante, ou seja, o montante acumulado cresce
O símbolo “%” é lido como “por cento”. “5%” lê-se “5 por cento”. como P.A. Deste modo, para o cálculo do montante, podemos usar a
“25%” lê-se “25 por cento”. seguinte fórmula:
Para se calcular uma porcentagem de um dado valor, basta multiplicar
a razão pelo valor desejado. ni 

M = C  1+ 
 100 
30
Ex.: 30% de 1500 → ⋅ 1500 = 450
100
Em que M é o montante, C o capital inicial, i a taxa de juros e n o
Aumento Percentual (ou redução): Dado um valor x (o preço de período de aplicação.
uma camisa, por exemplo), para calcular o valor após um aumento de
i 5.2.2 Juros compostos
i%, basta multiplicarmos x por (1 + ) . No caso de redução deve-se
100 No regime de juros compostos a taxa de juros é aplicada sempre sobre
i
multiplicar por 1 − . o montante atual, ou seja, temos juros sobre juros.
100
Como a taxa de juros é feita sobre o último valor, estamos sempre
Obs.: Um aumento de x% seguido de uma redução de x% não traz o valor i
de volta ao inicial. “pegando” o capital atual e multiplicando por 1 + de modo que o
100
Isto ocorre porque a redução é feita sobre um valor maior que o inicial. i
montante acumulado cresce como P.G. de razão 1 + .
100
Ex.: Se aumentarmos o preço de uma camisa em 10% e depois reduzirmos
em 10%, voltamos ao valor original? Vejamos: Logo, para calcularmos o montante, podemos usar a seguinte fórmula:
Seja x o preço da camisa, após o aumento de 10% ficamos com 1,1x. Ao
reduzirmos esse valor em 10% ficamos com 0,9 · 1,1. x = 0,99x, que n
 i 
corresponde a 99% do valor original. M = C  1+ 
 100 
5.2. Juros
Juro é a remuneração cobrada pelo empréstimo de dinheiro.
Obs.: Na verdade, tanto nos juros simples como nos juros compostos,
É expresso como um percentual sobre o valor emprestado (taxa de juro) os juros (J) adquiridos (ou cobrados) é a diferença entre o montante (M)
e pode ser calculado de duas formas: juros simples ou juros compostos. e o capital inicial (C)
O juro pode ser compreendido como uma espécie de “aluguel sobre
J=M–C

204 Vol. 1
Sequências

EXERCÍCIOS RESOLVIDOS

01 Em uma turma de Ciência da Computação formada de 40 rapazes e 05 (UF-PI) Uma quantia foi aplicada a juros simples de 6% ao mês,
40 moças, tem-se a seguinte estatística: 20% dos rapazes são fumantes; durante 5 meses e, em seguida, o montante foi aplicado durante mais
30% das moças são fumantes. Logo, a porcentagem dos que não fumam 5 meses, a juros simples de 4% ao mês. No final dos 10 meses, o
na turma é de: novo montante foi de R$ 234,00. Qual o valor da quantia aplicada
inicialmente?
Solução:
20% dos rapazes = 0,2.40 = 8 fumantes; 30% das moças = Solução:
0,3.40 = 12 fumantes. Sendo x a quantia inicial aplicada, após os cinco meses o montante
Logo o total de fumantes é 20 e o de não fumantes é 60 que corresponde era de:
a 60/80 = 75%. x(1 + 0,06 · 5) = 1,3x. Tendo aplicado esse montante com taxa de
juros de 4% a.m. durante 10 meses, o montante foi de 1,3x(1 + 0,04
02 Após se fazer uma promoção em um clube de dança, o número de · 10) = 1,3.1,4x = 234 donde x = R$128,57 (aproximadamente).
frequentadores do sexo masculino aumentou de 60 para 84 e, apesar
disso, o percentual da participação masculina passou de 30% para  in 
Obs.: Usamos a fórmula dos juros simples: M = C  1 +
24%. Considerando-se essas informações, é correto afirmar que o  100 
número de mulheres que frequentam esse clube, após a promoção,
06 (AFA – 03) Em julho de 2001, uma pessoa gastava 27,3% do seu
teve um aumento de:
salário com o pagamento da prestação da casa própria. Em 2002,
houve dois reajustes no seu salário: 40% em janeiro e 30% em junho.
Solução:
Se, em julho de 2002, o aumento daquela prestação foi de 130%, que
Seja x o total de frequentadores do clube antes da promoção, porcentagem de seu salário a pessoa passou a gastar?
têm-se:
0,3x = 60 donde x = 200. Assim, o número de frequentadores do Solução:
sexo feminino era 140. Sendo x o valor inicial do salário e y o valor da inicial da prestação,
Sendo y o número de frequentadores do clube após a promoção, têm-se sabe-se que y = 0,273x.
0,24y = 84 donde y = 350. Assim, o número de frequentadores do
Após os dois reajustes: 1,4 ∙ 1,3x = 1,82x. Já a prestação:
sexo feminino passou a ser 266.
2,3y = 2,3 ∙ 0,273x = 0,6279x.
Como 266/140 = 1,9 o aumento foi de 90%.
0 ,6279 x
Desse modo a prestação representa: = 0= ,345 34 ,5% do
03 (VUNESP) Uma mercadoria teve seu preço acrescido de 10%. salário. 1,82 x
Tempos depois, esse novo preço sofreu um desconto de 10%.
Denotando-se por pi o preço inicial e por pf o preço final da mercadoria, 07 (FUVEST – 90) Um país contraiu em 1829 um empréstimo de 1
p milhão de dólares, para pagar em cem anos à taxa de juros de 9% ao
determine f :
pi ano. Por problemas de balança comercial, nada foi pago até hoje, e a
Solução: dívida foi sendo “rolada”, com capitalização anual dos juros. Qual dos
valores abaixo está mais próximo do valor da dívida em 1989?
Basta lembrar que aumentos e reduções percentuais são feitos
Adote (1,09)8 ≅ 2.
através de multiplicação (não soma). Para aumentar 10% devemos
multiplicar por 1,10 e para reduzir 10% multiplicar por 0,90, deste modo:
p (A) 14 milhões de dólares.
p f = 1,1× 0 ,9 × pi → f = 0 ,99 (B) 500 milhões de dólares.
pi (C) 1 bilhão de dólares.
04 (CN – 99) As vendas de uma empresa foram, em 1998, 60% (D) 80 bilhões de dólares.
superiores às vendas de 1997. Em relação a 1998, as vendas de 1997 (E) 1 trilhão de dólares.
foram inferiores em:
Solução: Letra E.
(A) 62,5%. De 1829 até 1989 passaram-se 160 anos. Usando a fórmula de
(B) 60%. juros compostos:
(C) 57,5%.
(D) 44,5%. M = C(1 + i)n = 106(1 + 0,09)160 = 106 ∙ (1,098)20 = 106 ∙ 220 =
(E) 37,5%.
(1024)2 . 106 > 1012
Solução: Letra E.
Logo, a dívida passa de 1 trilhão de dólares.
Se x representa as vendas de 1998 e y as vendas de 1997, têm-se:
1
x = 1,6 y → y = x = 0 ,625 x , assim as vendas foram inferiores em
1,6
1 – 0,625 = 37,5%

IME-ITA 205
Matemática III – Assunto 1

6. Somatórios 8. Recorrências de 1a ordem


O símbolo de somatório serve para representar uma soma de parcelas Chamamos de recorrência qualquer sequência em que um termo pode
com mesma lei de formação, podendo ser uma soma finita ou infinita: ser obtido, através de alguma relação com os termos anteriores.
Quando numa recorrência, cada termo depende apenas do termo
n
imediatamente anterior dizemos que essa recorrência é de 1a ordem.
∑a i =1
i = a1 + a2 + ...+ an
Ex.: P.A. e P.G.
6.1 Propriedades dos somatórios Se (xn) é uma sequência tal que: xn = a · xn–1 + b, a ≠ 0 , dizemos
que esta é uma recorrência de 1a ordem com coeficientes constantes. Para
resolvê-la podemos utilizar a seguinte ideia:
Existem duas propriedades básicas de somatórios:
x x b x
xn = a · xn–1 + b, dividindo por an: nn = nn −−11 + n , seja y n = nn têm -se
I. O somatório da soma é a soma dos somatórios: a a a a
b b
y n = y n −1 + → y n − y n −1 = n como temos uma diferença de
n n n an a
∑ ( a + b ) = ∑a + ∑b
i =1
i i
i =1
i
i =1
i
consecutivos, vamos usar soma telescópica.

 b
Dem.:  y n − y n −1 = a n   1 n 
n
  − 1
∑(a + b ) = (a + b ) + (a + b2 ) + ... + ( an + bn ) = y − y = b
n
b b  a
i i
y n − y1 = ∑ k =     (soma da P.G.)
1 1 2
i =1  n −1
+ n−2
a n −1
k =2 a a  1 −1 
n n
     a 
= ( a1 + a2 + ... + an ) + ( b1 + b2 + ... + bn ) = ∑ai + ∑bi   
i =1 i =1

 b
 y 2 − y1 = a2
Voltando o problema para variável original:
II. Podemos colocar uma constante multiplicando para fora do somatório

n n x n x1 b  1 − a n   a  n −1  an − 1
− =  n   ⇒ x n = x1 ⋅ a + b ⋅ 
∑ λai = λ∑ai a n
a a  a   1− a   a−1 

i =1 i =1

EXERCÍCIOS RESOLVIDOS
Dem.: Basta pensar que colocamos λ em evidência.
1 1 1
01 Calcule a soma + + +…
1 1 + 2 1 + 2 + 3 1 + 2 + 3+4
+
7. Soma telescópica 1 + 2 + 3 +… + 2006
Solução:
Dada uma sequência (a 1, a 2, ...,a n) definimos a diferença de Veja que
consecutivos por: ∆n = an – an – 1, ∀n ∈ {2, 3, …, n} 2006
1 2006
2 2006
1 1 
S=∑ =∑ = 2∑  − =
k = 2 1 + 2 + ... + k k = 2 k ( k + 1) k =2  k k + 1
Repare que ao somarmos todas as diferenças de consecutivos cada 1 1  2005
termo aparece ora com sinal “+”, ora com sinal “–”, o que faz com = 2 − =
 2 2007  2007
que a maioria se cancele sobrando apenas o primeiro termo e o último,
chamamos essa soma de soma telescópica: 1 1
02 Deter mine o valor da soma: S = + +…
1 1⋅ 2 ⋅ 3 2 ⋅ 3 ⋅ 4
n +
∑∆a = ( a
i =2
i 2 − a1 ) + ( a3 − a2 ) + ... + (an − an −1) = an − a1 ⇒ n ( n + 1) ( n + 2 )

Solução:
n
Perceba que:
∑ ∆a i = an − a1
i =2  1 1 1 1 
 =  − 
 1 · 2 · 3 2  1 · 2 2 ·3
A ideia então é tentar transformar uma soma em uma diferença de 
modo que as diferenças tenham termos em comum, cancelando a maioria  1 = 1  1 − 1 
desses termos. + 2 · 3 · 4 2  2 · 3 3 · 4 

    
1 1 1  1  1 1  1 1 1 1 
+ + ... + =  1 −  +  −  + ...  =  − 
1⋅ 2 2 ⋅ 3 n(n + 1)  2   2 3   n · (nn + 1) · (n + 2) 2  n · (n + 1) (n + 1) · (n + 2) 
 1 1 1 n
+ −  = 1− = 1 1 1  1  n2 + 3 n + 2 − 2  n ( n + 3)
 n +1 n n+1 n+1 S=  − =  =
2  2 ( n + 1) ( n + 2 ) 
 2  2 ( n + 1) ( n + 2 )  4 ( n + 1) ( n + 2 )

206 Vol. 1
Sequências

2o Caso: p = 2
 x 0 = 2 , x1 = 5
03 Resolva a seguinte recursão:  x = 5 x − 6 x n
 n n −1 n−2
S2 = ∑ k2 = ?
Solução: k =1
Veja que podemos escrever essa recursão da seguinte forma: Usando a mesma ideia, diminuir o grau usando uma diferença:
(k + 1)3 – k3 = 3k2 + 3k + 1 e aplicando somatório dos dois lados:
xn – 2xn–1 = 3xn – 1 – 6xn – 2 = 3(xn – 1 – 2xn – 2)
Seja yn := xn – 2xn–1, têm-se: yn = 3yn–1 P.G., donde: n ( n + 1) 1 n ( n + 1) 
( n + 1)
3
− 1 = 3S2 + 3 + n ⇒ S2 =  n3 + 3 n2 + 2 n − 3 =
2 3 2 
yn = 3n–1 y1 = 3n–1(x1 – 2x0) = 3n–1(5 – 2.2) = 3n–1
2 n3 + 3 n 2 + n n ( n + 1) ( 2 n + 1)
= → S2 =
Logo: xn – 2xn–1 = 3n–1, dividindo por 2n (ideia conhecida para 6 6
recorrências de 1a ordem): 3o Caso: p = 3
n
x n x n −1 1  3  . Desse modo, seja z = x n , têm-se: n
− = ⋅  
2 n 2 n −1 3  2 
n
2n S3 = ∑ k3 = ?
k =1
1 3 
n Veja primeiro que (k + 1)4 = [(k + 1)2]2 = (k2 + 2k + 1)2 = k4 +
z n − z n −1 =   (diferença de consecutivos; soma telescópica) → 4k3 + 6k2 + 4k + 1
32
Assim, (k + 1)4 – k4 = 4k3 + 6k2 + 4k + 1
n
→ z n − z0 = 1 ⋅ ∑  3 
n

3 k =1  2  Aplicando somatório dos dois lados:


n ( n + 1) ( 2 n + 1) n ( n + 1)
( n + 1) − 1 = 4S3 + 6
4
  3 n  +4 +n⇒
 − 1 6 2
x n x0 1 3   2   → xn − 2 =
− = ⋅ ⋅ ⇒ 4S3 = (n4 + 4n3 + 6n2 + 4n) – (2n3 + 3n2 + n) – (2n2 + 2n) – n ⇒
2n 20 3 2  3 − 1  2n
 2 
 n ( n + 1) 
2
  n 4 + 2 n3 + n 2
⇒ S3 = ⇒ S3 =  
 3 n − 2n  4  2 
 → x n = 2.2 + 3 − 2 → x n = 2 + 3
n n n n n
= n
 2 
Caso Geral: Sp = ∑nk=1 kp

É fácil ver que as ideias usadas anteriormente continuam valendo


9. ∑nk=1 kp (somatório das para todo p natural, ou seja, o cálculo de Sp pode ser obtido da diferença
p-ésimas potências dos N (k + 1)p+1 – kp aplicando-se somatório na expressão, deste modo Sp
dependerá das somas anteriores e de (n + 1)p+1 – 1, onde Sp será um
primeiros naturais) polinômio de grau p + 1 sem termo independente.
Para determinar a soma das p-ésimas potências dos n primeiros
naturais, vamos analisar alguns casos particulares p = 1, p = 2 e p = 10. P.A. de ordem superior
3. A ideia é pensar em algum jeito de desenvolver esses somatórios, de
modo que a solução também funcione em um caso geral. Dizemos que uma sequência (a1, a2, a3, ..., an) forma uma P.A. de 2a
ordem se as diferenças ∆ai formam uma P.A. não estacionária.

1o Caso: p = 1
n Ex.: (1, 3, 6, 10, 15, 21) é P.A. de 2a ordem, pois suas diferenças são: (2,
S1 = ∑ k = ?. Poderíamos usar a soma da P.A., mas isso não serviria 3, 4, 5, 6) P.A. não estacionária.
k =1

para generalizar o problema. Temos que usar outra estratégia. De modo geral, uma P.A. de ordem k(k > 2) é uma sequência na qual
Pensando no que vimos na apostila, uma ideia para resolver somas as diferenças entre cada termo e o termo anterior formam uma P.A. de
é transformar cada termo numa soma de diferenças (soma telescópica). ordem k – 1.
Como k é um termo do 1o grau, podemos pensar na diferença de dois
termos do 2o grau: (k + 1)2 – k2 = 2k + 1 Ex.: (0, 0, 6, 24, 60, 120, 210, ...) é P.A. de 3a Ordem, pois suas diferenças
Definindo ak = k2, temos ak + 1 – ak = 2k+1 (diferença de consecutivos), valem:
aplicando somatório dos dois lados (0, 6, 18, 36, 60, 90, ...) P.A. de 2a Ordem, uma vez que olhando para
n n n as diferenças temos:
∑(a − ak ) = ∑ ( 2 k + 1) ⇒ ( n + 1) − 1 = n + 2∑k ⇒
2
k +1 (6, 12, 18, 24, 30, ...) P.A. não estacionária.
k =1 k =1 k =1

⇒∑
n
k=
(n 2
+ 2nn + 1) − 1 − n
=
n ( n + 1)
k =1 2 2

IME-ITA 207
Matemática III – Assunto 1

10.1 Teoremas importantes EXERCÍCIOS RESOLVIDOS


Teorema 1: 01 Obter o termo geral da sequência (1, 3, 7, 13, 21, ...)
Se (an) é uma P.A. não estacionária então an é um polinômio em n de
grau um e, reciprocamente, todo polinômio em n de grau um é termo geral Solução:
de alguma P.A. não estacionária. Olhando para a sequência formada pelas diferenças de termos
consecutivos (2, 4, 6, 8, ...) que é uma PA não estacionária, donde a
Dem.: sequência original é uma PA de 2a ordem.
Como toda PA de 2a ordem tem termo geral do 2º grau:
(⇒) Ida: Se (an) é P.A. não estacionária, então:
an = an2 + bn + c. Fazendo n = 1, 2 e 3:
an = a1 + (n – 1) · r = r · n + a1 – r (r ≠ 0) que é um polinômio do
1° grau em n.  a + b + c = 1(1)
 ( 2) − (1): 3 a + b = 2
4 a + 2 b + c = 3 ( 2 ) → → a = 1, b = −1, c = 1
(⇐) Volta: Se an = a · n + b, a ≠ 0, então an – an – 1 = (a · n + b) – 9 a + 3 b + c = 7 ( 3 ) ( 3 ) − ( 2 ) : 5 a + b = 4

[a · (n – 1) + b] = a = cte.
Assim, an = n2 – n + 1.
Assim (an) é P.A. não estacionária de razão a e a1 = a + b.
02 Determine a soma dos 20 primeiros termos da sequência (2, 4,
8, 14, 22 ...), que é uma PA de 2a ordem.
Teorema 2:
Seja Sn a soma dos n primeiros termos de uma sequência (an). Se Solução:
(an) é uma P.A. não estacionária, então Sn é um polinômio em n de grau A diferença dos termos consecutivos forma a sequência 2, 4, 6, 8,
dois desprovido de termo independente e reciprocamente, todo polinômio (...), que é uma PA não estacionária, donde a sequência original é
de grau dois desprovido de termo independente, é o valor da soma dos n uma PA de 2a ordem.
primeiros termos de alguma P.A. não estacionária. Assim, Sn (soma dos n primeiros termos) é um polinômio do 3o grau
sem termo independente.
Dem.:
Sn = an3 + bn2 + cn. Fazendo n = 1, 2 e 3.
(⇒) Ida: Se (an) é P.A. não estacionária, então:
a+ b+c=2 (1)
( a + a ) n ( a1 + a1 + ( n − 1) r ) n = r  ( 2) − 2 ⋅ (1): 6 a + 2b = 2
2a − r
⋅ n�( r ≠ 0)  8 a + 4 b + 2c = 2 + 4 = 6 ( 2) → →
= 1 n = ( 3 ) − 3 ⋅ (1) : 24 a + 6 b = 8
2
Sn ⋅n + 1
2 2 2 2 27 a + 9 b + 3c = 2 + 4 + 8 = 14 (3)

→ a = 1/3, b = 0, c = 5/3
Polinômio do 2º grau em n.
(⇐) Volta: Se Sn = a · n2 + b · n, a ≠ 0, então an = Sn – Sn – 1 = 1 3 5 1 5
Assim, Sn = n + n → S20 = ⋅ 203 + ⋅ 20 = 2700
(a · n2 + b · n) – [a · (n – 1)2 + b · (n – 1)] = 2a · n + b é polinômio do 3 3 3 3
1º grau em n e pelo Teorema anterior forma P.A. não estacionária. 03 Determine o valor da soma S = 1 · 2 + 2 · 3 + 3 · 4 + ... + 99 · 100.

Teorema 3: Solução (1):


Veja que estamos somando termos da forma k(k + 1) = k2 + k, polinômio
Se (an) é uma P.A. de ordem k então an é um polinômio em n de grau
do 2o grau. Assim, estamos somando termos de uma PA de ordem 2,
k e, reciprocamente, todo polinômio em n de grau k é termo geral de
donde a soma é um polinômio de grau 3 sem termo independente.
alguma P.A. de ordem k.
Sn = an3 + bn2 + cn. Fazendo n = 1, 2 e 3:
 a + b + c = 2(1)
Dem.:  ( 2) − 2 · (1): 6 a + 2b = 4 
 8 a + 4 b + 2c = 2 + 6 = 8 ( 2 ) → →
A demonstração desse resultado foge ao escopo do assunto. 27 a + 9 b + 3c = 2 + 6 + 12 = 20 ( 3 ) ( 3 ) − 3 · (1) : 24 a + 6 b = 14

Caso queira demonstrar a ideia é usar indução finita em k, o caso k
= 1 é o teorema 1, e para provar a passagem k → k + 1 deve usar que 1 2
→ a = ,b = 1,c =
Sp = ∑nk=1 kp é um polinômio de grau p + 1. 3 3
1 3 2 1 2
Assim,Sn = n + n2 + n → S99 = ⋅ 993 + 992 + ⋅ 99 = 333300.
Teorema 4: 3 3 3 3
A soma dos n primeiros termos de uma P.A. de ordem k é um polinômio Solução (2):
em n de grau k + 1 e termo independente nulo, reciprocamente, se Sn é a Podemos utilizar as conhecidas somas:
soma dos n primeiros termos de uma sequência e Sn é um polinômio de n
n ( n + 1) ( 2 n + 1) n
n ( n + 1)
grau k + 1 em n, então a sequência forma uma P.A. de ordem k. ∑k 2
=
6
e ∑k =
2
k =1 k =1

Dem.: junto às propriedades de somatório:


A demonstração foge ao escopo do assunto. 99 99 99
99 · 100 · 199 99 · 100
S = ∑k ( k + 1) = ∑k 2 + ∑k = + =
Caso queira demonstrar basta utilizar que ∑nk=1 kp é um polinômio de k =1 k =1 k =1 6 2
grau p + 1 sem termo independente junto ao Teorema 3.
= 328350 + 4950 = 333300

208 Vol. 1
Sequências

EXERCÍCIOS NÍVEL 1 07 (FUVEST) 500 moedas são distribuídas entre três pessoas, Antônio,
Pedro e Cristian, sentadas em círculo, da seguinte maneira: Antônio recebe
01 (AFA-90) Quantos números não múltiplos de 11 há no conjunto
uma moeda, Pedro recebe duas, Cristian três, Antônio quatro, Pedro cinco,
{x ∈ N| 51 ≤ x ≤ 1.500}?
Cristian seis, Antônio sete, e assim por diante, até não haver mais moedas
suficientes para continuar o processo.
(A) 1210.
Quantas moedas sobraram ao final do processo?
(B) 1318.
(C) 1406.
08 (ARGENTINA-88) Dados os números 7 e 15 determine um terceiro
(D) 1412.
número positivo tal que, ao se efetuar de todas as maneiras possíveis a
(E) n.r.a.
soma de dois quaisquer deles multiplicada pelo restante se obtenham três
números em progressão aritmética. Indique todas as soluções.
02 (AFA-94) O número formado por 3 algarismos em progressão
aritmética com soma 15 e que, adicionado a 396, dá como resultado ele
1 1 1
mesmo escrito em ordem inversa é: 09 Sabendo que (a, b, c) e ( , , ) estão em progressão aritmética,
b c d
(A) par. demonstre que 2ad = c(a + c).
(B) primo.
(C) múltiplo de 7. 10 Dada uma progressão aritmética na qual o primeiro termo é 12 e a
(D) divisível por 13. razão é 4, qual o valor de n, se a média aritmética dos n primeiros termos
dessa progressão é 50?
03 (AFA-88) A soma dos 15 primeiros termos da sequência (–2, 1, 4, 7,...)
vale: 11 (FUVEST) Em uma P.A. de termos positivos, os três primeiros termos
são: (1 − a,− a, 11 − a ). Qual o quarto termo desta P.A?
(A) 260.
(B) 285. (A) 2. (D) 5.
(C) 330. (B) 3. (E) 6.
(D) 345. (C) 4.

12 Determine cinco números em progressão aritmética sabendo que sua


5n − 4 soma é 40 e a soma dos inversos dos extremos, 1/3.
04 (AFA – 88) O termo geral de uma progressão aritmética é .A
soma dos n primeiros termos da progressão vale: 3
13 (FGV) Quantos termos devemos tomar na P.A. –7, –3, ... a fim de que
n2 − 5 n a soma valha 3.150?
(A) .
3
2
14 (FFCL USP-65) A soma de quatro termos consecutivos de uma P.A.
(B) 5 n − 3 n . é – 6 e o produto do primeiro deles pelo quarto é – 54 . Determine esses
6 termos.
5 n2 − 16 n
(C) . 15 (OLIMPÍADA DE MATEMÁTICA DE NATAL – 95) Os inteiros de 1 a
3
2
1.000 são escritos ordenadamente em torno de um círculo. Partindo de 1,
10 n − 8 n cada décimo quinto número é riscado (isto é, são riscados 1, 16, 31,...).
(D) .
6 O processo continua até se atingir um número já previamente riscado.
05 (ITA-96) As dimensões x, y e z de um paralelepípedo retângulo estão Quantos números sobrarão sem riscos?
em progressão aritmética. Sabendo que a soma dessas medidas é igual
a 33 cm e que a área total do paralelepípedo é igual a 694 cm2, então o (A) 800. (D) 862.
volume deste paralelepípedo, em cm3, é igual a: (B) 934. (E) Nenhuma correta.
(C) 933.
(A) 1200.
(B) 936. 16 (EUA) Se ƒ(x) = x2 + 3x + 2 e A = {1, 2, 3, ........., 1993}, para
(C) 1155. quantos elementos x, pertencentes ao conjunto A, ƒ(x) é divisível por 6?
(D) 728.
(E) 834. 17 Em uma progressão aritmética com um número par de termos, a soma
dos termos de ordem ímpar é 70 e a soma dos termos de ordem par é 85.
06 (ITA -88) Suponha que os números 2, x, y e 1.458 estão, nesta ordem, A soma dos extremos é 31. Forme a progressão.
em progressão geométrica. Desse modo o valor de x + y é:
18 Mostre que o produto de quatro termos consecutivos de uma
(A) 90. progressão aritmética de inteiros, aumentado da quarta potência da razão,
(B) 100. é um quadrado perfeito.
(C) 180.
(D) 360. 19 Os lados de um triângulo retângulo formam uma progressão aritmética
(E) 1.460. crescente. Mostre que a razão da progressão é igual ao raio do círculo
inscrito no triângulo e que os lados são diretamente proporcionais aos
números 3, 4 e 5.

IME-ITA 209
Matemática III – Assunto 1

20 Devemos colocar 500 bolas formando um triângulo, com uma bola 31 (ENEM-2010)
na primeira linha, duas na segunda linha, três na terceira etc. Em março de 2010, o Conselho Nacional de Desenvolvimento
Científico e Tecnológico (CNPq) reajustou os valores de bolsas de estudo
(A) Quantas bolas sobrarão? concedidas a alunos de iniciação científica, que passaram a receber R$
(B) Quantas linhas haverá? 360,00 mensais, um aumento de 20% com relação ao que era pago até
então. O órgão concedia 29 mil bolsas de iniciação científica até 2009, e
21 Em uma sequência (an), a soma dos n primeiros termos é, para todo esse número aumentou em 48% em 2010.
n, Sn = n2 + 2n . Determine an. (O Globo. 11/3/2010.)

22 Prove que se (a, b, c, d) estão em P.G., então vale a relação: (b – c)2 Caso o CNPq decidisse não aumentar o valor dos pagamentos aos
= ac + bd – 2ad. bolsistas, utilizando o montante destinado a tal aumento para incrementar
ainda mais o número de bolsas de iniciação científica no País, quantas
23 (Fuvest-10) Os números a1, a2, a3 formam uma progressão aritmética bolsas a mais que em 2009, aproximadamente, poderiam ser oferecidas
de razão r de modo que a1+ 3, a2 – 3, a3 – 3, estejam em progressão em 2010?
geométrica. Dado ainda que a1 > 0 e a2 = 2, conclui-se que r é igual a:
(A) 5,8 mil.
3 (B) 13,9 mil.
(A) 3 + 3 . (D) 3− . (C) 22,5 mil.
2
3 (D) 51,5 mil.
(B) 3 + . (E) 3− 3 . (E) 94,4 mil.
2
(C) 3 + 3 . 32 (UFPE) Segundo pesquisa recente, 7% da população brasileira é
4 analfabeta, e 64% da população de analfabetos é do sexo masculino. Qual
24 A espessura de uma folha de estanho é 0,1 mm. Forma-se uma pilha percentual da população brasileira é formado por analfabetos do sexo
de folhas colocando-se uma folha na primeira vez e, em cada uma das feminino?
vezes seguintes, tantas quantas já houverem sido colocadas anteriormente.
Depois de 33 dessas operações, a altura da pilha será, aproximadamente: (A) 2,52%.
(B) 5,20%.
(A) a altura de um poste de luz. (C) 3,60%.
(B) a altura de um prédio de 40 andares. (D) 4,48%.
(C) o comprimento da praia de Copacabana. (E) 3,20%.
(D) a distância Rio-São Paulo.
33 (PUC-CAMP) Um trabalhador comprou uma bicicleta, conseguindo
25 Existe alguma progressão geométrica que admite 8, 12 e 27 como um abatimento de 10% sobre o preço marcado. Do valor a ser pago, 40%
termos? foi dado como entrada e o restante foi pago em 5 parcelas sem juros, no
valor de R$ 41,04 cada. O valor do abatimento obtido foi:
26 Determine três números em progressão geométrica, sabendo que a
sua soma é igual a 52 e que o maior deles excede em 20 unidades a soma (A) R$ 32,00.
dos outros dois. (B) R$ 35,00.
(C) R$ 38,00.
27 Determine as geratrizes das dízimas periódicas: (D) R$ 40,00.
(E) R$ 42,00.
(A) 0,141414141... (D) 1,030503050...
(B) 0,345454545... (E) 1,711111111... 34 (FUVEST) Em certa população, 18% das pessoas são gordas, 30% dos
(C) 0,999999999... (F) 1,488888888... homens são gordos e 10% das mulheres são gordas. Qual é a porcentagem
de homens na população?
25 a2 2 2
28 (IME) Em uma P.G., tem-se a1 = e a4 = 2( a + 1) , com 35 (ESPM) Uma pessoa fez um investimento em ações. No primeiro
4( a2 + 1) 5a
a > 0. semestre, ela perdeu 30% do capital aplicado e no segundo semestre ela
recuperou 60% do que havia perdido. Em relação ao investimento inicial,
(A) Quais os valores de a para os quais a P.G. é decrescente? seu prejuízo nesses dois semestres foi de:

1 (A) 22%.
(B) Qual o limite da soma dos termos para q = a − ?
5 (B) 24%.
29 Uma pessoa pagou 20% de uma dívida. Se R$ 4.368,00 correspondem (C) 12%.
a 35% do restante a ser pago, determine a dívida total. (D) 16%.
(E) 18%.
30 Os capitais de R$ 20.000,00, R$ 30.000,00 e R$ 50.000,00 foram
aplicados à mesma taxa de juros simples mensal durante 4, 3 e 2 36 (Enem-11) Uma pessoa aplicou certa quantia em ações. No primeiro
meses, respectivamente. Obtenha o prazo médio de aplicação desses mês, ela perdeu 30% do total do investimento e, no segundo mês,
capitais, ou seja, o tempo por que seria necessário aplicar o capital total recuperou 20% do que havia perdido. Depois desses dois meses, resolveu
(R$ 100.000,00) à mesma taxa anterior para obtermos o mesmo retorno. tirar o montante de R$ 3.800,00 gerado pela aplicação.

210 Vol. 1
Sequências

A quantia inicial que essa pessoa aplicou em ações corresponde ao valor de: Esse veículo foi vendido pelo seu primeiro dono, após 5 anos de uso, por R$
24.000,00. Sabendo-se que o valor comercial do veículo atinge seu valor
(A) R$ 4.222,22. mínimo após 20 anos de uso, e que esse valor mínimo corresponde a 20%
(B) R$ 4.523,80. do valor que tinha quando era novo, então esse valor mínimo é, em reais:
(C) R$ 5.000,00.
(D) R$ 13.300,00. (A) menor que 4.500.
(E) R$ 17.100,00. (B) maior que 4.500 e menor que 7.000.
(C) múltiplo de 7.500.
37 (UFCE) José e João possuem uma empresa cujo capital é de (D) um número que NÃO divide 12.000.
R$ 150.000,00. José tem 40% de participação na sociedade e deseja
aumentar a sua participação para 55%. Se João não deseja alterar o EXERCÍCIOS NÍVEL 2
valor, em reais, de sua participação, o valor que José deve empregar na
empresa é: 01 (IME) Seja uma progressão aritmética de primeiro termo a1≠0 e último
1
termo a10 ≠ a1 ≠ 0. Seja a progressão aritmética de primeiro termo b1 =
(A) R$ 110.000,00. a1
1
(B) R$ 90.000,00. e último termo b10 = . É possível determinar a5 em função de a1 e a10?
a10 b6
(C) R$ 170.000,00.
(D) R$ 50.000,00. 02 (Uerj-05) O quadriculado abaixo deve ser preenchido por inteiros
(E) R$ 82.500,00. positivos de forma que cada linha e cada coluna formem uma progressão
aritmética. Qual deve ser o número na posição *?
38 (UERJ-08) João abriu uma caderneta de poupança e, em 10 de janeiro
de 2006, depositou R$ 500,00 a uma taxa de juros, nesse ano, de 20%. *
Em 10 de janeiro de 2007, depositou mais R$ 1.000,00. Para que João
tenha, nessa poupança, em 10 de janeiro de 2008, um montante de R$ 74
1.824,00, a taxa de juros do segundo ano deve corresponder a: 186
103
(A) 12%.
(B) 18%. 0
(C) 14%.
(D) 20%. 03 Calcule a soma de todos os inteiros compreendidos entre 100 e 400
(E) 16%. que não são divisíveis nem por 2, nem por 3, nem por 5.

39 (UFMG -2010) O preço de venda de determinado produto tem a 04 (IME) Calcule a soma dos n primeiros termos de uma P.A. cujo primeiro
seguinte composição: 60% referentes ao custo, 10% referentes ao lucro termo é a, sabendo que o quociente da soma dos n primeiros termos pela
e 30% referentes a impostos. Em decorrência da crise econômica, houve soma dos n seguintes é independente de n.
um aumento de 10% no custo desse produto, porém, ao mesmo tempo,
ocorreu uma redução de 20% no valor dos impostos. Para aumentar as 05 (ITA-93) Em uma progressão aritmética com 2n + 1 termos, a soma
vendas do produto, o fabricante decidiu, então, reduzir seu lucro à metade. dos n primeiros é igual a 50 e a soma dos n últimos é 140. Sabendo que
É correto afirmar, portanto, que, depois de todas essas alterações, o preço a razão desta progressão é um número inteiro entre 2 e 13, qual o último
do produto sofreu redução de: número?

(A) 5%. 06 Calcule o maior valor possível para a razão de uma P.A. que admita
(B) 19%. os números 32, 227 e 942 como termos da progressão.
(C) 10%.
(D) 25%. 07 Podem os números 2, 3e 5 ser termos de uma mesma
(E) 11%. progressão aritmética?

40 (PUC-SP-97) Um veículo de transporte de passageiros tem seu 08 Prove que em qualquer P.G. vale a seguinte relação:
valor comercial depreciado linearmente, isto é, seu valor comercial sofre (S(n))2 + (S(2n))2, = S(n)(S(2n)+S(3n)), em que S(n) é soma dos n
desvalorização constante por ano. Veja a figura seguinte. primeiros termos da P.G.

valor (R$) 09 (FUVEST) A soma dos cinco termos de uma P.G., de razão negativa,
1
é . Além disso, a diferença entre o sétimo termo e o segundo termo da
2
P.G. é igual a 3. Nessas condições, determine:

(A) a razão da P.G.;


(B) a soma dos três primeiros termos da P.G.

0 20 tempo

IME-ITA 211
Matemática III – Assunto 1

10 Sejam a = 111...1 (n dígitos iguais a 1) e b = 100...05 (n – 1 dígitos 21 As medidas dos lados de um triângulo são expressas por números
iguais a 0). Prove que ab + 1 é um quadrado perfeito e determine sua inteiros em P.G. e seu produto é 1.728. Calcule as medidas dos lados:
raiz quadrada.
22 (AIME) As duas progressões geométricas distintas a1, a2, a3,... e b1,
∞ ∞
11 (IME-82) Seja N = 44...488...89, em que há n 4’s e (n – 1) 8’s. Prove
que N é um quadrado perfeito.
b2, b3,... são tais que ∑ ∑ b = 1. Dado que a = 81 e a = b =
k =1
ak =
k =1
k 3 2 2

x > 0, determine x.
12 Larga-se uma bola de uma altura de 5 m. Após cada choque com o
solo, ela recupera apenas 4/9 da altura anterior. Determine: 23 Existe alguma progressão aritmética infinita de razão diferente de zero
que pode ser formada apenas por números primos?
(A) a distância total percorrida pela bola.
(B) o tempo gasto pela bola até parar. 24 Mostre que não existe uma progressão aritmética infinita cujos termos
são todos quadrados perfeitos.
13 (AIME-89) Um determinado dígito d é tal que 0,d25d25d25d25...=
n , em que n é um inteiro positivo. Determine n. 25 Seja Mn = {0, a1, a2, ...an|ai = 0 ou, 1≤ i ≤ n – 1 e an = 1} um conjunto
810 de frações decimais, Tn e Sn, respectivamente, o número de termos e a
∞ k
soma dos termos de Mn. Qual o valor da razão Sn , quando n tende ao
14 Calcule ∑k =1
 1
k ⋅  .
2
infinito? Tn

15 (UFMG) No período de um ano, certa aplicação financeira obteve um 26 Suprimindo-se um dos elementos do conjunto {1, 2,..., n}, a média
rendimento de 26%. No mesmo período, porém, ocorreu uma inflação de aritmética dos elementos restantes é igual a 16,1. Determine:
20%. Então, é correto afirmar que o rendimento efetivo da referida aplicação
foi de: (A) o valor de n;
(B) o elemento suprimido.
(A) 3%. (D) 4%.
(B) 6%. (E) 5,2%. 27 (IME -CG) Uma sequência a0, a1, a2, ... é tal que ai+1 – 2ai + ai –1 = K
(C) 5%. para todo i ≥ 1. Determine an em função de a0, a1, n e K.
16 (FGV) Uma empresa desconta do salário anual de seus funcionários
certa porcentagem para um plano de previdência privada. O desconto 28 A sequência {an}, satisfaz a1 = a2 = 1 e an+2 = 1 + an, para todo
é de p% sobre R$ 28.000,00 de renda anual, mais (p + 2)% sobre n. Determine a2010 . an+1
o montante anual do salário que excede R$ 28.000,00. João teve
desconto total de (p + 0,25)% do seu salário anual para o plano de 29 Krushenko salta 1 metro no primeiro salto, 2 metros no segundo,
previdência privada. O salário anual de João, em reais, sem o desconto 4 metros no terceiro, ..., 2n–1 metros no salto número n. Há alguma
do plano de previdência é: possibilidade de Krushenko escolher as direções dos seus saltos de modo
a conseguir voltar ao ponto de partida?
(A) R$ 28.000,00.
(B) R$ 42.000,00. 30 Calcule o valor da soma Sn = 1.3 + 2.4 + 3.5 +...+ n(n+2).
(C) R$ 32.000,00.
(D) R$ 56.000,00. 31 Os números (4, 6, 13, 27, 50, 84) estão em uma P.A. de ordem k.
(E) R$ 35.000,00. Determine o 30o termo.

17 Se (an) é uma progressão aritmética de termos não nulos, calcule, em 32 Seja A = {1,2, ... , p}. Calcule o valor da soma dos produtos que se
1 1 1 podem obter usando como fatores dois elementos distintos de A.
função de a1, an, e n, o valor de + + ... + .
a1. a 2 a2 . a3 an −1 .an
33 Determine o valor da seguinte expressão: 1 ∙ 100 + 2 ∙ 99 + 3 ∙ 98+
99 …+100 ∙ 1.
∑ ( k + 1)
1
18 Calcule .
k =1 k + k k +1 34 Seja (a1, a2, … , an) uma P.A. de 2a ordem, com bi := ai+1 – ai,
∀i ∈ {1, 2, …, n – 1}, e r = b2 – b1. Determine Sn = a1 + a2 + …+ an
121 em função de a1, b1, r e n.
19 Qual a P.G. de cinco termos cuja soma é e o produto é 243?
3
20 Calcule o valor da soma de n parcelas Sn= 1 + 11 +...+ 111...1. 1+ 2 1+ 2 + 3 1+ 2 + 3 + 4
35 (IIT) Calcule a soma S = + + + ...
13 + 23 13 + 23 + 33 13 + 23 + 33 + 43

212 Vol. 1
Sequências

EXERCÍCIOS NÍVEL 3

01 Seja a0, a1, ..., an,... uma sequência de números satisfazendo 04 Calcule a razão de uma P.G. de dez termos, sabendo que os seis
primeiros termos possuem exatamente quatro dígitos e que o último termo
(3 – an+1)(6 + an) = 18 e a0 = 3. Determine S = ∑ 1
n

possui cinco dígitos.


i = 0 ai

02 (YAGLOM) Considere a sequência a1, a2, a3, ... ,a100 de números reais ∞

∑ (3
tais que: 6k
05 (PUTNAM) Calcule .
a1 − 4 a2 + 3 a3 ≥ 0 k =1
k
− 2 ) ( 3 k +1 − 2k +1 )
k

a2 − 4 a3 + 3 a4 ≥ 0
min{m, n}
........................ 06 (OBM-U) Calcule ∑∑
m≥0
3 m+ n
n≥0
, em que min{m, n} é o menor
a98 − 4 a99 + 3 a100 ≥ 0 número dentre m e n. Por exemplo, min {3, 4} = 3.
a99 − 4 a100 + 3 a1 ≥ 0
a100 − 4 a1 + 3 a2 ≥ 0 2 bn
07 (IMC) Seja a0 = 2,b0 = 2,an +1 = 2 − 4 − an2 ,bn +1 = .
2 + 4 + bn2
an
Considerando que a1=1, determine os números a2, a3, ... , a100. Considere, então, a sequência cn = ; determine o valor de c11.
bn

03 (USAMO) A sequência xn é definida por x1 = 1 , x k +1 = x k2 + x k .


2
1 1 1
Encontre a parte inteira da soma + + ... + .
x1 + 1 x 2 + 1 x100 + 1

RASCUNHO

IME-ITA 213
Análise combinatória A ssunto
2
Matemática III

1. Introdução __ __ __ __ ⇒ 9 ∙ 10 ∙ 10 ∙ 10 = 9000 possibilidades


9 10 10 10
Talvez você já tenha se perguntado quantos são os resultados diferentes III. Caso no problema anterior os algarismos fossem todos distintos?
em uma loteria como a mega-sena ou quanto tempo seria necessário para
acertar uma senha caso fosse tentar todas as possibilidades. Solução:
Com o intuito de determinar o número de maneiras de ocorrer um __ __ __ __ ⇒ 9 ∙ 9 ∙ 8 ∙ 7 = 4536 possibilidades
dado evento e resolver problemas desse tipo, criou-se um ramo na 9 9 8 7
matemática conhecido como análise combinatória. Sua ideia principal é
agrupar problemas com ideias comuns, determinando assim os conceitos Repare que começamos a escrever o número da esquerda para a
principais necessários para resolução dos mesmos. direita, assim, na primeira casa temos nove possibilidade pois o zero
Neste assunto veremos os conceitos principais de combinatória, que não entra. Na casa seguinte continuamos com nove, uma vez que só não
são o Princípio Fundamental da Contagem (PFC) e o Princípio Aditivo, podemos utilizar o algarismo da primeira casa (o zero pode entrar) e depois
que basicamente são as ferramentas para o desenvolvimento de toda vamos sempre perdendo um algarismo.
teoria. Além disso, veremos as ideias principais de combinatória, como O que ocorreria se começássemos da direita para a esquerda no
as permutações, os arranjos e as combinações. problema anterior?
Encerrando este assunto, encontram-se tópicos menos tradicionais
como a Inclusão-Exclusão, os Lemas de Kaplansky e a Permutação __ __ __ __
Caótica. ? 8 9 10
Veja que, nesse caso, o número de possibilidades para o algarismo
Obs.: Este assunto possui diversos exemplos diferentes, uma vez que
das unidades de milhar não está definido, pois dependeria se o zero foi
boa parte do aprendizado em Combinatória está associado ao número de
utilizado anteriormente ou não. Se o zero tiver sido utilizado teremos sete
questões já vistas anteriormente.
possibilidades, caso contrário, teremos seis.
Isso ocorre uma vez que esta é a casa com maior restrição (o zero
2. Princípio fundamental da não entra), logo é importante que toda decisão com maior restrição seja
contagem (PFC) tomada primeiro.
Considere o seguinte problema: João decide sair de casa, abre então
seu armário e percebe que possui três calças e cinco blusas. De quantos 3. Princípio aditivo
modos diferentes João pode se vestir? Para resolver o tipo de problema que ocorreu no último exemplo,
Basta ver que para cada opção de calça João tem cinco opções de quando começamos pelo algarismo das unidades, podemos usar o
blusa. Como João pode escolher três calças diferentes, temos 3 ∙ 5 = princípio aditivo:
15 possibilidades.
De modo geral, se podemos tomar uma decisão de m maneiras e, se Dados dois conjuntos disjuntos A e B, temos: n(A∪B) = n(A) + n(B),
uma vez tomada essa decisão, podemos tomar outra de n maneiras, então onde n(X) denota o número de elementos de X.
o número de maneiras de tomar ambas as decisões é mn.
Vejamos então o exemplo anterior!
Ex.:
I. Sérgio deve viajar de uma cidade A para uma cidade B. Para isso,
Para determinar o número de possibilidades nas unidades de milhar,
ele possui oito opções distintas de estradas. Sabendo que ele deve
começando pelo algarismo mais a direita, devemos saber se o zero foi
ir de A para B em uma estrada e voltar por outra, de quantos modos
utilizado ou não anteriormente, assim temos dois casos:
diferentes Sérgio pode fazer o seu trajeto de ida e volta?

Solução: 1o Caso - se utilizarmos o zero antes.


Temos oito opções para ir e, uma vez escolhida essa opção, temos
sete opções para voltar, logo: 8 ∙ 7 = 56 • Primeiro, temos que definir aonde aparece o zero: 3 possibilidades,
• Segundo, completar o número:
II. Quantos números naturais de 4 algarismos existem no nosso sistema
__ 0 __ __ ⇒ 7 ∙ 1 ∙ 8 ∙ 9 = 504 possibilidades.
de numeração?
7 1 8 9
Solução:
Logo nesse caso temos: 3 ∙ 504 = 1512 números.
Uma ideia muito comum em combinatória é representar cada decisão
a ser tomada por um tracinho, colocando o número de possibilidades de
realizá-la abaixo de cada traço.

214 Vol. 1
Análise combinatória

2o Caso – sem utilizar o zero. 2o Caso – 1oQ ≠ 3oQ

• Basta escolher os algarismos do número.

__ __ __ __ ⇒ 6 ∙ 7 ∙ 8 ∙ 9 = 3024 possibilidades. λ–1 λ


6 7 8 9 ⇒ λ (λ – 1)(λ – 2)2
Como os casos são disjuntos: 1512 + 3024 = 4536 números, mesmo
λ–2 λ–2
resultado achado anteriormente.

Ex.:
I. Quantos números pares de 5 algarismos distintos podem ser Somando: λ (λ – 1) (λ – 2)2 + λ (λ – 1)2 = λ (λ – 1)(72 – 3λ + 3)
formados?

Solução: 4. Permutações simples


Repare que como o número deve ser par, ele deve terminar com 0, 2,
4, 6 ou 8. Como o fato de usar ou não o zero influencia na primeira casa Muitos problemas de combinatória estão associados a determinar o
(da esquerda para a direita), devemos abrir em casos: número de ordenações que se podem fazer dados n objetos. Chamamos
cada ordenação possível de uma permutação simples e representamos o
1o Caso - terminando em zero. número de permutações simples por Pn.

0 ⇒ 9 ∙ 8 ∙ 7 ∙ 6 ∙ 1= 3024 possibilidades.
__ __ __ __ __ Veja que na verdade isso é uma aplicação direta do PFC, uma vez que
9 8 7 6 1 basta determinar que elemento ocupa cada posição:

2o Caso - não terminando em zero. ... → Pn = n .( n − 1)... .1 := n ! → Pn = n!


n n −1 1
__ __ __ __ __ ⇒ 8 ∙ 8 ∙ 7 ∙ 6 ∙ 4= 10752 possibilidades.
8 8 7 6 4 (chamado também de fatorial n)

Somando: 3024 + 10752 = 13776 números.


Obs.: 0! = 1 por definição.

II. (Morgado) A figura abaixo mostra um mapa com quatro países:


Ex.:
I. Dada uma palavra qualquer, chamamos de anagrama qualquer
permutação simples de suas letras, mesmo que essa permutação
não tenha significado.

Sabendo disso, determine o número de anagramas da palavra


CADERNO?

Solução:
De quantos modos esse mapa pode ser colorido (cada país com uma Basta permutar suas letras, como existem 7 letras distintas:
cor, países com uma linha fronteira comum não podem ter a mesma cor) P7 = 7! = 5040.
se dispomos de λ cores diferentes?
II. Cinco casais desejam ocupar uma escada com cinco degraus para
Solução tirar uma foto. Sabendo que cada degrau deve ser ocupado por
exatamente um casal, determine o número de maneiras desses casais
Faremos menção a cada país como um quadrante do ciclo
se organizarem para essa foto:
trigonométrico.
Se começarmos a pintar o 1oQ, depois o 2oQ e assim por diante, o Solução:
número de possibilidades para o 4oQ não fica definido, uma vez que o 1oQ • Primeiro devemos permutar os casais nos degraus: P5 = 5! = 120.
e o 3oQ podem ser pintados da mesma cor ou não. Assim, devemos abrir • Segundo devemos decidir em cada casal quem fica à direita: 25 = 32.
em casos: • Pelo PFC: 120 ∙ 32 = 3840 possibilidades.

1o Caso – 1oQ = 3oQ III. Determine o número de anagramas da palavra HORTELÃ que possuem
todas as vogais juntas?
Solução:
λ–1 λ Sempre que um problema pedir para que alguns objetos fiquem juntos
⇒ λ (λ – 1)2 podemos tratá-los como um único objeto, já que podem ser vistos como
uma única caixa (bizu da caixinha).
1 λ–1

IME-ITA 215
Matemática III – Assunto 2

Assim, podemos considerar que essa palavra possui apenas cinco 6.1 Permutações completas (com repetição)
letras, sendo estas, as quatro consoantes e a caixinha com as vogais.
O que ocorre quando queremos determinar o número de anagramas de
• Primeiro: Permutando essas letras P5 = 5! = 120. uma palavra com letras repetidas? Por exemplo, quantos são os anagramas
• Segundo: Devemos permutar as vogais dentro da caixa: P3 = 3! = 6. da palavra CASA?
• Pelo PFC: 120 ∙ 6 = 720.
Considerando os A’s como letras distintas, temos 4! = 24
IV. Considerando a palavra do exemplo anterior, determine o número de permutações. Porém como os A’s são iguais, cada permutação é contada
anagramas que possuem H e R separados: duas vezes (CA1SA2 = CA2SA1), assim têm-se 12 anagramas.
De modo geral, se uma letra aparece n vezes, basta pegar o total de
Solução: permutações e dividir por n!, uma vez que fixadas as demais letras temos
Outra ideia importante em combinatória é olhar para o complementar n! maneiras de permutar essas letras iguais.
de um conjunto em relação ao total, uma vez que em muitos problemas é Chamamos de permutação completa cada ordenação de n objetos com
mais fácil fazer o contrário do que é pedido na questão. elementos repetidos ou não, e representamos por Pnα1, α2 , ..., αk o número
Neste problema, por exemplo, queremos saber o número de anagramas de permutações completas de n objetos, sendo α1 o número de objetos
que possuem H e R separados, porém pelo exemplo anterior já sabemos do 1o tipo, α2 do 2o tipo e assim por diante. Como os α’s representam a
resolver isso quando os objetos estão juntos, sendo assim: multiplicidade de cada objeto devemos ter α1 + α2 +...+ αk = n.

• Total de anagramas: P7 = 7! = 5040. Assim, o número de permutações completas será dado por:
• Anagramas com H e R juntos: P6 ∙ P2 = 6! ∙ 2!= 720 ∙ 2 = 1440.
• Resposta: 5040 – 1440 = 3600 anagramas. α ,α 2 ,...,α k n!
Pn 1
=
α1 ! α 2 !...αk !
Obs.: Essa ideia só foi rápida, porque ele queria apenas duas letras
separadas, no caso em que o problema peça mais de duas letras não
adjacentes existe outra maneira de resolver que será vista mais a frente.
6.2 Permutações circulares
E no caso de determinar o número de maneiras de colocar n pessoas
em um círculo com seus lugares equiespaçados, considerando iguais
5. Arranjos simples disposições obtidas através de rotação?
Muitos problemas em combinatória estão associados a determinar Fazendo o caso n = 4 (o caso geral é igual): Se a fila formada fosse
o número de ordenações em que alguns objetos podem ser distribuídos. em linha reta teríamos 4! = 24 maneiras de ordená-los, porém repare
Porém, nem sempre estamos interessados em utilizar todos os objetos que em um círculo temos configurações iguais (por rotação) como as
disponíveis, por exemplo, considere que em um parque de diversões existem representadas abaixo:
20 pessoas querendo entrar em numa montanha russa. Essa montanha
russa possui apenas quatro assentos, com disponibilidade para exatamente B A A D
uma pessoa. De quantos modos essa montanha russa pode ser composta?
Repare que, na verdade, assim como nas permutações, esse é apenas
mais um exemplo de aplicação direta do PFC: =
20 ⋅19 ⋅18 ⋅ ... ⋅ 1 20 !
⇒ 20 ⋅19 ⋅18 ⋅17 = =
20 19 18 17 16 ⋅15 ⋅ ... ⋅ 1 16 ! C D B C
Em um caso geral, se temos n objetos e queremos ordenar p desses
objetos, chamamos de arranjo de n escolhe p (An,p) o número de maneiras D C C B
de fazer essa ordenação:
n! =
⋅⋅⋅ → An, p = Anp = n ( n − 1) ⋅ ⋅ ⋅ ( n − p + 1) = →
n n −1 n − p+1 ( n − p)!
n!
→ An, p = A B D A
( n − p)!
Nesse caso devemos dividir o total de permutações simples por 4,
6. Outras permutações obtendo 6 permutações. No caso geral, com n objetos, teríamos n rotações
Ao resolver muitos problemas de combinatória é comum contarmos no círculo de onde basta dividir o total por n.
elementos que inicialmente são iguais erroneamente como distintos. Denotamos por (PC)n o número de permutações circulares que podem
Nesses casos, devemos tentar agrupar as soluções iguais, vendo quantas ser obtidos com n objetos, assim:
vezes cada objeto está sendo contado repetidamente.
Os casos mais clássicos em que isso ocorre são nas permutações n!
( PC)n = = ( n − 1)!
completas (com repetição) e nas circulares. n

216 Vol. 1
Análise combinatória

7. Combinações simples iguais, ou seja, cada escolha está sendo contada seis vezes, logo, basta
dividir o total por 6, obtendo 120 escolhas possíveis.
Existem alguns casos em que estamos interessados apenas em Ou seja, de modo geral, basta considerar as escolhas com ordem,
escolher um subconjunto de objetos dentre um conjunto maior disponível, e depois dividir pelo fatorial da quantidade de termos escolhidos para
não importando a ordem com que isso é feito. Um exemplo disso, supondo consertar isso.
que você possui um grupo de 10 amigos e deseja escolher três para fazer
uma viagem com você, de quantos modos isso pode ser feito? Sendo assim, se temos n objetos e queremos escolher p chamamos
de combinações simples o números de escolhas distintas:
Se a ordem fosse importante, a resposta seria A10,3 = 10 ∙ 9 ∙ 8 = 720.
Porém, como nesse caso a ordem não importa às escolhas: (A1, A2, A3); An,p n!
(A1, A3, A2); (A2, A1, A3); (A2, A3, A1); (A3, A1, A2); (A3, A2, A1) são todas Cn, p = =
p! ( n − p)! p!

EXERCÍCIOS RESOLVIDOS

01 (UFRJ) A mala do Dr. Z tem um cadeado cujo segredo é uma Solução:


combinação com cinco algarismos, cada um dos quais podendo variar Considere os irmãos A e B. Repare que, como os objetos do mesmo
de 0 a 9. Ele esqueceu a combinação que escolhera como segredo, mas tipo são idênticos, não é importante quais vamos escolher, o importante
sabe que atende às condições: é quantos vamos escolher. Então, temos:

• se o 1o algarismo é ímpar, então o último também é ímpar; • 1a etapa: Divisão das bolas: 3 opções – (3,5); (4,4); (5,3)
• se o 1o algarismo é par, então o último é igual ao 1o; • 2a etapa: Divisão das camisas: 3 opções – (2,4); (3,3); (4,2)
• a soma do 2o com o 3o é 5. • 3a etapa: Divisão das caixas: 7 opções – (2,8); (3,7); (4,6);...; (8,2)
• Resposta: 3 · 3 · 7 = 63 modos.
Quantas combinações diferentes atendem às condições estabelecidas 03 (UFRJ) Uma partícula desloca-se sobre uma reta, percorrendo
pelo Dr. Z? 1cm para a esquerda ou para a direita a cada movimento. Calcule de
quantas maneiras diferentes a partícula pode realizar uma sequência de
Solução: 10 movimentos terminando na posição de partida.
Sejam ABCDE os dígitos (nessa ordem). Vamos dividir o problema
em casos: Solução:
Representemos cada movimento para esquerda por E e cada
movimento para a direita por D. Veja que para terminar no ponto de partida,
1o caso – A é ímpar.
é necessário ter 5Ds e 5Es. Além disso, veja que cada maneira de realizar
• A: 5 opções (1, 3, 5, 7, 9) os 10 movimentos pode ser vista como uma sequência desses 5Ds e 5Es.
• B: 6 opções (0, 1, 2, 3, 4, 5) Então, o número de maneiras de se realizar esses movimentos é:
• C: 1 opção (fica determinado pela escolha de B)
• D: 10 opções 5,5 10 !
=
P10 = 252
• E: 5 opções (também é ímpar nesse caso) 5 !5 !
04 Oito crianças vão se dividir em dois times de 4 para disputar uma
No 1o caso, temos o total de 5 ∙ 5 ∙ 6 ∙ 1 ∙ 10 = 1500 combinações. partida de futebol. De quantas maneiras isso pode ser feito se:

2o caso – A é par. a. um time joga com camisa e o outro joga sem?


• A: 5 opções (0, 2, 4, 6, 8) b. os dois times jogam sem camisa?
• B: 6 opções
• C: 1 opção Solução:
• D: 10 opções a. Dentro de cada time, não importa a ordem na qual é feita a escolha,
• E: 1 opção (E = A) portanto precisamos usar combinação. Para o time com camisa, temos
8!
No 2o caso, temos o total de 5 ∙ 1 ∙ 6 ∙ 1 ∙ 10 = 300. C84
= = 70 . Para o time sem camisa, colocamos as crianças
4 !4 !
4 4!
restantes, o que só pode ser feito de=
C4 = 1 maneira possível.
Como dividimos em casos (disjuntos), devemos somar as respostas 0 !4 !
dos casos, obtendo assim 1500 +300 =1800 como resposta. Então, são 70 ∙ 1 = 70 maneiras.

02 Dois irmãos gêmeos ganharam de aniversário 8 bolas de futebol b. O que muda aqui é que a divisão passa a ser feita em grupos
iguais, 6 camisas iguais e 10 caixas de chocolate também idênticas. De indistinguíveis. Veja que se 1, 2, 3, 4, 5, 6, 7, 8 são as crianças,
quantos modos pode-se dividir esses presentes entre os dois de modo as divisões 1234/5678 e 5678/1234 são iguais (só porque os
que cada um receba, pelo menos, 3 bolas de futebol, 2 camisas e 2 dois times estão sem camisa – no item a, essas duas divisões
caixas de chocolate? são diferentes). Portanto, estamos contando duas vezes cada
configuração. Logo, precisamos dividir por 2 para contar apenas
70
uma vez, o que nos dá a resposta = 35 .
2

IME-ITA 217
Matemática III – Assunto 2

8. Soluções inteiras não negativas II. (Se a soma for menor ou igual a um número) Qual o número de
soluções inteiras não-negativas de x + y + z ″ 5?
Os conceitos iniciais de combinatória, como a permutação e a
combinação, envolvem a ordenação ou a escolha de determinados objetos. Solução:
E se quiséssemos distribuir objetos idênticos a um grupo de pessoas? Podemos interpretar esse problema da seguinte forma: temos cinco
Como isso deve ser feito? objetos e devemos entregar esses objetos (não necessariamente todos)
Como os objetos são idênticos, estamos interessados apenas em a três pessoas.
determinar quantos objetos cada pessoa irá ganhar. Então, por exemplo, Nesse caso, os objetos que não estão sendo entregues ficarão com
se temos cinco bombons e queremos distribuir a duas pessoas, basta ver o dono, logo, na prática, estamos distribuindo esses objetos entre quatro
o número de soluções inteiras não-negativas da equação: x + y =5, onde pessoas, assim o problema é equivalente a:
x e y representam o número de bombons que cada um ganhou.
x + y + z + f = 5 (onde f é a folga da equação, ou seja, o quanto
Nesse caso, temos as seguintes soluções: (5, 0); (4, 1); ...; (0, 5) falta para a soma chegar a cinco).
(6 soluções).
5,3 8!
Porém, como em muitos problemas de combinatória, se os números Desse modo, temos: P=
8
= 56 soluções.
5 !3 !
aumentarem um pouco fica impraticável listar todos os casos.
Então a pergunta é como determinar o número de soluções inteiras III. (Limitando uma variável por cima): Qual o número de soluções inteiras
não-negativas da equação: x1 + x2 + ... + xn = p não-negativas de x + y + z = 8, se x ″ 3 ?
Repare que basicamente o que se quer fazer é separar os p objetos
iguais em n grupos. Podemos representar cada objeto por um ponto, Solução:
e como eles devem ser separados em n grupos, deve-se inserir n –1 Nesse caso, como já se sabe limitar uma variável por baixo (ver ex. I),
divisórias entre esses pontos. O número de soluções é o número de trocas podemos calcular o total de soluções, sem restrições, e subtrair aquelas
possíveis entre as posições das divisórias (que iremos representar por que possuem x ≥ 4 .
barras) e dos pontos. 10 !
Total de soluções (sem restrição): = 45 soluções.
Para facilitar a visualização, considere o seguinte exemplo: 8 !2!
Soluções com x ≥ 4 Temos x = x’ + 4, donde x’ + y + z = 4 que
Quantas soluções existem para a equação: x + y + z = 3 vamos
representar cada solução de acordo com o que foi exposto acima: possui 6! = 15 soluções.
4!2!
(3, 0, 0) •••|| (0, 2, 1) |••|• Resposta: 45 – 15 = 30.

(0, 3, 0) |•••| (1, 0, 2) •||•• Obs.: No caso de termos mais de uma variável limitada por cima, o
problema fica um pouco mais difícil. Esse tipo de problema será visto
(0, 0, 3) ||••• (1, 2, 0) •|••| num tópico mais a frente chamado de inclusão-exclusão.

(1, 1, 1) •|•|• (2, 1, 0) ••|•| 9. Combinações com repetição


(0, 1, 2) |•|•• (2, 0, 1) ••||• (ou completa)
Já vimos o número de maneiras de escolher p objetos distintos
É fácil entender que existe uma bijeção entre as soluções e as dentre n disponíveis. E se pudéssemos escolher um mesmo objeto mais
representações por pontos e barras, ou seja, para determinar o número de de uma vez?
soluções da equação basta determinar o número de permutações existentes. Nesse caso, a pergunta a ser respondida é quantas vezes cada objeto
No caso geral, temos p pontos e n – 1 barras, logo temos Pnn+−p1,−p1 será escolhido, podendo alguns deles não serem escolhidos.
soluções inteiras não-negativas. O problema pode ser visto basicamente como o número de soluções
inteiras não-negativas de uma equação, assim se xi é o número de vezes
Ex.: que o objeto i é escolhido, e se chamarmos o número de escolhas possíveis
I. (Limitando as variáveis por baixo) Qual o número de soluções inteiras de ( CR ) p (combinação com repetição de n objetos tomados p a p) têm-se:
n
não-negativas de x + y + z = 5, se x ≥ 2 ?
p
x1 + x2 + ... + x n = p ⇒ ( CR ) n = Pnn+−p1,−p1 =
( n + p − 1)! = C p
Solução: p!( n − 1) ! n + p−1soluções.
Podemos simplesmente substituir variáveis. Se x ≥ 2 , então existe
Ex.: De quantos modos podemos comprar 3 refrigerantes em uma loja
x ' ≥ 0 inteiro tal que x = x’ + 2, assim a equação fica: onde há 5 tipos de refrigerantes?
3,2 5!
x’ + y + z = 3 que possui P= 5 = 10 soluções.
3 !2! Solução:
Uma interpretação para esse problema é pensar que queremos Devemos determinar quantas vezes cada refrigerante será
distribuir cinco objetos idênticos entre três pessoas, porém uma delas escolhido, ou seja, devemos ver quantas soluções possui a equação:
deve receber pelo menos dois. Podemos então entregar primeiro dois x1 + x2 + x3 + x4 + x5 = 3.
objetos a essa pessoa e depois distribuir três objetos de qualquer maneira n = (CR)5, 3 = C7, 3 = 35
entre as três pessoas.

218 Vol. 1
Análise combinatória

10. Lemas de Kaplansky Temos quatro espaços para cinco sinais “–“, assim devemos
determinar quantos sinais “–“ entram em cada espaço e como não existem
Existem diversos problemas de análise combinatória com restrições dois sinais “+” adjacentes, nos espaços do meio deve existir pelo menos
com relação a escolhas dos elementos. Um problema muito comum é um sinal de “–”. Assim o problema é equivalente: x + y + z + w = 5; com
determinar o número de maneiras de se escolher alguns objetos, não y ≥1 e z ≥1
podendo escolher objetos consecutivos dentro de uma ordem dada. Para Fazendo y = y’ + 1 e z = z’ + 1, têm-se: x + y’ + z’ + w = 3 que
a resolução desse problema, veremos uma ideia bem esperta. 6!
3,3
possui: P=
6
= 20 soluções.
10.1 Primeiro Lema de Kaplansky 3 !3 !

Quantos subconjuntos de p elementos existem no conjunto


Obs.: Apesar dessa solução ser mais trabalhosa que a primeira, é
{1, 2, 3, ..., n} sem elementos consecutivos?
importante conhecê-la, uma vez que essa ideia ajuda a resolver alguns
Vejamos, como exemplo, o caso n = 8 e p = 3. problemas mais gerais que o 1o Lema.
Uma forma de enxergar a escolha de elementos dentro de um conjunto
é pensar que, na verdade, devemos olhar para todos os elementos 10.2 Segundo Lema de Kaplansky
do conjunto e assim ir definindo quem entra e quem não entra no seu Veja o seguinte problema: de quantos modos um atleta pode escolher sua
subconjunto. rotina de treino semanal, se ele deve treinar três vezes na semana, porém não
Ou seja, para cada elemento pode-se atribuir um sinal “+” (ele entra pode treinar dias consecutivos para não ter fadiga muscular? (considere que
no subconjunto) ou um sinal “–“ (ele não entra no subconjunto). uma vez escolhida sua rotina, esta não pode ser alterada na semana seguinte).
Pensando assim, o que aconteceria se não existisse a restrição quanto O problema apresentado acima é parecido com o 1o Lema de
à escolha de elementos consecutivos? Kaplansky, pois temos sete dias na semana, e devemos escolher três
sem dois consecutivos para treinar. O que muda nesse problema é que o
Vejamos alguns subconjuntos de três elementos que podem ser
início de uma semana é consecutivo ao final de outra, ou seja, existe uma
{1, 2, 3, 4, 5, 6, 7, 8} {1, 2, 3, 4, 5, 6, 7, 8} situação circular no conjunto dado.
formados: {1, 2, 3} ⇒ {2, 5, 7} ⇒
+++−−−−− −+−−+−+− Mas geralmente, quer-se determinar o número de subconjuntos de p
É fácil entender que existe uma bijeção entre os subconjuntos e a elementos que se pode formar com n elementos distribuídos em círculo,
representação com os sinais, assim o total de subconjuntos (podendo como na figura a seguir: 1
n 2
existir elementos consecutivos) pode ser vista como o número de Desse modo, a única parte diferente do
5,3 8! problema é que devemos considerar 1 e n n–1 3
permutações com três sinais “+” e cinco “-“, que seria: P=8
= C83
5 !3 ! como elementos consecutivos.
(em um certo sentido escolher é equivalente a permutar e depois descontar Uma vez que a única mudança que ocorre
a ordem, ou seja, permutar com elementos iguais). é que ao entrar o elemento n o 1 não pode
No 1o Lema de Kaplansky, o que muda é que na hora de permutar mais ser escolhido, vamos abrir em casos:
não podemos ter sinais “+” adjacentes. E aí caímos em outro problema,
quantas permutações de cinco sinais “+” e três sinais “–“ existem sem
sinais “+” adjacentes? 1º Caso – O elemento n entra no subconjunto.

1a Solução: Fixando os sinais de “–“ Nesse caso, faltam escolher p – 1 elementos no conjunto {2, 3, 4, ...,
Podemos simplesmente pensar que os sinais “–” estão fixos e que n – 2} sem dois consecutivos, onde as extremidades não são consecutivas.
queremos espalhar os sinais “+”, nesse caso, têm-se: Pelo 1º lema isso pode ser feito de f ( n − 3, p − 1) = C(pn−−13)−( p−1)+1 = Cnp−−1p−1 .

2º Caso – O elemento n entra no subconjunto.


Ou seja, temos seis lugares para três sinais “+”, e como dois sinais
“+” não ficam juntos, devemos escolher exatamente três espaços, um Agora ainda faltam escolher os p elementos no conjunto {1, 2, 3, ...,
6! n – 1} sem dois consecutivos, onde as extremidades não são consecutivas.
para cada sinal “+”, ou seja, temos: C=6,3 = 20 escolhas.
3 !3 ! Pelo 1o lema isso pode ser feito de f ( n − 1, p) = C(pn −1)− p+1 = Cnp− p .
No caso geral, têm-se p sinais de “+” e n – p sinais “–“.
Definindo g(n, p) como resposta desse problema, chamado de 2o
Fixando os sinais “–“ teríamos n – p + 1 espaços para p sinais “+”, Lema de Kaplansky, têm-se:
ou seja, Cnp− p+1 escolhas. p n
g( n, p) = Cnp−−1p−1 + Cnp− p = Cnp− p + Cnp− p = Cnp− p ⇒
Representa-se a resposta do 1° Lema por f(n, p), assim: n− p n− p

n
f ( n, p) = C(pn − p+1) g( n, p) = Cnp− p
n− p
2ª Solução: Fixando os sinais de “+”
Deve-se distribuir os cinco sinais “–“ nos espaços representados abaixo:

+ + +

IME-ITA 219
Matemática III – Assunto 2

11. Permutação caótica EXERCÍCIOS RESOLVIDOS


Outro tipo de permutação conhecida é a permutação que não permite 01 (UFPB) Deseja-se pintar 6 esferas, recebendo cada uma, tinta
elementos em sua posição original. Por exemplo, imagine que temos de uma só cor escolhida entre 3 disponíveis. De quantas maneiras
10 livros em uma prateleira, e queremos reordená-los de modo que os pode-se pintar o conjunto de esferas?
livros não fiquem em sua posição inicial. De quantos modos podemos
organizá-los? (A) 30. (C) 28.
No caso mais geral, considerando o conjunto {1, 2, 3, ..., n}, queremos (B) 27. (D) N.R.A.
determinar o número de permutações de seus elementos, de modo que
nenhum elemento i caia na posição i. Solução: Letra C.
p
Uma ideia comum em combinatória é olhar para o problema contrário. Trata-se de uma combinação completa ( CR )n = Cn + p−1, p com n =
Nesse caso, vamos determinar o total de permutações e subtrair daquelas 6 6 8!
que tem pelo menos um elemento na posição original. 3 e p = 6, assim: ( CR )=
3 C=
8 = 28 soluções.
6 !2!
02 Quantas sãs as soluções inteiras não negativas de x + y + z + w <
• Total de permutações: n!
8?
• Permutações com pelo menos um elemento na posição original:
Solução:
Repare que temos que determinar o número de permutações que Como estamos interessados nas soluções inteiras se x + y + z +
possuem o 1 na primeira posição, ou o 2 na segunda posição, ou o 3 na w < 8, devemos ter: x + y + z + w  7que é equivalente a x + y
terceira, e assim por diante (ou não sendo no sentido excludente). Assim + z + w + f = 7.
queremos achar uma união de conjuntos. 11!
Neste caso devemos permutar 7 pontos e 4 barras: P=7,4
Chamando de Ai os conjuntos que possuem o elemento i em sua 11
= 330
7 !4 !
posição inicial, queremos determinar: n( A1 ∪ A2 ∪ ... ∪ An ) . soluções.

Pelo Princípio da inclusão-exclusão: 03 Quantas são as soluções inteiras positivas de x + y + z = 8?

Solução:
n( A1 ∪ A2 ∪ ... ∪ An ) = Soma(1a 1) − Soma( 2 a 2) + ... + ( −1)n Soma( n a n) Como as soluções são positivas, devemos ter:
x ≥ 1; y ≥ 1; z ≥ 1, em que x = x '+ 1; y = y '+ 1; z = z '+ 1 com x’,
Para determinar o número de elementos de uma interseção k a k, basta y’ e z’ inteiros não negativos, logo: x’ + y’ + z’ = 5.
notar que temos k elementos fixos e os outros permutando de qualquer 5,2 7!
Permutando cinco pontos e duas barras: P= 7 = 21 soluções.
maneira, logo temos (n – k)! possibilidades. Por exemplo: 5 !2!
04 (UFRJ) Uma estante de biblioteca tem 16 livros: 11 exemplares
Ex.: do livro “Combinatória é fácil” e 5 exemplares de “Combinatória
A1 ∩ A2 são as permutações em que o 1 e o 2 estão fixos e os demais não é difícil”. Considere que os livros com mesmo título sejam
permutando, logo possui (n – 2)! Elementos. indistinguíveis. Determine de quantas maneiras diferentes podemos
Finalmente como toda interseção k a k tem (n – k)! elementos a soma dispor os 16 livros na estante de modo que dois exemplares de
k a k é determinada pelo números de interseções k a k que podem ser “Combinatória não é difícil” nunca estejam juntos.
formadas

( Cnk ) vezes (n – k)!


Solução:
Representemos por F os livros ‘Combinatória é fácil’ e por D os
n
k +1 livros ‘Combinatória não é difícil’. Normalmente, em combinatória,
n( A1 ∪ A2 ∪ ... ∪ An ) = ∑ Cnk ( −1) ( n − k )! = começamos pela restrição. Este aqui é um dos poucos casos em
k =1 que a ordem de execução é contrária. Coloque os 11 F lado a lado:
n n
n! 1
=∑ ( −1)k +1 ( n − k )! = n ! ∑ ( −1)k +1 _F_F_F_F_F_F_F_F_F_F_F_
k =1 k !( n − k )! k =1 k !
Dos 12 espaços determinados acima, precisamos escolher 5 para
Finalmente sendo Dn o número de permutações caóticas, têm-se: colocarmos os D. Isso garante que os D não ficarão juntos, que é a
n restrição do problema. Veja que a ordem da escolha não é importante,
k +1 1 1 1 1 1
Dn = n !− n ! ∑ ( −1) = n !− n ! − + − ... + ( −1)n +1  ⇒ portanto, usamos combinação e temos = 5
C12
12!
= 792 .
k =1 k!  1! 2 ! 3 ! n ! 5 !7 !
05 Dado um octógono, quantos são os triângulos cujos vértices
são vértices não consecutivos do octógono?
1 1 1 1 ( −1)n 
Dn = n ! − + − + ...+ Solução:
 0 ! 1! 2! 3 ! n! 
   n 
Claramente o problema é o 2o Lema de Kaplansky  g( n, p) = Cnp− p 
 n− p 
8 3 8
com n = 8 e p = 3, logo: g(8, 3) = C5 = ⋅ 10 = 16 soluções.
5 5

220 Vol. 1
Análise combinatória

EXERCÍCIOS NÍVEL 1 10 (ITA – 83) Um general possui n soldados para tomar uma posição
inimiga. Desejando efetuar um ataque com dois grupos, um frontal com r
01 (IME) Determine quantos números de 4 algarismos diferentes podem soldados e outro da retaguarda com s soldados (r + s = n), ele poderá
ser formados com os algarismos 0, 1, 2, 3, 4, 5. dispor seus homens de:

Obs.: Considere os números iniciados com o algarismo 0 (por exemplo, n! 2n !


0123), número de 3 algarismos. (A) maneiras (D) maneiras
( r + s)! ( r + s)!
02 (UFRJ) Quantos números de 4 algarismos podemos formar nos quais n! 2n !
(B) maneiras (E) maneiras
o algarismo 2 aparece ao menos uma vez? r ! s! r ! s!
n!
03 (UFRJ) Um construtor dispõe de quatro cores (verde, amarelo, cinza (C) maneiras
e bege) para pintar cinco casas dispostas em uma rua, lado a lado. Ele ( )!
rs
deseja que cada casa seja pintada com apenas uma cor e que duas
casas consecutivas não possuam a menor cor. Determine o número de 11 (ITT JEE) Uma classe tem n alunos, temos que formar uma equipe
possibilidades diferentes de pintura. com eles, incluindo pelo menos dois estudantes e excluindo também, pelo
menos dois alunos. O número de maneiras de formar a equipe é:
04 (MORGADO) De um baralho comum (52 car tas) sacam-se
sucessivamente e sem reposição 3 cartas. Quantas são as extrações nas (A) 2n – 2n (C) 2n – 2n – 4
quais a primeira carta é de copas, a segunda é um rei e a terceira não é (B) 2n – 2n – 2
uma dama?
12 (ITA ) O número de arranjos de n + 2 objetos tomados cinco a cinco
05 (EFOMM) O código Morse usa “palavras” contendo de “1 a 4 letras”, as vale 180n. Nestas condições, concluímos que:
“letras” sendo ponto e traço. Quantas palavras existem no código Morse?
(A) n é número par. (D) n é um número ímpar.
06 (MORGADO) Quantos são os anagramas da palavra CAPÍTULO: (B) n é um número primo. (E) n é divisível por 5.
(C) n está compreendido entre 100 e 200.
a. que começam por consoante e terminam por vogal?
b. que têm as letras C, A, P juntas nessa ordem? 13 (ITA) Quantos anagramas com 6 caracteres distintos podemos formar
c. que têm as letras C, A, P juntas em qualquer ordem? usando as letras da palavra QUEIMADO, anagramas estes que contenham
d. que tem as vogais e consoantes intercaladas? duas consoantes e que, entre as consoantes, haja pelo menos uma vogal?
e. que têm a letra C no 1o lugar e a letra A no 2o lugar?
f. que têm a letra C no 1o lugar ou a letra A no 2o lugar? (A) 7.200. (D) 3.600.
g. que têm a letra C no 1o lugar ou a letra A no 2o lugar ou a letra P (B) 7.000. (E) 2.400.
no 3º lugar? (C) 4.800.

07 (OLIMPÍADA BELGA) Um número inteiro não negativo é dito 14 (MORGADO) Quantos são os números naturais de 7 dígitos nos quais
palíndromo se ele lido da esquerda para a direita é igual quando lido da o dígito 4 figura exatamente 3 vezes, e o dígito 8, exatamente 2 vezes?
direita para a esquerda. Por exemplo, 121, 0, 2002 e 4 são palíndromos.
O número de palíndromos que são menores que 1.000.000 é: 15 (VUNESP) A figura a seguir mostra a planta de um bairro de uma
cidade. Uma pessoa quer caminhar do ponto A ao ponto B por um dos
(A) 900. (D) 1999. percursos mais curtos. Assim, ela caminhará sempre nos sentidos “de
(B) 1991. (E) 2220. baixo para cima” ou “da esquerda para a direita”. O número de percursos
(C) 1993. diferentes
que essa pessoa poderá fazer de A até B é:
08 (ITA) Se colocarmos em ordem crescente todos os números de 5 (cinco)
algarismos, obtidos com 1, 3, 4, 6 e 7, a posição de número 61473 será:
(A) 95.040.
(A) 76o. (B) 40.635.
(B) 78o. (C) 924.
(C) 80o. (D) 792.
(D) 82o. (E) 35.
(E) N.D.A.
16 (UFRJ) Um grupo constituído por 4 mulheres e 4 homens deve ocupar
09 (ITA) Quantos números de seis algarismos distintos podemos formar as 8 cadeiras dispostas ao redor de uma mesa circular. O grupo deve
usando os dígitos 1, 2, 3, 4, 5 e 6 nos quais o 1 e o 2 nunca ocupam ser acomodado de modo que cada homem sente entre duas mulheres.
posições adjacentes, mas o 3 e o 4 sempre ocupam posições adjacentes? João e Maria estão nesse grupo de pessoas; entretanto, por motivos
de ordem estritamente pessoal, não podem sentar-se lado a lado.
(A) 144. (D) 288. Duas acomodações das pessoas ao redor da mesa são consideradas
(B) 180. (E) 360. diferentes quando pelo menos uma não tem o mesmo vizinho à direita,
(C) 240. nas duas acomodações. Determine o número de diferentes acomodações
possíveis dessas 8 pessoas ao redor da mesa circular.

IME-ITA 221
Matemática III – Assunto 2

17 Uma criança possui 96 blocos distintos. Cada bloco pode ser de 2 04 (OBM) Cinco amigos, Arnaldo, Bernaldo, Cernaldo, Dernaldo e Ernaldo,
materiais (plástico ou madeira), 3 tamanhos (pequeno, médio ou grande), devem formar uma fila com outras 30 pessoas. De quantas maneiras
4 cores (azul, verde, vermelho e amarelo) e 4 formatos (círculo, hexágono, podemos formar esta fila de modo que Arnaldo fique na frente de seus
quadrado e triângulo). Quantos desses blocos diferem do bloco “plástico 4 amigos? (Obs.: Os amigos não precisam ficar em posições consecutivas)
médio vermelho círculo” em exatamente dois quesitos? (Um exemplo seria
o bloco “madeira médio vermelho quadrado”.)  35 ! 
(A) 35! (D)   5!
 5 
18 (AFA) Em uma demonstração de paraquedismo, durante a queda livre, 35 !
participam 10 paraquedistas. Em certo momento, 7 deles devem dar as
(B) (E) eπ 163
5!
mãos e formar um círculo. De quantas formas distintas eles poderão ser 35 !
escolhidos e dispostos nesse círculo? (C)
5
(A) 120. (C) 86400. 05 (MORGADO) No quadro abaixo, de quantos modos é possível formar
(B) 720. (D) 151200. a palavra MATEMÁTICA, partindo de um M e indo sempre para a direita
ou para baixo?
19 (AFA) Dez balões azuis e oito brancos deverão ser distribuídos em
três enfeites de salão, de modo que um deles tenha 7 balões e os outros M
dois, no mínimo 5. Cada enfeite deverá ter 2 balões azuis e 1 branco, pelo M A
menos. De quantas maneiras distintas pode-se fazer os enfeites, usando
M A T
simultaneamente todos os balões?
M A T E
(A) 9. (C) 11. M A T E M
(B) 10. (D) 12. M A T E M A
20 Quantas são as peças de um dominó comum? M A T E M A T
M A T E M A T I
21 (UFRJ) Um campeonato de futebol foi disputado por 10 equipes em M A T E M A T I C
um único turno, de modo que cada time enfrentou cada um dos outros M A T E M A T I C A
apenas uma vez. O vencedor de uma partida ganha 3 pontos e o perdedor
não ganha ponto algum; em caso de empate, cada equipe ganha 1 ponto. 06 Para a Seleção Brasileira de Futebol foram convocados 22 jogadores,
Ao final do campeonato, tivemos a seguinte pontuação: os quais jogam em todas as posições, exceto dois deles, que só jogam
Equipe 1 – 20 pontos; Equipe 2 – 10 pontos; no gol. De quantos modos se podem selecionar os 11 titulares?
Equipe 3 – 14 pontos; Equipe 4 – 9 pontos;
Equipe 5 – 12 pontos; Equipe 6 – 17 pontos; 07 (ITA) Dispomos de seis cores diferentes. Cada face de um cubo será
Equipe 7 – 9 pontos; Equipe 8 – 13 pontos; pintada com uma cor diferente, de forma que as seis cores sejam utilizadas.
Equipe 9 – 4 pontos; Equipe 10 – 10 pontos. De quantas maneiras isto pode ser feito, se uma maneira é considerada idêntica
à outra, desde que possa ser obtida a partir desta por rotação do cubo?
Determine quantos jogos desse campeonato terminaram empatados.
08 (JOSÉ PLÍNIO – PRC) De quantas maneiras 8 contas distintas podem
ser colocadas num cordão elástico de modo a formar uma pulseira?
EXERCÍCIOS NÍVEL 2
01 (MORGADO) Um campeonato é disputado por 12 clubes em rodadas 09 (UFRJ – 08) Para montar um sanduíche, os clientes de uma lanchonete
de 6 jogos cada. De quantos modos é possível selecionar os jogos da podem escolher:
primeira rodada?
– um dentre os tipos de pão: calabresa, orégano e queijo;
02 (ITA) Uma escola possui 18 professores, sendo 7 de Matemática, 3 de – um dentre os tamanhos: pequeno e grande;
Física e 4 de Química. De quantas maneiras podemos formar comissões de – de um até cinco dentre os tipos de recheio: sardinha, atum, queijo,
12 professores de modo que cada um contenha exatamente 5 professores presunto e salame, sem possibilidade de repetição de recheio em um
de Matemática, com no mínimo 2 de Física e no máximo 2 de Química? mesmo sanduíche.

(A) 875. (D) 2877. Calcule:


(B) 1877. (E) N.D.A.
(C) 1995. a. quantos sanduíches distintos podem ser montados;
b. o número de sanduíches distintos que um cliente pode montar, se ele
03 (JOSÉ PLÍNIO - PRC) De quantas maneiras um grupo de 7 pessoas não gosta de orégano, só come sanduíches pequenos e deseja dois
pode ser agraciado com 4 prêmios diferentes: (todos os prêmios devem recheios em cada sanduíche.
ser distribuídos)
10 Tem-se 5 pontos sobre uma reta r e 8 pontos sobre uma reta r’ paralela
a. se nenhuma pessoa puder receber mais que um prêmio; a r. Quantos quadriláteros convexos com vértices em 4 desses 13 pontos
b. se cada pessoa puder receber qualquer número de prêmios (até quatro existem?
naturalmente);
c. se o vencedor do primeiro prêmio não puder receber outro prêmio, mas 11 (ITA – 95) Considere todos os números de cinco algarismos formados
vencedores de outros prêmios puderem receber mais de um prêmio. pela justaposição de 1, 3, 5, 7 e 9 em qualquer ordem, sem repetição.

222 Vol. 1
Análise combinatória

Calcule a soma de todos esses números. (A) 1.365. (D) 120.


(B) 840. (E) 35.
(A) 5 · 106 e 6 · 106. (D) 9 · 106 e 10 · 106. (C) 240.
(B) 6 · 106 e 7 · 106. (E) 10 · 106 e 11 · 106.
(C) 7 · 106 e 8 · 106. 03 (UESPI) Um supermercado oferece 10 variedades de sopas em
pacotes. De quantas maneiras um consumidor pode escolher 4 pacotes
12 (ITA) Considere (P) um polígono regular de n lados. Suponha que os de sopas, se pelo menos 2 pacotes devem ser da mesma variedade?
vértices de (P) determinem 2n triângulos, cujos lados não são lados de
(A) 500. (D) 515.
(P). O valor de n é:
(B) 505. (E) 520.
(C) 510.
(A) 6. (D) 20.
(B) 8. (E) Não existe este polígono.
04 (UFF) Quinze (15) pessoas, sendo 5 homens de alturas diferentes e
(C) 10.
10 mulheres também de alturas diferentes, devem ser dispostas em fila,
obedecendo ao critério: homens em ordem crescente de altura e mulheres
13 (Morgado) No início de uma festa há 6 rapazes desacompanhados e
em ordem decrescente de altura. De quantos modos diferentes essas 15
10 moças desacompanhadas. Quantos são os estados possíveis no fim
pessoas podem ser dispostas nessa fila?
da festa?
05 (Morgado) Quantas soluções inteiras não negativas de x1 + x2 + x3 +
14 (Morgado) São dados n pontos em círculo. Quantos n-ágonos (não
x4 + x5 + x6 = 20 existem nas quais exatamente 3 incógnitas são nulas?
necessariamente convexos) existem com vértices nesses pontos?
Em quantas pelo menos três são nulas?
15 (José Plínio – PRC) Quantos números distintos podem ser formados
06 (Morgado) Qual é o número máximo de termos de um polinômio do
pelo produto de dois números ou mais do multiconjunto {3, 4, 4, 5, 5,
grau p com n variáveis?
6, 7, 7, 7}. (Obs.: Um multiconjunto é um conjunto em que o número de
cópias de um elemento é relevante.)
07 (ITT) O número de soluções inteiras não-negativas de x1 + x2 + x3 +
4x4 = 20 é:
16 (Morgado) Quantas são as permutações simples dos números 1, 2,
..., n nas quais o elemento que ocupa a k-ésima posição é maior que k – 3, (A) 530. (C) 532.
para todo k? (B) 534. (D) 536.

17 Determine o número de triplas ordenadas de conjuntos (A, B, C) tais 08 (José Plínio – PRC) De quantas maneiras as letras da palavra
que: A ∪ B ∪ C = {1, 2, 3, ..., 2013} e A ∩ B ∩ C = ∅. INDIVISIBILIDADE podem ser permutadas de modo que duas letras “I”
nunca fiquem juntas?
18 Calcule o número de subconjuntos de três elementos escolhidos de
{21, 22, 23, ..., 22013} tais que eles formem uma P.G. 09 (EN) Quantos são os anagramas da palavra ESCOLA nos quais
nenhuma letra ocupa o seu lugar primitivo?
19 (José Plínio – PRC) Determine o número de pares ordenados (a, b)
tais que MMC (a, b) = 233557: (A) 719. (D) 100.
(B) 265. (E) 249.
20 (EN – 91) A partir de um conjunto de 19 atletas, formam 57 times (C) 197.
de 4 atletas cada. Todos os atletas participam de um mesmo numero de
times e cada par de atletas fica junto no mesmo time um mesmo numero EXERCÍCIOS NÍVEL 2
x de vezes . O valor de x é?
01 (OBM) Dizemos que uma palavra Q é quase anagrama de outra palavra
(A) 1. (D) 4. P quando Q pode ser obtida retirando-se uma letra de P e trocando a ordem
(B) 2. (E) 5. das letras restantes, resultando em uma palavra com uma letra a menos do
(C) 3. que P. Um quase-anagrama pode ter sentido em algum idioma ou não. Por
exemplo, RARO, RACR e ARCO são quase anagramas de CARRO. Quantos
são os quase anagramas da palavra BACANA que começam com A?
Soluções inteiras e não negativas
(A) 48.
EXERCÍCIOS NÍVEL 1 (B) 60.
01 (AFA) O número de soluções inteiras e não negativas da equação (C) 72.
x + y + z + t = 6 é igual a: (D) 96.
(E) 120.
(A) 84. (D) 90.
(B) 86. (E) N.R.A. 02 (Olímpiada de Maio) Cada um dos seis
(C) 88. segmentos da figura ao lado deve ser pintado de
uma entre quatro cores de modo que segmentos
02 (EN) Uma livraria vai doar 15 livros iguais a 4 bibliotecas. Cada vizinhos não tenham a mesma cor. De quantas
biblioteca deve receber ao menos dois livros. O número de modos que maneiras podemos fazer isso?
esses livros podem ser repartidos nessa doação é igual a:

IME-ITA 223
Matemática III – Assunto 2

03 (UFPE – Adaptado) No mapa abaixo estão esboçadas as ruas de um 03 Quantos números de 5 algarismos são divisíveis por 3 e também
bairro. As ruas verticais são paralelas entre si e a distância entre duas ruas possuem 6 como um de seus algarismos?
consecutivas é a mesma; o mesmo acontece com as ruas horizontais.
Calcule o número de formas de sair de A e chegar até B percorrendo a 04 (Morgado) Escrevem-se números de cincos dígitos (inclusive os
menor distância possível. começados por zero) em cartões. Como 0, 1 e 8 não se alteram de cabeça para
baixo e como 6 de cabeça para baixo se transforma em 9, um só cartão pode
B representar dois números (por exemplo, 06198 e 86190). Qual é o número
mínimo de cartões para representar todos os números de cinco dígitos?

05 (AIME) Dois quadrados de um tabuleiro 7 x 7 são pintados de amarelo


e o restante de verde. Dizemos que duas colorações são equivalentes se
uma puder ser obtida através da outra por uma rotação. Quantas colorações
não equivalentes existem?

06 (Morgado) De quantos modos podemos escolher 3 números, não


necessariamente distintos, no conjunto {1, 2, ... ,150} de modo que a
soma dos números escolhidos seja divisível por 3? E se os números
A devessem ser distintos?
04 (Morgado) Quantas permutações de 7 letras A e 7 letras B, nas quais 07 (Morgado) Para n ≥ k , determine o número de funções sobrejetoras
não há 3 letras A adjacentes, existem?
f : A → B , em que #A = n e #B = k.
05 (José Plínio – PRC) Em uma caixa há três tipos de objetos, cada um 08 (OBM) Um quadrado 4 x 4 é dividido em 16 quadrados unitários. Cada
em uma quantidade de 2n, de modo que ao final, ao todo, são 6n objetos. um dos 25 vértices desses quadrados deve ser colorido de vermelho ou
De quantas maneiras podemos dividi-lo entre duas pessoas de modo que azul. Ache o número de colorações diferentes tais que cada quadrado
cada um fique com 3n objetos? unitário possua exatamente dois vértices vermelhos.
06 Dados oito anéis distintos, determine o número de maneiras de 09 De quantos modos se podem sentar em fila 3 russos, 3 ucranianos e
usarmos 5 anéis em 4 dedos (o polegar não entra) de uma única mão. (A 3 bielorrussos, de modo que não fiquem dois compatriotas juntos?
ordem dos anéis é significante, mas não é necessário que todos os dedos
possuam um anel). 10 (Eureka) Quantas são as sequências de n termos, todos pertencentes
a {0, 1, 2}, que não possuem dois termos consecutivos iguais a zero?
07 Em um elevador entram 6 pessoas. De quantos modos essas 6
pessoas podem saltar no 3o, 5o e 6o andares, de maneira que em cada 11 Uma bandeira circular de n setores deve ser pintada utilizando-se
um desses andares salte pelo menos uma pessoa? k cores. De quantos modos isso pode ser feito, sabendo que regiões
adjacentes não podem ser pintadas com a mesma cor?
08 Quantas são as sequências de n termos, todos pertencentes a {0,1},
que possuem um número ímpar de termos iguais a 0? 12 (Morgado) Depois de ter dado um curso, um professor resolve se
despedir de seus 7 alunos oferecendo, durante 7 dias consecutivos, 7
09 Dado o alfabeto com três letras (a, b, c), encontre o número de palavras jantares para 3 alunos cada. De quantos modos ele pode fazer os convites
com n letras contendo um número par de a’s. se ele não deseja que um mesmo par de alunos compareça a mais de um
jantar?
EXERCÍCIOS NÍVEL 3
13 (AIME) Em um torneio, cada jogador jogou exatamente um jogo contra
01 (Morgado)
cada um dos outros jogadores. Em cada jogo, o vencedor recebe 1 ponto,
o perdedor recebe 0 ponto e, se houve empate, cada um dos jogadores
a. Qual a soma dos divisores inteiros e positivos de 720?
recebe 0,5 ponto. Ao fim do torneio, notou-se que exatamente metade dos
b. De quantos modos 720 pode ser decomposto em um produto de dois
pontos conquistados por cada jogador foram ganhos em jogos contra os
inteiros positivos?
dez jogadores com menor pontuação. (Em particular, cada um dos dez
c. De quantos modos 720 pode ser decomposto em um produto de três
piores jogadores marcou metade de seus pontos contra os outros nove
inteiros positivos?
desses dez). Qual o total de participantes do torneio?
d. De quantos modos 144 pode ser decomposto em um produto de dois
inteiros positivos?

02 (Morgado) Nove cientistas trabalham em um projeto sigiloso. Por


questões de segurança, os planos são guardados em um cofre protegido
por muitos cadeados de modo que só é possível abri-los todos se houver
pelo menos 5 cientistas presentes.

a. Qual o número mínimo de cadeados?


b. Na situação do item a., quantas chaves cada cientista deve ter?

224 Vol. 1
Trigonometria A ssunto
1
Matemática IV

Introdução 1.2 Linhas trigonométricas notáveis


A trigonometria surgiu com o objetivo de estabelecer relações entre
ângulos (normalmente fáceis de medir) e comprimentos (às vezes Ângulo Seno Cosseno Tangente
difíceis de mensurar, como no caso da largura de um rio extenso ou do
comprimento de um morro/prédio muito alto) em figuras geométricas. 1 3 3
30°
Hoje, a trigonometria também tem aplicações como ferramenta puramente 2 2 3
algébrica, útil para descrever fenômenos físicos e para simplificações
matemáticas. 2 2
45° 1
Os seus objetivos nesta seção incluem memorizar as definições 2 2
trigonométricas e as relações algébricas entre elas, memorizar
transformações trigonométricas e identificar oportunidades de aplicação 3 1
(usualmente a parte mais difícil e mais importante), resolver equações e 60° 3
2 2
inequações trigonométricas e compreender o comportamento das funções
trigonométricas e suas inversas.
Demonstração: basta aplicar as definições às figuras abaixo:
1. Definições e relações básicas
Triângulos retângulos com um ângulo comum α sempre são
semelhantes (caso AA) e, portanto, têm a mesma razão entre lados 45°
l 2
30°
correspondentes. Essas razões recebem nomes especiais, definidos
l
abaixo:
l l 3
2
hipotenusa 45° 60°
cateto
cateto oposto l l
senα = oposto
hipotenusa a 2
cateto adjacente cateto
cosα =
hipotenusa adjacente 1.3 Relações entre ângulos complementares
cateto oposto senα
taanα = = sen(90 – a) = cosa
cateto adjacente cosα
hipotenusa cos(90 – a) = sena
cateto
tan(90 – a) = cota
oposto
a
cateto Demonstração: Basta ver que o cateto oposto ao ângulo a é adjacente
adjacente ao ângulo 90° – a .
1 1 1 cosα
cscα = , secα = , cot α = =
senα cosα tanα senα 1.4 Linhas trigonométricas para ângulos
quaisquer (ciclo trigonométrico)
Obs.: Em um triângulo qualquer, vale a lei dos senos, que relaciona um
Podemos estender as definições de linhas trigonométricas para ângulos
ângulo a, o lado a oposto a esse ângulo e o raio R do círculo circunscrito
maiores que 90 (ou menores que zero) com auxílio do ciclo trigonométrico
de um triângulo pela fórmula a = 2R.sena
(circunferência de raio 1, como nas figuras).
1.1 Relações fundamentais Para ângulos entre 0 e 90 (1o quadrante), temos por definição (fazendo
sen2a + cos2a = 1 1 hipotenusa = 1) que o cosseno é a projeção do raio no eixo horizontal
o

e o seno é a projeção no eixo vertical. Estendendo esta ideia, definimos


tan2a + 1 = sec2a o cosseno (seno) de um ângulo x qualquer como o tamanho da projeção
cot2a + 1 = csc2a no eixo horizontal (vertical) do raio que forma um ângulo x com o eixo
horizontal. As demais linhas trigonométricas continuam definidas em
função do seno e do cosseno como nos ângulos agudos.
Demonstração: Substituindo as definições, vemos que todas essas
relações são equivalentes ao teorema de Pitágoras.

IME-ITA 225
Matemática IV – Assunto 1

sen sen Demonstração: O primeiro caso segue da definição, dado que x e


1o Q 2o Q x+2p têm a mesma representação no ciclo trigonométrico. O segundo
sen( x + π) − senx
caso segue de tan(x + π) = = = tan x .
sen cos( x + π) − cosx
a a
cos cos cos cos EXERCÍCIOS RESOLVIDOS

01 Calcule, para todo x, 3(sen4 x + cos4 x) – 2(sen6 x + cos6 x).

Solução:
Não basta atribuir valores para x, já que queremos determinar a expressão
para todo x. Elevando ao quadrado a relação fundamental, temos que
sen > 0 sen > 0 (sen2x + cos2x)2 = 1, que nos dá sen4 x + cos4x = 1 – 2sen2x cos2x
cos > 0 cos < 0 (repare que usamos (a + b)2 = a2 + 2ab + b2). Agora, elevando ao
cubo a relação fundamental, temos que (sen2x + cos2x)3 = 1, que nos dá
sen6x + cos6x = 1 – 3sen2x cos2x (sen2x + cos2x) = 1 – 3sen2x cos2x
sen sen
(repare que usamos (a + b)3 = a3 + b3 + 3ab(a + b)). Daí, a expressão
dada é igual a: 3 · (1 – 2sen2x cos2x) – 2 · (1 – 3sen2x cos2x) = 1.

a a 02 Simplifique a expressão sen4 x + 4 cos2 x − cos4 x + 4sen2 x .


cos cos
Solução:
Quando só temos seno e cosseno, é inevitável usarmos a relação
fundamental.
3o Q 4o Q O 1o radical é igual a:

(1 − cos x )
sen < 0 2
sen < 0 2
+ 4 cos2 x = 1 − 2 cos2 x + cos4 x + 4 cos2 x =
cos < 0 cos > 0

(1 + cos x )
2
= 1 + 2 cos2 x + cos4 x = 2
= 1 + cos2 x
1.5 Redução ao 1o– quadrante
Casos principais: De forma análoga, temos que o 2o radical é igual a 1 + sen2 x.
Ângulos suplementares (2o para 1o quadrante): sen(180 – x) = senx, Portanto, a expressão dada é igual a: (1 + cos2x) – (1 + sen2x) =
cos(180 – x) = – cosx cos2x – sen2x = cos2x.
Ângulos explementares (3o para 1o quadrante): sen(180 + x)= – senx,
4 5senα + 7 cos α
cos(180 + x) = – cosx 03 Sendo tan α = , calcule F = .
15 6 cos α − 3senα
Ângulos replementares (4o para 1o quadrante): sen(360 – x)= – senx,
cos(360 – x) = cosx Solução:
Em F, basta dividir o numerador e o denominador por cosα:
Demonstração: Basta desenhar o ponto no ciclo trigonométrico, senα cos α 4
observar a orientação dos eixos cosseno/seno e compará-lo (visualmente 5 +7 5⋅ + 7
cos α cos α 5 tan α + 7 125
ou por meio de congruência de triângulos) com o ponto correspondente F= = = 15 =
cos α senα 6 − 3 tan α 4 78
no 1o quadrante. 6 −3 6 −3⋅
cos α cos α 15

Obs.: Na prática, a melhor forma de resolver este tipo de problema é 04 Sabendo que 9 cos2x – 5 senx · cosx + 4 sen2x = 3 , determine
usando as fórmulas de adição e subtração de arcos que veremos em 2.1. tanx.

1.6 Paridade Solução: Usando que 3 = 3 (sen2x + cos2x), observamos que


sen(– x)= – senx essa é uma equação homogênea de grau 2 em senx, cosx (reveja
o conceito na apostila de álgebra básica). Dividindo por cos2x ,
cos(– x)= cosx
obtemos 9 – 5 tanx + 4tan2x = 3 sec2x (= 3 + 3 tan2x), ou seja,
tan(– x)= – tanx tan2x – 5 tanx + 6 = 0. Resolvendo a equação do 2o grau, obtemos
tanx = 2 ou tan x = 3.
Demonstração: São as simetrias (congruências) no ciclo trigonométrico.
05 Simplifique a expressão sen(270° + a) + sen(450° – a).
Obs.: Usaremos, a partir de agora, indiscriminadamente as unidades grau
e radiano para representar ângulos (180 graus equivalem a p radianos). Solução: Escrevendo 270 = 3 · 90° e 450 = 5 · 90o, podemos
desenhar os pontos no ciclo trigonométrico e concluir que:
1.7 Periodicidade sen(270° + a) = –cosa e sen (450o – a) = sen(90° – a) = cosa.
Seno e cosseno têm período 2p: sen( x + 2 p) = senx e cos( x + 2p) Logo, a expressão vale zero.
= cosx para todo x.
Tangente tem período p: tan(x + p) = tan x para todo x.

226 Vol. 1
Trigonometria

2. Transformações trigonométricas C

Para manipular expressões trigonométricas (simplificar expressões,


resolver equações/inequações), é importante compreender e memorizar
transformações importantes como adição/subtração de arcos, arco duplo, 1
arco metade, fatoração (transformação soma em produto), transformação
produto em soma e outras que veremos a seguir. β
+
α
2.1 Adição e subração de arcos
sen(a ± β) = sena ∙ cosβ ± senβ ∙ cosa 0 P
cos(a ± β)=cosa ∙ cosβ sena ∙ senβ
PC
tan α ⋅ tan β sen(a + b) = = AB + QC = sena ∙ cosb + senb ∙ cosa
tan( α ± β) = 1
1 tan α ⋅ tan β OP
cos(a + b) = = OA – BQ = cosa ∙ cosb – sena ∙ senb
1
Demonstração: Inicialmente, sobreponha um triângulo retângulo de
ângulo a a um de ângulo b e hipotenusa 1 como na sequência de figuras Dividindo uma pela outra e, em seguida, dividindo numerador e
abaixo. Em seguida, pense no seno como a projeção da hipotenusa no denominador por cosa ∙ cosb:
cateto separado e no cosseno como a projeção no cateto colado para
obter as seguintes relações: senα ⋅ cos β senβ ⋅ cos α
+
sen( α + β) cos α ⋅ cos β cos α ⋅ cos β tan α + tan β
C tan( α + β) = = =
cos( α + β) cos α ⋅ cos β − senα ⋅ senβ 1 − tan α ⋅ tan β
cos α ⋅ cos β cos α ⋅ cos β
α
90 – α
Para as fórmulas de subtração, basta escrever a – b = a + (– b)
1 α B e usar paridade.
90 – α
Obs.: Como usamos somente a definição de seno e cosseno como
β projeções, esta demonstração funciona mesmo que a e b não sejam
α ângulos agudos.
0 P A
2.2 Arco duplo
C
cos 2 a = cos2 a − sen2 a = 2 cos2 a − 1 = 1 − 2sen2 a
senβ sen( 2 a) = 2sena ⋅ cos a
2 tan( a)
1 B tan( 2 a) =
1 − tan2 a
β
cos
β Demonstração: Basta tomar b = a na fórmula de adição de arcos.
α
0 A Observação: De forma similar, temos:
C cos( 3 a) = 4 cos3 a − 3 cos a, sen(3 a) = − 4sen3 a + 3sena e
senβ · cosα
3 tan a − tan3 a
α senβ tan(3a)=
senβ · senα 1 − 3 tan2 a
Q α B
2.3 Arco metade
β
cos cosβ · senα a 1 + cos a
cos = ±
α P 2 2
0 cosβ · cosα A a 1 − cos a
sen = ±
2 2
No triângulo OBC, temos: OB = cosb e BC = senb. a 1 − cos a
tan = ±
No triângulo OAB, temos: OA = cosa ∙ hipotenusa = cosa · cosb e 2 1 + cos a
AB = sena ∙ cosb.
No triângulo QBC, temos: BQ = hipotenusa ∙ sena = senb ∙ sena e
QC = senb ∙ cosa. Demonstração: Trocando 2a por a na fórmula do cosseno do arco
duplo, obtemos as duas primeiras fórmulas. Dividindo uma pela outra,
Logo, olhando para o triângulo OPC, temos: obtemos a terceira.

IME-ITA 227
Matemática IV – Assunto 1

a a 1 + cos a 2.5 Transformação produto em soma


cos a = 2 cos2 − 1 ⇒ cos2 =
2 2 2 Em algumas ocasiões (por exemplo, quando um produto está sendo
a a 1 − cos a somado a uma parcela) é interessante fazer o contrário da fatoração, i.e.,
cos a = 1 − 2sen2 ⇒ sen2 =
2 2 2 transformar um produto em uma soma.
Obs.: Em algumas situações, é interessante utilizar essas 1
sena ⋅ cos b = ⋅ ( sen ( a + b ) + sen ( a − b ) )
fórmulas ao contrário, fatorando expressões da forma 1 ± cosa 2
 2 a 1
 por exemplo, 1 − cos a = 2sen 2  ou 1 ± sena (neste caso, primeiro cos a ⋅ cos b = ⋅ ( cos ( a + b ) + cos ( a − b ) )
  2
faz-se sena = cos(90 – a)). 1
sena ⋅ senb = − ⋅ ( cos ( a + b ) − cos ( a − b ) )
2.4 Fatoração (ou transformação 2
soma em produto) Demonstração: basta observar as contas intermediárias da
demonstração do teorema 10.
Fatorar (transformar soma em produto) é uma das principais
ferramentas algébricas para simplificação de expressões e resolução de 2.6 Truque do triângulo retângulo
equações, de forma que estes resultados merecem atenção especial.
Para lidar com combinações lineares de senos e cossenos de um
mesmo arco (ex.: senx+ cosy, 3senx + 2cosx, 5senx + 12cosx, etc.)
x±y xy
senx ± sen y = 2sen ⋅ cos reescrevemos a expressão como:
2 2
a ⋅ senx + b ⋅ cos x = R ⋅ sen( x + α), em que
x+y x−y b
cos x + cos y = + 2 cos ⋅ cos R = a2 + b2 , tan α =
2 2 a
x+y x−y Demonstração: Considerando um triângulo retângulo de catetos a e
cos x − cos y = − 2sen ⋅ sen
2 2 b como na figura abaixo:
tana ± tanb = sen(a ± b) · seca · secb
cota ± cotb = sen(a ± b) · csca · cscb R = a2 + b 2
b
Demonstração: Fazendo x = a + b, y = a – b em 2.1., temos
x+y x−y α
a= ,b= e:
2 2
a
senx + seny = sen( a + b) + sen( a − b) = 2sena ⋅ cos b = Temos a = R · cosa, b = R · sena
x+y x−y Logo,
= 2sen ⋅ cos
2 2 a · senx + b · cosx = R · (senx · cosa + sena · cosx) =
cos x + cos y = cos( a + b) + cos( a − b) = 2 cos a ⋅ cos b = = R · sen(x + a)
x+y x−y
= 2 cos ⋅ cos
2 2 Obs.: Embora a intuição geométrica utilizada só funcione para
cos x − cos y = cos( a + b) − cos( a − b) = − 2sena ⋅ senb = ângulos agudos, é possível demonstrar o resultado no caso geral.
a a
x+y x−y Como ≤ 1 , basta escolher a tal que cosα = e
= − 2sen ⋅ sen a2 + b 2 a2 + b 2
2 2
b
Trocando x por – x, obtemos as demais fórmulas. sen α = e substituir.
Substituindo a definição de tangente e cotangente: a + b2
2

sena senb sena ⋅ cos b + senb ⋅ cos a 2.7 Parametrização em função do arco
tan a + tan b = + = = metade
cos a cos b cos a ⋅ cos b
= sen( a + b) ⋅ sec a ⋅ sec b É possível escrever todas as seis linhas trigonométricas de um
x
cos a cos b senb ⋅ cos a + sena ⋅ cos b ângulo x em função de uma única variável t = tan . Desta forma, todas
cot a + cot b = + = = 2
sena senb sena ⋅ senb as expressões trigonométricas podem ser transformadas em frações
= sen( a + b) ⋅ csc a ⋅ csc b envolvendo apenas polinômios (nem sempre de grau baixo).
2t
Trocando b por – b, obtemos as demais fórmulas. senx =
1+ t 2
2t
Obs.: Para fatorar expressões com linhas trigonométricas diferentes (por tan x =
exemplo, sena + cosb), basta transformar cosseno em seno (por exemplo, 1− t 2
fazendo cosb = sen(90 – b)) e usar 2.4. 1− t 2
cos x =
1+ t 2

228 Vol. 1
Trigonometria

Demonstração: A primeira expressão é a mais difícil de demonstrar: EXERCÍCIOS RESOLVIDOS


partindo do seno do arco duplo, multiplicando em cima e embaixo por cosseno
para forçar o aparecimento da tangente, e usando a relação fundamental: 01 Calcule sen 15o e sen 75o:
x x x
sen 2 tan 2 tan Solução:
x x 2 2 x 2 2
senx = 2sen ⋅ cos = 2 ⋅ cos = = Utilizando as fórmulas de subtração e adição de arcos, temos:
2 2 x 2 sec2 x 1 + tan2 x
cos
2 2 2 sen 15o = sen(45o – 30o) = sen 45o cos 30o – sen 30o cos 45o=
A segunda expressão é simplesmente a tangente do arco duplo, e a 2 3 1 2 6− 2
= . − ⋅ = .
terceira expressão pode ser obtida dividindo-se a primeira pela segunda. 2 2 2 2 4
Analogamente, sen 75o = sen(45o + 30o) = sen 45o cos 30o + sen
2.8 Soma de senos/cossenos de arcos em
progressão aritmética 30o cos 45o = 6 + 2
4
C = cos b + cos( b + r ) + cos( b + 2 r ) + ... + cos( b + nr ) = Obs.: Das relações entre ângulos complementares (1.2.), obtemos
 r cos15°= 75° e cos75° = sen15°.
sen  ( n + 1) ⋅ 
 2  r
= ⋅ cos  b + n ⋅ 
sen r ( )2
 2 3
02 Sabendo que sen x + cos x = , determine os possíveis valores
4
de sen(2 x) e tan(2 x).
S = senb + sen( b + r ) + sen( b + 2 r ) + ... + sen(bb + nr ) =
 r Solução:
sen  ( n + 1) ⋅ 
 2  r Elevando a expressão dada ao quadrado para que apareça o termo
= ⋅ sen  b + n ⋅ 
sen ( )
r
2
 2  2 sen x · cos x da fórmula do sen(2 x), obtemos
9 −7
sen2 x + 2 sen x · cos x + cos2 x = ⇒ 1 + sen2 x = .
Demonstração: Uma técnica geral para resolver somas fechadas é 16 6
tentar encontrar uma “soma telescópica”, i.e., tentar quebrar cada parcela Para calcular a tangente, basta calcular o valor de cos 2x pela relação
como uma diferença de termos sucessivos se cancelem. fundamental:
r
Neste caso, podemos multiplicar a primeira equação por sen   e
2 sen22x + cos22x = 1
transformar produto em soma.
49 207
r r r cos2 2x = 1 − =
C ⋅ sen   = sen   ⋅ cos( b) + sen   ⋅ cos( b + r ) + ... 256 256
2
  2
  2
207
r |cos 2 x| =
sen   ⋅ cos( b + nr ) 16
2 −7
|tan 2 x | = 16 = 7 207
r  r  r  3r  207 207
2C ⋅ sen   = sen  b +  − sen  b −  + sen  b + 
2  2  2  2 16
 r  ( 2 n + 1)r   ( 2 n − 1)r 
− sen  b +  + ... + sen  b +  − sen  b + 
 2  2   2  03 (Lei das tangentes) Em um triângulo não retângulo, prove que
são iguais a soma e o produto das tangentes dos ângulos internos.
Cancelando os termos comuns:
r  r  r Solução:
2C ⋅ sen   = sen  b + ( 2 n + 1)  − sen  b − 
2
   2   2  Como A, B e C são ângulos de um mesmo triângulo, temos
A + B + C = 180° (Em todo problema com esse dado ‘A, B e C
Transformando o lado direito em produto: ângulos de um triângulo’, essa é obviamente a primeira ideia). Daí
 r A + B = 180° – C e, aplicando a função tangente dos dois lados,
sen  ( n + 1) ⋅  tan A + tan B
 2  r temos que tan(A + B) = tan(180° – C), logo = − tan C
C= ⋅ cos  b + n ⋅ 
sen r ( )
2
 2 1 − tan A tan B
. Eliminando o denominador, chegamos a tan A + tan B + tan C =
Trocando b por 90 – b e usando 1.2, temos:
tan A tan B tan C.
 r
sen  ( n + 1) ⋅ 
S( b) = C ( 90 − b ) =  2  ⋅ cos  90 − b + n ⋅ r  = 04 Sendo A, B e C ângulos de um mesmo triângulo, prove que:
sen( )r
2

 2 
A B C
sen A + sen B + sen C = 4cos cos cos .
 r 2 2 2
seen  ( n + 1) ⋅ 
 2  r
⋅ sen  b + n ⋅ 
sen ( ) r
2
 2

IME-ITA 229
Matemática IV – Assunto 1

Para o caso geral, a ideia é deixar um lado igual a zero, fatorar o outro lado
Solução: e utilizar quatro sinais para lidar com cada fator conforme resultados abaixo:
Assim como no exemplo anterior, temos A + B = 180° – C, o que dá
sen C = sen(A + B). Então: sen A + sen B + sen C = sen A + sen
B + sen(A + B) = sen z > 0 ⇔ z ∈ ∪ ( 2 κπ, π + 2 κπ )
κ∈
 A +B  A −B  A +B  A +B
= 2sen   cos   + 2sen   cos  = (i.e., z ∈ ... ∪ ( −2π, − π) ∪ (0, π) ∪ ( 2 π, 3 π) ∪ ...)
 2   2   2   2 
 A + B   A −B  A + B 
= 2sen    cos   + cos    (*) .  π π 
 2   2   2  cos z > 0 ⇔ z ∈ ∪  − + 2 κπ, + 2 κπ 
κ∈  2 2 
 A +B   C C
É fácil ver que sen   = sen  90 − 2  = cos 2 . Além disso,  5π 3π   π π   3π 5π 
 2    (i.e., z ∈ ... ∪  − , −  ∪  − ,  ∪  ,  ∪ ...)
 2 2   2 2  2 2 
 A −B  A +B A B A B A B
cos 
2  + cos  2  = cos 2 cos 2 + sen 2 sen 2 + cos 2 cos 2 −
   
EXERCÍCIOS RESOLVIDOS
A B  A −B  A +B A B
−sen sen , ou seja, cos   + cos  2  = 2cos 2 cos 2 .
2 2  2    01 Utilize a equação sen 3x = sen 2x para obter o valor de cos36°.
Substituindo em (*), temos o resultado.
Solução:
Veja que x = 36° é solução dessa equação, pois sen108° = sen72°.
Inicialmente, vamos provar que sen3x = 3senx – 4sen3 x:
3. Equações e inequações sen3 x = sen(2x + x) = sen2 x cos x + sen x cos 2x = 2sen x cos x
A primeira etapa para resolver equações ou inequações trigonométricas cos x + sen x(cos2 x – sen2 x) = 2sen x(1 – sen2 x) + sen x(1 – 2sen2
é simplificar as expressões por meio de transformações, com o objetivo x)= 3sen x – 4sen3 x.
de chegar a uma igualdade ou desigualdade simples entre linhas
trigonométricas iguais. Agora, no problema, temos a equação 3sen x – 4sen3 x = 2sen x cos
x. Para sen x ≠ 0, temos 3 – 4sen2 x = 2 cos x ⇔ 3 – 4(1 – cos2 x)
3.1 Igualdade entre mesmas linhas
= 2cos x ⇔ 4 cos2 x – 2cos x – 1 = 0. Como cos36° > 0, segue
trigonométricas 1+ 5
cos x = cos y ⇒ x = ± y + 2 kp, k ∈ Z que cos36 = .
4
tan x = tan y ⇒ x = y + kp , k ∈ Z Obs.: Em alguns problemas, é conveniente saber de antemão o valor
sen x = sen y ⇒ x = y + 2 kp ou x = – y+(2 k +1)p, k ∈ Z de cos36°.

Demonstrações: Observando o ciclo trigonométrico e o resultado 02 Resolva a equação senx + 3 cos x = 1.


sobre redução ao 1º quadrante:
Solução:
Cossenos iguais implicam ângulos congruentes (x – y = 2 kp) ou Esse problema pode ser resolvido de outra forma, mas a melhor
replementares (x + y = 2 kp). solução é utilizando o truque do triângulo retângulo. Dividindo ambos
Tangentes iguais implicam ângulos congruentes (x – y = 2 kp) ou 1 3 1
os lados da equação por 2, temos que senx + cos x = , ou seja,
explementares (x – y = p + 2 kp). Combinando as expressões, obtemos 2 2 2
x = y, x = y + p , x = y + 2p, etc. π π 1
cos senx + sen cos x = . Usando a fórmula de seno da soma,
Senos iguais implicam ângulos congruentes (x – y = 2 kp) ou 3 3 2
suplementares x + y = (2 k + 1)p. temos

3.2 Caso geral de equações trigonométricas  π 1 π π


sen  x +  = , que dá x + = + 2 k π ou x + =
π 5π
+ 2 k π, para
No caso geral, deve-se sempre utilizar substituições, transformações  3 2 3 6 3 6
e reduções de quadrante para se obter uma situação de igualdade entre  π π 
inteiro. Então S = − + 2 k π, + 2 k π k ∈ Z .
mesmas linhas trigonométricas.  6 2 
Ex.: para resolver sen x = cos 2x, podemos inicialmente escrever 03 Determine todos os valores de x∈[0,2p] que satisfazem sen(3x) <
π  cos(4x).
sen  − x  = cos x ; para encontrar a solução de sen2 x – 3sen x + 2
2 
=0, fazemos a substituição t = sen x; e para resolver uma equação do Solução:
Colocando tudo no lado esquerdo e transformando seno em cosseno
tipo sen x + sen 3x = cos x, começamos por transformar o lado esquerdo (usando 1.2) temos:
em um produto.
π 
sen(3 x ) − cos( 4 x ) < 0 ⇔ cos  − 3 x  − cos( 4 x ) < 0
3.3 Inequações trigonométricas 2 
Para casos simples (ex: inequações em que os argumentos das
funções estão limitados ao intervalo [0,2 p]), basta desenhar o ciclo
trigonométrico e identificar os intervalos que funcionam.

230 Vol. 1
Trigonometria

Demonstração: Basta substituir e usar as expressões de redução ao


Fatorando o lado esquerdo da desigualdade (usando 2.4): 1o quadrante.
1 π  1 π 
−2 ⋅ sen  ⋅  − 3 x + 4 x   ⋅ sen  ⋅  − 3 x − 4 x   < 0 ⇔
2 2  2 2  4.2 Gráfico
π π  π 7π  Para esboçar o gráfico de uma função trigonométrica, é aconselhável
sen  +  · sen  − >0 determinar, se possível, o período, as raízes, os pontos de máximo/mínimo
4 2 4 2 
e o comportamento próximo aos pontos fora do domínio da função. Os
Fazendo o quadro de sinais, concluímos que a expressão será negativa gráficos das funções principais são:
em dois casos. Seno:
π x  π 7x 
Caso 1: sen  +  > 0 e sen  −  > 0. Usando 3.3 :
4 2 4 2 
π x
+ ∈ ... ∪ ( −2π, − π) ∪ (0, π) ∪ ( 2π, 3π) ∪ ... ⇒
4 2
 9π 5π   π 3π   7π 11π 
⇒ x ∈ ... ∪  − , −  ∪  − ,  ∪  − , ∪ ...
 2 2   2 2   2 2 
π 7x
− ∈ ... ∪ ( −2π, − π) ∪ (0, π) ∪ ( 2π, 3π) ∪ ... ⇒ Cosseno:
4 2
 17π 13π   9π 5π   π 3π 
⇒ x ∈ ... ∪  + ,+  ∪+ , +  ∪  + , −  ∪ ...
 14 14   14 14   14 14 
Para encontrar a interseção, desenhamos a reta numérica de 0 até 2p.
3 p/2
0 p/14 5p/14 9p/14 13p/14 17p/14 21p/14 25 p/14 2p

Logo, o conjunto solução no 1o caso é:


 π   5π 9π   13π 17π 
S1 = 0,  ∪  ,  ∪  ,  Tangente:
 4   14 14   14 14 

π x  π 7x 
Caso 2: sen  +  < 0 e sen  −  < 0
4 2 4 2 
Para o 2o caso, olhamos os complementos das partes hachuradas na
 25π 
reta numérica: S2 =  , 2π 
 14 
Logo, a resposta do problema é

 π   5π 9π   13π 17π   25π 


S = S1 ∪ S2 =  0,  ∪  ,  ∪  , ∪ , 2π 
 14   14 14   14 14   14 
4.3 Inversa
4. Funções trigonométricas Restringindo o domínio e o contra-domínio das funções trigonométricas,
pode-se garantir que elas sejam bijetoras. Por exemplo, as funções
Como definimos seno e cosseno de maneira única para todo  π π  π π
número real x, podemos definir funções f :  → , f ( x ) = cos x e sen :  − ,  → [−11 , ] cos : [0, π] → [−11
, ] e tan :  − ,  →  são
 2 2  2 2
g :  → , g( x ) = sen x . E, restringindo domínios de forma apropriada,
todas bijetoras e contínuas. Definem-se as funções trigonométricas
podemos definir funções para todas as linhas trigonométricas:
inversas como:
 π 
tan :  x ∈ ; x ≠ + k π, k ∈   → , cot : { x ∈ ; x ≠ k π, k ∈ } →   π π
 2  , ] →  − ,  , definida por y = arc sen x ⇔ x = sen y
arc sen : [−11
 2 2
 π 
sec :  x ∈ ; x ≠ + k π, k ∈   → , csc : { x ∈ ; x ≠ k π, k ∈ } →  , ] → [0, π] , definida por y = arc cos x ⇔ x = cos y
arc cos : [−11
 2 
 π π
arc tan :  →  − ,  , definida por y = arc tan x ⇔ x = tan y
4.1 Período  2 2
Tipo de função Período
sen( ax + b), cos( ax + b), 2π Obs.: de forma similar, define-se arc cot, arc sec e arc csc com contra-
sec( ax + b), csc( ax + b) a π  π π
domínios (0, π), [0, π] −   , e  − ,  − {0} respectivamente.
2  2 2
π
tan( ax + b), cot( ax + b)
a

IME-ITA 231
Matemática IV – Assunto 1

EXERCÍCIOS RESOLVIDOS π 7π 7π
03 (EFOMM-96) Sabendo que A = 6 tg + 4 sen − cos , então o
1 1 6 3 6
07 Calcule x = arctan + arctan . valor de A é igual a:
2 3
Solução: 3 4 12 6
(A) (D) 12
2
 1 4
α = arctan 2 3
Defina  (na grande maioria dos problemas que (B) 3
(E)
β = arctan 1 2
 3 4
3
envolvem funções inversas trigonométricas, isso é uma boa (C)
2

2
ideia!). Essas definições nos dão as seguintes informações: 04 (AFA-2000) Simplificando a expressão (cossec x ) − 2 , para
 1 cossec x ≠ 0, obtemos: (cossec x )2
 tan α = 2  π π
 e α,β ∈  − ,  .
 tan β = 1  2 2 (A) cos x
 3 (B) cos2 x
(C) sen2 x
tan α + tan β (D) cos 2x
Então, x = α + β ⇒ tan x = tan ( α + β ) = ⇒
1 − tan α tan β
1 1 05 (AFA-2000) O acesso ao mezanino de uma construção deve ser
+ feito por uma rampa plana, com 2 m de comprimento. O ângulo a
⇒ tan x = 2 3 = 1.
1 1 que essa rampa faz com o piso inferior (conforme figura) para que
1− ⋅ nela sejam construídos 8 degraus, cada um com 21,6 cm de altura, é,
2 3
aproximadamente, igual a:
 π π
Como α,β ∈  − , , temos que x = α + β ∈ (–π, π). Como tan x
 2 2
π 2m
tan x = 1, temos que x = .
4

Definições, relações básicas e adição de arcos


(A) 15O
EXERCÍCIOS NÍVEL 1 (B) 30O
(C) 45O
(D) 60O
01 (EFOMM-1999) A soma das raízes da equação 4 · cos2q = 1 é:
(0 < q < p)
06 A soma de dois arcos é 400º. Calcule seus arcos sabendo que seus
π cossenos são números simétricos.
(A) p. (D) .
7
07 Simplificar as expressões:
3 π
(B) . (E) π.
2 2 (A) cos (90° + a) · cos(180° – a) + sen (180° + a) · (90° + a);
π
(C) .
(B) sen(360° + a) + cos a · cos(90° – a) + sen(90° – a) · sen(360° – a);
4 sen( − a) tan(90º + a) cos a
(C) − + .
sen(180º + a) cot a sen(90º + a)
02 (EN-2000) Em um triângulo retângulo, a hipotenusa é o triplo de um
cos( 90º + a) ⋅ sec( − a) ⋅ tan(180º − a)
dos catetos. Considerando o ângulo oposto ao menor lado, podemos (D) .
afirmar que tan α + cos α é igual a: sec( 360º + a) ⋅ sen(180º + a) ⋅ cot(90º −a)
sen( 270º − a) ⋅ tan(180º − b) cot( 450º − a) ⋅ sen( c − 90º )
5 (E) +
(A) ⋅ 11 2 cot( b − 270º ) ⋅ cos( 540º + a) cos(180º + c) ⋅ tan(1260º + a)
6 (D) ⋅
4
11 2 15π
(B) ⋅ (E) 12 + 2 ⋅ 08 Calcule para x = o valor de:
12 4 4
(C) 2 ⋅ (A) 3sec2 x + sen2 x – 2tan x+ cos2 x
(B) 2sen2 x – 2tan x + sec2 x

232 Vol. 1
Trigonometria

09 Se sen x + sen2 x = 1, calcule E = cos2 x + cos4 x.  π 2


03 ( ENEM-2001) Se x ∈  0,  e cos2 x − sen2 = , o valor de
 4  5
10 (EFOMM-02) O resultado da simplificação da expressão
tg x ⋅ cotg x é: cos2 x + 4 sen2 x + 5 sen x cos x é:
sec2 x −
cossec 2 x − 1
19 + 5 21
(A) 13 + 21 (C)
(A) sen x. (D) 1.
(B) cos x. (E) 0. 10
(C) –1.
(B) 17 + 3 21 (D) 21 + 2 21
10 3
11 Verifique as identidades:
04 Se a + b = 135º, mostre que (1 + cot a) (1 + cot b) = 2.
(A) sen6 x + cos6 x – 2sen4 x – cos4 x + sen2 x = 0
(B) sec4 x – sec2 x = tan2 x + tan4 x a b−a π
05 Se tan x = e tan y = , mostre que x + y = k π + , k ∈ Ζ
1 − 2 cos2 x b b+ a 4
(C) = tan x − cot x
sen x ⋅ cos x 
sec x − cos x  tan x + tan y = tan a
(D) = tan3 x 
csc x − sen x 06 Elimine x e y nas equações: cot x + cot y = cot b
12 Mostre que se tan2 a = 1 + 2tan2 b, então, cos2 b = 2cos2 a.  π
x + y =
 4
13 Se sen x . cos x = m, determine em função de m:
sen x + cos x  π
07 Demonstrar que: = tan  x +  .
(A) y = sen x + cos x cos x − sen x  4
(B) y = sen4 x + cos4 x
08 Dados os arcos Â, B , C^ e D^ todos do primeiro quadrante, e tais
sec x + csc x 1  = 1/5, tan C^ = 1/7 e tan D^ = 1/8, verificar
que tan  = 1/3, tan B
14 Determine o valor de sabendo que senx = .  + C + D = π/4.
^ ^
1 + tan x 5 se  + B
3
15 Determine sen x e tan x sabendo que cos x = − e x ∈ 3° Q.
5 09 (ITA-77) Considere um triângulo ABC cujos ângulos internos A, B, C
16 (EFOMM-1998) Resolvendo sen 15° – sen 75°, encontra-se:  B+C
verificam a relação sen A = tan   . Então, podemos afirmar que:
 2 
(A) Com os dados do problema, não podemos determinar A nem B nem C.
(A) − 3 . (D)
2.
(B) Um desses ângulos é reto.
2 π 5π
(C) A = , B + C =
2 3. 6 6
(B) . (E) π 5π 5π
2 2 (D) A = , B = , C =
6 12 12
(E) n.d.a.
(C) − 2 .
2 π π
10 (ITA-79) Se a e b são ângulos complementares, 0 < a < , 0 < b <
EXERCÍCIOS NÍVEL 2 sen a + sen b  3a  2 2
e = 3 , então sen   + cos ( 3 b ) é igual a:
01 (EFOMM-2000) As raízes da equação 2x2 + 3x – 1 = 0 são tg B e sen a − sen b  5 
tg C, sendo B e C ângulos de um triângulo. O ângulo A desse triângulo vale:
(A) 3 (D) 2
(A) 30°. (D) 90°.
2
(B) 45°. (E) 120°. (B) 3 (E) 1
(C) 60°. 3
(C) 2
02 (AFA-1998) O valor da expressão cos 35º (sen 25° + cos 55º) +
sen 35º (cos 25º – sen 55º) + tg 31 + tg 14 é:
o o
 π
1 − tg 31o ⋅ tg 14 o 11 (ITA-81) Seja f :  0,  → ℜ definida por f ( x ) = sec2 x + csc2 x .
 2
 π a
(A) 2 − 3 Se α ∈  0,  é tal que tanα = , então f( α) é igual a:
2  2  b

(B) 3 + 2 a+ b 2 2
2 (A) (D) a + b
2 ab
1 2
(C) 2 + 3 (B) a + b2 (E) nenhuma das anteriores
2 2
2 2
(C) a − b
(D) 2 − 3 ab
3

IME-ITA 233
Matemática IV – Assunto 1

b π
12 Ache a relação que deve existir entre os arcos x e y para que tan2 x = 02 (EFOMM-94) Sendo 0 < x < e sen x = 3 sen 2 x , então tg 2x vale:
a 2
a+ b
e sec2 y = .
a (A) 0 (D) 6
a b (B) 1 (E) 35
13 Se = , calcule, em função de a e b, o valor de sen x . cos x.
sen x cos x (C) p
14 Determinar a condição que deve ser imposta a b para que seja possível
03 (EFOMM-01) O valor de ( tg 10° + cot g 10°) sen 20° é:
 tan x + tan y = 2
o sistema 
2 2
sec x + sec y = b (A) 0,5 (D) 2,5
(B) 1 (E) 4
15 Calcule tan x em 7cos2 x + 3sen x . cos x – sen2 x = 5.
(C) 2
16 Determine k de modo que os ângulos agudos de um triângulo retângulo
04 (EFOMM-1995) Se sen2a = x e sen2b = y, então sen(a + b) cos(a–
sejam raízes da equação 3 tan x + k2 cot x = 4k e calcule os dois ângulos
b) é igual a:
agudos.
(A) x + y (D)
x² + y²
sen x ⋅ cos y = a

17 Eliminar x e y entre as equações: sen x ⋅ sen y = b (B) 2(x + y)
cos x = c

x+y
(C) x – y (E)
 a sen2 x + b cos2 x = m 2

18 Eliminar x e y entre as equações:  b sen2 y + a cos2 y = n
 a tan x = b tan y  π 1
 05 (ITA-99) Se x ∈ 0,  é tal que 4 tan4 x = + 4 , então o valor
 2  cos4 x
19 Sendo tan a e tan b as raízes da equação x2 + px + q = 0, calcule o de sen(2x) + sen(4x) é:
valor da expressão:
15 1
(A) (D)
E = sen2(a + b) + p . sen(a + b) cos(a + b) + q . cos2(a + b) 4 2
15
tan 1° ⋅ tan 2° ⋅ tan 3° ⋅  ⋅ tan 89° (B) (E) 1
20 Simplifique: . 8
cos 4° + cos 8° + cos 12° +  + cos 356°
3 15
π (C)
21 (ITA) Seja a real com 0 < a < . 8
2
  3π   3π  π  π 3
A expressão sen  + a  + sen  − a   ⋅ sen  − a  é idêntica a: 06 (EFOMM-1999) Sabendo que < θ < π e que sen θ = , o valor
2 5
  4   4  2 
2 cot 2 a
(A) (D) 1 + 3 cot a π 
de cos  + θ  –− sen ((p
π −– 20
2θ)) é igual a:
1 + cot 2 a 2 2 

2 cot a 1 + 2 cot a (A) 9 . (D) 4 + 5 .


(B) (E) 25 25
2 1 + cot a
1 + cot a
39
(B) − . (E) 3 − 2 .
2 25 9
(C)
1 + cot 2 a
(C) 2 − 2 .

Transformações trigonométricas 07 (AFA-1999) O valor da expressão cos 15O + sen 105O é


EXERCÍCIOS NÍVEL 1
(A) 6+ 2
1 4
01 (EFOMM-96) Sabendo que sen x = + cos x, então sen 2x vale:
5
(B) 6− 2
4
25 −2 6+ 2
(A) (D) (C)
24 5 2
9 24 6− 2
(B) (E) (D)
25 25 2
−9
(C)
25

234 Vol. 1
Trigonometria

13π 11π 27 3
08 (EFOMM-01) O valor numérico de y = sen ⋅ cos é: 16 Resolver a equação: cos 4 x = 7 cos4 x − cos2 x + .
12 12 4 4

(A)
1
8
(
1+ 3 1+ 3 )( ) 17 Dada a equação: 3sen 2x + cos 2x = 1, calcule tan x.

18 (EN -1999) Coloque (F) falso ou (V) verdadeiro nas proposições abaixo
1
(
(B) −1 + 3 1 + 3
8
)( ) e assinale a opção correta.
1
(
(C) −1 + 3 3 − 3
8
)( ) I. (1 + cot x ) (1− cos x ) = 1 ∀x ∈ , x ≠ kπ , k ∈ 
2 2

2 π
(D) tg π
11 II. (1 + sec 4 x ) = 2 sec2 x + tan4 x, ∀x ∈ , x ≠ 2
+ kπ , k ∈ 
π
(E) 13π 11π 1
12 III. sen cos =
12 12 4
09 (ITA -2001) Sendo α e β os ângulos agudos de um triângulo retângulo,
e sabendo que sen2 (2β) – 2 cos(2β) = 0, então sena é igual a:
(A) F – V – V
2 4
8 (B) F – F – V
(A) . (D) . (C) V – V – F
2 4
(D) V – V – V
4
2 (E) V – F – V
(B) . (E) zero.
2
4 EXERCÍCIOS NÍVEL 2
(C) 8 .
2  x
01 Mostre a expressão E ( x ) = cos x ⋅  1 + tan x ⋅ tan  tem valor
 π constante, independente do arco x.  2 
10 (ITA-87) O valor de x > 0 que satisfaz a equação x = tan  
é:  12 
02 Se a + b = 45º, mostre que tan a + tan a . tan b + tan b = 1.
(A) x = 4 3 . (D) x = 7 − 4 3.
3 + cos 4 x
(B) x = 5 − 4 3 . (E)
x = 9 − 4 3. 03 Demonstrar a identidade tan2 x + cot 2 x = 2 ⋅ .
1 − cos 4 x
(C) x = 7 − 3 .
04 (EFOMM-00) A função cos a + cos 3a + cos 5a + cos 9a equivale a:
 tan( x − y ) = 3
11 (ITA-87) Suponha x e y números reais, tais que  . (A) 4cos 4a cos 3a cos 2a (D) 3sen a cos 2a sen 2a
Calcule o módulo do número S = tan x + tan y.  tan x ⋅ tan y = 1 (B) 3cos 3a cos 2a cos a (E) 2sen 3a cos a
(C) 2cos 2a 2cos a
≠ 3≠
12 Calcule tan e tan .
8 8 x 7 π
05 Calcule tan se sen x + cos x = e0< x < .
13 Prove que para todo arco x cada uma das relações abaixo é verdadeira: 2 2 6
06 Sendo sen x + sen y = a e cos x + cos y = b, calcule sen(x + y).
 2π   4π 
(A) sen x + sen  x +  + sen  x + =0
 3   3  A sen B
 + senC
07 Demonstrar que, em um triângulo ABC, cot =
 + cos C
2 cos B
 2π   4π 
(B) cos x + cos  x +  + cos  x + =0
 3   3  08 Os ângulos agudos de um ∆ABC, verificam a relação 2.
tan A = tan B + tan C, mostre que:
14 Calcule o valor da expressão: E = tan 20° + tan 40° + 3 tan 20° · tan40°.
(A) tan B · tan C = 3.
5π π
15 Sendo y = sen cos , o valor numérico de y é: (B) cos A = 2 · cos B · cos C.
12 12
09 Sendo a + b + c = 180°, calcular:
1 3 cos( a − b) cos( a − c) cos( b − c)
(A) + 3 +2
(D) y= + +
2 4 sen a sen b sen a sen c sen b sen c

3 10 Sejam A, B, C os ângulos de um triângulo. Mostre que sen 2A + sen


(B) 2( 3 + 1)
(E) 2B + sen 2C = 4 sen A sen B sen C.
2

1
(C)
2

IME-ITA 235
Matemática IV – Assunto 1

11 (EN-1997) Sabendo-se que tan x = a, tan y = b, pode-se reescrever 21 Verifique as igualdades:


sen( 2 x ) + sen( 2 y )
z= como:
sen( 2 x ) − sen( 2 y ) (A) tan 50º + cot 50º = 2 · sec10º.
π 3π 3
(B) cos4 + cos4 = .
8 8 4
 1 − ab   a − b 
(A)  ⋅  (C) tan 50º – tan 40º = 2 · tan 10º.
 1 + ab   a + b 
π 3π 7π 9π
 1 + ab   a + b  22 Calcule o valor da expressão: E = tan − tan − tan + tan .
(B)  ⋅  20 20 20 20
 1 − ab   a − b 
 1 − ab   a + b  π 5π 7π 11π
23 Determine o valor de P = sen sen sen sen .
(C)  ⋅  24 24 24 24
 1 + ab   a − b 
24 (ITA-2002) Se x, y e z são os ângulos internos de um triângulo ABC
 1 + ab   a − b 
(D)  ⋅  e sen x = sen y + sen z ,
 1 − ab   a + b  prove que o triângulo ABC é retângulo.
cos y + cos z
12 (2a Lei dos Cossenos) Sendo A, B e C ângulos de um ∆ABC, mostre que:
Equações e inequações trigonométricas
cos2 A + cos2 B + cos2 C = 1 – 2cos A · cos B · cos C.
EXERCÍCIOS NÍVEL 1
13 (ITA-2003) Para todo x real, a expressão cos2 ( 2 x ) ⋅ sen2 ( 2 x ) ⋅ sen x
01 (EFOMM-95) Se x ∈ [0,2p], o número de soluções da equação
é igual a:
2sen3x – senx + 1 = cos2x é igual a:
(A) 2−4 sen( 2 x ) + sen(5 x ) + sen( 7 x ) (A) 1. (D) 4.
(B) 2−4 2 sen( x ) + sen(7 x ) − sen( 9 x ) (B) 2. (E) 6.
(C) 3.
(C) 2−4  − sen( 2 x ) − sen(3 x ) + sen( 7 x )
(D) 2−4  − sen x + 2 sen(5 x ) − sen(9 x ) 02 (AFA-2000) Se (senx, sen2x, cosx) é uma progressão geométrica
estritamente crescente, com 0 < x < 2p, então o valor de x é:
(E) 2−4 sen x + 2 sen(3 x ) + sen( 5 x )
≠ ≠
A 5  20 (A) (C)
C B 12 8
14 Em um ∆ABC, calcule , sabendo que tan = = e tan = .
2 2 6 2 37
≠ ≠
(B) (D)
15 Mostre que: se em um triângulo ABC vale a relação: 10 6
cos( B − C) 03 ( A FA - 1 9 9 8 ) O c o n j u n t o s o l u ç ã o , e m ℜ , d a e q u a ç ã o
= tan B, então o triângulo é retângulo com ângulo reto (cos x)(sen 2x) = (sen x)(1 + cos 2x), é:
sen A + sen( C − B)
em A. (A) ∅
(B) ℜ
16 Em um triângulo ABC, vale a relação 9BC2 + 9CA2 – 19AB2 = 0. Qual o
(C) {x ∈ ℜ l x = 2k p ± p /2, k ∈ Z}.
cot C
valor de cot A + cot B ? (D) {x ∈ ℜ l x = 2k p ± p /3, k ∈ Z}.

04 Resolva as equações abaixo:


17 Sejam  o lado de um polígono regular de n lados, r e R,
respectivamente, os raios dos círculos inscrito e circunscrito a este (A) sen 2x = sen 5x. (F) cos 2x + cos 8x = 0.
 π. (B) sen 3x = cos 4x. (G) tan 5x = tan 2x.
polígono. Prove que r + R = cot
2 2n (C) sen 4x + sen x = 0. (H) tan 5x + tan x = 0.
18 Mostre que, se os ângulos de um triângulo ABC verificam a igualdade (D) cos 5x = cos 7x. (I) tan x · tan 3x = 1.
sen4A + sen4B + sen4C = 0, então o triângulo é retângulo. (E) cos 3x + sen 5x = 0.

05 Ache as expressões gerais das medidas algébricas de todos os arcos


que verificam as igualdades:
 tan x + tan y = 6
2 2


19 Resolva o sistema  tan x tan y (A) 2sen2 x + 3sen x – 2 = 0
 tan y + tan x = −6 (B) tan4 x – 4tan2 x + 3 = 0

20 27 3
π π 06 Resolver a equação: cos 4 x = 7 cos4 x − cos2 x + .
(A) Prove que: tan( − x ) ⋅ tan x ⋅ tan( + x ) = tan 3 x . 4 4
3 3
07 Resolva:
(B) Usando (a), mostre que tan 20° · tan 30° · tan 40° = tan 10°.
1 1 1 1 1 1
− − − − − = −3
sen² x cos² x tan² x cot ² x sec² x csc² x

236 Vol. 1
Trigonometria

08 (EFOMM-01) O conjunto solução da equação sen x + cos x = 1 é:  π


15 (ITA- 2000) Para x no intervalo 0,  , o conjunto de todas as soluções
 2
(A) S = { x ∈ ℜ / x = π + 2 hπ , h ∈Z}
 π
da inequação sen( 2 x ) − sen  3 x +  > 0 é o intervalo definido por:
(B) S = { x ∈ ℜ / x = hπ ou x = π + 2 hπ , h ∈Z}  2
π π
(A) <x<
 π  10 2
(C) S =  x ∈ ℜ / x = 2 hπ ou x = + 2 hπ , h ∈ Z 
 2  π π
(B) <x<
(D) S = { x ∈ ℜ / x = 2 hπ , h ∈ Z} 12 4
π π
 3π  (C) <x<
(E) S =  x ∈ ℜ / x = + 2 hπ , h ∈ Z  6 3
 2 
π π
3  π (D) <x<
09 (AFA -2003) Dado que sen x + cos x = , tem-se que cos  x −  4 2
vale: 3  4
π π
(E) <x<
4 3
−1 + 3 6
(A) (C) EXERCÍCIOS NÍVEL 2
2 3

2 6 01 (ITA-87) O número de raízes reais da equação sen2x + sen4x + sen6x


(B) − (D) + sen8x + sen10x = 5 é:
3 6
10 (AFA-02) O conjunto dos valores reais de x que tornam verdadeira a (A) um número maior do que 12.
desigualdade cos2 (x – p) ≥ p é: (B) zero.
(C) 2.
(A) { x ∈ ℜ / x ≤ – π ou x ≥ π } (D) 10.
(B) { x ∈ ℜ / – π ≤ x ≤ π } (E) 1.
(C) ℜ
(D) ∅  1
02 (EN-2003) O número de soluções reais da equação sen  = x −2
é igual a n; assim, pode-se concluir que:  x
1 2
11 (AFA-1998) O conjunto solução da inequação ″ sen x · cos x< ,
4 2
para 0 ≤ x ≤ p, é: (A) n = 0
(B) n = 1
π π 5π 5π (C) n = 2
(A) { x ∈ ℜ | ≤ x < }. (C) { x ∈ ℜ | ≤x≤ }.
12 6 12 6 (D) n =3
(E) n > 3
π 5π π 5π
(B) { x ∈ ℜ | <x< }. (D) { x ∈ ℜ | ≤x≤ }.
12 3 12 12 03 (ITA-88) Sobre a equação tan x + cot x = 2 sen( 6 x ), podemos afirmar que:
12 Resolva no intervalo (0, 2), a desigualdade 2 sen²x – 3senx + 1 <0.
π
(A) apresenta uma raiz no intervalo 0 < x <
1 4
13 Resolva: senx + cos x < .
π
cos x (B) apresenta duas raízes no intervalo 0 < x <
3 sec x − cos x 2
14 (EN-1997)Se x ∈ 0, 2π  , oconjuntosoluçãode ≤ <1
é: 9 csc x − sen x π
(C) apresenta uma raiz no intervalo < x <π
2
  π π   7π 4π   3π
(A)  x ∈  : x ∈  ,  ∪  ,  (D) apresenta uma raiz no intervalo π < x <
  6 3   6 3   2
(E) não apresenta raízes reais.
  π π   5π 4π  
(B)  x ∈  : x ∈  ,  ∪  , 
 4 3   4 3   1
 04 (ITA-88) Seja a equação sen3 x ⋅ cos x − sen x ⋅ cos3 x = , onde m
é um número real não nulo. Podemos afirmar que: m
  π π   7π 5π  
(C)  x ∈  : x ∈  ,  ∪  ,   (A) A equação admite solução qualquer que seja m não nulo.
  6 4   6 4  
(B) Se| m|< 4, esta equação não apresenta solução real.
(C) Se m > 1, esta equação não apresenta solução real.
  π π   5π 4π   (D) Se| m|> 2, esta equação sempre apresenta solução real.
(D)  x ∈  : x ∈  ,  ∪  , 
  4 3   4 3   (E) Se m < 4, esta equação não apresenta solução real.

IME-ITA 237
Matemática IV – Assunto 1

05 (AFA-2000) Os valores de m ∈ ℜ para os quais a equação Funções trigonométricas


2 (sen x – cos x) = m2 – 2 admite soluções, são:
EXERCÍCIOS NÍVEL 1
(A) –1 ≤ m ≤ 1.
(B) –2 ≤ m ≤ 2. 01 Calcule arcsen(sen150°).
(C) 0 ≤ m 2 .
(D) – 2 ≤ m ≤ 2 . 02 Calcule:

06 (EN-1999) O produto das soluções da equação 2sen3x + 5cos2x + 1


(A) y = arcsen .
 π 5π  2
4senx + 2tan2x = 4 + 2sec2x no intervalo  ,  é:
 12 6   
(B) y = arctan 3 + arccos  − 3  .
2  2 
2
5π π  
(A) (D)  1
12 6 (C) y = cot  arcsen  .
 3
2
π3 π (D) y = arcsen x + arccos x.
(B) (E)
12 12
03 (EN) Seja x = arccos 3 , x ∈[0, ]. Então, sen(2x) é igual a:
5π3 5
(C)
72
(A) 24 (D) 6
25 5
07 ( I T A - 8 8 ) A r e s p e i t o d a s o l u ç ã o d a e q u a ç ã o
sen x + 3 cos x = 2, 0 ≤ x < 2π , podemos afirmar que: (B) 4 (E) 2
5 5
(A) existe apenas uma solução no primeiro quadrante. (C) 16
(B) existe apenas uma solução no segundo quadrante. 25
(C) existe apenas uma solução no terceiro quadrante.
(D) existe apenas uma solução no quarto quadrante. 04 (AFA) O valor de cotg arcsen 2 2 é
(E) existem duas soluções no intervalo. 3

08 Resolva a equação: 4 · sen2x + 3 · cos2x = 3. 2 . (C) 2.


(A)
2 4
09 Resolva a equação: 5sen2 x – 3sen x cos x + 4cos2 x = 3.
(B) 2 2 . (D) 3 2 .
10. Resolva a equação cos 3x – 2cos 2x + 1 = 0. 4
1
11. Resolva a equação: 2(sen x – cos x) + 2 sen x cos x = 1. 05 (AFA-2000) O valor de sen arc cos + arc sen 1 é:
2 3

12 Resolva a inequação: 4 sen² x − 2 (1 + 2 ) senx + 2 < 0 (A) 2 2 − 1 . (C)


2 3 −1 .
2 3
2sen2 x + cos x − 1 (B) 2 6 − 1 . (D)
−2 6 − 1 .
13 Resolva no intervalo [0,2π]; ≥0
senx − cos x − 2 6 6
14
06 (AFA-1999) O valor real que satisfaz a equação arcsenx + arcsen2x
cos 2 x + cos x − 1
(A) Resolva a seguinte desigualdade: ≥ 2 para 0 ≤ x ≤ π; = p/2, para x pertencente ao intervalo (0,1), é:
cos 2 x
1 1
(A) . (C) .
(B) Resolva a inequação 2 cos x + 2 sen x + 2 < 0. 5 2
cos x − sen x
(B) 5 . (D) 2.
15 (AFA-2000) Os valores de α, 0 ≤ α < 2p, que satisfazem a 5 2
1
desigualdade – x2 + <senα, para todo x real, pertencem ao intervalo: 1− x 2
07 (EN-2002) Seja f(x) = definida nos reais e seja g(x) = tan x
2 1+ x 2
 π π
π 5π definido no intervalo aberto  − ,  . Se x ∈ ]– p, p[, então o valor da
(A) 0 < α < (C) <α<π  2 2
2 6
função composta no número x/2 é igual a:
π π 5
(B) 0 < α < (D) <α< π
6 6 6 (A) cos (2x) (C) sen x
(B) tan x (D) cos x

238 Vol. 1
Trigonometria

08 (EN-2002) Sejam A, B e C os pontos de interseção da curva com os 05 (ITA) Sendo z = cos (arctan(a2 + b2) + arccot (a2 + b2)), podemos
eixos coordenados conforme a figura abaixo, em que k e w são constantes afirmar que:
reais.
y (A) z = 0
(B) z = 1
B y = k cos(wx) 3
(C) z =
A C x 2
(D) cos (a2 + b2), se a2 + b2 ≤ 1
Supondo que o triângulo de vértices A, B e C tem 3p unidades de área e (E) é impossível determinar o valor de z
que k + w – 14 = 0, o valor de (k – w) é:
06 (ITA) Seja k uma constante real e considere a equação em x: arcsen
(A) – 14. (C) 10.  1+ x2 
(B) – 10. (D) 12.   = k, x ≠ 0. Podemos afirmar que:
 2x 
 
09 (ITA-80) Sobre a função f(x) = sen2 x, podemos afirmar que:
(A) Para cada k real, a equação admite uma única solução.
(A) é uma função periódica de período 4p. (B) Para cada k real, a equação admite duas soluções.
(B) é uma função periódica de período 2p. (C) Existe k real tal que a equação admite uma infinidade de soluções.
(C) é uma função periódica de período p. (D) Não existe k real tal que a equação admita solução.
(D) é uma função periódica onde o período pertence ao intervalo (p, 2p). (E) Existe k real tal que a equação admite uma única solução.
(E) não é uma função periódica.
07 (ITA) Seja f(t) = 4 + 3cos(pt) + 4sen(pt) a função definida nos reais.
10 Determine o período das seguintes funções trigonométricas: Sobre esta função das alternativas abaixo é correta?
x (A) f(t) é função par.
(A) y = sen . (D) y = sen2 x.
3 (B) f(t) é função ímpar.
2x π (C) o maior valor que f(t) assume é 9.
(B) y = 3 tan( − ). (E) y = tan2 x.
3 4 (D) o maior valor que f(t) assume é – 3.
x
(C) y = 4 – 3 sec(–πx). (F) y = cos3 . (E) o maior valor que f(t) assume é – 1/2.
2
11 (ITA-88) O conjunto imagem da função f: [0,1] → [0, p], f(x) = 08 (AFA) O gráfico que melhor representa a função y = |sen x + cos
3x − 1 x|, com 0 ≤ x < 2p, é:
arccos é:
2
y
(A) (C) y
 π 2π   2π  2 2
(A) 0, ,  (D)
0, 3 
 4 3   
 π 1
(B) [0, p] (E) 0, 2  1
 
 π 3π  x x
(C)  , 
4 4  p 2p p 2p

(B) y (D) y
EXERCÍCIOS NÍVEL 2
2 2
1  1− x 
01 Determine x na equação arctan x = arctan  .
2  1+ x  1 1
1 1
02 Resolva a equação: arctan − arctan = arctan a.
x −1 x +1 x x
p 2p p 2p
arcsen xy − arcsen 1 − xy = π / 6
03 Resolva o sistema 
arctan 2 x + arctan 2 y = arctan 2
09 (ITA-81) Seja g uma função não nula dos reais nos reais que
x π satisfaz, para todo x e y, g(x + y) = g(x) + g(y). Se f real for definida por
04 (ITA) A solução da equação arctanx + arctan = definida no
x +1 4
conjunto dos reais diferentes de – 1 é: f(x) = sen  2g( x )  , a ≠ 0, então podemos garantir que:
 a 
(A) 1. (A) f é periódica com período pa.
(B) ½. (B) Para a = n, n natural, temos f(n) = 2sen(g(1)).
(C) ½ e 1. (C) Se g(1) ≠ 0, então g(1) = f(0).
(D) 2. (D) Se g(T) = pa, então T é período de f.
(E) 2 e 1. (E) g(T) = 2p, então T é período de f.

IME-ITA 239
Matemática IV – Assunto 1

10 Determine o período e a amplitude da função: y = 12senx – 5 cos x.  π π


16 (ITA) Encontre todos os valores de a ∈  − ,  para os quais a
equação na variável real x:  2 2
11 Seja ƒ(x) = 5sen2 x + 3senx cos x + 7cos2 x, ƒ:  → . Determine
o conjunto-imagem da função ƒ.
 ex   ex 
12 (ITA) Seja a ∈ , 0 < a < 1 e f uma função real de variável real arctan  2 − 1 +  + arctan  2 − 1−  = a admite solução.
 2   2 
1  

definida por f ( x ) =
( 2
a x − a2 ) 2

. EXERCÍCIOS NÍVEL 3
cos ( 2πx ) + 4 cos( πx ) + 3
Sobre o domínio A desta função podemos afirmar que: 01 (ITA) Dados A, B e C ângulos internos de um triângulo, tais que 2B + C ≠ p
 α−C
(A) (–∞, – 2) ∩  ⊂ A. (D) {x ∈  / x ∉ , x ≥ 2} ⊂ A.  4 π 5π   5π  sen A + sen B = sen  
e α ∈  ,  ∪  , 2π , o sistema   2 
(B) A = [– 2, 2] ∩ . (E) A ⊂ [– 2, 2]  3 3   3  − cos A + cos B = cos  α − C 
(C) (– 2, 2) ⊂ A.   2 
 
admite como solução:
13 (AFA) Analise as alternativas seguintes e classifique-as como
verdadeiras (V) ou falsas (F). α α 2π 2π
(A) A = π − ,B = − eC=
2 2 3 3
( ) O período e o conjunto-imagem da função f: → definida por
1  1 1 α α
f(x) = senx · cosx são, respectivamente, 2p e  − ,  . (B) A = π − ,B = e C = 0
4 2 2
 4 4
( ) A função y = 2 arccos4x tem por domínio o conjunto de todos os 2π α π α
(C) A = ,B = e C = −
 1 3 2 3 2
valores de x pertencentes a 0, 4,  .
  α 2π α 2π
(D) A = π − ,B = eC= −
 π π 2 3 2 3
( ) Para todo x ∈  − ,  , o valor de (tg2x + 1) · (sen2x – 1) é – 1.
 2 2 α α
A opção que corresponde à classificação anterior é: (E) A = π, B = eC=−
2 2

(A) F – V – F. (C) F – F – V. 02 (ITA) Seja a uma constante real. Eliminando q das equações abaixo:
(B) V – V – F. (D) V – F – V.
 x ⋅ sen θ + y ⋅ sen θ = 2 a ⋅ sen 2θ
 , obtemos:
14 (EN) A função que melhor se adapta ao gráfico abaixo é:  x ⋅ cos θ − y ⋅ sen θ = a ⋅ cos 2θ
y
2 2 2
3 (A) ( x + y ) 3 + ( x − y ) 3 = 2 a 3
2 2 2
(B) ( x + y ) 3 − ( x − y ) 3 = 2 a 3
2
2 2 2
(C) ( x + y ) 3 + ( x − y ) 3 = a 3
2 2
x a3
(D) ( x + y ) 3 + ( x − y ) 3 =
– 3p –p –p p p 3p 2
4 2 4 4 2 4
(E) n.d.a.
x x
(A) y + sen = 3. (D) y + sen = 3 – 2/2. cos( 2 x )
2 2 03 (AFA) Considere a função real definida por y = e as
1 + sen( 2 x )
x seguintes afirmações:
(B) y + sen = 3 + 2/2. (E)
y + |sen2x| = 3.
2
I. A função é decrescente em todo seu domínio
(C) y + |cos2x| = 4. π
II. O gráfico da função apresenta assíntotas em arccos + k π, k ∈ 
2
15 (ITA) Considere os contradomínios das funções arco seno e arco  π
III. A função é negativa em 0, 
 π π  4
cosseno como sendo  − ,  e [0,p], respectivamente. Com respeito
 2 2  π
à função f: [–1,1] → f(x) = acrsen x + arccos x, temos que: IV. A função admite inversa em 0, 
 2
(A) é não crescente e ímpar. (D) é injetora. São verdadeiras somente as afirmações contidas nos itens:
(B) não é par nem ímpar. (E) é constante.
(A) I e II. (C) III e IV.
(C) é sobrejetora.
(B) II e III. (D) I e IV.

240 Vol. 1
Trigonometria

π 
04 Resolva a equação 2sen2  cos2 x  = 1 − cos ( πsen2 x ) 13 Determine os valores de x e y que satisfazem as equações:
2 
x + y = π / 5

05 Calcule: sen 10° · sen 50° · sen 70°.  2 2 π.
sen x + sen y = 1 − cos 5
06 Achar os valores de x que satisfazem a equação:
π2 − 4 x 2 = arcsen(cos x ). 14 Sejam a, b e g ângulos internos de um triângulo oposto aos lados a,

k≠ b, c respectivamente.
07 Prove que os valores da expressão sen , k inteiro, são todos
diferentes. 7 α a
a. Mostre que sen ≤ .
2 b+c
08 α β γ 1
n
sen 2n a b. Mostre que sen ⋅ sen ⋅ sen ≤ .
(A) Prove que: ∏ cos 2
k −1
a= n . 2 2 2 8
k =1 2 sen a
15 Sejam a, b e g ângulos internos de um triângulo.
(B) Usando(a),mostre que 1 ⋅ 1 + 1 ⋅ 1 ⋅ 1 + 1 ⋅ 1 + 1 1  = 2
2 2 2 2 2 2 2 2 2 π α β γ
a. Mostre que cos α + cos β + cos γ = 1 + 4 sen ⋅ sen ⋅ sen
2 2 2
 π 2
09 Dada a equação cos  2 x +  − msen x = 0 determine a condição 3
 6 b. Mostre que cos α + cos β + cos γ ≤
que deve satisfazer m para que ela tenha pelo menos uma solução xo tal 2
que 0 < xo < 2π. 16 Resolva sen 18x + sen 10x + sen 2x = 3 + cos2 2x.
1 1 1 Cˆ
10 Calcule 
sen 1 ⋅ sen 2 
+ 
sen 2 ⋅ sen 3
+ ... +
sen n ⋅ sen( n + 1)
 =
17 Prove que um triângulo satisfaz a + b tan
2
( )
a tan Aˆ + b tan Bˆ é
isósceles.
11
(A) Mostre que é possível expressar tan 3α em função de tan α = x. 2π 3π π
18 Mostre que csc + csc = csc .
(B) Utilize o item anterior para determinar as soluções da equação x3 – 7 7 7
3mx2 – 3x + m = 0 onde m é um número real dado. 19 Mostre que cos1° é irracional.
12
(A) Resolva a equação m cos x – (m + 1)sen x = m, m ∈ ∗; 1+ xn
20 Considere uma sequência definida por x0 = 2014 e x n +1 =
(B) Determine m de modo que essa equação admita as raízes x’ e x” cuja para n > 0. Calcule x2014. 1− xn
diferença seja π/2.

RASCUNHO

IME-ITA 241
Matemática IV – Assunto 1

RASCUNHO

242 Vol. 1
Introdução à geometria plana euclidiana A ssunto
1
Matemática V

1. Conceitos primitivos e axiomas


Na geometria euclidiana, trabalha-se com noções de elemento e
conjunto, embora denotemos por alguns nomes diferentes. Os conceitos
primitivos são os objetos com os quais iremos trabalhar, mas não
definiremos formalmente. São eles:
• o ponto, um objeto adimensional, e que deve ser lidado como um
‘elemento’.
• a reta, um objeto de dimensão 1, que é um ‘conjunto’ de pontos.
• o plano, um objeto de dimensão 2, também um ‘conjunto’ de pontos.

A partir daqui, começaremos o estudo da geometria plana. Todos os


objetos estarão contidos em um mesmo plano, ou seja, serão coplanares.

EXERCÍCIOS RESOLVIDOS

01. Prove que existem infinitas retas.

Solução:
Como existem infinitos pontos, sejam A e B dois deles. A partir deles,
definimos a reta AB. Como em uma reta existem infinitos pontos, bem
como fora dela, tome fora dela o ponto X. A cada ponto P da reta AB,
seja a reta XP. Como X não pertence à reta AB, todas as retas XP são
diferentes de AB. Agora, dados dois pontos P e Q na reta AB, se XP
e XQ fossem uma mesma reta, então seriam a reta PQ, que é a reta
AB, o que é um absurdo, pois X não está na reta AB. Logo todas as
retas do tipo XP são diferentes entre si, variando P. Como são infinitos
Apesar de não podermos defini-los, podemos estabelecer notações e pontos P, são infinitas retas no plano.
relações entre eles, através dos axiomas. Os axiomas são como “regras
do jogo”, as verdades que não podemos provar, e que servem pra iniciar
o estudo desse sistema.
2. Outros objetos iniciais e
Existem cinco grupos principais de axioma na geometria euclidiana,
segundo a axiomatização de Hilbert. Seguem alguns dos principais axiomas definições
da geometria:
Segmento de reta AB é o conjunto de pontos que estão entre os
pontos A e B. Chamamos A e B de extremidades do segmento. Todo ponto
• Existem infinitos pontos; que está entre A e B está na reta AB, logo o segmento de reta AB está
• dois pontos distintos determinam uma reta que os contém; 
contido na reta AB . A cada segmento de reta, associamos uma medida,
• numa reta existem infinitos pontos, fora dela também;
que é um número real positivo. A união de dois segmentos adjacentes
• três pontos distintos que não estejam em uma mesma reta determinam
tem por medida a soma das medidas de ambos.
um plano que os contém; 
• dados uma reta e um ponto fora dela, existe e é única uma segunda Formalmente, dados dois pontos A e B, chamamos de semirreta AB
reta que contém o ponto dado e não intersecta a reta dada, embora o conjunto dos pontos X que estão entre A e B ou são tais que B está
esteja no mesmo plano. [Ax. De Euclides] entre A e X. Para entender, pense em A dividindo a reta em dois conjuntos
infinitos em sentidos diferentes, um dos quais contém o ponto B. Esse

A essa reta, chamamos de paralela. Foram suprimidos alguns axiomas, conjunto é a semirreta AB . O vértice (ou origem) dela é o ponto A. Quando
para facilitar o entendimento. duas semirretas possuem o mesmo vértice e a sua união é a reta suporte
delas, dizemos que são semirretas opostas.
O ponto M entre A e B tal que os segmentos AM e MB têm a mesma
medida é chamado de ponto médio.

IME-ITA 243
Matemática V – Assunto 1

Dizemos que um conjunto do plano é convexo se, e somente se, Dizemos que dois ângulos são opostos pelo vértice quando os lados
para todo par de pontos A e B do conjunto, o segmento AB está contido de um são semirretas opostas aos lados do outro. Prova-se que se dois
no conjunto também. Caso existam dois pontos do conjunto tais que o ângulos são opostos pelo vértice, então são congruentes.
segmento com extremidade neles não está contido no conjunto, dizemos
que o conjunto é não-convexo.

β α

Conjunto convexo Conjunto não-convexo

EXERCÍCIOS RESOLVIDOS
Os ângulos α e β são opostos pelo vértice α = β
01. Sejam A, B e P pontos colineares tais que B está entre P e A. Sejam
M e N pontos médios dos segmentos AP e PB respectivamente. Calcule
a medida de MN, sabendo as medidas de AP = a, PB = b. Segue a nomenclatura dos ângulos quanto às suas medidas:

Solução: • Ângulo Agudo: ângulo maior que 0° e menor que 90°.


Como AP = a, e M é médio de AP, então MP = a/2. Como PB = • Ângulo Reto: ângulo de 90°.
b, e N é médio de PB, então NP = b/2. Agora, como N está entre • Ângulo Oblíquo: ângulo maior que 90° e menor que 180°.
M e P, tem-se que MN = MP – PN = (a – b)/2. • Ângulo Raso: ângulo de 180°.
• Ângulo Côncavo ou Reentrante: ângulo maior que 180° e menor que 360°.

3. Ângulos • Ângulos complementares: ângulos que somam 90°.


• Ângulos suplementares: ângulos que somam 180°.
• Ângulos replementares: ângulos que somam 360°.
Quando duas semirretas OA e OB possuem mesmo vértice, elas
• Ângulos explementares: ângulos cuja diferença é de 180°.
determinam uma região do plano que chamamos de ângulo [denotamos
AÔB], e as semirretas chamamos de lados do ângulo. Associamos a
cada ângulo uma medida, que é um número real positivo. Quando dois 4. Paralelismo e teorema angular
ângulos têm por interseção apenas um lado em comum, dizemos que
são adjacentes, e a medida da união é a soma das medidas. O ângulo
de Tales
formado por duas semirretas opostas é associado à medida em graus Com o conteúdo de triângulos, e após a formalização de alguns
de 180°. Existem outras unidades de medida, como o radiano e o grado. teoremas, podemos concluir o seguinte teorema: na figura, se r//s, então
Para converter, é só fazer uma regra de três com a seguinte equivalência: os ângulos a e b são congruentes. Na verdade, vale a recíproca também,
180° = πrad. = 200gr a qual servirá como um bom critério de paralelismo entre retas.
Dado um ângulo AÔB, chamamos a semirreta interna a ele OX de
bissetriz, se, e somente se, os ângulos XÔA e XÔB são congruentes, isto
é, têm a mesma medida. α
r
B
^ ^
AO X = XO B
^
OX é bissetriz de AO B
x
O
β
s
A

Na figura, r//s ⇔ α = β

244 Vol. 1
Introdução à geometria plana euclidiana

Na figura, temos que são ângulos congruentes: EXERCÍCIOS NÍVEL 1

a = c = e = g, b = d = f = h. 01 São dados os pontos A, B, C, D e E, nessa ordem, sobre uma reta.


Sabe-se que AB + CD = 3 · BC e DE = AB. Sendo M médio de BE, tem-se
que MD = 2 e AE = 16. Calcule MC.

(A) 2 (D) 5
(B) 3 (E) 6
(C) 4

02 São dados os pontos A, B, C e D, nessa ordem, sobre uma reta, de


forma que AD = 20 e BC = 12, sendo AB menor que a metade de CD.
Calcule a distância entre os pontos médios de AB e CD.

03 Efetue:

a. 23°45’19” + 37°32’43”
Alguns pares de ângulos, como na figura anterior, recebem um nome b. 87°18’32” – 54°37’42”
pela posição relativa às retas paralelas e à transversal. São eles: c. 5°23’47” · 4
d. 56°25’33” ÷ 3
• Alternos internos: (c, e), (d, f)
• Alternos externos: (a, g),(b, h) 04 Sendo dado um ângulo de medida a, escreva simplificadamente uma
• Colaterais internos: (c, f),(d, e) fórmula que calcule:
• Colaterais externos: (a, h), (b, g)
• Correspondentes: (a, e),(b, f),(c, g),(d, h). a. o suplemento de a;
b. o complemento da metade de a;
c. o replemento de um terço do suplemento de a;
Como consequência, tem-se o Teorema Angular de Tales: em
d. o suplemento do dobro do complemento da metade de a.
um triângulo, a soma dos ângulos internos é constante e igual a 180°.
Analogamente, podemos concluir a relação do ângulo externo: cada ângulo
05 Um ângulo é igual ao dobro do complemento do seu quádruplo. Quanto
externo de um triângulo mede a soma dos outros dois ângulos internos
mede esse ângulo?
não-adjacentes a ele.
06 Dois ângulos suplementares são tais que um é o triplo do complemento
do outro. Quanto vale a razão entre esses ângulos?

07 Dois ângulos AÔB e BÔC são adjacentes, e o ângulo AÔC mede 120°.
Calcule a medida do ângulo formado pelas bissetrizes de AÔB e BÔC.

08 Em um relógio de ponteiro convencional, qual é o ângulo formado


pelos ponteiros quando marcam:

a. 5:20h
b. 10:44h

09 As semirretas OA, OB, OC e OD formam os ângulos adjacentes AÔB, BÔC


e CÔD. Sabendo que OA e OD são semirretas opostas, e que o ângulo BÔC
mede 130°, quanto mede o ângulo formado pelas bissetrizes de AÔB e CÔD?

10 Nas figuras, prove as relações “dos bicos”.

IME-ITA 245
Matemática V – Assunto 1

Quais são verdadeiras?

(A) apenas I. (D) II e III.


(B) apenas II. (E) I e II.
(C) apenas III.

05 Pelo vértice de um ângulo AÔB reto, traça-se uma reta r qualquer,


externa a ele. Calcule o ângulo formado pelas bissetrizes dos ângulos
agudos que as semirretas OA e OB formam com a reta r.

06 Os ângulos AÔB e BÔC são adjacentes, sendo a medida do primeiro


igual a 70˚. Calcule o ângulo formado pelas bissetrizes dos ângulos AÔC
e BÔC.
11 (EPCAR-2004) Considere as retas r e s (r//s) e os ângulos ê, î e â da
figura abaixo: 07 Após ver um relógio de ponteiro às 13:00, o ponteiro das horas
percorreu um ângulo de 42°. Qual é o horário indicado pelo relógio após
r
â esse movimento?

08 Após as 15:00, qual é o primeiro horário em que os ponteiros das


horas e dos minutos formam um ângulo de 130°?
î
ê 09 As semirretas OA, OB, OC e OD formam ângulos adjacentes AÔB, BÔC,
s CÔD e DÔA, nessa ordem, tais que os três primeiros são proporcionais a
Pode-se afirmar que: 1, 3 e 6 respectivamente. Sabe-se que OD é semirreta oposta à bissetriz
do ângulo BÔC. Calcule o ângulo AÔD.
(A) ê + î + â = 270° (C) ê + î = â
(B) ê + î + â = 180° (D) ê + î = â + 90° 10 Os ângulos AÔB e BÔC são adjacentes, sendo AÔC = 100°. Sendo
OX, OY e OZ semirretas bissetrizes, respectivamente, de AÔB, BÔC e XÔY,
12 Na figura, prove que vale a relação x = a + b + c. e sendo BÔZ=10°, então o maior dentre AÔB e BÔC mede:

(A) 60°.
(B) 70°.
(C) 80°.
(D) 90°.
(E) faltam dados.

11 Dados os ângulos adjacentes AÔB, BÔC, CÔD e DÔA, traçam-se as


bissetrizes OX, OY e OZ dos ângulos AÔB, CÔD e XÔY, respectivamente. Sabe-se
que XÔC + XÔD – 4 · BÔZ = 80° e que BÔZ mede 50°. Calcule o ângulo CÔD.

(A) 10°. (D) 60°.


(B) 20°. (E) 80°.
EXERCÍCIOS NÍVEL 2 (C) 40°.

01 São dados os pontos A, B, C e D, nessa ordem, sobre uma reta. 12 (OBM) Três quadrados são colados pelos seus vértices entre si e a
Sabe-se que AB e CD são congruentes. Prove que o ponto médio de AD dois bastões verticais, como mostra a figura.
é também ponto médio de BC.

02 Sobre uma reta marcam-se os pontos M, A e B. Sendo O o ponto


médio de AB, calcule k para que valha a seguinte relação: MA2 + MB2 =
k · (MO2 + AO2)

03 O suplemento da terça parte de um ângulo excede o complemento do


seu triplo em 130°. Quanto mede o replemento do quíntuplo desse ângulo?

04 As medidas de quatro ângulos replementares estão em progressão


aritmética. Analise as afirmativas a seguir: A medida do ângulo x é:

I. Dois deles são complementares. (A) 39º. (D) 44º.


II. Existe um que é o dobro do outro. (B) 41º. (E) 46º.
III. Existem dois deles que são suplementares. (C) 43º.

246 Vol. 1
Introdução à geometria plana euclidiana

13 Na figura que segue, quanto vale a soma a + b + c + d + e? EXERCÍCIOS NÍVEL 3

(A) 180°.
01 Quantas vezes os ponteiros das horas e dos minutos são perpendiculares
(B) 270°.
em um dia de funcionamento?
(C) 360°.
(D) 540°.
(A) 44.
(E) faltam dados.
(B) 46.
(C) 48.
(D) 23.
(E) 24.

02 Os ângulos AÔB e BÔC são adjacentes. Sendo OX, OY, OZ e OW


semirretas bissetrizes, nessa ordem, de AÔB, BÔC, XÔY e AÔC, prove
que OZ é bissetriz de BÔW.

03 Dê o valor numérico de x, em graus, na figura abaixo:

14 Nas figuras, calcule a medida de x:

04 Na figura, quanto vale x?

IME-ITA 247
Polígonos A ssunto
2
Matemática V

Neste bloco, você verá uma pequena introdução ao conceito de Pode ser ainda calcular o número de diagonais de um polígono de
polígonos. Aqui analisaremos principalmente a parte qualitativa de gênero n, pela seguinte expressão:
polígonos. Mais adiante, com ferramentas mais avançadas, você poderá
deduzir as principais relações métricas e angulares envolvidas nas questões n( n − 3)
D=
mais comuns do assunto. 2
Dem: De fato que de cada vértice do polígono partem n – 3 retas
1. Definição e nomenclatura uma vez que, um vértice não se liga a si mesmo, nem aos seus dois
Chamamos de polígono a região delimitada pela união de segmentos vizinhos(lados).
não-colineares consecutivamente, desde que tal linha seja fechada. Como o polígono tem gênero n, teríamos n(n – 3) reatas, porém cada
Aos segmentos chamaremos ‘lados’, aos extremos dos segmentos reta é contada duas vezes(uma em cada vértice).
chamaremos ‘vértices’, e ao número de lados, que é igual ao número de
vértices, chamaremos de ‘gênero’ do polígono. Qualquer segmento com 3. Polígonos regulares
extremidades em dois vértices é chamado de ‘diagonal’, desde que não
Dizemos que um polígono convexo é equilátero quando todos os seus
seja um lado do polígono.
lados são congruentes.
Dizemos que um polígono é simples
quando não existem dois lados não-
consecutivos que se intersectem
mutuamente. Caso existam dois lados
não-consecutivos com interseção,
dizemos que o polígono é complexo.
Caso seja simples, definimos como
‘ângulo interno’ qualquer ângulo formado
por dois lados consecutivos, definido na
região interna ao polígono. Observe que
o número de ângulos internos é igual ao gênero do polígono. Dizemos que um polígono convexo é equiângulo quando todos os
seus ângulos internos são congruentes.
Se o polígono é simples, então podemos classificá-lo como convexo
ou não-convexo. Caso seja convexo, chamamos de ‘ângulo externo’ cada
menor ângulo formado por um lado e um prolongamento de outro lado
adjacente. Considere sempre que para cada vértice existe um ângulo
externo, já que na verdade são dois ângulos opostos pelo vértice, logo
são congruentes. Dessa maneira, o número de ângulos externos também
é igual ao gênero do polígono.

Observe pelas figuras que um polígono equilátero não necessariamente


Pentágono ABCDE Pentágino IJFGH não Pentágono MLKPN
é equiângulo, e vice-versa. Dessa maneira, existem os polígonos regulares:
complexo convexo convexo
são os polígonos equiláteros e equiângulos simultaneamente. Todo
2. Fórmulas importantes polígono regular admite um centro, que é um ponto que equidista dos
vértices, bem como dos lados. Logo, sempre existe um círculo circunscrito
Por triangulação, que consiste em quebrar um polígono em triângulos a um polígono regular, bem como um inscrito nele.
através de suas diagonais, podemos concluir que a soma dos ângulos
internos de um polígono simples é dada pela fórmula:
S = 180o · (n –2),
sendo n o gênero do polígono.

Obs.: É fácil provar que todo polígono simples é triângulorizável por


indução.
Para polígonos convexos, a soma de
um ângulo interno com um externo sempre
será um ângulo raso, logo podemos concluir
que, para polígonos convexos, a soma
dos ângulos externos de cada vértice é
constante e igual a 360 graus.

248 Vol. 1
Polígonos

Num polígono regular, todos os ângulos internos são iguais a 02 ABCDE é um pentágono regular, e ABIJK é outro pentágono regular.
Calcule o ângulo AÎC.
180o ( n − 2) 360o
αi = , e todos os ângulos externos são iguais a α e =
n n 03 Em um quadrilátero ABCD convexo, os ângulos internos em A e B
medem 120° e 80° respectivamente. Calcule o ângulo agudo formado
. Ao traçar as diagonais, cada ângulo interno fica dividido em partes pelas bissetrizes internas nos vértices C e D:
congruentes, iguais à metade do ângulo externo.
(A) 60°. (D) 75°.
(B) 65°. (E) 80°.
(C) 70°.

04 Um polígono regular tem 20 diagonais. Quanto mede seu ângulo


interno?

05 Aumentando o número de lados de um polígono em 3 unidades, seu


número de diagonais aumenta em 21. Determine o número de diagonais
do polígono.

06 Calcule o número de diagonais que não passam pelo centro de um


polígono regular de 2n lados.

07 A soma dos gêneros de dois polígonos é 16. Se os seus números de


No polígono regular de gênero n temos ainda os seguintes fatos conhecidos: diagonais diferem de 26, a diferença entre seus gêneros é:

n (A) 1. (D) 4.
• n é par: o número de diagonais que passam pelo centro é e que (B) 2. (E) 5.
2
(C) 3.
n ( n − 3)
n n ( n − 4)
não passam é = − .
2 2 2 08 A soma dos ângulos internos de dois polígonos regulares é 1980°, e
• n é ímpar: nenhuma diagonal passa pelo centro. a diferença entre seus gêneros é 3. Determine os polígonos.

EXERCÍCIOS RESOLVIDOS 09 Dois polígonos regulares são tais que o gênero de um excede o do outro
em 3 unidades, e o número de diagonais de um é o quádruplo do número de
01 ABCD é um quadrado, e ABP e BCQ são triângulos equiláteros,
diagonais do outro. Determine os ângulos internos desses dois polígonos.
o primeiro interno ao quadrado, e o segundo externo a ele. Prove que
D, P e Q são colineares.
10 Um polígono regular convexo tem o seu número de diagonais expresso
Solução: por n2 – 10n + 8, onde n é o seu número de lados. O seu ângulo interno
x é tal que:
Considerando que ABCD e ABP possuem o mesmo lado, então o
triângulo APD é isósceles, e Â=30° implica PDA=75°. Logo PDC=15°. (A) x < 120º. (D) 140º < x <150º.
Analogamente, considerando ABCD e BCQ, tem-se que o triângulo QCD (B) 120º < x < 130º. (E) x > 150º.
é isósceles, com C=150°. Logo QDC=15°. Como PDC e QDC são (C) 130º < x < 140º.
iguais a 15°, então os pontos D, P e Q são colineares.
EXERCÍCIOS NÍVEL 2
02 Um polígono regular de gênero desconhecido ABCDEF é tal que o
ângulo ACE mede 150°. Calcule o número de diagonais do polígono. 01 ABCDE é um pentágono regular, e ABI é um triângulo equilátero, sendo
I interno ao pentágono. Calcule o ângulo CÎD.
Solução:
ABC é um triângulo isósceles. Sendo x=BCA, tem-se que o ângulo 02 Em um polígono convexo de 15 vértices, são escolhidos quatro de
BCD mede x + 150° + x. Além disso, o ângulo externo do polígono é seus vértices, sem que haja dois consecutivos. Quantas são as diagonais
dado por 2x. Como o ângulo interno é suplementar do externo, tem-se que partem desses 4 vértices?
que (2x + 150°) + 2x = 180°, logo 2x = 15°. Sendo n o gênero, tem-
-se que n = 360° / 15° = 24. Logo D=24 · 21 / 2 = 252 diagonais. 03 Em um polígono convexo, dois ângulos internos medem 130°, e todos
os outros medem 128°. Determine o gênero desse polígono.
EXERCÍCIOS NÍVEL 1
04 Em um polígono regular, as mediatrizes de dois lados consecutivos
01 ABCD é um quadrado e ABP é um triângulo equilátero. Calcule o ângulo formam um ângulo de 20°. Calcule o ângulo formado entre as duas
CDP nos seguintes casos: diagonais menores que partem do mesmo vértice.
a. o triângulo é interno ao quadrado.
b. o triângulo é externo ao quadrado.

IME-ITA 249
Matemática V – Assunto 2

05 As diagonais de um polígono regular convexo são medidas, e 12 (ITA-1998) Considere as afirmações sobre polígonos convexos:
apresentam os valores: {m1, m2, m3, ... , m14}. Calcule o ângulo externo I. Existe apenas um polígono cujo número de diagonais coincide com
desse polígono, sabendo que, em graus, ele apresenta um valor inteiro. o número de lados.
II. Não existe polígono cujo número de diagonais seja o quádruplo do
06 Em um polígono regular ABCDE..., as bissetrizes externas traçadas número de lados.
de A e C são perpendiculares. Qual é o gênero do polígono? III. Se a razão entre o número de diagonais e o de lados de um polígono
é um número natural, então o número de lados do polígono é ímpar.
07 Dois polígonos regulares são tais que a razão entre seus ângulos Então:
internos é 5/4, e a razão entre seus ângulos externos é 1/2. Calcule o
número de diagonais do polígono com maior gênero. (A) Todas as afirmações são verdadeiras.
(B) Apenas I e III são verdadeiras.
08 Um ladrilho com formato de polígono regular é tal que, rotacionado em (C) Apenas I é verdadeira.
torno do centro de 40° no sentido horário ou de 60° no sentido anti-horário, (D) Apenas III é verdadeira.
fica encaixado perfeitamente no espaço vago deixado antes de rotacionar. (E) Apenas II e III são verdadeiras.
Determine o número mínimo de lados que tal ladrilho pode possuir.
13 Um hexágono ABCDEF convexo é tal que AB//DE, BC//EF e CD//AF.
Prove que os ângulos internos nos vértices opostos são iguais.
09 Dado um polígono convexo regular ABCDEF... de gênero desconhecido,
considere as bissetrizes de seus ângulos internos A e D. Sabendo que o ângulo EXERCÍCIOS NÍVEL 3
3
formado por essas bissetrizes é igual a da soma de todos os ângulos
40 01 (ITA-Adaptado) Considere um polígono convexo não necessariamente
internos do polígono, pede-se para calcular quantas diagonais ele possui.
regular de gênero x. Sabe-se dele que a soma de x – 1 ângulos internos
desse polígono é 2014°. Calcule o número de diagonais desse polígono.
10 O número de gêneros de polígonos regulares tais que quaisquer duas
de suas diagonais, que passam pelo seu centro, formam entre si ângulo 02 Prove que, em um polígono convexo, o número máximo de ângulos
expresso em graus por um número inteiro, é: internos agudos é 3.

(A) 17. (D) 23. 03 Um hexágono ABCDEF é equiângulo. Sabendo que AB = 7, BC = 8,


(B) 18. (E) 24. CD = 6, DE = 10, calcule os lados EF e FA.
(C) 21.
04 Prove que em qualquer pentágono convexo, existem dois ângulos
11 (ITA-2003) Considere três polígonos regulares tais que os números que internos consecutivos cuja soma é maior ou igual a 216°.
expressam a quantidade de lados de cada um constituam uma progressão
aritmética. Sabe-se que o produto desses três números é igual a 585 e que
a soma de todos os ângulos internos dos três polígonos é igual a 3780o. O
número total das diagonais nesses três polígonos é igual a:

(A) 63. (D) 97.


(B) 69. (E) 106.
(C) 90.

RASCUNHO

250 Vol. 1
Triângulos A ssunto
3
Matemática V

1. Definição e propriedades Além do mais, deduz-se o Teorema da Envolvente: se dois caminhos


convexos de A para B são tais que a região definida por um [o envolvente]
iniciais contém a região definida pelo outro [o envolvido], então o comprimento
daquele será maior que o desse.
Triângulo é o polígono de gênero 3. Não possui diagonais, e é sempre
convexo. Possui portanto três lados, três vértices, três ângulos internos
[que somam 180˚] e três ângulos externos.

Podemos classificar os triângulos quanto às medidas dos lados:

I. Escaleno: todos os seus lados são diferentes;


II Isósceles: possui pelo menos dois lados iguais;
III Equilátero: possui todos os lados iguais.

Podemos classificá-los também quanto aos ângulos internos: Uma desigualdade muito útil é a seguinte: num triângulo, o maior
lado é sempre oposto ao maior ângulo, e o menor lado é sempre oposto
I. Acutângulo: todos os seus ângulos internos são agudos; ao menor ângulo.
II. Retângulo: possui um ângulo interno reto;
III. Obtusângulo: possui um ângulo interno obtuso.
3. Congruência de triângulos
Se um triângulo é retângulo, chamamos o lado oposto ao ângulo reto Dizemos que dois triângulos são congruentes se, e somente se, os
de hipotenusa, e os outros dois lados de catetos. lados e os ângulos internos de um são homologamente congruentes aos
lados e ângulos internos do outro. Formalmente, dizemos:

∆ABC = ∆DEF ⇔ A = D
, B
 = E , C
 = F , AB = DE, AC = DF, BC = EF.

Para demonstrar
que dois triângulos são
congruentes, basta testar
se vale um dos cinco
casos de congruência que
seguem:
2. Desigualdade triangular –
teorema da envolvente I. LAL – Lado/ângulo/lado: Triângulos com par de lados iguais formando
ângulos congruentes são congruentes entre si.
A desigualdade triangular estabelece o seguinte: dados dois pontos,
A e B, e um ponto X qualquer variável, sempre vale que AB ≤ AX + XB, x
com igualdade se, e somente se, X está entre A e B. Essa desigualdade
pode ser estendida: dada uma poligonal fechada, um lado é menor que a a
soma de todos os outros lados. y

II. ALA – Ângulo/lado/ângulo: Triângulos com par de ângulos iguais com


os lados comuns a eles congruentes são congruentes entre si.

x a
b

III. LAAo – Lado/ângulo/ângulo oposto: Triângulos com lado, ângulo


A partir disso, podemos concluir a condição de existência de um adjacente ao lado e ângulo oposto a ele congruente um aos do outro
triângulo: dados os lados de um possível triângulo, ele existe se, e somente são congruentes.
se:
a < b + c x a
 b
b < c + a ⇔ a − b < c < a + b

c < a + b
IV. LLL – Triângulos com todos os lados congruentes um aos do outro
são congruentes.

IME-ITA 251
Matemática V – Assunto 3

y c. Altura [e o ortocentro]: se D sobre a reta BC é tal que AD é


perpendicular a BC, dizemos que AD é altura relativa a BC. A altura pode
ser uma ceviana externa ou até mesmo um lado, como nas figuras. As
z
x três alturas traçadas a partir de cada vértice são concorrentes num
ponto chamado ortocentro.

V 90˚HC – Caso especial para triângulos retângulos: Triângulos


retângulos que possuam hipotenusas iguais e um cateto de um igual
a um do outro são congruentes.

a
b

Obs.: LLA não é caso de congruência!


Os triângulos ABC e ABD d. Bissetriz externa [e o exincentro]: se o triângulo ABC é escaleno,
possuem dois pares de lados então existe D sobre a reta BC tal que AD é bissetriz do ângulo externo
em comum e os ângulos em A. Chamamos AD de bissetriz externa. São três bissetrizes externas,
opostos a um deles iguais. uma para cada vértice. Duas retas bissetrizes externas e uma interna
Eles não são congruentes: um do vértice remanescente são concorrentes num ponto chamado
está dentro do outro exincentro relativo àquele vértice. São três exincentros, e eles são
centros de círculos tangentes às retas suportes dos lados do triângulo.
4. Cevianas notáveis – pontos
notáveis Mediatriz
A mediatriz de um segmento AB
Dado um triângulo ABC, dizemos que o segmento AD é uma ceviana se
é a reta perpendicular a AB no
o ponto D está sobre a reta suporte do lado BC. Caso D esteja sobre o lado
seu ponto médio. Num triângulo,
BC, dizemos que AD é ceviana interna. Caso contrário, AD é ceviana externa.
as mediatrizes dos lados nem
sempre são cevianas, já que não
Algumas cevianas possuem propriedades impor tantes, e têm
passam necessariamente pelo
nomenclatura especial.
vér tice oposto. Porém, no caso
a. Mediana [e o baricentro]: do triângulo, as mediatrizes dos
se M é ponto médio de lados são concor rentes num
BC, chamamos AM de ponto chamado circuncentro, que
mediana relativa ao lado equidista dos vértices, e, portanto,
BC. Cada triângulo possui é centro de uma circunferência
três medianas, que são circunscrita ao triângulo.
concorrentes num ponto
chamado baricentro. O
baricentro G divide uma 5. Triângulo isósceles
mediana AM na razão Podemos provar que um triângulo tem uma das propriedades abaixo
AG : GM = 2. Traçadas se, e somente se, ele é isósceles. Seguem:
as medianas, o triângulo
• Os ângulos das bases são congruentes;
original fica dividido em seis
• a altura do vértice principal também é mediana;
triângulos de áreas iguais.
• a altura do vértice principal também é bissetriz;
• a bissetriz do vértice principal também é mediana.
b. Bissetriz interna [e o incentro]: se D sobre o lado BC é tal que AD
bissecta o ângulo interno em A, chamamos AD de bissetriz interna
relativa ao vértice A, ou relativa ao lado BC. Cada triângulo possui
três bissetrizes internas, que são concorrrentes num ponto chamado
incentro. O incentro equidista dos lados do triângulo, logo é centro de
uma circunferência inscrita no triângulo.

252 Vol. 1
Triângulos

6. Lugares geométricos iniciais


Dada uma propriedade Ω, dizemos que certo conjunto é o lugar
geométrico (LG) de Ω se, e somente se, todos os pontos que satisfazem
a Ω estão no conjunto, e vice-versa. Inicialmente, podemos deduzir três P está na mediatriz de AB,
importantes LG’s. logo PA = PB
a. Par de retas bissetrizes: dadas duas retas concorrentes r e s, o LG
dos pontos P que equidistam de r e s é o par de retas bissetrizes dos
ângulos formados pelas retas.

c. Par de paralelas: dada uma reta r e uma distância d, o LG dos pontos


P que distam d da reta r é o par de paralelas a uma distância d.

P e Q pertencem a r’ e r” paralelas distando x de r.


b. Mediatriz: dado um segmento AB, o LG dos pontos P que equidistam dist(P, r) = PP’ = x = QQ’ = dist(Q, r)
de A e B é a mediatriz de AB.
EXERCÍCIOS RESOLVIDOS

01 Truque do simétrico – Dados dois pontos A e B no mesmo semiplano 03 A, B e C são pontos colineares, com B entre A e C. Constroem-se,
determinado por uma reta r, determine o ponto P que minimiza a soma num mesmo semiplano gerado por AB, os triângulos equiláteros ABX
PA + PB, sendo P sobre a reta r. e BCY. Sendo M e N médios de AY e CX, prove que o triângulo BMN é
equilátero.
Solução: Seja A* o simétrico de A em relação a r. Logo, por simetria
[mediatriz], AP + PB é igual a A*P + PB para todo P na reta. Logo, para Solução: Observe que os triângulos ABY e XBC são congruentes pelo
minimizar PA + PB, basta minimizar PA* + PB, o que ocorre quando A*, caso LAL. (AB = XB, BY = BC e os ângulos B medem 120°). Logo, as
P e B são colineares. Logo, o ponto P que minimiza a soma PA + PB medianas relativas aos lados AY e XC são congruentes [ou seja, BM = BN].
é a interseção do segmento A*B com a reta r. Além disso, o ângulo entre elas é de 60° [por argumento de rotação],
[A ideia do simétrico é muito útil em problemas de mínimos caminhos.] logo o triângulo BMN é equilátero.

02 Seja P interno ao triângulo ABC. Prove que AP + PB < AC + CB. 04 ABC é um triângulo em que Â=120°. Sejam D, E e F pés das
[Teorema da Envolvente] bissetrizes internas de A, B e C, respectivamente. Prove que:
a. E é exincentro do triângulo ABD, bem como F, de ACD.
Solução: Considere prolongar o segmento AP até X sobre BC. Por b. triângulo EFD é retângulo [em D].
desigualdade triangular temos:
Solução: Considerando o triângulo ABD, tem-se que BE é bissetriz interna
No triângulo ACX: AP + PX < AC + CX. e que AE é bissetriz externa [já que, prolongando AB de AX, tem-se
No triângulo PXB: PB < PX + XB. EÂD = EÂX = 60°]. Logo, E é exincentro do triângulo ABD.
Consequentemente, DE é bissetriz externa. Analogamente, F é exincentro
Somando e usando a lei do corte, tem-se a conclusão de que é um do triângulo ACD, e DF é bissetriz externa desse triângulo. Como os
caso particular do teorema da envolvente. Usando a ideia de prolongar ângulos ADB e ADC são suplementares, essas bissetrizes DE e DF são
o segmento, podem-se provar as versões poligonais generalizadas. perpendiculares entre si. Logo, o triângulo EFD é retângulo em D.

EXERCÍCIOS NÍVEL 1

01 Em um triângulo ABC, tomam-se sobre os lados AB e BC os pontos IV. os segmentos CD e AE são congruentes.
D e E, respectivamente, tais que BD = BE, e a medida dos ângulos BCD
e BAE são iguais. Analise as afirmativas: Quantas afirmativas são verdadeiras?

I. os triângulos BEA e BDC são congruentes; (A) Nenhuma. (D) Apenas três.
II. os ângulos BDC e BEA são congruentes; (B) Apenas uma. (E) Todas.
III. os segmentos BE e AD são congruentes; (C) Apenas duas.

IME-ITA 253
Matemática V – Assunto 3

02 Sobre os lados AB e AC de um triângulo ABC, tome os triângulos 13 Em um triângulo ABC, calcule, em função do ângulo interno em A, os
equiláteros ABP e ACQ, externos ao triângulo ABC. Prove que: ângulos formados:

a. CP e BQ são congruentes; (A) pelas bissetrizes internas em B e C.


b. as retas CP e BQ formam um ângulo de 120°. (B) pelas alturas traçadas de B e C.
(C) pelas bissetrizes externas de B e C.
03 Sobre os lados AB e AC de um triângulo ABC, tomam-se os quadrados (D) pelas bissetrizes interna em B e externa em C.
ABDE e ACFG. Prove que os segmentos CE e BG são congruentes e
perpendiculares entre si. 14 ABC é um triângulo isósceles tal que AB = AC = 13, BC = 10. Sobre
o lado BC, toma-se um ponto P, e sobre os lados AB e AC tomam-se X e Y,
04 Prove que, num triângulo ABC, a reta suporte da mediana AM equidista respectivamente, tais que PX // AC e PY // AB. Calcule a soma PX + PY.
dos vértices B e C.
15 Na figura, tem-se AB = BC e AC = CD = DE = EF. Calcule .
05 Em um triângulo ABC, AD é bissetriz. Sobre a semirreta AD, tomam-se
os pontos E e F tais que AB = AE e AC = AF. Prove que BF = CE.

06 Dadas as seguintes proposições, analise se são elas verdadeiras (V)


ou falsas (F):

I. Dois triângulos retângulos são congruentes se possuem dois lados


congruentes.
II. Se em um quadrilátero ABCD, BC = CD, e BÂC = DÂC, então os 16 ABC é um triângulo isósceles com AB = AC. Sobre os lados BC e
triângulos ABC e ADC são congruentes. AC, tomam-se os pontos P e Q, de forma que AP = AQ. Calcule o ângulo
III. Se no quadrilátero ABCD, AC = BD e os ângulos ABD e ACD são retos, QPC, sabendo que BÂP = 30°.
concluímos que AB = CD.
(A) 10°. (D) 25°.
(B) 15°. (E) 30°.
Então, tem-se:
(C) 20°.
(A) V-F-V.
17 ABC é um triângulo no qual o ângulo interno em A é o dobro do em B.
(B) F-V-V.
Sejam X e Y pontos sobre BC e AC tais que AB = AX = XY = YC. Calcule
(C) V-V-V.
os ângulos do triângulo ABC.
(D) V-V-F.
(E) F-F-V.
18 Dado um triângulo ABC, marca-se um ponto D sobre AC de forma que
AB = AD. Sabendo que o ângulo interno em B excede o em C em 30°,
07 Em um triângulo ABC, prolongam-se as medianas BM e CN de
calcule a medida do ângulo CBD.
comprimentos iguais [MB’ = BM, NC’ = CN]. Prove que A é ponto médio
de B’C’.
EXERCÍCIOS NÍVEL 2
08 No triângulo ABC, AB = 5, AC = 7, e os ângulos internos são
crescentes na seguinte ordem: C < A < B. Diga quanto mede BC, dado 01 Dado um triângulo ABC, sobre os lados AC e BC são marcados os
que é inteiro. pontos M e N, respectivamente, tais que AB = MC, MB = MN, e os ângulos
ABM e CMN são congruentes. Sabendo que BÂC = 50°, quanto vale o
(A) 4. (D) 7. ângulo ACB?
(B) 5. (E) 8.
(C) 6. (A) 50°. (D) 25°.
(B) 40°. (E) 75°.
09 Em um triângulo ABC isósceles, dois lados medem 14 cm e 4 cm. (C) 60°.
Calcule o perímetro do triângulo.
02 Na figura, os triângulos ABC e CDE são congruentes, de forma que
(A) 18 cm. (D) Podem ser 22 cm ou 32 cm. AC = CD. Se EC = 5, EF = 2, então quanto mede AF?
(B) 22 cm. (E) Faltam dados.
(C) 32 cm.

10 Prove que, em um quadrilátero convexo, a soma das medidas das


diagonais é maior que a soma das medidas de dois lados opostos.

11 Entre que valores está compreendido um dos lados de um quadrilátero


que possui lados medindo 3 cm, 5 cm e 11 cm?

12 Seja ABC um triângulo e AD, uma ceviana interna. Prove que AD é (A) 2. (D) 5.
menor que o semiperímetro do triângulo. (B) 3. (E) 7.
(C) 4.

254 Vol. 1
Triângulos

03 Na figura, AB = BD e os triângulos ABC e BDE são congruentes. 10 Considere ABC um triângulo equilátero. Prolongando BC de um
Calcule a razão entre os ângulos BCA e DAC. segmento CP qualquer, toma-se, sobre a altura de B no triângulo ABP,
um ponto Q tal que QÂB = 30°. Calcule a medida do ângulo QPC.

(A) 15°. (D) 35°.


(B) 20°. (E) 45°.
(C) 30°.

11 Em um triângulo isósceles ABC de base AC e ângulo interno em B =


40°, traça-se a ceviana interna CM e marca-se o incentro I do triângulo
MCB. Os ângulos AMC e IAC são congruentes. Calcule BÂI.
(A) 1:1. (D) 1:3.
(B) 3:2. (E) 2:1. (A) 20°. (D) 50°.
(C) 1:2. (B) 40°. (E) 30°.
(C) 10°.
04 Considere os triângulos ABC e A’B’C’, sobre os quais são feitas as
seguintes afirmações: 12 ABC é um triângulo equilátero. Prolonga-se BC de um segmento BP,
I. Se AB = A’B’, BC = B’C’ e  = Â’, então os triângulos são de forma que o ângulo APB meça 20°. Sobre o segmento AP, toma-se um
congruentes. ponto Q tal que o triângulo PQC é isósceles. Calcule a medida do ângulo
II. Se AB=A’B’, BC = B’C’ e B = B’, então os triângulos são congruentes. QBA.
III. Se AB = A’B’, BC = B’C’, Â = Â’, e BC > AB, então os triângulos
são congruentes. 13 Sobre os lados AB e AC de um triângulo ABC, tomam-se os pontos M
e N, tais que MN // BC e MN passa pelo incentro de ABC. Calcule a medida
As afirmações verdadeiras são: de MN, sabendo que AB = 10, BC = 11 e AC = 12.

(A) apenas II. 14 ABC é um triângulo equilátero, X é um ponto interno, e M, N e P estão


(B) apenas I e II. sobre os lados AB, AC e BC, de forma que XM // AC, XN // BC e XP // AB.
(C) apenas II e III. Sabendo que XM = 3, XN = 4 e XP = 5, calcule o lado do triângulo ABC.
(D) apenas I e III.
(E) todas. 15 Em um triângulo ABC, AB = AC, e o ângulo  = 40°. Tomam-se os
pontos D e E sobre AB e AC, respectivamente, tais que os ângulos DCA e
05 ABCD é um quadrilátero convexo, de perímetro 2p. Podemos garantir EBC medem 15° e 35°, nessa ordem. Calcule BÊD.
então que AC + BD está entre:
16 O triângulo ABC é equilátero de lado 3 cm. Toma-se sobre o lado BC o
(A) 0 e p. ponto P. Seja Q o pé da perpendicular de P a AB, R o pé da perpendicular
(B) p e 2p. de Q a AC e S o pé da perpendicular de R a BC. Calcule a distância PB
(C) 2p e 3p. para que os pontos P e S coincidam.
(D) 3p e 4p.
17 Prove que a soma das distâncias de um ponto qualquer da base de
06 Prove que a medida da mediana traçada de um vértice em um triângulo um triângulo isósceles aos dois lados congruentes é constante e igual às
qualquer está entre a semidiferença e a semissoma dos dois lados consecutivos alturas iguais do triângulo.
a ela. [Dica: considere prolongar uma mediana AM de MA’ = MA.]
EXERCÍCIOS NÍVEL 3
07 Em um quadrilátero ABCD, no qual os ângulos ABC e ADC são retos,
01 Seja ABC um triângulo acutângulo, e BH e CJ alturas. Prolonga-se BH
tem-se que o ângulo ACD é o dobro do ângulo ACB, e também AB = 2.
de um segmento BX = AC, e prolonga-se CJ de um segmento CY = AB.
Calcule a medida de AD, sabendo que é um valor inteiro.
Prove que o triângulo XAY é retângulo isósceles.
(A) 4.
(B) 2. 02 Em um grande salão de baile há várias pessoas espalhadas, e as
(C) 3. distâncias entre elas são todas distintas. Uma pessoa A vai falar com outra
(D) 6. pessoa B se, dentre todas, B for a mais próxima de A, e depois retorna ao
(E) 5. lugar de origem. Uma pessoa por vez sai do seu lugar, e depois retorna.
Qual é o número máximo de saudações de pessoas diferentes que alguém
08 Seja ABC um triângulo qualquer, e P um ponto interno qualquer. Prove pode receber?
que a soma PA + PB + PC é maior que o semiperímetro e menor que o
perímetro do triângulo. (A) 2.
(B) 3.
09 Mostre que são menores que o perímetro de um triângulo:
(C) 4.
(D) 5.
a. a soma das alturas.
(E) 6.
b. a soma das medianas.

IME-ITA 255
Matemática V – Assunto 3

03 Dada uma reta, r, e dois pontos, A e D, em um mesmo semiplano 05 Dadas duas retas concorrentes, determine o lugar geométrico dos
gerado por r, determine os pontos B e C sobre r tais que BC = k, dado, e pontos P tais que a soma das distâncias de P a essas retas é constante
o comprimento da linha poligonal AB + BC + CD seja mínimo. e igual a k, dado.

04 No triângulo ABC, retângulo em A, a bissetriz interna de B intersecta a 06 Dadas duas retas concorrentes, determine o lugar geométrico dos
altura AH em E e o lado AC em M. A bissetriz interna de C intersecta AH em pontos P tais que a diferença das distâncias de P a essas retas é constante
F e o lado AB em N. Se AM = 2 cm, NA = 6 cm, o valor de EF, em cm, é: e igual a k, dado.

(A) 3. (D) 5.
(B) 4. (E) 6.
(C) 4,5.

RASCUNHO

256 Vol. 1
Quadriláteros A ssunto
4
Matemática V

1. Definições e De fato, também é possível provar que, se uma dessas propriedades


é satisfeita, então o quadrilátero é necessariamente um paralelogramo.
propriedades iniciais Além disso, vale que: se o quadrilátero convexo ABCD é tal que AB e CD
Quadrilátero é o polígono de gênero 4. Sempre possui duas diagonais. são segmentos paralelos e congruentes, então ele é um paralelogramo.
Pode ser polígono complexo, ou simples, e aí, convexo ou não convexo D C
como nas figuras abaixo. Caso seja simples, a soma dos seus ângulos
internos é 360°.
E K M
D
I
A
A
G B
H J L Na figura, ABCD é um paralelogramo.
B C F
ABCD é um quadrilátero EFGH é um quadrilátero IJKL é um quadrilátero
simples e convexo simples e não convexo auto intersectante 4. Retângulo
Dizemos que um quadrilátero é ‘retângulo’ se for um quadrilátero
Diferente dos triângulos, os quadriláteros possuem uma estrutura não equiângulo. Dessa maneira, todos os seus ângulos serão retos. Como
tão bem definida a partir de poucas informações [dessa maneira, é difícil todos os ângulos são congruentes, em particular os opostos são iguais;
inclusive estabelecer critérios de congruência de quadrilátero, de forma logo, todo retângulo é paralelogramo, herdando suas propriedades.
que não discutiremos a respeito disso]. Por outro lado, alguns quadriláteros
Uma propriedade que o retângulo tem a mais, diferente dos
possuem definições interessantes que levam a propriedades bastante úteis,
paralelogramos, é que suas diagonais são sempre congruentes.
e então recebem nomes especiais.
D C
2. Trapézio
Dizemos que se o #ABCD é convexo tal que AB//CD, então o O
quadrilátero é um ‘trapézio’ de bases AB e CD. Caso os outros lados
opostos não sejam paralelos, a eles chamaremos lados oblíquos.
A B
Existem 3 tipos especiais de trapézio:
ABCD é um retângulo de centro O, o ponto O equidista dos vértices
→ Trapézio isósceles: os lados oblíquos são congruentes;
→ Trapézio escaleno: os lados oblíquos não são congruentes; Obs.: Todo triângulo retângulo pode ser obtido através do traço de uma
→ Trapézio retângulo: um lado oblíquo é perpendicular às bases [costuma diagonal em um retângulo. Logo, pode-se deduzir que a mediana relativa
ser chamado de altura]. à hipotenusa sempre mede a metade dela. [Na figura, o triângulo ABD é
B retângulo, AO é mediana relativa à hipotenusa e vale a metade da diagonal,
C E H I L
ou seja, vale a metade de BD.]

5. Losango
A D F G J K Dizemos que um quadrilátero é ‘losango’ se for um quadrilátero
equilátero. Dessa maneira, todos os seus lados são congruentes. Em
ABCD é um trapézio isósceles, EFGH é um trapézio retângulo, particular, tem-se que os lados opostos são congruentes; logo, todo
IJKL é um trapézio escaleno.
losango é um paralelogramo, herdando assim suas propriedades.
Uma estratégia comum em resolução de problemas com trapézio é Uma propriedade que o losango tem a mais, diferente dos
traçar alguma paralela, normalmente a algum lado oblíquo, quebrando um paralelogramos [e, portanto, também dos retângulos], é que suas diagonais
trapézio em um triângulo e um paralelogramo. são perpendiculares. Também são bissetrizes dos ângulos internos.

D C
3. Paralelogramo
Dizemos que se o #ABCD é tal que AB//CD e AD//BC, então o
I
quadrilátero é um ‘paralelogramo’. Através de congruência de triângulos,
podemos provar as seguintes propriedades:

→ Os pares de ângulos internos opostos são congruentes; A B


→ Os pares de lados opostos são congruentes;
→ As diagonais do paralelogramo se bissectam, isto é, se cortam no ABCD é um losango, o ponto I equidista dos lados
ponto médio.

IME-ITA 257
Matemática V – Assunto 4

6. Quadrado A B
Dizemos que um quadrilátero é ‘quadrado’ se for um quadrilátero
regular, isto é, se for equiângulo e equilátero ao mesmo tempo. Dessa M N
maneira, todo quadrado é retângulo e losango ao mesmo tempo, portanto,
possui as propriedades deles. D C

D C MN é a base média no trapézio ABCD


AB + CD
MN//AB//CD E MN =
2
O

45° Observe que a base média passa pelos pontos médios das diagonais
do trapézio. O segmento formado pelos pontos médios das diagonais
B
A do trapézio é chamado de ‘Mediana de Euler’, e mede o módulo da
semidiferença entre as bases do trapézio.
ABCD é um quadrado de centro O
B
A B
A
7. Base média de triângulo
Dado um triângulo ABC, sejam M e N os pontos médios dos lados AB Q Q P
P
e AC. Dizemos que o segmento MN é uma base média relativa ao lado BC.
Nesse caso, valem as seguintes afirmativas: MN//BC e 2 · MN = BC, ou
D C D C
seja, a base média é paralela e igual à metade do lado ao qual ela é relativa.

A PQ é mediana de Euler em ABCD Sendo ABCD um trapézio, PQ é med. de Euler,


CD − AB
vale que PQ//AB//CD e PQ =
2
M N EXERCÍCIOS RESOLVIDOS

01. No trapézio ABCD, as bases AB e CD medem 17 e 32,


respectivamente. Sabendo que o ângulo interno em B é o dobro do
B C
encontrado em D, calcule a medida de BC.
MN é a base média do ABC
BC Solução
MN//BC e MN = Considere P sobre a base CD de forma que BP//AD. Dessa maneira,
2
#ADPB é paralelogramo, e, por transporte de ângulos via paralelismo,
os ângulos ADP, BPC e PBA são congruentes. Como B é o dobro de D,
Se, em um triângulo ABC, M é ponto médio de AB, e N está sobre AC
o ângulo PBC é igual também. Com isso, o triângulo BPC é isósceles,
de forma que MN seja paralelo a BC, então necessariamente N é ponto
com BC = CP = CD – PD = CD – AB = 32 – 17 = 15.
médio de AC.

A P 02. ABCD é um quadrilátero convexo qualquer. Prove que o quadrilátero


formado pelos pontos médios dos lados de ABCD é um paralelogramo.
K
Solução
M N Sejam M, N, P e Q médios de AB, BC, CD e DA, respectivamente.
J
No triângulo ABC, MN é base média relativa a AC; logo, MN//AC e
MN = AC/2. Analogamente, no triângulo ACD, PQ // AC e
B C Q I R PQ = AC/2. Logo, MN//PQ e MN = PQ, o que caracteriza #MNPQ
como paralelogramo.
M ponto médio de AB JK é a metade de QR
MN//BC implica N médio de AC JK não é a base média
EXERCÍCIOS NÍVEL 1
8. Base média de trapézio e 01 Em um trapézio retângulo, as bases medem 8 cm e 18 cm. Se um
dos ângulos internos do trapézio mede 45°, então a altura do trapézio é:
mediana de Euler
Usando argumentos de base média de triângulo, podemos provar (A) 12 cm.
o seguinte resultado: se AB e CD são bases de um trapézio de lados (B) 18 cm.
oblíquos AD e BC, e M e N são pontos médios desses lados oblíquos, (C) 13 cm.
então chamamos MN de base média do trapézio, e vale que MN//AB// (D) 10 cm.
CD e MN mede a semissoma das bases, ou seja, MN = (AB + CD)/ 2. (E) 9 cm.

258 Vol. 1
Quadriláteros

02 ABCD é um paralelogramo tal que AB = 6 e AD = 8. A bissetriz do ângulo 07 Seja ABCD um quadrilátero convexo, e M, N, P e Q pontos médios
interno em A corta BC no ponto E. Calcule a medida do segmento CE. de AB, BC, CD e DA, respectivamente. Estabeleça condições sobre as
diagonais AC e BD para que o quadrilátero MNPQ seja um:
03 Sobre os lados CD e AD de um paralelogramo ABCD, constroem-se,
externamente a ABCD, triângulos equiláteros CDE e ADF. Prove que o a. retângulo;
triângulo BEF também é equilátero. b. losango;
c. quadrado.
04 Prove que qualquer reta que passa pelo ponto I de interseção das
diagonais de um paralelogramo intersecta os lados opostos em dois pontos 08 ABC é um triângulo acutângulo, H é pé da altura traçada de A a BC, e
M e N tais que IM = IN. M, N e P são os pontos médios de AB, AC e BC, respectivamente. Calcule
o perímetro do quadrilátero MNPH, sabendo que AB = 8, BC = 12 e BH =
05 Dado o retângulo ABCD cuja diagonal mede 10 cm, inscreve-se nele 3.
um paralelogramo que possui os lados paralelos às diagonais do retângulo.
Qual é o perímetro do paralelogramo? 09 No interior do triângulo ABC, toma-se um ponto I sobre a bissetriz de
A de forma que o ângulo AÎB é reto. Sendo M ponto médio de BC, calcule
06 Duas retas perpendiculares entre si cortam os lados AB, BC, CD IM, sabendo que AB = 15, AC = 19 e BC = 20.
e AD de um quadrado nos pontos X, Y, Z e W. Prove que XZ e YW são
congruentes. 10 No quadrilátero convexo ABCD, AB e CD são congruentes. Prove
que a reta da mediana de Euler forma com esses lados ângulos também
07 As bases de um trapézio escaleno medem 3 cm e 9 cm. Os segmentos congruentes.
determinados pelas diagonais do trapézio sobre a base média são
proporcionais aos números: 11 Em um triângulo ABC, AM é mediana. No triângulo ABM, traça-se a
mediana BP, que corta AC no ponto Q. Calcule a razão AQ : QC.
(A) 1,1,1
(B) 1,2,1 (A) 1 : 1 (D) 2 : 1
(C) 1,3,1 (B) 1 : 2 (E) 2 : 3
(D) 1,4,1 (C) 1 : 3
(E) 2,3,4
12 São dadas duas paralelas. De um ponto A de uma delas, traça-se a
08 Em um paralelogramo ABCD, se E e F são os pontos médios de AB perpendicular comum AC e uma reta oblíqua AB. Uma reta traçada de B
e CD, prove que as retas DE e BF dividem a diagonal AC em três partes intersecta AC em E e a outra paralela em D, de forma que ED = 2 · AB.
congruentes. Sabendo que o ângulo ABC é de 60°, qual é a medida do ângulo DBC?

EXERCÍCIOS NÍVEL 2 13 ABCD é um paralelogramo, E é ponto médio de AD e F é a projeção


01 No trapézio ABCD retângulo, a base menor AB mede a metade do de B sobre EC. Prove que o triângulo AFB é isósceles.
lado oblíquo BC. Sendo M médio de BC, tem-se que o ângulo DMB mede
120°. Calcule o ângulo interno em C no trapézio ABCD. EXERCÍCIOS NÍVEL 3
01 ABCD é um quadrado de lado 2. Sobre os lados BC e CD marcam-se
02 As bases de um trapézio medem 10 cm e 8 cm e os lados não paralelos os pontos M e N de forma que MÂN = 45°. Calcule o perímetro do triângulo
medem 5 cm e x cm. Quais são os possíveis valores de x? MCN.

03 As diagonais AC e BD de um quadrilátero convexo ABCD se cortam 02 ABCD é um quadrilátero convexo qualquer. Prove que os segmentos
num ponto P. Os perímetros dos triângulos ABC e ABD são iguais, bem que unem os pontos médios dos lados opostos de ABCD e a mediana de
como os dos triângulos ACD e BCD. Mostre que ABCD é um trapézio Euler de ABCD são concorrentes no ponto médio deles.
isósceles.
03 Seja ABCD um trapézio de bases AB e CD, com AB < CD. Sabe-se
04 ABCD é um paralelogramo de lados medindo 7 cm e 10 cm. Calcule que o segmento que une os pontos médios das bases e a mediana de
o comprimento das diagonais do quadrilátero formado pelas bissetrizes Euler do trapézio são congruentes. Prove que os ângulos DAC e DBC são
dos ângulos internos de ABCD. obtusos.

05 Constroem-se quadrados sobre os lados de um paralelogramo, 04 Uma reta r contém o vértice A de um triângulo ABC. Sejam B’ e C’ as
externamente a ele. Prove que o quadrilátero formado pelos seus centros é projeções de B e C na reta r. Determine a posição de r que torna máxima
um quadrado, cujas diagonais são concorrentes com as do paralelogramo. a soma BB’ + CC’.

06 Em um quadrado ABCD, as retas r e s passam pelo vértice A, 05 Seja ABC um triângulo, e sejam P, Q e R simétricos do circuncentro
intersectando os lados do quadrado. Perpendiculares BB’, BB’’, DD’ e do triângulo ABC em relação aos lados AB, AC e BC, respectivamente.
DD’’ são traçadas em relação a essas retas. Prove que B’B’’ e D’D’’ são Prove que os triângulos ABC e PQR são congruentes.
congruentes e perpendiculares entre si.

IME-ITA 259
Círculos A ssunto
5
Matemática V

1. Definição e nomenclatura Um teorema importante: se P é um ponto externo a um círculo, e PA e


PB são segmentos tangentes traçados de P ao círculo, então PA e PB são
Dizemos que a circunferência de centro O e raio r é o conjunto {P|OP = r}. congruentes. Além disso, o segmento OP, que une o centro do círculo ao
O círculo de centro O e raio r é o conjunto {P|OP ≤ r}. O círculo é a união da ponto P, é bissetriz do ângulo APB.
circunferência com a região interna a ela, que, prova-se, é uma região convexa.
O comprimento de uma circunferência de raio r é dado por 2πr, sendo π
um número irracional, aproximado por 3.1415926535 (existem expressões
precisas para o cálculo de π).

3. Posição relativa entre círculos


De acordo com a posição entre dois círculos, podemos classificá-los
como:
2. Posições relativas a uma reta
• Concêntricos: são círculos que possuem o mesmo centro.
Como o círculo é uma região convexa, uma reta pode intersectar a
• Internos: um está totalmente contido no interior do outro.
circunferência em zero, um ou dois pontos, no máximo. Dizemos, nessa
• Tangentes internos: um tangencia o outro internamente. (Suas
ordem, que ela é externa, tangente ou secante ao círculo, de acordo com
circunferências só se intersectam em um ponto, no qual existe uma
o número de interseções com a circunferência (0,1 ou 2).
reta tangente comum.)
• Secantes: as circunferências se intersectam em dois pontos.
r: reta externa • Tangentes externos: um tangencia o outro externamente. (No único
s: reta secante ponto de interseção das circunferências, existe uma reta tangente
l: reta tangente comum.)
• Externos: os círculos não se intersectam.

No caso em que círculos são tangentes, interna ou externamente, os


centros e o ponto de tangência são sempre colineares. Dessa maneira, é
fácil calcular a distância entre os centros.
No caso em que círculos são externos, o menor segmento e o maior
segmento ligando os dois conjuntos estão contidos na reta que contém
os centros.

4. Ângulos no círculo
Uma vantagem dos círculos é a estrutura de transporte e cálculo
facilitado de ângulos que podemos deduzir a partir de algumas definições.
Demonstra-se que a reta tangente Para o que segue, seja um círculo de centro O.
a um círculo é perpendicular ao raio
traçado no ponto de tangência. Ou seja,
Ângulo central
se r é tangente a uma circunferência Se A e B são pontos sobre a circunferência,
de centro O no ponto T, então r é chamamos o ângulo AÔB de ‘ângulo central’. C
perpendicular a OT. Identificamos cada arco AB com seu respectivo
ângulo central AÔB, de forma que diremos que a B α
A
medida do arco é igual à medida do ângulo.

260 Vol. 1
Círculos

Consequências imediatas: A’ F
B’
• A circunferência mede 360˚. F’
P
• Um diâmetro define dois arcos de 180˚, que chamamos de E K
semicircunferência. K’
G’
• Se uma corda tem medida igual ao raio da circunferência, então o J
A B L
arco correspondente a ela mede 60˚. G
• O comprimento de um arco é proporcional ao ângulo central que o
determina.
^
Vejamos o caso em que A PB é excentrico interno.

B’ A’ Traçando a reta AB’ e usando ângulos inscritos



AB 
AB ' .
temos:
= AB ' B = e B ' AA '
P 2 2
^
Como A PB é externo do ∆PAB’ : APB = AB’B +
AB + AB
A B’AA’ =
2
Ângulo inscrito B
Obs.: Os outros casos podem ser demonstrados com a mesma ideia.
Se A, P e B são pontos da circunferência, dizemos que o ângulo
APB é ângulo inscrito na circunferência (seu vér tice está sobre a Em problemas de polígonos regulares, ângulos entre diagonais podem
circunferência). Por consequência da definição anterior, junto com o ser facilmente calculados através dos ângulos excêntricos. Lembre-se de
teorema do bumerangue, temos que o arco AB mede o dobro do ângulo que cada lado determina arcos iguais, e, fazendo as contas, de medida
APB, como mostra a figura. igual ao ângulo externo.
Dessa maneira, fixado um certo arco AB, todos os ângulos inscritos
que ‘olham’ para esse arco têm a mesma medida [ideia do arco-capaz]. Ângulo de segmento
Em particular, todos os ângulos inscritos em uma semicircunferência
medem 90˚. Se uma reta XA é tangente a um círculo no B
P3 ponto A, e AB é uma corda, dizemos que o ângulo 0
P P2
C XÂB é ‘ângulo de segmento’, e mede a metade do
P1 P4 arco AB contido na sua região interna.
α A x
P5 E
D De fato, basta considerar que a tangente é o caso limite da secante, de
modo que vale o mesmo resultado do ângulo inscrito.
B
A 5. Arco capaz
Dado um segmento AB fixo, e um ângulo  constante, o lugar
Vejamos o caso em que O é interno ao ângulo: geométrico dos pontos P tais que o ângulo APB mede  é o arco capaz
de  sobre AB. Ele é a únião de dois arcos de circunferência congruentes,
P Como OA = OP temos OÂP = O PA.
^
com extremidades em A e B, como na figura.
^ ^ ^
Da figura, O PB = A PB – O PA e como OP = OB P
^ ^ ^ ^
temos: O BP = O PB = A PB – O PA.
A ^
Do bumerangue ÂOB = OÂP + A PB + O BP =
^
O ^ 30° O
2 · A PB. __
Arco capaz de 30° sobre AB
__
B
Os pontos P e P’ “olham AB
A B
Obs.: O caso em que O externo pode ser feito de modo similar. segundo um ângulo de 30°

Ângulos excêntricos Sendo O centro do arco capaz,


temos AÔB = 2α = 60°
Se duas cordas AC e BD se intersectam no ponto P, interno ao círculo,
dizemos que o ângulo APB é ‘excêntrico interno’, e vale a semissoma 30°
dos arcos AB e CD para os quais tal ângulo olha. Caso as retas AC e BD
se intersectem fora do círculo no ponto P, dizemos que o ângulo APB é P’
‘excêntrico externo’, e mede a semidiferença entre os arcos AB e CD,
em módulo. Observe que um círculo de diâmetro AB, excetuando os pontos A e B,
é arco capaz de 90˚ sobre AB.

IME-ITA 261
Matemática V – Assunto 5

6. Quadrilátero inscritível EXERCÍCIOS RESOLVIDOS


Através dos resultados anteriores, podemos adquirir uma técnica para 01 Fixam-se os pontos B e C, e faz-se variar um ponto A de forma
incluir pontos em uma circunferência. Além do mais, essa ferramenta será que BÂC = 60°. Determine o lugar geométrico do incentro do triângulo
útil para transportar ângulos. ABC.
Primeiro, observe que três pontos colineares (ou ainda, um triângulo)
sempre determinam um círculo que passa por eles (círculo circunscrito Solução:
a eles). Nem sempre, portanto, quatro pontos estão em um círculo por Tem-se que, como  = 60°, então B + C = 120°, logo (B+C)/2
exemplo. Veremos duas condições interessantes para que isso ocorra. = 60°. Sendo I o incentro de ABC, no triângulo BIC, tem-se que BÎC
Dizemos que o #ABCD é inscritível quando existe uma circunferência = 180° – (B+C)/2 = 120°. Como B e C são fixos, e o ângulo BÎC
que passa pelos vértices. Para isso, existem dois critérios angulares é constante e igual a 120°, tem-se que I varia em um arco capaz de
iniciais: 120° sobre o segmento BC. (dado Â, tem-se BÎC=90° + A/2, como
visto no bloco de triângulos)
I. Um quadrilátero é inscritível se, e somente se, a soma dos ângulos
internos opostos vale 180˚. 02 O triângulo ABC está inscrito em um círculo, e uma corda KL
II. Um quadrilátero é inscritível se, e somente se, atende à “ideia do arco corta os lados AB e AC nos pontos P e Q. Sabendo que o #PQCB é
capaz”, ou seja, #ABCD é inscritível se, e somente se, os ângulos inscritível, prove que o triângulo AKL é isósceles.
BAC e BDC são iguais.
Solução:
D D Já que o #PQCB é inscritível, então os ângulos APQ e ACB são
A A
congruentes. Por soma de arcos, tem-se que, daí, os arcos AK e AL
β são congruentes, portanto as cordas AK e AL são congruentes. Segue
α α
daí o resultado.

B β B 03 O círculo exinscrito relativo a BC no triângulo ABC tangencia a


C reta AC em P. Prove que o segmento AP mede o semiperímetro do
C
triângulo.
α + β = 180° α=β
Solução:
Os ângulos opostos são Ângulos “olhando” o mesmo Sejam X e Q os pontos de tangência do exincírculo com os lados
complementares. arco iguais. BC e AB, respectivamente. Por segmentos tangentes iguais, tem-se:
CP = CX = x, BQ = BX = y, AP = AQ = c + x = y + b. Logo, no
7. Quadrilátero circunscritível triângulo ABC, 2p = AB + BC + AC = c + (x+y) + b = 2AP. Daí,
AP = p, o semiperímetro do triângulo.
Da mesma forma, podemos estabelecer critérios para que um
quadrilátero tenha seus lados tangentes a uma circunferência. 04 O círculo inscrito no triângulo ABC tangencia o lado AB no ponto
Todo triângulo possui um círculo inscrito, já que suas bissetrizes F. Prove que AF = p – a, sendo p o semiperímetro e BC = a.
são concorrentes. Em um quadrilátero, nem sempre isso ocorre. Caso
ocorra, o ponto de encontro é equidistante dos lados, logo é centro de Solução:
uma circunferência que tangencia os lados. Sejam D, E e F os pontos de tangência do incírculo com os lados
Existe o Teorema de Pitot: o #ABCD é circunscritível, ou seja, seus BC, AC e AB, respectivamente. Então: AE = AF = x, BD = BF = y,
lados tangenciam uma circunferência, se, e somente se, vale a seguinte CE = CD = z. Logo, no triângulo ABC, 2p = 2x + 2y + 2z, logo x
relação: AB + CD = AD + BC (as somas dos lados opostos são iguais). + y + z = p. Como BC = a = y + z, tem-se que AF = x = p – a.

05 Prove que o simétrico do ortocentro de um triângulo em relação


a qualquer lado pertence ao círculo circunscrito ao triângulo.

Solução:
Seja ABC o triângulo (considere por ora acutângulo), de ortocentro
H, e seja H* o simétrico de H com relação ao lado BC. Tem-se então que
são congruentes os ângulos HBC e HAC (os dois são complementares
do ângulo C interno). Por simetria, HBC e H*BC são congruentes, logo
H*BC e HAC são congruentes. Como A, H e H* são colineares, tem-se
H*BC = H*AC. Logo o quadrilátero H*BAC é inscritível, ou seja, H*
pertence ao circuncírculo de ABC. O caso em que ABC é obtusângulo
pode ser tratado por analogia.

262 Vol. 1
Círculos

EXERCÍCIOS NÍVEL 1 (A) 40°. (D) 140°.


(B) 110°. (E) 70°.
01 É dado um círculo A de raio 10 cm e dois círculos B e C tangentes a (C) 50°.
A internamente, e externamente entre si. Calcule o perímetro do triângulo
formado pelos centros dos círculos A, B e C. 13 Prove que entre cordas paralelas de uma circunferência estão arcos
congruentes.
02 AB é diâmetro de um círculo de centro O e raio 9 cm. Prolonga-se AB
de um segmento BP, e traça-se uma secante PMN, com M e N sobre o 14 Dois círculos são tangentes externamente em A. Traça-se uma secante
círculo, de forma que PM = OA. Calcule o comprimento de NA, sabendo aos círculos por A, que intersecta cada círculo em B e C, respectivamente.
que o ângulo BPM mede 20°. Prove que as tangentes em B e C são paralelas.

03 Um triângulo acutângulo ABC está inscrito em um círculo, de forma 15 Em um triângulo ABC, une-se o ponto médio da base BC aos pés H e
que AB é congruente ao lado do triângulo equilátero inscrito nesse círculo, H’ das alturas BH e CH’. Prove que o triângulo MHH’ é isósceles, e calcule
e BC é congruente ao lado do quadrado inscrito nesse círculo. Calcule o seus ângulos em função do ângulo Â.
maior ângulo interno do triângulo ABC.
16 ABC é um triângulo equilátero e BCD um triângulo retângulo e isósceles
04 O quadrilátero ABCD é convexo inscritível, e sua região interna contém em D, com D externo ao triângulo ABC. Sendo M médio de AB, determine
o centro do círculo no qual está inscrito. Sabe-se que AB e CD têm medidas o ângulo DMB.
iguais às do quadrado e do eneágono inscritos nesse círculo. Calcule o
ângulo formado pelas diagonais de ABCD. 17 Sobre um círculo marcam-se dois arcos AB e AC menores que o
semicírculo e uma corda DE que liga os pontos médios D e E dos arcos
05 ABCDEFGHIJKL é um polígono regular. Calcule o ângulo formado: AB e AC determinando-se sobre as respectivas cordas dois pontos F e G.
Prove que AF = AG.
(A) pelas diagonais AC e BD.
(B) pelas diagonais BE e DH. 18 Dois círculos de centro O e O’ se intersectam em A e B. Se AOC e
(C) pelos prolongamentos dos lados CD e HI. AO’D são dois diâmetros desses círculos, prove que:
(D) pelos prolongamentos das diagonais BD e HK.
a. CD é perpendicular a AB;
06 Um triângulo ABC está inscrito num círculo de raio 6 cm, e seu b. os pontos C, B e D são colineares.
perímetro mede 16 cm. Sabendo que  = 30°, então a soma AB + AC é
EXERCÍCIOS NÍVEL 2
igual a:

07 Um quadrilátero convexo ABCD está inscrito em uma circunferência, e 01 Considere um triângulo ABC de ângulos internos iguais a 50°, 60° e
suas diagonais se intersectam perpendicularmente no ponto P. Prove que 70°. Sendo DEF o triângulo cujos vértices são os pontos de tangência do
a altura traçada de P no triângulo ABP é colinear com a mediana traçada incírculo de ABC com seus lados, calcule os ângulos do triângulo DEF.
de P no triângulo PCD.
02 ABC é um triângulo cujo perímetro é 10 cm, e BC mede 4 cm. Sejam
08 Um hexágono regular ABCDEF e um pentágono regular AXYZW D e E sobre os lados AB e AC, respectivamente, tais que DE é tangente ao
estão inscritos em uma mesma circunferência. Sabendo que o arco CY incírculo de ABC. Calcule o perímetro do triângulo ADE.
é menor que 90°, e que CY é lado de um polígono regular inscrito nessa
circunferência, determine o número de diagonais desse polígono. 03 ABC é um triângulo escaleno retângulo em A, e AD é a bissetriz interna,
com D sobre BC. Toma-se o segmento DE perpendicular a BC, com E sobre
09 Prove que, em um triângulo retângulo de perímetro 2p e hipotenusa o lado AC. Calcule o ângulo BÊD.
a, o raio do círculo inscrito mede r = p – a.
04 É dado um círculo de centro C, e um ponto O externo a ele. Os segmentos
10 ABC é um triângulo isósceles de base BC, e ABPQ é um quadrado traçados de O tangentes ao círculo são OX e OY. Traça-se também uma secante
construído externamente a ele. Calcule a medida do ângulo QCB. OBA, cortando a circunferência em B e A, com B entre O e A. Sabendo que o
arco AX é o dobro do arco XB e o ângulo XOY é reto, calcule o ângulo BÔX.
11 A e B são dois pontos de uma circunferência que a dividem em arcos
de medidas proporcionais a 3 e 7. As tangentes traçadas por A e B formam 05 É dada uma circunferência e um ponto P, externo ao círculo, a partir
um ângulo igual a: do qual se traçam os segmentos tangentes PC e PD. Marca-se sobre a
circunferência um ponto A, e sobre AD um ponto B tal que AB = BC. Dessa
(A) 60°. maneira, BÂC = 80°. Calcule o ângulo PBC.
(B) 66°.
(A) 50°. (D) 60°.
(C) 72°.
(B) 40°. (E) 65°.
(D) 78°.
(C) 80°.
(E) N.R.A.
06 No triângulo acutângulo ABC, H é pé da altura de A. Os pontos M e N são
12 Em um quadrilátero ABCD convexo, os vértices B, C e D são
as projeções de H sobre os lados AB e AC, e os pontos P e Q são as projeções
equidistantes do vértice A. Se  = 140°, o ângulo C mede:
de M e N sobre os lados AC e AB, nessa ordem. Prove que PQ // BC.

IME-ITA 263
Matemática V – Assunto 5

07 Duas circunferências de raios R e r se cortam nos pontos A e B. 12 Em uma triângulo escaleno ABC, Â = 60°. Sejam H, I e O, respecti-
Em cada circunferência, a partir do ponto B, traçam-se cordas BQ e BN, vamente, ortocentro, incentro e circuncentro de ABC. Prove que:
que cortam a outra circunferência nos pontos P e M [de forma que Q e
M estão na circunferência de raio R, e P e N estão na circunferência de a. os pontos B, H, I, O e C estão num arco de circunferência.
raio r]. Sabe--se que AB é bissetriz do ângulo QBN. Assinale a proposição b. HI = IO.
verdadeira:
EXERCÍCIOS NÍVEL 3
(A) se R > r, então PQ > MN. 01 ABCD é um quadrado, e sobre os lados BC e CD tomam-se pontos M
(B) PQ = MN se, e somente se, R > r. e N de forma que MÂN = 45°. Os pontos P e Q são os pés das alturas de
(C) sempre PQ = MN. M e N no triângulo AMN. Prove que os pontos B, P, Q e D são colineares.
(D) AQ = NA e AM = AP.
(E) N.R.A. 02 Em um círculo de centro O, toma-se uma corda AB, e dentro dele
traça-se o triângulo ABC equilátero, com O interno ao triângulo. Sobre a
08 Dois círculos são tangentes no ponto A. O segmento BC é tangente circunferência marca-se o ponto D, tal que AD = AB, e a reta CD corta o
comum aos círculos. Prove que o ângulo BÂC é reto. círculo novamente em E. Calcule o comprimento de EB em função do raio
R do círculo.
09 A, B e C são três pontos de um círculo. A bissetriz do ângulo ABC
intersecta o círculo em D. Do ponto D traça-se uma corda paralela a AB que 03 Reta de Simson-Wallace – Prove que as projeções de um ponto do
intersecta o círculo em E. Se DE = 3 cm, quanto mede o segmento BC? circuncírculo de um triângulo sobre as retas suportes dos lados desse
triângulo são colineares.
10 Os círculos inscrito e exinscrito relativo a BC tangenciam o lado BC
do triângulo ABC nos pontos M e N. Prove que BM = BN. 04 (IME) Quatro retas concorrentes duas a duas determinam 4 triângulos.
Prove que os seus circuncírculos são concorrentes em um ponto.
11 Em uma circunferência de centro O, marcam-se os pontos P e Q, e
traçam-se tangentes AP e AQ ao círculo. Por um ponto M do menor arco 05 Seja ABC um triângulo e sejam P, Q e R pontos quaisquer nos lados
PQ, traça-se a tangente, que corta AP e AQ nos pontos B e C. Prove que, BC, AC e AB, respectivamente. Prove que as circunferências circunscritas
variando M, não variam: aos triângulos ARQ, BPR e CPQ se encontram em um ponto.

a. o perímetro do triângulo ABC.


b. o ângulo BÔC.

RASCUNHO

264 Vol. 1
Gabarito V olume
1
Matemática I 04 -
05 Letra C.
Assunto 1 06 Letra B.
07 Letra C.
Exercícios Nível 1 08 Letra D.
01 {2, 3} e {1}. 12 15. 09 Letra A.
02 Letra E. 13 Letra B. 10 Letra C.
03 Letra B. 14 Letra B. 11 Letra D.
04 Letra B. 15 Letra C. 12 Letra B.
05 Letra C. 16 Letra B. 13 Letra E.
06 200. 17 Letra B.
Exercícios Nível 2
07 9. 18 Letra B.
01 Letra D.
08 Letra A. 19 Letra B.
02 Letra A.
09 Letra A. 20 Letra B.
03
10 23. 21 Letra D.
a. –
11 48.
b. –
Exercícios Nível 2 c. –
01 Letra C. 08 Letra C. d. –
02 Letra D. 09 Letra E. 04 –
03 Letra D. 10 Letra D. 05 –
04 18. 11 Letra D. 06
05 Letra C. 12 Demonstração. a. –
06 Letra B. 13 Letra C. b. –
07 Letra C. 14 Letra D. 07 –
08 –
Exercícios Nível 3 09 –
01 Demonstração. 10 Soma: 15; número central: 5.
02 10%. 11 –
03 100%. Exercícios Nível 3
04 6. 01 –
05 Demonstração. 02 B é sempre maior ou igual a A (pode ser que A e B coincidam).
06 Demonstração. 03 O multiplicando é 1237 e o multiplicador é 893.
07 04 9567 + 1085 = 10652.
a. 101. 05 M(N – 1).
b. 102. 06 –
c. 97. 07 –
08 –
Assunto 2
09 –
Exercícios Nível 1 10 –
01 11 –
a. Falso. 12 –
b. Falso. 13 –
c. Verdadeiro.
Assunto 3
02
a. Se um quadrilátero não é um retângulo, então ele não é um quadrado. Exercícos Nível 1
b. Se o quadrado de um inteiro é par, então ele também é par. 01
c. ƒ(x1) = ƒ(x2)  x1 = x2. a. R = {(3, 2)}.
03 Conectivos errados: primeiro e quarto. b. R–1 = {(2, 3)}.
02 (0, 1), (0, 2), (0, 3), (1, 0), (1, 1), (1, 2), (2, 0), (2, 1), (3, 0).

IME-ITA 487
Gabarito

03 R1: de equivalência e de ordem; R2: simétrica e transitiva; 41 Letra E.


R3: antissimétrica; R4: transitiva, antissimétrica. T
04 Todos os múltiplos de 4. 42 .
a
05 –
06 43 Letra D.
a. {(1, 1), (2, 2), (3, 3), (1, 2), (2, 1)}, por exemplo. 44 Letra C.
b. [1]R={(1, 1), (1, 2)}; 45 –
[2]R={(2, 2), (2, 1)}; 46 –
[3]R={(3, 3)}. 47 Letra A.
c. {[1]R, [2]R, [3]R}. 48 Letra E.
07 49 –
a. F é função 50 Letra C.
b. FoG = {(1, 3), (1, 4), (2, 3), (3, 3), (4, 4)}. Exercícios Nível 2
08 – 01
09 Letra C. a. {-1, 1}.
10 Letra E. b. [2, + ∞).
11 Letra D. c. (0, 1].
12 Letra A.  1 1
d.  − ,  .
13 Letra C.  2 2
14 Letra A. 02 r(5) = 0; q(5) = 2.
15 Letra D. 03 –
16 Letra C. 04 Injetora e não-sobrejetora.
17 Letra C. 05 1000.
06 Letra B.
18 Letra B.
07 Letra A.
19 Letra D. 08 Letra C.
20 Letra D. 09 Letra C.
21 Letra A. 10 Letra E.
22 Letra A. 11 Letra D.
23 Letra C. 12 Letra C.
24 Letra C. 13 Letra B.
25 Letra B. 14
26 Letra D. a. –
27 b. ƒ(0) ou ƒ(0) = 1.
12 x + 3 −3 x + 15
a. g  f ( x ) = , f  g( x )= . c. –
5x − 3 x + 12 1 1
d. ƒ(2) = 2; ƒ(4) = 4; f( −2) = ; f ( − x ) .
27 9 2 f (x )
b. g  f ( 2) = , f  g( 2)= . 15
7 14 a. ƒ(1) = 0.
28
a. ƒ(x +1) = ax2 + (2a + b) x + a + b + c.  1
b. ƒ   = – ƒ(x).
b. ƒ(– x)= ax2 – bx + c. x
c. a = – b.
2x − 4 c. –
29 f (x − 1)= 2 .
x − 4x + 8 16 Letra C.
x2 17 Letra A.
30 g(x )= .
4 18 Letra B.
19 Letra E.
31 Letra B.
20 Letra C.
32 Letra C.
n3 − 8 n2 + 21n + 34
33 Letra B. 21 f (n) = .
n2 − 2 n − 2
34 Letra A.
35 Letra D. 22 Letra A.
36 y = 5 x − 1 . 23 Letra A.
24 Letra D.
3
37 f −1 = x + 3 x 2 + 3 x − 2 ; g-1(x )= 5 x + 1 ; g(g(x ))= 7 x − 8 . 25 Letra C.
2 3 − 2x 16 x + 23 26 Letra C.
38 I = [3, + ∞). 27 Letra C.
39 Letra A. 28 y = – 1 – x − 2 .
40 Letra E. 29 Letra A.

488 Vol. 1
Gabarito

30 Letra E. 18 Letra B
31 T = 4ª. 19
b − dx a. (a – 1)(a + 1)(a2 + 1)
32 ƒ–1(x) = .
cx − a b. (a– 1)(a + 1)(a2 + a + 1)(a2 – a + 1)
c. (a2 + 1)(a2 + a + 1)(a2 – a + 1)
33 Letra E.
d. (a – 3)2 (a + 3)2
34 Letra B.
e. (a – 1)2 (a + 1)2 (a2 + a + 1)2(a2–a+1)2
35 Letra E.
f. (a – 1)(a2 + 1)(a2 + a + 1)
36 Letra D.
g. (a – 1)(a + 1)3
37 Letra A.
h. (a – b+c)(a + b – c)(–a + b+c)(a + b+c)
38 Letra A.
i. (c – 1)(c + 1)(c2 – ab)
39 Letra B.
j. (a2 + 1)(2a2 + a + 2)
k. (a2 – 2)(a2 + 3a + 6)
Exercícios Nível 3 20 -
01 Letra E. 21 -
02 ƒ (x) = 4x2 – 115. 22 Letra A
03 – 23 -
04 Letra E. 24 -
05 2011. a. 3 2 − 1
06 0.
1+ 1+ 4a 1
07 x = ;a ≥ − b. 1 – 3 2
2 4
1 3 25 2
08 x = − a ± a2 − a + 26 b ≠ 2a
2 16
1 18
09 27
1999 7
10 x + 1 28
11 Não. a
x2 + x − 1 a.
12 a2 − 1
2x
( a + b + c)2
13 61. b.
14 990. 2 bc
c. 0
d. 0
Matemática II 1
e.
a+c
Assunto 1
29 -
Exercícios Nível 1 30 Letra D.
01 0 31 Letra A.
1
02 s = −1; 4; 2; −  32 Letra E.
 5
33 Letra E.
03 x = 1
34
5 a. (a2 + ab + b2)(a2 – ab + b2)
04 ∅ b. (a + 1)(a – 1)(a2 + 5)
05 ∅ c. (a2 + 1)(4a2 + 1)
06 Letra E. d. (a + 1)(a2 + a + 1) (a2 – a + 1)
07 Letra A. e. (a2 + 6a +18)(a2 – 6a + 18)
08 2x² f. (a2 + a + 1)(a2 – a + 1)
09 10¯¹0 g. (a2 + 6 a + 3)(a2 – 6 a + 3)
10 o segundo h. (a2 + 2ab + 2b2)(a2 – 2ab + 2b2)
11 Letra A. i. 3(a + b)(b + c)(c + a)
12 Letra B. j. 3(a – b)(b – c)(c – a)
13 Letra D. k. 3(a + c)(a – c)(a2 + b2)(b2 + c2)
14 Letra C. l. 5ab(a + b)(a2 + ab + b2)
15 - 35 (a + b)(b + c)(c + a)
16 - 36 |a|
17 -

IME-ITA 489
Gabarito

37 a2 19
2− 4 8
38 2 a.
39 2 2
40 b. 3
4 −1
3 2 + 2 3 + 30
a.
12 c. 1 + 3 2 + 3 4
( 4 5 + 4 3 )( 5 + 3 )
b.
2 d. 3
3−32

c. 2 ( 3 + 5)
2 e. 2 + 3 2 ( 3 3 − 2 )( 4 + 2 3 9 + 3 3 3 )
20 Letra B.
21 − 15 − 14 + 10 21 Letra D.
d.
2 22 Letra B.
23 Letra C.
41 |a+2b+3c|
24 502
42 9
25
43 A 2 2 2 2
a. ( a + a + 1)( a − a + 1)( a + 3 a + 1)( a − 3 a + 1)
44 2 / 3 2 2
a +1 b. ( a + 1)( a + a + 1)( a − a + 1)
a. c.
−6 b
4 c. ( a + b)(b+ c)(c+ a)
2 4 ab b d. ( a + 2 b)( 2 b− c)( a − c)
b. d.
6
4
a+4 b a e. ( a − 2c)(b− 2c)( a + b)
f. a( a − 1)(a + 1)( a − 2)( a + 2)(a − 3)( a + 3)
45 Letra C.
g. ( a − b)( b − c)( c − a)( ab + bc + ca)
46 Letra C.
h. ( b + c)( 2 a − b)( 2 a + c)( 2 a + b − c)
47 546 2
i. (a − 1)(a + 3)
48 2 2
2
2 a2 + 3 j. (a +3a + 1)
a. 52 a + 4 e. 2
a + a+1 3 a2 − 3 k. ( a + 2)( a + 6)( a + 8 a + 10)
2 2
l. (3 a + 4 a − 1)(3 a + 2 a + 1)
a4 + 1 5 a2 − 3 b m. ( a − b)( b − c)( c − a)( ab + bc + ca)
b. f.
a+1 a2 + 2 n. ( a + b + c)( ab + bc + ac)
2 2
a2 − 1 1 o. ( a + 3 a + 1)
c. g.
a − 2 a2 + 4
4
a2 − b 2 p. ( a + b + c)( a + b − c)( a − b + c)( a − b − c)
q. ( a − ab − b2 )( a2 + 3 ab + b2 )
2
d. a − 4  3 4 3 4   3 3 2 3 3  
a2 + 5 26 S =  ,  ,  ,  

49 —  2 2   3 3  

Exercícios Nível 2 27 S = {(1, 2, 3 )}


01 Letra C. 28 Dica: analise inicialmente a 1a equação separadamente.
02 Letra E. 29
16 a15
03 - a.
1 − a16
04 Letra B.
05 u2 = u 2 − 2 , u3 = u 3 − 3u , u4 = u 4 − 4 u 2 + 2 , u5 = u 5 − 5u 3 + 5u 32
06 S = {(63, 58),( 459, 58),( −63, −58),( −459, −58)} b.
1 − a32
07 Letra E.
08 bc 5
09 Letra D. c.
a(a + 5)
10 -
11 - 30 Dica: o dado do problema equivale a ( x + a)( x + b)(a + b) = 0 .
12 - 31
13 - a. 0
14 - b. XXXXXXXXXXXX
15 - c. 2a
16 todos a e b reais 32 Letra B.
17 Letra B.
18 Letra D.

490 Vol. 1
Gabarito

Exercícios Nível 3 Matemática III


01 Letra C.
02 1057 Assunto 1
03 -
Exercícios Nível 1
04
a. ( a + b + c)(a 2 + b2 + c2 − ab − bc − ca) 01 Letra B.
b. = 3 3 02 Letra C.
x a= , y b= , z c3 .
05 Letra A. 03 Letra B.
06 Letra B. 04 Letra B.
07 2 05 Letra C.
08 Dica: escreva 1 = ( a + b + c)
2
06 Letra C.
3 3 07 4.
09 08 105/11 ou 105/23.
2 09 –
a.
2 10 20.
−3 + 7 3 2 − 3 4 11 Letra B.
b.
23 12 4, 6, 8, 10, 12 ou 12, 10, 8, 6, 4.
10 13 42.
a. - 5
14 .
b. - 2
c. Dica: tome conjugado dos dois lados. 15 Letra A.
1 1 1 1 1 1 16 1.329.
11 Dica: faça x = ,y = ,z = e use que + + = 0 . 17 2, 5, 8, 11, 14, 17, 20, 23, 26, 29.
a−b b−c c−a x y z
18 –
12 -
19 –
13 0 (a expressão é constante e igual a 2)
14 - 20
1 a. 4.
15 A expressão é igual a b. 31.
16 2010 a + b +c
21 2n+1.
17 Letra D.
22 –
18 S = {(111
, , ,),( −1, −1, −1)}
23 Letra E.
  1 a + 1  24 Letra D.
19 S = (111
, , ),  a, − ,  , a ∈ ℜ − {0, −1}
  a + 1 a   25 Sim.
26 4, 12 e 36.
 1 27
(obs: a solução  −2,1, −  encontrada já está sendo contada.)
 2 a. 14/99. d. 10.304/9.999.
b. 19/55. e. 77/45.
20 S=∅
c. 1. f. 67/45.
21 Letra D.
28
22 ( a + b)2 ( a2 − 4 ab − b2 )
23 1
a. < a < 2. b. 125 .
a. (x 2 − x + 3)(x 2 + x − 1) 2 8
b. (x 2 + x + 2)(x 2 + 3 x + 3)
c. (x 2 + 2 x + 1)(x 2 − 2 x + 2) Exercícios de matemática financeira
24 29 R$ 15.600,00.
a. (x 2 + x + 1)(x 3 − x 2 + 1) 30 2 meses e 21 dias.
b. (x 2 + x + 1)(x 8 − x 7 + x 5 − x 4 + x 3 − x + 1) 31 Letra C.
c. (x + 1)(x 2 + 1)(x 8 − x 7 + x 4 + x 4 − x 2 + 1) 32 Letra A.
25 Letra B. 33 Letra C.
16 34 40%.
26
11 35 Letra C.
27 Letra E 36 Letra C.
37 Letra D.
38 Letra C.
39 Letra A.
40 Letra B.

IME-ITA 491
Gabarito

Exercícios Nível 2 06 9 / 32.


a 07 6 − 2 .
01 Sim, 5 = a1a10 .
a6
02 142. Assunto 2
03 19.800. Combinação, arranjo e permutação
04 an2.
05 34. Exercícios Nível 1
06 65. 01 300.
07 Não. 02 3.168.
08 – 03 4.464.
09 a. – 2. b. 3/22. 04 2.350.
10 n + 2 05 30.
10 = 33334 (n dígitos).
3 06
11 – a. 11.520. e. 720.
12 a. 13 m. b. 5 s. b. 720. f. 9.360.
13 750. c. 4.320. g. 13.080.
14 2. d. 1.152.
15 Letra C. 07 Letra D.
16 Letra C. 08 Letra A.
17 (n – 1) / (a1 . an). 09 Letra A.
18 9 / 10. 10 Letra B.
19 – 11 Letra B.
10 n+1 − 10 − 9.n 12 Letra A.
20 .
81 13 Letra A.
21 (12, 12, 12) ou (8, 12, 18). 14 12.960.
5 −1 15 Letra D.
22 . 16 72.
8
17 62.
23 Não. 18 Letra C.
24 – 19 Letra C.
1 20 28.
25 .
18
21 17.
26 a. 31. b. 13.

27 a0 + n ( a1 − a0 ) +
( n − 1) n K . Exercícios Nível 2
2 01 10.395.
2009


1 02 Letra D.
28 (1 + ).
j =1
j +1 03
29 Não. a. 840.
n( n + 1)( 2 n + 7) b. 2401.
30 .
6 c. 1512.
31 9.400. 04 Letra C.
32 p(p + 1)(p – 1)(3p + 2) / 24. 05 512.
11
06 C22 9 .
33 171.700 − C20
b1 r
34 a1n + n ( n − 1) + n ( n − 1) ( n − 2 ). 07 30.
2 6
08 2.520.
35 2.
09
a. 186.
Exercícios Nível 3 b. 20.
1 n+2
01 (2 − n − 3). 10 280.
3
11 Letra B.
02 a2 = a3 = ... = a100. 12 Letra B.
03 1. 13 42.4051.
3 14 ( n − 1)!
04 r = .
2 2
05 2. 15 138.

492 Vol. 1
Gabarito

16 2·3n – 2. Matemática IV
17 62013.
18 1.012.036. Assunto 1
19 2835.
20 Letra B. Definições, relações básicas e adição de arcos
Exercícios Nível 1
Suluções inteiras e não-negativas 01 Letra A.
Exercícios Nível 1 02 Letra B.
03 Letra A.
01 Letra A.
04 Letra D.
02 Letra E.
05 Letra D.
03 Letra B.
06 {(x,y) ∈ 2|(k . 180º + 290º, –k . 180º + 110º), k ∈ Ζ}.
04 3.003.
07
05 a. 3.240. b. 3.711.
a. 0.
06 (n + p)!/n!p!.
b. sen α.
07 Letra D.
c. 3.
08 279.417.600.
d. –1.
09 Letra B.
e. 2.
08
Exercícios Nível 2 a. 9.
01 Letra B. b. 5.
02 96. 09 1.
03 735. 10 Letra D.
04 1.016. 11 Demonstração.
05 3n2 + 3n + 1. 12 Demonstração.
06 376.320. 13
07 540. a. ± 1 + 2m .
08 2n – 1 b. 1 – 2m2.
n
09 3 + 1 . 14 5.
2 15 – 4/5, 4/3.
16 Letra C.
Exercícios Nível 3
01
a. 2.418. Exercícios Nível 2
b. 15. 01 Letra B.
c. 48. 02 Letra C.
d. 8. 03 Letra C.
02 a. 126. b. 70. 04 Demonstração.
03 12.504. 05 Demonstração.
04 98.475. 06 tan a = 1 – tan a . tan b.
05 300. 07 Demonstração.
06 191.300. 08 Demonstração.
k k
07 ∑ ( −1)i   ( k − i )n 09 Letra B.
i =0 i  10 Letra C.
11 Letra D.
08
12 y ± x = kπ; k ∈ Ζ.
a. Fn+2 .
13 ab/(a2 + b2).
b. Cnk− k +1 .
14 b ≥ 4.
09 37.584.
10 - 3 ± 57
15 .
11 - 12
12 151.200. 16 k = 3, 30º e 60º.
13 25. 17 a2 + b2 + c2.
18 a2(m – b)/(m – a) = b2 (n – a) = b2 (n – a)/(n – b).
19 q.
20 –1.
21 Letra A.

IME-ITA 493
Gabarito

Transformações trigonométricas Equações e inequações trigonométricas


Exercícios Nível 1 Exercícios Nível 1
01 Letra E. 01 Letra E.
02 Letra E. 02 Letra A.
03 Letra C. 03 Letra B.
04 Letra E. 04
05 Letra C. a. {x ∈ U|x = 2kπ|3 ∨ x = (2k + 1)π/7; k ∈ Ζ}.
06 Letra A. b. {x ∈ U|x = 2kπ/7 + π/14 ∨ x = 2kπ – π/2; k ∈ Ζ}.
07 Letra C. c. {x ∈ U|x = 2kπ|5 ∨ x = (2k + 1)π/3; k ∈ Ζ}.
08 Letra B. d. {x ∈ U|x = kπ/6; k ∈ Ζ}.
09 Letra C. e. {x ∈ U|x = kπ – π/4 ∨ x = kπ/16; k ∈ Ζ}.
10 Letra D. f. {x ∈ U|x = kπ/5 + π/10 ∨ x = kπ/3 + π/6; k ∈ Ζ}.
11 4. g. {x ∈ U|x = kπ/3; k ∈ Ζ}.
π 3π
12 tan = 2 − 1 e tan 2 + 1.
= h. {x ∈ U|x = kπ/3 ∨ x =(2k + 1)π/6; k ∈ Ζ}.
8 8 i. {x ∈ U|x = kπ/4 + π/8; k ∈ Ζ}.
13 Demonstração. 05
14 3. a. {x ∈ U|x = kπ + (–1)kπ/6; k ∈ Ζ}.
15 Letra A. b. {x ∈ U|x = kπ ± π/4 ∧ π = kπ ± π/3; k ∈ Ζ}.
16 Demonstração. 06 {x ∈ U|x = kπ ± π/3 ∨ x =kπ; k ∈ Ζ}.
17 0 ou 3. 07 S = {x ∈ |x = kπ + π/4; k ∈ Ζ}.
18 Letra E. 08 Letra C.
Exercícios Nível 2 09 Letra D.
01 Demonstração. 10 Letra D.
11 Letra D.
02 Demonstração.
03 Demonstração. 12 ]π/6, π/2[∪]π/2, 5π/6[.
π π π
04 Letra A. 13 − + 2 k π < x < 2 k π, + 2 k π < x < + 2 k π, π +
2 4 2
7 −2
05 . 5π
3 2k π < x < + 2 k π. k inteiro.
2ab 4
06 2 . 14 Letra C.
a + b2
15 Letra A.
07 Demonstração.
08 Demonstração.
Exercícios Nível 2
09 4.
01 Letra A.
10 Demonstração. 02 Letra B.
11 Letra B. 03 Letra E.
12 Demonstração. 04 Letra B.
13 Letra B. 05 Letra B.
2 06 Letra E.
14 .
5 07 Letra A.

15 Demonstração. 08 {x ∈ U|x = kπ ∨ x = kπ + arctan 4/3; k ∈ Ζ}.


5 09 S = {x ∈ U|x = kπ + arctan 1/2 ∨ x = kπ + π; k ∈ Ζ}.
16 .
9 10 S = {x ∈ |x = 2kπ∧x =+ π/6 + kπ; k∈ Ζ}.

17 Demonstração. 11 S = {x ∈ |x = kπ + π/4; k ∈ Ζ}.


18 Demonstração. 12 S = {x ∈ |2kπ + π/6 < x <2kπ + π/4 ∨ = 2kπ + 3π/4 < x < 2kπ
19 b ≥ 4. + 5π/6; k ∈ Ζ}.
20 1. 13 ]π/6, π/2[∪]π/2, 5π/6[.
21 Demosntração. 14
22 4. a. S = ]π/4, π/3]∪[π/2, 3π/4[;
1
23 . b. S = {x ∈ U|2kπ + π /4 < x < 2kπ + 11π/12 ∨2kπ + 5π/4 < x< 2kπ
16
+ 19π/12;k e Ζ}.
24 – 15 Letra D.

494 Vol. 1
Gabarito

Funções trigonométricas senn


10 .
sen1 ⋅ sen ( n + 1)

Exercícios Nível 1
11
01 30°.
a. Demonstração.
02
b. S = {x ∈ |x = tan(kπ/3 + (arctan m)/3); k = 0,1, 2}.
a. π/6.
12 Letra E.
b. 7π/6.
13 Letra C.
c. 2 2 .
14 –
d. π/2.
15 –
03 Letra A.
16 ∅
04 Letra C.
17 a = b.
05 Letra B.
18 Demosntração.
06 Letra B.
19 –
07 Letra D.
20 − 1
08 Letra C. 2014
09 Letra C.
10
a. 6π.
Matemática V
b. 3π/2.
c. 2. Assunto 1
d. π. Exercícios Nível 1
e. π.
01 Letra B.
f. 4π
02 16.
11 Letra D.
03 –
Exercícios Nível 2 a. 61° 18’ 2’’.
3
01 . b. 32° 40’ 50’’.
3 c. 21° 35’ 8’’.
02 – d. 18° 48’ 31’’.
03 ∅. 04
04 Letra B. a. 180° – α.
α
05 Letra A. b. 90° − .
2
06 Letra E.
α
07 Letra C. c. 300° + .
3
08 Letra A. d. α.
09 Letra D. 05 20°.
10 = 2π e A = 13. 06 45°.
 13 13  07 60°.
11 6 − ,6 + . 08
 2 2 
a. 40°.
12 Letra E. b. 58°.
13 Letra C. 09 155°.
14 Letra E. 10 –
15 Letra E. 11 Letra A.
 π 12 –
16 a∈  0, 
 4
Exercícios Nível 2
Exercícios Nível 3
01 –
01 Letra A.
02 k = 2.
02 Letra A.
03 285°.
03 Apenas a afirmativa IV é verdadeira.
π  04 Letra C.
04  + k π : k ∈ Z 
12
  05 135°.
05 .
8 06 35°.
06 x = ±π /2.
07 13h 07min.
07 Demosntração.
08 Aproximadamente 13h 4min 33s.
08 Demonstração.
09 120°.
sen n°
09
sen2 1° sen( n + 1)

IME-ITA 495
Gabarito

10 Letra A. 03 –
11 Letra E. 04 –
12 Letra A. 05 –
13 Letra A. 06 Letra E.
14 07 –
Figura 1: x = 45°. 08 Letra C.
Figura 2: x = 65°. 09 Letra C.
Exercícios Nível 3 10 –
01 Letra A. 11 3 < x <19
02 – 12 –
03 135°. 13
04 24°. a. 90° + A/2 c. 90° – A/2
b. 180° – a. d. A/2
14 26.
Assunto 2 15 a = 20°
Exercícios Nível 1 16 Letra B.
17 18°
01
18 15°
a. 15°.
b. 75°.
02 54°. Exercícios Nível 2
03 Letra E. 01 Letra A. 10 Letra C.
04 135°. 02 Letra B. 11 Letra C.
05 14. 03 Letra E. 12 30°
06 n(2n – 4). 04 Letra C. 13 22/3
07 Letra D. 05 Letra B. 14 12
08 Eneágono e hexágono. 06 – 15 75°
09 108° e 135°. 07 Letra C. 16 2 cm
10 Letra E. 08 – 17 –
09 –
Exercícios Nível 2
01 84°. Exercícios Nível 3
02 24. 01 –
03 7. 02 Letra D.
04 140°. 03 –
05 12°. 04 Letra B.
06 8. 05 É um retângulo cujas diagonais estão apoiadas sobre as retas dadas.
07 54. 06 –
08 18.
09 20. Assunto 4
10 Letra C.
11 Letra D. Exercícios Nível 1
12 Letra B. 01 Letra D.
13 – 02 CE = 2
03 –
04 –
Exercícios Nível 3 05 20 cm
01 77. 06 –
02 – 07 Letra B.
03 EF = 5 e FA = 9. 08 –
04 – Exercícios Nível 2
01 80°
Assunto 3 02 3 < x < 7
Exercícios Nível 1 03 –
01 Letra D. 04 3 cm e 3 cm
02 05 –
a. – 06 –
b. –

496 Vol. 1
Gabarito

07 10 45°
a. AC perpendicular a BD 11 Letra C.
b. AC = BD 12 Letra B.
c. AC = BD e AC perpendicular a BD 13 –
08 17
14 –
09 2
15 180° – 2A, A e A
10 –
11 Letra B. 16 45°
12 10° 17 –
13 – 18 –
Exercícios Nível 3
01 4
Exercícios Nível 2
02 – 01 55°, 60° e 65°.
03 – 02 2 cm
04 A reta r deve ser perpendicular à mediana relativa a BC. 03 45°
05 – 04 15°
05 Letra C.
06 –
Assunto 5 07 Letra C.
Exercícios Nível 1 08 –
09 3 cm
01 20 cm
10 –
02 9 cm
11 –
03 75°
12 –
04 65°
05
a. 30° Exercícios Nível 3
b. 75° 01 –
c. 30° 02 –
d. 15° 03 R
06 10 cm 04 –
07 – 05 –
08 90
09 –

ANOTAÇÕES

____________________________________________________________________________________________________________________

____________________________________________________________________________________________________________________

____________________________________________________________________________________________________________________

____________________________________________________________________________________________________________________

____________________________________________________________________________________________________________________

____________________________________________________________________________________________________________________

____________________________________________________________________________________________________________________

____________________________________________________________________________________________________________________

____________________________________________________________________________________________________________________

____________________________________________________________________________________________________________________

____________________________________________________________________________________________________________________

____________________________________________________________________________________________________________________

IME-ITA 497
Binômio – Trinômio – Modular A ssunto
4
Matemática I

1. Função do 1o Grau 1.3 Crescimento e decrescimento


(Função afim) Teorema 1 (monotonismo em função do sinal do
coeficiente angular):
1.1 Definição Dada uma função ƒ(x) = ax + b (a ≠ 0), temos:
É uma função da forma ƒ(x) = ax + b, com a, b ∈ e a ≠ 0. O real I. ƒ é estritamente crescente se a > 0;
a é chamado de coeficiente angular de ƒ, enquanto b é o coeficiente linear. II. ƒ é estritamente decrescente se a < 0.

Demonstração: Faremos o caso I e deixaremos II como exercício.


Ex.: ƒ1(x) = 2x + 1, ƒ2(x) = – π2x + 3
I. Devemos provar que se x > y, então ƒ(x) > ƒ(y). Temos que
Obs. 1: As raízes (ou zeros) de uma função ƒ real são os valores que ƒ(x) – ƒ(y) = ax + b – ay – b = a(x – y). Como a > 0 e x > y,
1 segue que ƒ(x) – ƒ(y) > 0 ⇔ ƒ(x) > ƒ(y).
anulam tal função. Por exemplo, a raiz de 2x + 1 é x = − . Mais
b 2
geralmente, a raiz de ƒ(x) = ax + b é x = − .
a
1.4 Sinal do binômio
b
Obs. 2: Uma função do 1o grau é dita linear se b = 0, ou seja, uma função Usando o teorema 1 e considerando que f ( x ) = 0 ⇔ x = − ,
concluímos que: a
é linear se é da forma ƒ(x) = ax, a ≠ 0.

1.2 Gráfico  b
O gráfico de uma função afim é uma reta. Desta forma, para efetuar  x > − a ⇒ f ( x ) > 0
a construção de tal gráfico, basta que conheçamos dois de seus pontos. I. Se a > 0, então: 
x < − b ⇒ f ( x) < 0
 a
Obs.: Graficamente, o coeficiente angular é dado pela tangente do ângulo
de inclinação da reta, e o coeficiente linear é a ordenada do ponto de  b
interseção da reta com o eixo y.  x > − a ⇒ f ( x ) < 0
II. Se a < 0 , então: 
Ex.: Construa o gráfico de y = 3x + 2 . x < − b ⇒ f ( x) > 0
 a
Para fazer a construção, basta conhecermos dois pontos. Neste
caso, escolhemos as interseções da reta com os eixos coordenados:
 2  Podemos montar o seguinte esquema:
A =  − , 0  e B = ( 0, 2 ).
 3 
a > 0:
4 +

3

2 B
–b
a

1
a < 0:
A 0
–3 –2 –1 0 1 2 3 +
–1


–2 –b
a

IME-ITA 97
Matemática I – Assunto 4

2. Função do 2o grau (função Também temos que

quadrática) ( −b + ∆ )( −b − ∆ ) = b 2
− ∆ 4 ac c
P = x1 x 2 = 2
= 2 = .
2.1 Definição 4a 4 a2 4a a
É uma função da forma ƒ(x) = ax + bx + c, com a, b, c ∈  e a ≠ 0 .
2
−b + ∆ + b + ∆ ∆
Finalmente, segue que D = x1 − x 2 = =
2a a
Obs.: a é dito coeficiente líder da função quadrática.
Obs.: As seguintes identidades podem ser úteis:
Ex.: ƒ3(x) = 3x2 – 5x + 10
x12 + x 22 = ( x1 + x 2 ) − 2 x1x 2 = S 2 − 2 P e x13 + x23 =
2

2.2 Gráfico
= ( x1 + x 2 ) − 3 x1x 2 ( x1 + x 2 ) = S3 − 3SP.
3

O gráfico de uma função quadrática é uma parábola. Isso será


demonstrado na apostila de Matemática IV, na parte de cônicas.
2.4 Forma canônica
Obs.: Quando a > 0, a parábola tem concavidade voltada para CIMA e Em Matemática, é comum transformar o formato de determinados
quando a < 0, a parábola tem concavidade voltada para BAIXO. objetos para que seja mais conveniente de se trabalhar e de onde possam
ser extraídas informações relevantes. Por exemplo, escrevemos um número
2.3 Raízes inteiro positivo como produto de primos, escrevemos a equação de um
Como encontrar os valores de x tais que ax2 + bx + c = 0? Para círculo na forma (x – h)2 + (y – k)2 = r2. Essas são chamadas formas
determinados valores de a, b, c, essa tarefa é relativamente simples. Por canônicas. Para uma função quadrática, podemos escrever:
exemplo, x2 – 5x + 6 =0 é fácil de ser resolvido, pois podemos usar
o Produto de Stevin e chegar a (x – 2)(x – 3) = 0 ⇔ x = 2 ∨ x = 3.  b c  b   b2 − 4 ac  
2
f ( x ) = ax 2 + bx + c = a  x 2 + x +  = a  x +  −   =
Entretanto, o caso geral é um pouco mais sofisticado e então recordaremos  a a  2 a   4 a2  

aqui a demonstração vista na apostila 1 de Matemática II. Usaremos a
técnica de completar quadrados: ax2 + bx + c = 0 ⇔ 4a2x2 + 4abx +  b 
2
∆ 
4ac = 0. Agora, temos que (2ax + b)2 = b2 – 4ac. Sendo ∆ = b2 – 4ac = a  x +  − 2  , sua forma canônica. Nos tópicos seguintes,
 2a  4 a 
(discriminante), chegamos a: 

Teorema 2 (Fórmula de Bhaskara) veremos como extrair informações relevantes da forma canônica.
As raízes da equação do 2o grau ax2 + bx + c = 0, a ≠ 0, são dadas
−b ± ∆ 2.5 Máximos e mínimos
por , em que ∆ = b2 – 4ac. 2
2a Veja que f ( x ) = a  x + b  − ∆ . Logo, se a > 0, temos que
Obs.: A fórmula de Bhaskara também é verdadeira quando os coeficientes  2a  4 a
da equação são complexos! 2
 b  ∆
Teorema 3 (Número de raízes reais em função do a  x +  ≥ 0 e, portanto, f ( x ) ≥ − , com igualdade se e somente
 2a  4a
discriminante)
b ∆
Dada uma equação do 2o grau com coeficientes reais, temos: se x = − . Da mesma forma, se a < 0, segue que f ( x ) ≤ − com
2a 4a
b
I. ∆ > 0: a equação possui duas raízes reais distintas. igualdade se e somente se x = − . Temos então:
2a
II. ∆ = 0: a equação possui duas raízes reais iguais (raiz dupla).
III. ∆ < 0: a equação não possui raízes reais.
Teorema 5 (Máximos e Mínimos)
2.3 Soma, produto e diferença entre raízes
Teorema 4 (Relações entre coeficientes e raízes) I. Se a > 0, a função quadrática y = ax2 + bx + c admite valor mínimo
∆ b
yV = − (“y do vértice”) e tal valor mínimo ocorre para x = xV = −
4a 2 a
Dada uma equação do segundo grau ax2 + bx + c = 0 de raízes x1
b c ∆ (“x do vértice”).
e x2, temos que S = x1 + x 2 = − , P = x1x 2 = e D = x1 − x 2 = . II. Se a < 0, a função quadrática y = ax2 + bx + c admite valor máximo
a a a ∆
yV = − (“y do vér tice”) e tal valor máximo ocorre para
4a
Demonstração: Basta usar a fórmula de Bhaskara:
b
x = xV = − (“x do vértice”).
2a
−b + ∆ −b − ∆
x1 = e x2 = .  b ∆ 
2a 2a Obs.: O ponto ( xV , yV ) =  − , −  é chamado de vértice da parábola.
 2a 4 a 
Ex.: A função quadrática y = x2 + 4x + 7 possui vértice no ponto
Logo, S = x1 + x 2 = − b + ∆ + − b − ∆ = − 2 b = − b  −4 −12 
2a 2a a  ,−  = ( −2, 3 ), como mostrado na figura.
 2 4 

98 Vol. 2
Binômio – Trinômio – Modular

10
9
8
7
6
5
4
V
3
2
1 Demonstração: Na forma canônica, temos que ƒ(x) = a(x – xV)2 + yV.
Logo, ƒ(xv + k) = ak2 + yV e ƒ(xV – k) = a(–k)2 + yV = ak2 + yV, o que
0
conclui a prova.
–7 –6 –5 –4 –3 –2 –1 0 1 2 3 4 5 6 7

2.6 Crescimento e decrescimento 2.8 Forma fatorada


A partir da forma canônica (ou analisando o gráfico da função Dada uma função quadrática ƒ(x) = ax2 + bx + c de raízes x1 e x2,
 b c
quadrática), segue: temos que f ( x ) = a  x 2 + x + . Usando o teorema 4, segue que
 a a
Teorema 6 (Crescimento e decrescimento) ƒ(x) = a(x2 – (x1 + x2) x + x1x2) = a(x – x1)(x – x2). Temos então:
I. Se a > 0, a função quadrática y = ax2 + bx + c é crescente à direita
do vértice e decrescente à esquerda do vértice.
Teorema 8 (Forma fatorada)
Dada uma função quadrática ƒ(x) = ax2 + bx + c de raízes x1 e x2,
podemos fatorar ƒ(x) = a(x – x1)(x – x2).

Obs.: Para polinômios de graus maiores, há uma forma fatorada análoga,


crescente conforme será visto na apostila 3 de Matemática II.
decrescente
2.9 Sinal do trinômio
Estamos interessados em saber quando ƒ(x) = ax2 + bx + c > 0.
V Para isso, dividiremos a nossa análise em três casos: I. ∆ < 0, II. ∆ = 0
e III. ∆ > 0 .
II. Se a < 0, a função quadrática y = ax2 + bx + c é crescente à esquerda
I. ∆ < 0:
do vértice e decrescente à direita do vértice.
Nesse caso, escrevendo o trinômio na forma canônica:
 b 
2
∆ 
f ( x ) = a  x +  − 2  . O termo dentro dos colchetes é sempre
 2a  4 a 

positivo, pois é soma de um quadrado com um termo positivo. Dessa
forma, ƒ(x) tem o mesmo sinal de a.
crescente II. ∆ = 0:
decrescente Mais uma vez, escrevendo na forma canônica, temos:
 b 
2
∆   b 
2
f ( x ) = a  x +  − 2  = a  x +  . Nesse caso, ƒ(x) tem o
 2a  4 a   2a 

b
Obs.: Dessa forma, se a > 0, a imagem da função quadrática y = ax2 + mesmo sinal de a, exceto quando x = − , quando ƒ(x) se anula.
2a
 ∆   ∆ Nesse caso, o gráfico da parábola é tangente ao eixo das abscissas.
bx + c é  − , +∞  e se a < 0, a imagem é  −∞, −  .
 4a   4a  III. ∆ > 0
Aqui, o trinômio possui duas raízes reais distintas x1 < x2. Usaremos
2.7 Eixo de simetria então a forma fatorada: ƒ(x) = a(x – x1)(x – x2). Assim, temos que
ƒ(x) tem o mesmo sinal de a quando x < x1 ou x > x2 e ƒ(x) tem sinal
Teorema 7 (Eixo de Simetria)
contrário ao de a quando x1 < x < x2.
Sendo ƒ(x) = ax2 + bx + c uma função quadrática com vértice
no ponto V = (xV, yV), vale que ƒ(xV – k) = ƒ(xV + k) para todo k real. Temos então:
Visualmente, pontos do gráfico cujas abscissas equidistam de xv estão à
mesma altura, ou ainda a reta x = xv é um eixo de simetria da parábola.

IME-ITA 99
Matemática I – Assunto 4

Teorema 9 (Sinal do trinômio) III. Da direita para a esquerda, trocar o sinal sempre que se passa por
Seja ƒ(x) = ax + bx + c uma função quadrática.
2 um ponto não sublinhado e manter o sinal quando se passa por um
ponto sublinhado.
I. a > 0:
( x − 1) ( x + 2) ( x − 3 ) ( x + 6 ) ≤ 0.
3 4 5
– ∆ < 0: Ex.: Resolver a inequação
x 2 ( x − 7)
3
ƒ(x) > 0 para todo x real
– ∆ = 0
b
ƒ(x) ≥ 0 para todo x real e f ( x ) = 0 ⇔ x = − Solução: Marcamos na reta real os pontos de descontinuidade e os zeros:
2a – 6 < – 2 < 0 < 1 < 3 < 7. Os pontos –6, –2, 1, 3 são fechados e os
– ∆ > 0 (digamos que as raízes reais são x1 < x2): pontos 0 e 7 são abertos. Além disso, devemos sublinhar – 2 e 0. Quando
x → +∞, todas as parcelas são positivas e, portanto, a função é positiva.
ƒ(x) < 0 para x < x1 ou x > x2 Executando o passo 3, temos:

ƒ(x) > 0 para x1 < x < x2 + – – – + – +

a > 0 –6 –2 0 1 3 7
– –
+ + + + + + + + Assim, o conjunto-solução é S = [–6, 0) ∪ (0, 1] ∪ [3, 7).

(∆ < 0) (∆ = 0) – 2.11 Posição de um


(∆ > 0)
número com relação às raízes
O objetivo aqui é saber se um real t está à esquerda das raízes, entre
II. a < 0: as raízes, à direita das raízes ou se é uma raiz de uma função quadrática
– ∆ < 0: de forma simples (sem necessidade de calcular as raízes da função
ƒ(x) < 0 para todo x real efetivamente).
– ∆ = 0:
b Teorema 10
ƒ(x) ≤ 0 para todo x real e f ( x ) = 0 ⇔ x = −
2a Seja ƒ(x) = ax2 + bx + c uma função quadrática com ∆ > 0, raízes
– ∆ > 0 (digamos que as raízes reais são x1 < x2): reais x1 < x2 e seja t um real qualquer.
ƒ(x) < 0 para x < x1 ou x > x2
ƒ(x) > 0 para x1 < x < x2 I. a · ƒ(t) < 0 ⇔ x1 < t < x2
(∆ > 0)
(∆ < 0) (∆ = 0) +  b 
II.  a ⋅ f ( t ) > 0 ∧ t > −  ⇔ t > x 2
– – – – –  2 a
– – – III.  a ⋅ f ( t ) > 0 ∧ t < − b  ⇔ t < x1

 2 a 
2.10 Inequações
IV. a · ƒ(t) = 0 ⇔ (t = x1 ∨ t = x2)
Estaremos interessados em estudar o sinal de funções que são
produtos e quocientes de funções do 1 o grau, ou seja, estamos 2.12 Fórmula de Newton
interessados em estudar o sinal de funções da forma
Você já se perguntou como calcular de maneira rápida
( x − a1 ) 1 ( x − a2 ) 2 ( x − ak ) k
n n n

f (x) = ( 2 + 2 )10 + ( 2 − 2 )10? Uma maneira de se fazer isso é usando a


m , em que n1,..., nk, m1, ..., mp
( x − b1 ) 1 ( x − b2 ) 2 ( x − bp ) p
m m
fórmula de Newton, que serve para calcular a soma de potências de um
mesmo expoente de raízes de uma equação do 2o grau.
são inteiros positivos. Nos pontos x = a1,..., ak , a função se anula (zeros) Teorema 11 (Fórmula de Newton)
e os pontos x = b1,...,bp são chamados pontos de descontinuidade. Para
estudarmos o sinal de ƒ(x), marcamos na reta real todos os zeros e os Sejam x1 e x2 as raízes (não necessariamente reais) de uma equação
pontos de descontinuidade. Esses pontos dividem a reta em k + p + 1 do 2o grau ax2 + bx + c. Defina a sequência S0 = 2 e Sn = x1n + x 2 n para
intervalos e é possível provar que, em cada um desses intervalos, o sinal n inteiro positivo (podemos definir a sequência para n inteiro negativo, se
da função é constante. Para determinar qual é o sinal em cada um desses as raízes são não nulas). Então, vale que aSn+2 + bSn+1 + cSn = 0 para
intervalos, há um método simples (MÉTODO DA RETA REAL): todo n inteiro não negativo (mais uma vez, a relação é válida para n inteiro
negativo, se as raízes são não nulas).
I. Colocar todos os pontos na reta retal em ordem crescente e sublinhar
aqueles associados a expoentes pares. Além disso, os pontos de Demonstração: Como x 1 é raiz de ax 2 + bx + c, segue que
descontinuidade devem ser sempre abertos (indicando que eles não ax12 + bx1 + c = 0.
pertencem ao conjunto-solução). Os zeros devem ser fechados quando M u l t i p l i c a n d o a m b o s o s l a d o s p o r x 1 n, s e g u e q u e
o sinal da inequação for ≤ ou ≥.
ax1n + 2 + bx1n +1 + cx1n = 0 ( * ) . A n a l o g a m e n t e , s e g u e q u e
II. Verificar o sinal da função quando x → +∞ (essa análise é bastante
simples como veremos nos exemplos). ax 2 n + 2
+ bx 2 n +1 + cx 2 n = 0 (**). Somando (*) e (**), temos que

100 Vol. 2
Binômio – Trinômio – Modular

( ) ( ) ( )
a x1n + 2 + x 2 n + 2 + b x1n +1 + x 2 n +1 + c x1n + x 2 n = 0, o que nos dá 3
finalmente aSn+2 + bSn+1 + cSn = 0, como desejado.

3. Função modular 2

3.1 Definição
1
A função modular (também conhecida como função valor absoluto) é
 x , se x ≥ 0 0
tal que f ( x ) =  . Representamos f (x) =|x|.
− x , se x < 0 –3 –2 –1 0 1 2 3
Obs.: Veja que ƒ(x) ≥ 0 para todo x real.

3.2 Efeitos do módulo –1


nos gráficos de uma função
–2
Dada uma função ƒ(x), há duas outras funções interessantes a serem
consideradas: |ƒ(x)| e ƒ(|x|). Entenderemos como fazer o gráfico dessas
funções a partir do gráfico de ƒ(x). 2o Passo: Ignoramos a parte à esquerda do eixo y e refletimos com
I. |ƒ(x)|: relação a tal eixo a parte que está à direita
Nesse caso, toda a parte do gráfico que está abaixo do eixo x deve
ser refletida com relação a tal eixo. 2

Ex.: Determine o gráfico da função y = |x2 – 1|. 1


1o Passo: Desenhamos o gráfico da função y = x2 – 1. A 0
–3 –2 –1 0 1 2 3
0.4
–1
0.2

–1.0 –0.5 0.5 1.0 –2


–0.2
–0.4 3.3 Propriedades do módulo
–0.6
I. x2 = x
–0.8
II. |xy| = |x||y|
–1.0 III. |x2| = |x|2 = x2
2o Passo: Refletimos a parte do gráfico que está abaixo do eixo x x x
IV. = , se y ≠ 0
2.0 y y
V. x ≤ |x| e – x ≤ |x|
VI. (Desigualdade Triangular) |x + y| ≤ |x| + |y|, com igualdade se,
1.5
e somente se, x e y têm o mesmo sinal.

1.0 Demonstração: Como os dois lados são positivos,


( ) ≤( x + y )
2 2
x+y ≤ x + y ⇔ x+y ⇔
0.5
⇔ ( x + y ) ≤ x + 2 x y + y . Esta última é equivalente a x2 + 2xy
2 2 2

+ y2 ≤ x2 + 2 |xy| + y2 ⇔ xy ≤ |xy|, o que é verdade pela propriedade


–1.5 –1.0 –0.5 0.5 1.0 1.5 v. Ademais, a igualdade ocorre quando xy = |xy|, ou seja, quando xy é
II. ƒ(|x|) positivo, ou ainda quando x e y têm o mesmo sinal.
Devemos ignorar a parte do gráfico de ƒ(x) que está à esquerda do 3.4 Equações e inequações modulares
eixo y e refletir a parte que está à direita com relação a tal eixo.
Há três propriedades úteis a serem usadas:
Ex.: Determine o gráfico de sen (|x|) para – π ≤ x ≤ π.
I. |x| = |y| ⇔ x = y ∨ x = – y
II. |x| > k ⇔ x > k ∨ x < – k (k > 0)
1o Passo: Desenhamos o gráfico de sen x para – π ≤ x ≤ π.
III. |x| < k ⇔ –k < x < k (k > 0)

Quando não for possível usar uma das três propriedades anteriores
diretamente, podemos usar a força bruta e dividir o problema em vários
casos.

IME-ITA 101
Matemática I – Assunto 4

Ex.: (x – 2)2 = (3 – 2x)2 ⇒ x2 – 4x + 4 = 9 – 12x + 4x2 ⇒ 3x2 – 8x + 5 = 0.


I. Resolva a equação |3x – 1| = |2x + 3|
Nesse caso, basta termos 3x – 1 = 2x + 3 ⇔ x = 4 ou 3x – 1 = – 2x 5 5
Resolvendo a equação do 2o grau, temos = x 1=
ou x . Mas não
2  2 satisfaz a restrição (*), por isso, S = {1}. 3 3
– 3 ⇔ x = − . Logo, o conjunto-solução é S = 4, − .
5  5
II. Resolva a equação |x – 2| = 3 – 2x. III. Resolva a inequação |2x + 1| < 3.

 x − 2, se x ≥ 2 Basta que – 3 < 2x + 1 < 3 ⇔ – 2 < x < 1.


1a solução: Usaremos x − 2 =  . Logo, S = (– 2, 1).
2 − x , se x < 2
5 IV. Resolva a inequação 2x – 7 + |x + 1| ≥ 0.
1o caso (x ≥ 2): x − 2 = 3 − 2 x ⇔ x = . Veja que esse número não
satisfaz x ≥ 2, por isso não é solução! 3
Devemos dividir o problema em 2 casos:
2 caso (x < 2): 2 – x = 3 – 2x ⇔ x = 1. Veja que esse número
o

satisfaz x < 2, portanto é solução! 1o caso: x ≥ – 1


Aqui, a inequação se reduz a 2x – 7 + x + 1 ≥ 0 ⇔ x ≥ 2.
Então, S = {1}. Fazendo a interseção com a restrição, temos x ≥ 2 .

2a solução: Outra forma de eliminar o módulo é elevando ao quadrado. 2o caso: x < – 1


Mas, antes disso, precisamos obrigar os dois lados a terem o mesmo
3 Aqui, a inequação se reduz a 2x – 7 – x – 1 ≥ 0 ⇔ x ≥ 8.
sinal: 3 – 2x ≥ 0, que dá x ″ (*). Então, elevando ao quadrado, temos
2 Fazendo a interseção com a restrição, não encontramos soluções.
|x – 2|2 = (3 – 2x)2. Daí segue que: Juntando os dois casos, temos que S = [2, +∞).

EXERCÍCIOS RESOLVIDOS

01 Determine a sabendo que a soma dos quadrados das raízes da 03 Determine todos os valores de m para os quais x2 + mx + 3m > 0
equação x2 + 3x + a = 0 é igual a 4. para todo x real.

Solução: Sendo x1, x2 as raízes, devemos ter x12 + x 22 = 4 . Esta Solução: Como o coeficiente líder da expressão quadrática é positivo (a
relação pode ser escrita como (x1 + x2)2 – 2x1x2 = 4 (*). As relações > 0), para que o trinômio seja sempre positivo, seu discriminante deve
x + x = − 3 ser negativo (faça o gráfico!): ∆ = m2 – 12m < 0. Agora temos outro
de soma e produto nos dão que  1 2 . Em (*), temos que problema. Precisamos determinar os valores de m que tornam (m – 12)
5  x1 x 2 = a
( − 3 ) − 2a = 4 ⇒ a = .
2
m < 0 verdadeira. Como as raízes de (m – 12)m são m = 12, m = 0 e
2
a concavidade é positiva, devemos ter 0 < m < 12 (faça o desenho!).
x 04 Considere as equações ax2 + bx + c = 0 (i) e x2 + bx + ac = 0
02 Determine a imagem da função real y = .
x2 + 1 (ii). Exiba a relação existente entre as raízes de (i) e (ii).
Solução: Para determinar a imagem, precisamos explicitar todos os
Solução: Veja que as duas equações têm o mesmo discriminante ∆ =
valores possíveis para y. A igualdade é equivalente a yx2 – x + y = 0
(*). Veja que y = 0 é possível, basta tomar x = 0. Caso y ≠ 0, (*) é uma b2 – 4ac. Por isso, as equações de (i) são − b ± ∆ e as raízes de (II)
equação do 2o grau e, para possuir soluções reais, precisa possuir 2a
d i s c r i m i n a n t e n ã o n e g a t i v o :
1 1 1 1 são − b ± ∆ . Então, podemos dizer que as raízes de (I) são iguais às
∆ = 1− 4 y 2 ≥ 0 ⇔ y 2 ≤ ⇔ y ≤ ⇔− ≤y≤ . 2
4 2 2 2 raízes de (II) divididas por a.
Juntando isso com o y = 0 que já tinha sido encontrado, temos que a
Comentário: Este problema dá uma maneira de reduzir equações gerais
 1 1
imagem é o intervalo  − ;  . do 2o grau a equações com coeficiente líder igual a 1.
 2 2
Obs.: Também é possível resolver fazendo uma substituição 05 Calcule a para que as equações x2 + x + a = 0 e x2 + ax +1 = 0
trigonométrica x = tan θ. De fato, como a função tangente pode assumir possuam pelo menos uma raiz real comum.
qualquer valor real, podemos fazer tal substituição. Assim, ficamos com
tan θ tan θ sen 2θ Solução: Neste tipo de problema, uma boa ideia é considerar uma raiz
y= 2
= 2
= sen θ cos θ = . Como a imagem de sen comum às duas equações, ou seja t. Daí, temos que t2 + t + a = 0
1 + tan θ sec θ 2
e t2 + at + 1 = 0. Agora, para diminuir o grau, devemos subtrair as
1 1
2θ é [–1, 1], segue que a imagem pedida é  − ;  . igualdades, obtendo at + 1 – t – a = 0 ⇔ (a – 1)(t – 1) = 0. Agora,
 2 2 há dois casos:

102 Vol. 2
Binômio – Trinômio – Modular

1o caso: a = 1
Aqui, as duas equações são x2 + x + 1 = 0, que não possui raiz real. 09 Resolva a equação 2 + x − 5 − 13 − x = 0.
Assim, este caso não fornece solução.
2o caso: t = 1 Solução: Esta é uma equação irracional. Uma técnica simples e útil para
Daqui, obtemos que a = – 2 e 1 é uma raiz comum às duas equações. resolver tal equação é elevá-la ao quadrado de maneira conveniente de
Logo, a = – 2. modo que os radicais desapareçam. Devemos tomar bastante cuidado
06 Resolva a inequação
( x − 3 ) ( x + 2) < 1. com isso, pois, ao elevar ao quadrado, podemos introduzir raízes
x2 − 1 estranhas (por exemplo, na equação x = 3, ao elevarmos ao quadrado,
Solução: Um ERRO muito comum que muitos cometem é pensar que, obtemos x = 3 ou x = – 3 e essa última é uma raiz estranha!). Assim,
em problemas como esse, basta passar o x2 – 1 multiplicando para o ao fim de todo o processo, devemos testar as soluções para verificar
outro lado! Não podemos fazer isso porque x2 – 1 poderia ser negativo se elas são de fato soluções.
e, quando multiplicamos uma inequação por um negativo, o sinal se Aqui, podemos reescrever a equação como 2 + x − 5 = 13 − x .
inverte!! Assim, devemos proceder como a seguir: Elevando ao quadrado, segue que 2 + x − 5 = 13 − x ⇒ 11 − x = x − 5 .
Elevando ao quadrado mais uma vez, temos que x2 – 22x + 121 = x – 5
( x − 3 ) ( x + 2) < 1 ⇔ ( x − 3 ) ( x + 2) − 1 < 0 ⇔ ⇒ x2 – 23x + 126 = 0 e então x = 9 ou x = 14. Voltando à equação
x2 − 1 x2 − 1 original, vemos que 9 é solução, enquanto 14 não (veja a importância
2 2 da verificação!).
x − x −6− x +1 −x − 5
⇔ <0⇔ 2 < 0. Esta última é equivalente a Logo, S = {9}.
2
x −1 x −1
( x + 5 ) < 0 ⇔ x + 5 > 0. Usando o método da reta real visto
− 2 10 Resolva a inequação x 2 − 4 x > x − 3.
x −1 ( x + 1) ( x − 1)
em 2.10, segue que o conjunto-solução é S = (–5, –1) ∪ (1, +∞). Solução: Está é uma inequação irracional. A heurística para a solução é
basicamente a mesma das equações irracionais (elevar ao quadrado para
07 Determine k para que as raízes de kx2 – 2(k + 1)x + (k + 2) = 0 eliminar radicais!). Entretanto, aqui devemos ser ainda mais cautelosos,
sejam positivas. porque, como vimos na apostila 1 de Matemática II, só podemos elevar
Veja que se k = 0, a única raiz da equação é x = 1, que é positiva. Podemos ao quadrado a > b se ambos os lados da inequação são não negativos.
considerar k = 0 como solução (alguns preferem não considerar, pois o
enunciado fala de raízes no plural). Suporemos então k ≠ 0. Primeiramente, em qualquer inequação irracional, devemos fazer as
restrições. Aqui, há apenas uma restrição: x2 – 4x ≥ 0 ⇔ x ≤ 0 ∨ x ≥ 4.
Solução: Uma maneira de se abordar este problema é usando a teoria de
posição de um número com relação às raízes. Primeiramente, devemos Agora, temos dois casos:
garantir a existência de raízes reais e, portanto, o discriminante deve ser 1o caso: x – 3 < 0
positivo. Logo, 4(k + 1)2 – 4k(k + 2) > 0 ⇔ 4 > 0, o que é sempre Como x 2 − 4 x ≥ 0, nesse caso, a inequação é verdadeira.
verdade. Agora, para garantir que as raízes são positivas, devemos
garantir que 0 está à esquerda das raízes. Pelo teorema 10, devemos Fazendo a interseção com a restrição, temos x ≤ 0.
2 ( k + 1)
ter k · ƒ(0) > 0 e 0 < . Logo, devemos ter k(k + 2) > 0 e 2o caso: x – 3 ≥ 0
2k
( k + 1) > 0. Logo, temos que k < –2 ou k > 0. Agora, ambos os lados da inequação são não negativos e então
k podemos elevá-los ao quadrado:
Assim, o conjunto-solução é (–∞, –2) ∪ [0, +∞). 9
x2 − 4 x > x2 − 6x + 9 ⇔ 2x > 9 ⇔ x > .
2
Comentário: Poderíamos ter resolvido o problema de outra maneira: 9
Fazendo a interseção com a restrição, temos x > .
atentando para o fato de que as raízes são positivas se, e somente se, 2
o discriminante é positivo, a soma das raízes é positiva e o produto das
9 
raízes é positivo. Além disso, neste problema especificamente, é fácil Assim, o conjunto-solução é S = ( −∞, 0  ∪  , +∞ .
2 
ver que 1 é raiz da equação e, portanto, a outra raiz é k + 2 . Como 1 é
k +2 k 11 Determine a soma das quintas potências das raízes da equação
positivo, basta forçarmos que seja positivo. x2 + 3x – 1 = 0.
k
08 Determine os valores de m para os quais 1 é exterior ao intervalo
das raízes da equação (m + 3)x2 – x + 2 = 0. Solução: É possível obter uma solução na força bruta calculando as
raízes. No entanto, utilizaremos a fórmula de Newton, que possibilitará
Solução: Nesse tipo de problema que cita o “intervalo das raízes”, uma abordagem mais elegante. Sendo x1, x2 as raízes e definindo
inicialmente, é necessário forçarmos a existência de duas raízes reais Sn = x1n + x 2n , temos que Sn+2 + 3Sn+1 – Sn (fórmula de Newton).
distintas (para haver o intervalo). Isso nos dá a primeira restrição: Escrevendo Sn+2 = Sn – 3Sn+1 e percebendo que S0 = 2 e S1 = –3,
23 podemos calcular os termos seguintes:
∆ = − 8 m − 23 > 0 ⇔ m < − (i).
8 S2 = S0 − 3S1 = 2 − 3 ( −3 ) = 11, S3 = S1 − 3S2 = −3 − 3 ⋅ 11 = −36
Agora, definindo ƒ(x) = (m + 3)x2 – x + 1, para 1 ser exterior ao intervalo S4 = S2 − 3S3 = 11 − 3 ( −36 ) = 119, S5 = S3 − 3S4 = −36 − 3 ⋅ 119 = −393
das raízes, precisamos ter a · ƒ(1) > 0, o que nos dá (m + 3) · (m + 4) > 0.
Essa é uma inequação do 2o grau de concavidade positiva e raízes – 3 e – 4;
Então, a soma das quintas potências das raízes é – 393.
então, a 2a restrição é m < – 4 ou m > – 3 (ii). Fazendo a interseção de (i) e
(ii), ficamos com m < – 4.

IME-ITA 103
Matemática I – Assunto 4

EXERCÍCIOS NÍVEL 1 Pode-se afirmar que m + n é expresso por:


01 (ITA 77) Supondo a < b, onde a e b são constantes reais, considere a 3 abc + b3
função g(x) = a + (b – a)x definida no intervalo (0,1). Podemos assegurar (A) 3 abc − b . (D) .
que: a2 c c2 a
3 abc + b3 abc − b3
(A) g não é uma função injetora. (B) . (E) .
a2 c a2 c
(B) dado qualquer y0 < b, sempre existe um x0 ∈ (0,1) tal que g(x0) = y0.
(C) para cada a < y0 < b, corresponde um único real x0 ∈ (0,1) tal que 3 abc − b3
(C) .
g(x0) = y0. c2 a
(D) não existe uma função real h, definida no intervalo (a, b), satisfazendo
a relação h(g(x)) = x para cada x ∈ (0,1). 06 Qual é a soma das raízes quadradas das raízes da equação do 2o grau
(E) n.d.a. x2 – 6x + 2 = 0?

02 (ITA 94) Dadas as funções reais de variável real ƒ(x) = mx + 1 e g(x) (A) 6 + 2 2 . (D)
3+2 3 .
= x + m, onde m é uma constante real com 0 < m < 1, considere as
afirmações:
(B) 6 + 2 3 . (E)
3+3 2.
I. ƒ o g(x) = g o ƒ(x), para algum x real.
II. ƒ(m) = g(m) (C) 3 + 2 2 .
III. Existe a real tal que ƒ o g(a) = ƒ(a)
IV. Existe b real tal que ƒ o g(b) = mb 07 Determine uma equação de segundo grau de coeficientes racionais
V. 0 < g ° g(m) < 3 em que uma das raízes é 3 + 5 .

Podemos concluir que: 08 Determine m e n para que as equações (2n + m)x2 – 4mx – 3 = 0 e
(6n + 3m)x2 – 3(n – 1)x – 9 = 0 tenham as mesmas raízes.
(A) todas são verdadeiras.
(B) apenas quatro são verdadeiras. 09 Determine p e q na equação x2 + px + q = 0, sabendo que suas
(C) apenas três são verdadeiras. raízes aumentadas de uma unidade são as raízes de x2 – px + pq = 0.
(D) apenas duas são verdadeiras.
(E) apenas uma é verdadeira. 10 Para m ≠ 1, mostre que existe uma relação independente de m entre
a soma e o produto das raízes da equação (1 + m)x2 – (1 + m2)x +
03 (ITA 84) Os coeficientes do trinômio x2 + bx + c constituem, nesta m(1 – m) = 0.
q
ordem, uma progressão aritmética de razão não nula r = , onde q é a
2 11 Calcule m e n para que as raízes da equação (n + m)x2 – 4mx – 3 =
razão da progressão aritmética b2 – 1, c2 – b2. Nestas condições podemos 0 sejam os inversos das raízes da equação 9y2 + 3(n – 1)y – (6n + m)
afirmar que o trinômio apresenta: = 0.
12 As raízes de x² + ax + b = 0 são a e b. Sabendo que b é não-nulo,
(A) uma raiz nula. então a – b é igual a:
(B) duas raízes reais distintas.
(C) duas raízes iguais. (A) 0. (D) 3.
(D) duas raízes complexas não reais. (B) 1. (E) 4.
(E) uma raiz irracional. (C) 2.

04 (ITA 2009) Sejam a, b, c constantes reais com a ≠ b formando, nesta 3 x + 3 , x ≤ 0


13 (ITA 96) Seja ƒ:  →  definida por f ( x ) =  2 .
ordem, uma progressão aritmética e tais que a soma das raízes da equação  x + 4 x + 3 , x > 0
Podemos afirmar que:
ax2 + bx + c = 0 é − 2. Então, uma relação válida entre b e c é:
 2
(A) ƒ é bijetora e f  f  −  = f −1 ( 21).
(A) c =
b
2
( 2 − 1).  3

(B) c = b ( 2 − 2 ) .  2
(B) ƒ é bijetora e f  f  −  = f −1 ( 99 ).
 3
(C) c = b ( 2 − 1) . (C) ƒ é sobrejetora, mas não é injetora.
(D) ƒ é injetora, mas não é sobrejetora.
(D) c = b 2.
(E) ƒ é bijetora e f  f  − 2  = f −1 ( 3 ).
 3
(E) c =
b
2
(4− 2 . ) 14 O conjunto dos valores inteiros e positivos de m para os quais a
equação x2 – 5mx + 2m = 0 tem ambas as raízes reais e distintas é:
05 ( CN 2007) A menor raiz da equação ax 2 + bx + c = 0,
com abc ≠ 0, é a média geométrica entre m e a maior (A) {0, 1, 2, ... } (D) {1, 2, 3, ... }
raiz. A maior raiz é a média geométrica entre n e a menor raiz. (B) {4, 5, 6, ... } (E) n.r.a.
(C) {1, 2, 3}

104 Vol. 2
Binômio – Trinômio – Modular

15 Três máquinas P, Q e R, trabalhando juntas, fazem um trabalho em x 20 ( UNIFESP) De um car tão retangular de base 14 cm e altura
horas. Trabalhando sozinha, P necessita de 6 horas adicionais para fazer 12 cm, deseja-se recortar um quadrado de lado x e um trapézio isósceles,
o trabalho; Q, uma hora adicional e R, x horas adicionais. O valor de x é: conforme a figura, onde a parte hachurada será retirada.

(A) 2/3. (D) 2.


(B) 11/12. (E) 3.
(C) 3/2. x
x
16 Se = x − y , onde x e y são reais, y ≠ 0, então:
y
12 cm
(A) x ≥ 4 ou x ≤ 0.
(B) y pode ser igual a 1.
(C) x e y devem ser irracionais.
(D) x e y não podem ser inteiros.
(E) x e y são necessariamente racionais.

17 (ITA 95) Os dados experimentais da tabela abaixo correspondem às 14 cm


concentrações de uma substância química medida em intervalos de 1 segundo.
Assumindo que a linha que passa pelos três pontos experimentais é uma O valor de x em centímetros, para que a área total removida seja mínima, é
parábola, tem-se que a concentração (em moles) após 2,5 segundos é:
(A) 3. (D) 1.
Tempo(s) Concentração(moles) (B) 2. (E) 0,5.
1 3,00 (C) 1,5.
2 5,00
21 (ESPCEX) Um curral retangular será construído aproveitando-se um
3 1,00 muro prexistente no terreno, por medida de economia. Para cercar os
outros três lados, serão utilizados 600 metros de tela de arame. Para que
(A) 3,60. (D) 3,75. a área do curral seja a maior possível, a razão entre as suas menor e maior
(B) 3,65. (E) 3,80. dimensões será:
(C) 3,70.

18 (AFA 1986) “Um terreno retangular de área 875 m2 tem o comprimento (A) 0,25. (D) 1,00.
excedendo em 10 metros a largura”. Assinale a equação que representa o (B) 0,50. (E) 1,25.
problema acima: (C) 0,75.

comp = y 22 Dada a função real ƒ definida por ƒ(x) = x2, considere a função real g
definida por g(x) = ƒ(x + m) + k, sendo m e k reais. É incorreto afirmar
larg = x que:

(A) o gráfico da função g em relação ao gráfico da função ƒ é deslocado k


(A) x2 + 10x + 875 = 0. (C) x2 + 10x – 875 = 0.
unidades para cima, se k > 0, e m unidades para a direita, se m < 0
(B) x2 + 875x – 10 = 0. (D) x2 – 875x + 10 = 0.
(B) se m = 0 e k = 1, então o conjunto imagem de g é dado por Im =
{y real / y ≥ 1}
19 (AFA 2001) O retângulo, com base no eixo das abscissas, está inscrito
(C) a equação do eixo de simetria da parábola que representa g é dada
numa parábola, conforme figura abaixo. O valor de x que faz esse retângulo
por x = m
ter perímetro máximo é:
(D) se m = –2 e k = –3, então as coordenadas do vértice da parábola
que representa g são (– m, k)
y
8 23 (CN 2008) O gráfico de um trinômio do 2o grau y tem concavidade
para cima e intersecta o eixo das abscissas em dois pontos à direita da
origem. O trinômio –y tem um valor:

(A) mínimo e raízes positivas.


(B) mínimo e raízes negativas.
(C) máximo e raízes positivas.
(D) máximo e raízes negativas.
(E) máximo e raízes de sinais opostos.

–2 –x x 2 x 24 O conjunto dos valores de p para os quais a inequação x2 + 2x + p > 10


é verdadeira para qualquer x real é dado por:

(A) p > – 9. (D) p < – 9.


(A) 1. (C) 0,25. (B) p < 11. (E) n.r.a.
(B) 0,5. (D) 0,125. (C) p > 11.

IME-ITA 105
Matemática I – Assunto 4

25 Resolva a inequação x4 + x2 – 20 > 0. 2


34 (ITA 99) Considere as funções ƒ, g definidas por f ( x ) = x − para
x
x
26 Dado o trinômio do segundo grau y = kx2 + (k – 1)x + (k – 1): x ≠ 0 e g( x) = para x ≠ 1. O conjunto de todas as soluções da
x +1
(A) Não há nenhum valor de k que torne o trinômio negativo para qualquer inequação g o ƒ(x) < g(x) é:
valor de x.
(B) O trinômio é negativo para qualquer valor de x se –1/3 < k < 1. (A) [1, +∞[. (D) ]– 1,1[.
(C) k > 3 torna sempre nulo o trinômio. (B) ]– ∞, – 2[. (E) ]–2, – 1[∪]1, +∞[.
(D) Para que o trinômio seja sempre negativo, só convirão os valores de (C) [– 2, – 1[.
k < –1/3
(E) n.r.a. 35 (ITA 2001) O conjunto de todos os valores de m para os quais a função

f (x) =
(
x 2 + ( 2 m + 3 ) x + m2 + 3 ) está definida e é não negativa para
27 Calcule m para que a inequação (m – 3)x + 4x + m < 0 seja válida
2

para todos os valores de x, com exceção de um só. x + ( 2 m + 1) x + m + 2


2 2

todo x real é:
(m + 3) x 2 + (m + 3) x + 19
28 Determine m para que se tenha > 0, para
todo x real. x2 + x + 6 1 7 
(A)  , . (D)
7
4 4  −∞, 4  .
   
29 Determine m de modo que a desigualdade x2 – (8m – 2)x + 15m2 –
1 7
2m – 7 > 0 seja satisfeita para qualquer valor de x. (B)  1 ,+∞ . (E)
4,4.
 4   
30 Considere o trinômio y = x2 + (2a – 1)x + a2. Assinale dentre as
condições abaixo a que torna o trinômio sempre positivo: (C)  0, 7  .
 4
 
(A) a > b. (D) a > – 1/2.
x2 + x + 3
(B) a < 1/2. (E) a > 1/4. 36 (AFA 1989) A solução da inequação ≤ 3 é dada pelo conjunto:
(C) a < – 1/4. x +1
(A) {x ∈  / 0 ∈ x ≤ 2} (C) {x ∈  / x > –1 ou 0 ≤ x ≤ 2}
31 A inequação x2 + (m – 2)x + (m2 – m + 4) > 0 é satisfeita qualquer (B) {x ∈  / x ≤ –1 ou 0 < x ≤ 2} (D) {x ∈  / x < –1 ou 0 ≤ x ≤ 2}
que seja x:
37 (AFA 1988) Considere o polinômio p(x) = ax2 + bx + c, satisfazendo
(A) Só para m > 2 e m < – 2. as condições a < 0, c < 0 e p(1) > 0. Se as suas raízes forem reais,
(B) Só para –2 < m < 2. então elas serão:
(C) Só para m = 2.
(D) Para todo m. (A) nulas. (C) positivas.
(E) Não existe m tal que a inequação seja satisfeita qualquer que seja x. (B) negativas. (D) de sinais contrários.

32 (ITA 87) Considere a função y = ƒ(x) definida por ƒ(x) = x3 – 2x2 + 5x, x 2 + ax − 2
38 (EN 1988) Para todo x real, − 3 < < 2 se e só se:
para cada x real. Sobre esta função, qual das afirmações abaixo é verdadeira? x2 − x + 1
(A) –3 < a < 2. (D) –1 < a < 7.
(A) y = ƒ(x) é uma função par.
(B) –1 < a < 2. (E) –6 < a < 2.
(B) y = ƒ(x) é uma função ímpar.
(C) –6 < a < 7.
(C) ƒ(x) ≥ 0 para todo real x.
(D) ƒ(x) ≤ 0 para todo real x. 39 (CN 2009) O conjunto-solução de números reais tal que
(E) ƒ(x) tem o mesmo sinal de x, para todo real x ≠ 0.
( x − 5 ) ( 2 x − 1)
15 10

≥ 0 é:
( 3 x + 1)
8
33 (ITA 96) Considere as funções reais ƒ, g definidas por:
1+ 2 x x  1
f (x) = , x ∈ ¡ − {−1,1} , e g( x) = , x ∈ ¡ − −  .  1 1  1 1
1− x 2 1 + 2 x  2 (A) 5, +∞  ∪ − ,  (D)
3 2  − 3 , 2  ∪ 5, +∞ 
   
O maior subconjunto de , onde pode ser definida a composta ƒ o

 1 

g, tal que ƒ o g(x) > 0 é: (B)  −∞,  ∪ 5, +∞  (E)
1   ∪ 5, +∞ 
 2     2   

 1  1 1 (C) 
(A)  −1, −  ∪  − , −  (D) ]1, +∞[
 2   3 4
40 (CN 2009) Quantos são os números inteiros com os quais é possível,
 1 1
(B)  −∞, −1 ∪  − , −  (E)
1 1
− 2 ,− 3  no conjunto dos números reais, calcular o valor numérico da expressão
 3 4  
algébrica 103 x − x 2 − 300 ?

 1  (A) 100. (D) 97.


(C)  −∞, −1 ∪  − ,1 (B) 99. (E) 96.
 2  (C) 98.

106 Vol. 2
Binômio – Trinômio – Modular

p 53 (EN 1990) A equação |2x + 3| = ax + 1:


41 Determine os valores possíveis da razão , onde p e h são positivos,
de modo que p2 − 2 ph − h2 seja real. h
(A) não possui solução para a < – 2.
(B) possui duas soluções para a > 2.
42 As raízes de x2 + bx + c = 0 são reais e maiores que 1. Então, 2
b + c + 1: (C) possui solução única para a < .
3
2
(D) possui solução única para −2 < a < .
(A) pode ser negativo. (D) é negativo. 3
(B) pode ser nulo. (E) está compreendido entre –1 e 1.
2
(C) é positivo. (E) possui duas soluções para −2 < a < .
3
43 Resolva a equação 2 x 2 + 3 x − 3 + 2 x 2 + 3 x + 9 = 30. 54 (UFPE) Considere a função ƒ(x) = |x + 1|–|x – 1|, definida para x
real. Analise as seguintes afirmações sobre ƒ:
44 Resolva a inequação x − 5 < x 2 + 25 . ( ) ƒ é par.
( ) ƒ é positiva.
45 Resolva a inequação x − 2 < x − 2. ( ) ƒ é injetora.
( ) A imagem de ƒ é o intervalo [–2, 2].
1
46 Resolva a inequação x −1< .
x +1 55 (FUVEST) Seja ƒ(x) = |x| – 1. Determine os valores de x para os
2
quais ƒ(ƒ(x)) = 5.
47 A solução de x − 6 x + 8 < 8 − 3 x é:
56 (UFRJ) Considere a função ƒ:  →  definida por ƒ(2x) = |1 – x2|.
(A) x ≤ 2 (D) 2 < x < 8/3 Determine os valores de x para os quais ƒ(x) = 2.
(B) x < 4 (E) n.r.a.
(C) x < 8/3 57 (UFC) Dadas as funções ƒ:  →  e g:  →  definidas por ƒ(x)
= |1 – x2| e g(x) = |x|, o número de pontos na interseção do gráfico
48 Resolva a inequação 2( x − 1) < x . de ƒ com o gráfico de g é igual a:

(A) 5.
x2 + 3 x 3
49 Todas as raízes reais da equação − = são (B) 4.
2
x x +3 2 (C) 3.
(A) x1 = 3, x2 = – 3. (D) A equação não tem raízes reais. (D) 2.
(B) x1 = 3, x2 = 3. (E) n.r.a. (E) 1.
(C) x1 = 3, x2 = 3.
58 (UFRGS) A interseção dos gráficos das funções ƒ e g definidas por
ƒ(x) = |x| e g(x) = 1 – |x| os quais são desenhados no mesmo sistema de
50 (ITA 80) Considere a equação |x| = x – 6. Com respeito à solução
coordenadas cartesianas, determina um polígono. A área desse polígono é:
real desta equação, podemos afirmar que:
(A) 0,125. (D) 1.
(A) a solução pertence ao intervalo [1,2].
(B) 0,25. (E) 2.
(B) a solução pertence ao intervalo [–2, –1].
(C) 0,5.
(C) a solução pertence ao intervalo (–1, –1).
(D) a solução pertence ao complementar da união dos intervalos anteriores.
59 Se |x| + x + y = 10 e x + |y| – y = 12, determine x + y.
(E) a equação não tem solução.
60 Uma certa função de x é igual a x2 para x ≥ 0 e igual a – x2 para x < 0.
51 (ITA 88) Sabendo-se que as soluções da equação |x|2 – |x| = 6
Dê uma expressão única definindo esta mesma função para todo e qualquer
são raízes da equação x2 – ax + b = 0, podemos afirmar que:
valor real de x.
(A) a = 1 e b = 6.
61 O maior valor de x2 – |x| + 1 no intervalo [–3,3] é:
(B) a = 0 e b = – 6.
(C) a =1 e b = – 6.
(A) 2. (D) 6.
(D) a = 0 e b = – 9.
(B) – 3. (E) 7.
(E) não existem a e b tais que x2 – ax + b = 0 contenha todas as raízes
(C) 0.
da equação dada.
62 A equação |x + 1| – |x| = x + 2:
52 (ITA 2002) Os valores de x reais para os quais a função real dada por
f ( x ) = 5 − 2 x − 1 − 6 esta definida formam o conjunto: (A) possui duas soluções reais cuja soma é 2.
(B) possui somente uma solução real.
(A) [0,1]. (C) possui três soluções reais cuja soma é – 3.
(B) [–5,6]. (D) possui uma infinidade de soluções reais distintas.
(C) [–5,0] ∪ [1,+∞]. (E) não possui solução real.
(D) ]–∞,0] ∪ [1,6].
(E) [–5,0] ∪ [1,6].

IME-ITA 107
Matemática I – Assunto 4

63 A equação |x – 1| = |x| + 1: 72 (AFA – 2000) O gráfico que melhor representa a função y = |sen x
+ cos x|, com 0 ≤ x < 2π, é:
(A) não tem solução.
(B) tem uma única solução. (A) y
(C) tem somente duas soluções. 2
(D) tem uma infinidade de soluções.
(E) n.r.a.
1
x 1 1
64 Resolva a inequação − < .
x + 1 2 10
65 Resolva a inequação |x – 1|<|x – 3|. 0 π 2π x

66 Resolva a equação |x + 1|+|x – 4| = 5


(B)
y
67 Resolva equação ||3 – 2x|–1|= 2|x|. 2

68 Resolva a equação |x – 2| = 7 – 7x.


1
69 Resolva a equação |2x + 1| – |3 – x| = |x – 4|.

70 Qual é o valor mínimo da expressão |x – 1| + |x – 3|?


0 π 2π x
1
71 (AFA) O gráfico que melhor representa a função f (x) = ( x − x ) é: (C)
2 y
2
y

1
(A)

x
0 π 2π x

(D)
y
y
2

(B) 1

0 π 2π x
y
73 O gráfico que melhor representa a função dada por ƒ(x) = |sen x| +
|cos x| é:
(C)

x
(A)

(D)

x
(B)

108 Vol. 2
Binômio – Trinômio – Modular

(C) | x 2 − 4 x + 3 |
 + 2 x − 1 se x ≠ 3
75 (EN) O gráfico da função f (x) =  x −3 é:
0 se x = 3

(A)
7

(D)
0 1 3
–2

(B)
7

1, se 0 ≤ x ≤ 2 3
74 (AFA) Considere a função f (x) =  . A função
−2, se − 2 ≤ x < 0
g(x) = |ƒ(x)| – 1 terá o seguinte gráfico:
0 1 3
(A) y –2

(C)
1
7
–2 0 2 x

3
(B) y

1 0 1 3
–2 0 2 x
–2

(D)
(C)
y 7

1
3
–2 0 2 x

0 1 3

(D) –2
y

2 (E)
2
7
–2 0 x
–1

0 1 3

–2

IME-ITA 109
Matemática I – Assunto 4

76 (EN-01) Assinale o gráfico que melhor representa a função real: EXERCÍCIOS NÍVEL 2
x | x − 1|
f (x) = + 2 | x + 1| se x ≠ 1, ƒ(1) = 0
x −1
01 Há muito tempo, quando poucas pessoas eram versadas na arte
de contar, houve uma grande tempestade no oceano. Um navio, colhido
(A) pelo tufão, foi salvo graças ao trabalho excepcional de dois marinheiros.
ƒ(x)
Terminada a borrasca, o capitão, decidido a recompensar seus dois
comandados pelo serviço bem executado, anunciou que dividiria entre
4 eles no dia seguinte o conteúdo de um pequeno baú com moedas de
ouro, tendo encarregado o seu imediato desta tarefa. Acontece que os
3 dois marinheiros eram muito amigos e, querendo evitar o constrangimento
de uma partilha pública, um deles teve a ideia na madrugada de pegar a
2
sua parte do prêmio. Indo ao baú, este marinheiro separou as moedas
em dois grupos idênticos e, para sua surpresa, sobrou uma moeda. Não
1
x sabendo como proceder, jogou-a ao mar para agradecer aos deuses a
–2 –1 0 1 2
sua sobrevivência e pegou a parte que lhe cabia. Porém, mais tarde o
(B) segundo marinheiro teve exatamente a mesma ideia. Indo ao baú, ele
ƒ(x)
separou as moedas em dois montes iguais e, para surpresa sua, sobrou
uma moeda. Jogou-a ao mar como agradecimento pela sua sorte e tomou
5 a parte que lhe cabia da recompensa. Pela manhã os dois marinheiros se
sentiram constrangidos em comunicar o procedimento noturno. Assim, o
4
imediato separou as moedas em dois grupos e verificou que sobrava uma.
3 Deu a cada marinheiro a sua parte do prêmio e tomou para si a moeda
restante como paga pelos seus cálculos. Sabendo-se que a razão entre as
2 moedas ganhas pelo primeiro e pelo segundo marinheiros foi de 29/17, o
1 número de moedas que havia originalmente no baú era:

x (A) 99. (D) 87.


–2 –1 0 1 2
(B) 95. (E) n.d.a.
(C) (C) 135.
ƒ(x)
02 Para que valores de k as equações x2 – 5x + k = 0 e x2 –7x + 2k =
5 0 admitem soluções de modo que uma das raízes da segunda equação
4 seja o dobro de uma das raízes da primeira?

3 03 Calcule p para que as equações x2 + 11x + p = 0 e x2 + 17x +


2p = 0 possuam uma raiz comum.
2
04 (CN 2011) A soma das raízes de uma equação do 2o grau é 2 e o
1
a3 − b3 − 2 ab2
produto dessas raízes é 0,25. Determine o valor de ,
x a2 − b 2
–2 –1 0 1 2
sabendo que a e b são as raízes dessa equação do 2o grau e a > b:
(D)
ƒ(x) 1 1
(A) . (D)
2+ .
2 4
5 1
(B) 3 − 2 . (E)
2− .
4 4 4
3 (C) – 1.

2
05 (CN 2006) O produto de dois números reais x e y é igual a 150. Assim
1 sendo, x + y não pode ser igual a:
x
–2 –1 0 1 2 (A) 31,71.
(B) 28,27.
(C) 25,15.
(D) 24,35.
(E) – 26,95.

110 Vol. 2
Binômio – Trinômio – Modular

06 (ITA 80) No sistema de coordenadas cartesianas ortogonais, a curva A soma dos comprimentos dos intervalos nos quais ela é verdadeira é igual a:
y = ax2 + bx + c passa pelos pontos (1, 1), (2, m) e (m, 2), onde m é
um número real diferente de 2. Sobre esta curva, podemos afirmar que: 11
(A) 3 . (D) .
4 6
1 3 3 7
(A) ela admite um mínimo para todo m tal que <m< . (B) . (E) .
2 2 2 6
(B) ela admite um mínimo para todo m tal que 0 < m < 1. 7
(C) .
1 1 3
(C) ela admite um máximo para todo m tal que − <m< .
2 2
1 3 15 (AFA 2004) Seja ƒ(x) = ax2 + bx + c uma função real definida para
(D) ela admite um máximo para todo m tal que < m < .
2 2 todo número real. Sabendo-se que existem dois números x1 e x2, distintos,
(E) ela admite um máximo para todo m tal que 0 < m < 1. tais que ƒ(x1) · ƒ(x2) < 0, pode-se afirmar que:

07 (ITA 86) Sejam a, b, c números reais dados com a < 0. Suponha (A) ƒ passa necessariamente por um máximo.
que x1 e x2 sejam as raízes reais da função y = ax2 + bx + c e x1 < x2. (B) ƒ passa necessariamente por um mínimo.

Sejam x3 = −
b
e x4 = −
(
2 b + b2 − 4 ac )
. Sobre o sinal de y, podemos
(C) x1 · x2 é necessariamente negativo.
(D) b2 – 4ac > 0.
2a 4a
afirmar que: 16 A equação x2 – 3mx + 4m2 = 0 tem as soluções entre –1 e 1 para
os seguintes valores de m:
(A) y < 0, ∀x ∈ , x1 < x < x3.
(B) y < 0, ∀x ∈ , x4 < x < x2. (A) m = 1. (D) para nenhum valor de m.
(C) y > 0, ∀x ∈ , x1 < x < x4. (B) 1 < m < 2. (E) n.r.a.
(D) y > 0, ∀x ∈ , x > x4. (C) m ≤ 0.
(E) y < 0, ∀x ∈ , x < x3.
17 Determine m para que o número 2 seja interno ao intervalo das raízes
08 (CN 2005) As raízes do trinômio do 2o grau y = ax2 + bx + c são de x2 – 2mx + m = 0.
1000 e 3000. Se quando x vale 2010, o valor numérico de y é 16, qual é 18 Se ambas as raízes de x2 + m(x – 1) = 0 são maiores que 1, qual o
o valor numérico de y quando x vale 1990? maior valor que o parâmetro m pode assumir?

(A) 64. (D) 8. 19 Calcule p para que o gráfico do trinômio y = x2 – px – 3 corte o eixo
(B) 32. (E) 4. dos x apenas no interior do segmento de abscissas externas –2 e 2.
(C) 16.
20 Determine m para que o número 2 seja exterior ao intervalo das raízes
09 Determine m para que o trinômio y =(1 – m)x2 – (1 + m)x + 2(m – 4) da equação (m – 1)x2 + (1 – 2m)x – 3 = 0.
seja negativo para todo x.
21 Dois barcos partem num mesmo instante de lados opostos de um rio de
10 Determine m para que x2 – 7x + 28 – 4m seja positivo para todo x negativo. margens paralelas. Viajam cada qual, perpendicularmente às margens, com
velocidade constante. Supondo que um deles é mais rápido que o outro, eles
11 Qual é o valor máximo de 21n – n2, para n inteiro? se cruzam num ponto situado a 720 m da margem mais próxima; completada
a travessia, cada barco fica parado no respectivo cais por 10 minutos. Na volta
12 Determine o valor real de k tal que o mínimo de 2x2 – 3x + k, para x eles se cruzam a 400 m da outra margem. Qual é largura do rio?
inteiro, seja 10.
x
7 22 Resolva a equação: x+ x − x− x =2 .
13 (ITA 85) Considere as seguintes funções: f ( x ) = x − e x+ x
2
1
g ( x ) = x 2 − definidas para todo x real. Então, a respeito da solução da 23 Determine os valores reais de λ para os quais a equação
4 x 2 + 1 = λ x − 1 admite solução real.
inequação |g o ƒ(x)| > g o ƒ(x), podemos afirmar que:
24 Resolva a equação 3
x − a + 3 x + a = 3 x.
(A) nenhum valor de x real é solução
(B) se x < 3, então x é solução 3
25 Resolva a equação x + 1 − x + 2 + 1 = 0.
7
(C) se x > , então x é solução
2 26 Se 3 x + 9 − 3 x − 9 = 3, x2 está compreendido entre:
(D) se x > 4, então x é solução
(E) se 3 < x < 4, então x é solução (A) 55 e 65.
(B) 65 e 75.
14 (ITA 2000) Sendo I um intervalo de números reais com extremidades (C) 75 e 85.
em a e b com a < b, o número real b – a é chamado de comprimento de I. (D) 85 e 95.
Considere a inequação: 6x4 – 5x3 – 7x2 + 4x < 0 (E) 95 e 105.

IME-ITA 111
Matemática I – Assunto 4

27 A solução de 2 x − 1 − x ≥ 2 é: 35 A área da região do plano cartesiano cujos pontos (x, y) satisfazem


|x| + |y| + |x + y| ≤ 2 é igual a:
(A) 0 ≤ x ≤ 1.
(B) Não existe x que satisfaça a inequação. (A) 2,5.
(C) x = 1. (B) 3.
(D) x ≥ 1. (C) 2.
(E) n.r.a. (D) 4.
(E) 3,5.
28 2 + x > 1 − x + 6 se e só se:
36 Resolva as inequações abaixo:
(A) x ≥ –2. (D) x < 1/4.
(B) x ≤ 6. (E) n.r.a. a. 5 x 2 − 2 x + 1 < 1
(C) – 2 ≤ x ≤ 6.
b. x 2 + x − 10 ≤ 3 x 2 + 7 x + 2
29 Seja ƒ(m) o número de raízes reais da equação x + m = x (onde
ƒ:  → ). x −3
c. ≥2
x2 − 5x + 6
a. Quantos elementos possui a imagem da função ƒ?
b. Exiba a função ƒ. x 2 − 7 x + 10
d. <0
x2 − 6x + 9
30 Resolva as inequações abaixo:
x 2 − x − 12
e. ≥ 2x
x −3
a. ( x − 3) x 2 − 4 ≤ x 2 − 9
x − 1 + x + 2 ≤ 1 d.
f. 2x + 1 − 5 > 2
b. 2 7 + x − 2 7 − x > 4 28 e. 1 + x 2 ( ) 2
x +1> x −1 2

g. x −2 − x +3 <5
x −4
c. < x +8
x +2 x2 − 2x + 1 x −1
h. 2
+ − 12 < 0
x − 4x + 4 x −2
31 Resolva a equação 2 n 1 − x + n 1 + x = 32 n 1 − x 2 .
37 Prove que:
32 Resolver a equação: x + x + 5 + 2 x = 25 − 2 x 2 + 5 x .
a. x − z ≤ x − y + y − z
33 (FUVEST) Seja m ≥ 0 um número real e sejam ƒ e g funções reais
definidas por ƒ(x) = x2 – 2 |x| + 1 e g(x) = mx + 2m. b. x − y ≤ x−y

a. Esboçar, no plano cartesiano representado a seguir, os gráficos de ƒ c. a − b < ε ⇒ a < b + ε


1
e de g quando m = e m = 1.
4 EXERCÍCIOS NÍVEL 3
y 01 Seja p > 0 um número real tal que x2 – 3px – p = 0 possui duas raízes
distintas x1 e x2. Prove que 3 px1 + x 22 − p > 0.

02 Mostre que dados 2n reais a1, a2,...,an, b1,..., bn sempre se tem:


( a12 + a22 + ... + an2 ) ⋅ ( b12 + b22 + ... + bn2 ) ≥ ( a1b1 + a2 b2 + ... + an bn )2
x
(Desigualdade de Cauchy-Schwarz).
Sugestão: Considere o trinômio P(x) = (a1 + b1x)2 + (a2 + b2x)2 + ...
+ (an + bnx)2 .

03 Determine os valores reais de x para os quais se tem


4x2
b. Determinar as raízes de ƒ(x) = g(x) quando m = 1. < 2 x + 9.
2 (1 − 1 + 2 x )2
c. Determinar, em função de m, o número de raízes da equação ƒ(x) = g(x).
x2 + 7
04 Determine o menor valor positivo que pode assumir a expressão .
34 (UFRJ) Seja ƒ a função real dada por ƒ(x) = ax2 + bx + c, com a > 0. x +1
Determine a, b e c sabendo que as raízes da equação |ƒ(x)| = 12 são – 2, 05 Calcule m para que (m2 – 1)x2 – (m + 2)x + 1 tenha um só de seus
1, 2 e 5. Justifique. zeros interno ao intervalo (–1, 1).

112 Vol. 2
Binômio – Trinômio – Modular

ax + b
06 Considere a função f : ℜ → ℜ , f (x) = , onde a, b, c, d são 15 Determine todas as soluções reais da equação x2 + 2y2 – 2xy + 2x
cx + d – 4y + 2 = 0 .
constantes reais. Mostre que se ƒ(x) = x para três valores distintos de x,
então necessariamente temos a = d e b = c = 0. 16 Quantas soluções inteiras possui a equação x2y + xy + x2 + y = 4?

07 Dados 3 reais a, b, c tais que a + b + c = 0, mostre que 17 Determine todos os x e y inteiros positivos tais que 2x2 + 3xy + y2 =
a2 + b2 + c2 a3 + b3 + c3 a5 + b5 + c5 5x + 2y + 3.
⋅ = .
2 3 5
18 As seguintes operações são permitidas com a função quadrática
ƒ(x) = ax2 + bx + c:
08 Dados 3 reais a, b, c tais que a + b + c = 0, mostre que
a2 + b2 + c2 a5 + b5 + c5 a7 + b7 + c7 . I. trocar a e c de lugar;
⋅ = II. trocar x por x + t, onde t é um número real;
2 5 7
Repetindo estas transformações, é possível transformar x2 – x – 2 em
x2 – x – 1?
09 Determine as soluções reais da equação (x2 – 3x – 2)2 – 3(x2 – 3x – 2) –
2–x=0. 19 Sejam a, b, c reais com a diferente de 0 tais que a e 4a + 3b + 2c
tem o mesmo sinal. Prove que a equação ax2 + bx + c = 0 não pode ter
10 Sendo a, b e c números ímpares, prove que a equação ax + bx 2
as duas raízes no intervalo (1,2) simultaneamente.
+ c = 0 não admite raízes racionais.
20 Encontre todos os números reais x para os quais vale
11 Os números inteiros a e b são tais que para dois valores inteiros
x −7 x −6 x −5 x − 1989 x − 1990 x − 1991
consecutivos a função ƒ(x) = x2 + ax + b assume valores quadrados + + = + +
1989 1990 1991 7 6 5 .
perfeitos, também consecutivos. Prove que ƒ(x) assume valores quadrados
perfeitos para todo valor inteiro de x.
21 Determine todos os valores inteiros de m para os quais mx2 – (m + 5)
x + 5m = 0 só admite raízes inteiras.
12 Prove que para todo p real o gráfico do trinômio y = x2 – px – 3 corta o
eixo dos x em algum ponto do interior do segmento de abscissas externas
22 Considere as equações x2 – (a – b)x + (b – c) = 0, x2 – (b – c)x +
–2 e 2.
(c – a) = 0 e x2 – (c – a)x + (a–b) = 0, onde a, b, c são reais. Mostre
que pelo menos uma das equações só possui raízes reais.
13 Sejam P1, P2 e P3 polinômios quadráticos com coeficientes líderes
positivos e raízes reais. Mostre que, se cada par de polinômios tem uma
23 a, b, c e d são números reais distintos tais que a e b são as raízes da
raiz em comum, então o trinômio P1 + P2 + P3 possui raízes reais.
equação x2 – 3cx – 8d = 0 e c e d são as raízes da equação x2 – 3ax – 8b
= 0. Calcule a soma a + b + c + d.
14 João e Gilberto jogam o seguinte jogo:

– João escreve num quadro dois números reais; 24


– Em seguida, Gilberto escreve um número real no quadro; a. Para a < b < c < d, determine o valor mínimo da função
– Então, João monta uma equação do 2o grau com coeficientes iguais ƒ(x) = |x – a| + |x – b| + |x – c| + |x – d|.
aos três números escritos no quadro (os coeficientes ficam numa b. Para a < 106 < c, determine o valor mínimo da função ƒ(x) = |x – a|
ordem escolhida por João). + |x – 106| + |x – c|.
Mostre que Gilberto sempre pode escolher o número que vai escrever
no quadro de modo que a equação escrita por João possua raízes 25 (ESTÔNIA) Determine o valor de a para que a equação |x – 1| +
reais. |x – 2|+...+|x – 2001| = a possua exatamente uma solução.

RASCUNHO

IME-ITA 113
Matemática I – Assunto 4

RASCUNHO

114 Vol. 2
Números complexos A ssunto
2
Matemática II

1. Motivação 3.3 Divisão


ac + bd bc − ad
Há mais de uma maneira de iniciarmos este assunto, no entanto, z/w = + i
vamos nos ater à ordem histórica. c2 + d 2 c2 + d 2

2
Ex.: z = 2 + 4 i , w = 3 − 2 i → z ÷ w =
Ao tentar determinar as raízes de uma equação como x + 2 x + 2 = 0,
2·3 + 4·( −2) 4·3 − 2·( −2) 2 16
percebemos que seu discriminante é negativo. Já sabemos que, neste + 2 i= + i
32 + ( −2)2 3 + ( −2)2 13 13
caso, a equação não possui raízes reais. Mas se usássemos a fórmula de

Bhaskara assim mesmo? Assim, chegaríamos a


−2 ± −4
. Veja que, de 4. Igualdade (igualar partes reais
2
fato, esse número não pertence ao conjunto dos Reais, já que há ali um e igualar partes imaginárias)
−4 .=Se2 pudermos
−1 escrever −4 = 2 −1 , teremos que x = −1 ± −1 . Sejam z e w números complexos tais que z = a + bi e w = c + di.,
com a, b, c e d reais:
Veja que este x possui uma parte que é real (o –1) e também uma z=w⇔a=ceb=d
parte que não é (o −1) .
a−c
Demonstração: De fato, se b ≠ d, podemos escrever i = . Isso é
d−b
Assim, definimos a chamada unidade imaginária, que é chamada de uma contradição, pois o lado direito da equação é real e o direito não.
i = −1 . (É importante ficar muito claro que i não é um número real.
Logo, b = d e, assim, a = c.
Como i 2 = −1 < 0 , isso é verdade.)
Por conta disso, numa equação com números complexos temos, na
2. Definição verdade, duas equações: podemos igualar as partes reais e, também,
Definimos  = {a + bi | a, b ∈ } como o conjunto dos números igualar as partes imaginárias.
complexos. Para um elemento z=a+bi, com a e b reais, denotamos por
5. Conjugado
Re(z) = a (parte real de z)
Im (z) = b (parte imaginária de z) Se z=a+bi é um número complexo, definimos como z − a − bi o
seu conjugado.
Se Im(z) = 0, temos que z é real (ou seja, o conjunto dos complexos Propriedades:
contém o dos reais).
I. z=z
Se Re(z) = 0 e Im(z) ≠ 0, dizemos que z é um imaginário puro.
II. z + w = z + w
3. Operações algébricas III. zw = z·w
(sem surpresas) z z
 =
As operações são feitas tratando i como uma variável e trocando IV.  w  w
sempre i2 por – 1. Sendo z = a + bi e w = c + di complexos, definem-se: z+z z−z
V. Re( z ) = e Im( z ) =
2 2i
3.1 Adição e subtração
VI. z é real ⇔ z = z
z ± w = (a ± c) + (b ± d)i
 z = − z
VII. z é imaginário puro ⇔ 
Ex.: z = 2 +4i, w = 3 – 2i →  z ≠ 0
→ z + w = (2 + 3) + (4 + (–2))i = 5 + 2i
→ z – w = (2 – 3) + (4 – (–2))i = –1 + 6i 6. Módulo
3.2 Multiplicação Dado um complexo z = a + bi, definimos o módulo de z como
z · w = (ac – bd) + (bc + ad)i |z|= a2 + b2 .

Ex.: z = 2 +4i, w = 3 – 2i → z · w = (2 · 3 – 4 · (–2)) + (4 · 3 + 2 · Mais à frente, veremos este conceito de uma forma geométrica e tudo
(–2))i = 14 + 8i fará mais sentido.

IME-ITA 115
Matemática II – Assunto 2

Propriedades: Por isso, é natural definir cisθ = cos θ + isenθ . Então, temos a
I. z ∈ ¡ + chamada forma trigonométrica de um complexo:
II. z = z
III. z = z z = ρ·cisθ
2
IV. z = z· z (uma boa maneira de eliminar um módulo é elevando-o ao
quadrado) Ex.: z = 1 + i
1 2 1 2 π
V. zw = z w ρ = 12 + 12 = 2, cos θ = = , senθ = = ⇒ θ = , então:
2 2 2 2 4
z z π
VI. = z = 2 . cis
w w 4
VII. z + w ≤ z + w (desigualdade triangular) 8.1 Conjugado
Dado z = ρ·cisα , veja que seu conjugado é dado por z = ρ·cis( −α) .
7. Potências de i Para isso, basta lembrar que cos( −α) = cos α e sen( −α) = − senα .

(deixe o resto do expoente por 4) 8.2 Igualdade


Em muitas situações, é necessário elevar i a um número grande. Sendo z = ρ1·cisα , w = ρ2 ·cisβ tem-se:
0
i = 1, 1
i = i, 2
i = −1, 3 2
i = i ·i = − i, =
i (=
i ) 1 4 2 2 z = w ⇔ (ρ 1 = ρ 2 ) ∧ α − β = 2 k π, k ∈ Z

i 4k ) (=
Mais geralmente, como (= i 4 )k 1 tem-se, para todo k inteiro:
9. Propriedades do cis
π
i 4=
k 0
i= 1; i 4 k +1 = i 1 = i; i 4 k + 2 = i 2 = −1; i 4 k +3 = i 3 = −i I. cis0 = 1, cis =i
2
II. cisα·cisβ = cis( α + β)
Ou seja, basta deixar no expoente o seu resto na divisão por 4. 1
III. cis( −α) =
cisα cis α
Ex.: i273 = i4.68+1 = i1 = i IV. = cis( α − β)
cisβ
8. Plano de Argand-Gauss V. (cis α)n = cis(n α) , para n inteiro (1a Fórmula de DeMoivre).
e forma trigonométrica Demonstrações:
I. definição.
Como cada complexo z = a + bi está definido por 2 parâmetros (a e b)
II. cisα·cisβ = (cos α + isenα)(cos β + isenβ) = (cos α cos β − senαsenβ) + i( senα cos β
de forma única, podemos fazer uma associação direta entre números
cisα·cisβ = (cos α + isenα)(cos β + isenβ) = (cos α cos β − senαsenβ) + i( senα cos β + senβ cos α) = cos( α + β) + isen( α + β) = cis( α + β) .
complexos e pontos no plano:
III. Basta fazer β = −α .
a + bi → ( a, b)
cisα 1
IV. = cisα· = cisα·cis( −β) = cis( α − β)
Assim, representaremos cada complexo z = a + bi por um ponto no cisβ cisβ
chamado plano de Argand-Gauss. V. para n positivo, basta usar II várias vezes; para n negativo, basta usar
III antes.
Im(z) P(z=a+bi)
10. Multiplicação e divisão na
ρ
forma trigonométrica
b
Sejam z e w números complexos tais que z = ρ1cisα e w = ρ2 cisβ .
a
• Multiplicação
θ
z·w = ρ1ρ2 cis( α + β)
O Re (z)
• Divisão
θ: argumento z ρ1
→ = cis( α − β)
P : afixo
w ρ2
|OP|= ρ
a • Potência
cos θ = → a = ρ cos θ
ρ z n = ρ n cis(n α)
b • Interpretação geométrica
senθ = → b = ρsenθ
ρ Ao multiplicarmos um complexo z por u = cisα , onde u é unitário,
geometricamente estamos girando o vetor z (isto é, o vetor que sai da
ρ = a2 + b 2 origem e termina em z) de um ângulo α (sentido anti-horário).
• Distância
Daí, z = a + bi = ρ cos θ + iρ sen θ ⇒ z = ρ(cos θ + i sen θ) A distância entre os afixos dos complexos z e w é igual a | z − w | .
(Isso pode ser muito útil para resolver alguns problemas!)

116 Vol. 2
Números complexos

11. Radiciação 13. Transformação arco-metade


(2a fórmula de De Moivre) É muito comum, em problemas, aparecerem as expressões cisθ + 1 e
cisθ − 1. O interessante é que conseguimos fatorar essas expressões
θ θ
 α + 2k π  I. cisθ + 1 = cis ·2 cos
w n = ρ · cisα ⇒ w = n p · cis  , k = 0, 1, 2, ..., n − 1 2 2
 n 
θ θ
II. cisθ − 1 = cis ·2 i ·sen
2 2
Demonstração: Seja w = rcisθ um complexo tal que w n = ρcisα . Na
forma trigonométrica: Podemos escrever, de forma equivalente, utilizando a fórmula de Euler:

θ iθ
r n cis(n θ) = ρ cis α e i θ − 1 = 2 i ·sen ·e 2
α + 2k π 2
Logo, r = n ρ e α − nθ = 2 k π , k ∈ Z , o que dá ∴ θ = . θ i2θ
n iθ
e + 1 = 2·cos ·e
2
Para ver que só precisamos tomar k = 0,1, 2,..., n − 1 , veja que
α + 2k π EXERCÍCIOS RESOLVIDOS
θ= assume valores côngruos de n em n.
n
Propriedade: As raízes n-ésimas do complexo ρcisα determinam um 01 Determine todos os complexos z tais z + 2 i z = 1 + 2 i .
polígono regular de n lados inscrito em uma circunferência de raio ρ .
n

Solução: Seja z = a + bi , com a e b reais. A equação é equivalente


Isso acontece porque os argumentos das raízes estão em P.A.
a ( a + bi ) + 2 i( a − bi ) = 1 + 2 i , ou seja, ( a + 2 b) + ( 2 a + b)i = 1 + 2 i .
Agora, podemos igualar as partes reais e igualar as partes imaginárias.
Obs.: As raízes n-ésimas de 1 são comumente chamadas de “raízes da
2k π  a + 2b = 1
unidade”. Usando a fórmula acima, veja que z n = 1= cis0 implica z = cis Com isso, obtemos o sistema  , que tem como solução
k
n 2 a + b = 2
 2k π   2k π  2π a = 1 e b = 0. Portanto, a única solução é z = a + bi = 1 + 0 i ·
, k = 0,1, 2,...,n− 1 . Como cis   =  cis n  , fazendo n = ξ , as
 n    ∴ S = {1} .
raízes n-ésimas da unidade são ξ0 , ξ1, ξ2 ,..., ξ n−1 . 1 2 3 2011
02 Calcule o produto P = i ·i ·i ·...·i .

2 kπ Solução: Podemos somar os expoentes e obter que o produto


Ex.: 1 = cis
5
(abuso de notação)
5 é igual a P = i 1+ 2 + 3 +...+ 2011 . Somando a PA do expoente, temos
2011· 2012
k = 0 → z1 = 1
=P i=
2
i 2011·1006 . Então, P = ( i 2 )2011·503 = ( −1)ímpar = −1.
2π 2π 2π
k = 1 → z2 = cis = cos + isen 03 (Mack) Se z = (a + bi)4 é um número real estritamente negativo,
5 5 5
podemos ter:
4π 4π 4π
k = 2 → z3 = cis = cos = isen
5 5 5 (A) a + b = 0
6π 6π 6π (B) a + 2b = 0
k = 3 → z4 = cis = cos + isen
5 5 5 (C) 2a + b = 0
5 8π 8π 8π (D) a + 4b = 0
k = 4 → z = cis = cos + isen
5 5 5 (E) 4a + b = 0

Obs.:
Solução: Como (a + bi)2 = (a 2 − b2 ) + 2abi , elevando ao quadrado,
I. Qualquer outro valor de k dará uma solução repetida.
II. As cinco raízes são os vértices de um pentágono regular centrado na origem. temos que z = (a + bi)4 = (a 2 − b2 )2 − 4 a2 b2 + 4 ab(a 2 − b2 )i .
Para que esse número seja real, devemos ter 4 ab(a 2 − b2 ) = 0 , ou seja,
a = 0 ou b = 0 ou b = a ou b = –a.
12. Fórmula de Euler
Precisamos definir o que é uma potência com expoente complexo. Temos:
É possível formalizar esta parte com os conceitos de fórmula de Taylor ou a = 0 ⇒ z = b4 > 0
de equações diferenciais, no entanto, ficaremos apenas com o resultado:
b = 0 ⇒ z = a4 > 0
eιθ = cis( θ) a = ± b ⇒ z = −4 a4

Veja que as propriedades do cis são compatíveis com as da exponencial,


como, por exemplo: cis( α + β) = e i ( α + β ) = e i α + iβ = e i α e iβ = cisα·cisβ

IME-ITA 117
Matemática II – Assunto 2

Portanto, para que z seja estritamente negativo, podemos ter a + b = 0 06 No plano de Argand-Gauss, qual é o formato do lugar geométrico
ou a – b = 0. dos afixos dos z tais que |z – i| = |z – 2|?

Resposta: Letra A. Solução: Poderíamos fazer a conta normalmente neste problema. No


n entanto, como é um problema qualitativo, podemos dar um argumento
04 Determine o menor natural n, n > 1, tal que ( 3 + i) é um número direto. Lembrando que | z − i |= “distância de z até i” e | z − 2|= “distância
real positivo. de z até 2”, queremos que o afixo de z equidiste de i e 2. Portanto, o L.G.
é a mediatriz do segmento definido pelos pontos (0,1) e (2,0).
Solução: Como queremos efetuar uma potência, uma alternativa é colocar a
3 + 12 = 2 e tanθ = 1 .
2
3
base na forma trigonométrica. Veja que ρ = 07 (VUNESP – adaptada) Quais são as raízes da equação z = i ?
3
π
Como 3 + i está no 1o quadrante, temos 3 + i = 2cis . Portanto, Solução: Primeiramente, o lado direito da equação deve ser colocado
6 nπ π
usando a fórmula de DeMoivre, temos que ( 3 + i ) = 2 cis . Para
n n
na forma trigonométrica: z 3 = cis .
6 2

que esse número seja real e positivo, devemos ter = 2 k π , com k
6  π + 2k π 
inteiro. Daí, segue que n = 12k, ou seja, n é múltiplo de 12. Portanto, Usando a 2a fórmula de DeMoivre: Z = cis  2  = cis  π + 2 k π  , k = 0,1, 2.
 3  6 3 
o menor valor natural > 1 existente é n = 12. 
 
 π + 2k π 
 = cis  π + 2 k π  , k = 0,1, 2. Substituindo os valores de k, as raízes são: cis ≠ , cis 5≠ , cis 3≠ .
cis  de2 Argand-Gauss,
05 Qual é o lugar geométrico, noZ =plano  3   6 dos3 
  6 6 2
complexos z tais que | z − 2 + 3 i |= 4 ?    3 1 3 1 
Então, S =  + i; − i; − i .
Solução: Podemos escrever a equação dada como |z – (2 – 3i) = 4.  2 2 2 2 
Lembrando que |z – w| é igual à distância entre os afixos de z e w,
temos que o afixo de z dista 4 unidades do afixo de 2 – 3i. Portanto, o 08 Um quadrado está centrado na origem do plano de Argand-Gauss.
lugar geométrico é uma circunferência de raio 4 e centro no ponto (2, –3). Se um dos vértices é o afixo do complexo 2 + i, qual é o produto de
todos os complexos associados aos vértices?
Obs.: O aluno que já está habituado à equação da circunferência pode
Solução: Seja w = 2 + i. Como as diagonais do quadrado são
escrever z = x + yi, com x e y reais e ver que | z − 2 + 3 i |=|( x − 2) + ( y + 3)i |= (x + 2)2 + (y + 3)2 e de mesmo comprimento, se cortando ao meio,
perpendiculares
+ ( y + 3)i |= (x + 2)2 + (y + 3)2 . Isso gera a equação , que representa exatamente temos que os vértices são w, wi, –w, –wi. Daí, o produto pedido
a equação de centro (x + 2) + (y + 3) = 16 (2,–3) e raio 4.
2 2 é igual a −w 4 = −( 2 + i)4 . Como ( 2 + i)2 = 3 + 4 i , temos que
( 2 + i)4 = (3 + 4 i )2 = −7 + 24 i . Portanto, o produto pedido é igual a
7 − 24 i .

EXERCÍCIOS NÍVEL 1
01 Determine reais x e y tais que:
05 (ITA 85) Seja a um número real. Os valores de z∈C que satisfazem
 a + z 10   a + z 
10

a. (1 − 2 i) x + (1 + 2 i)y = 1 + i    ∈ ℜ são:
x −3 y −3  1 + i   1 − i 
+ =i
b. 3 + i 3 − i
(A) z = − a + i 10 | a|
16 8 (B) não é possível determiná-los.
 1+ i   1− i 
02   +  . (C) z = − i 10 | a|
 1− i   1+ i  (D) não existe z∈C tal que isto aconteça
03 Se z e w são números complexos, prove que: (E) todo z ∈ ℜ

a. z + w = z + w 06 Resolva as equações :
b. zw = z·w a. iz 2 + ( 2 + 2 i ) z + 2 − i = 0
c. |zw|=|z||w| 2
b. z + z + 1 = 0
1
d. | z |= 1 ⇔ z =
z 07 Um imaginário puro é um complexo cuja parte real é nula. Determine
2 + ai
04 Prove que os números abaixo são reais, para todo n natural: a real para que seja um imaginário puro.
1− i
(2 + i 5 ) + (2 − i 5 )
n n

a. 2014

n
 19 + 7 i   20 + 5 i 
n 08 Determine o valor do somatório S = ∑i k
, em que i = −1 .
 9 − i  +  7 + 6i 
k =0

b.    
09 Resolva o sistema a seguir:  z + wi = w

 i z + w = 2 i − 1

118 Vol. 2
Números complexos

10 Resolva o sistema de equações abaixo, em que z e w são complexos: 17 Determine os possíveis valores de 3 + 4i .
 iz + (1 + i)w = 1

(1 + i ) z − (6 + i)w = −4 − 8 i 18 (AFA 03) Dado o número complexo z tal que z + 2 z − 9 = 3 i , é correto
afirmar que:
2
11 (ITA 93) Resolvendo a equação z = 2 + z no conjunto dos números
complexos, conclui-se sobre as suas soluções que: (A) |z|= 3 10 .
 7π 7π 
(A) nenhuma delas é um número inteiro. (B) z = 3 2  cos + isen  .
(B) a soma delas é 2.  4 4 
(C) estas são em número de 2 e são distintas. (C) z = 9 − 3 i .
(D) estas são em número de quatro e são 2 a 2 distintas. 1+ i
−1
(E) uma delas é da forma z = bi com b real não nulo. (D) z = .
3
12 (ITA 94) Sejam x e y números reais, com x ↑ 0 , satisfazendo 19 (AFA 99) A representação trigonométrica do conjugado do número
( x + yi )2 = ( x + y )i . Então: complexo z = (1 + 3 i )5 , sendo i a unidade imaginária e k inteiro, é:

(A) x e y são números irracionais. π  π 


(B) x > 0 e y < 0. (A) 32 cos  + 2 k π  − 32 isen  + 2 k π  .
3  3 
(C) x é uma raiz da equação x 3 + 3 x 2 + 2 x − 6 = 0 .
(D) x < 0 e x = y.  5π   5π 
(B) 32 cos  + 10 k π  − 32 isen  + 10 k π  .
1  4   4 
(E) x 2 + xy + y 2 = .
2  5π   5π 
(C) 32 cos  + 10 k π  − 32 isen  + 10 k π  .
13 (ITA 87) Seja N o número de soluções reais da senx = | 2 + 3i|.  6   6 
Então, temos:
 5π   5π 
(D) 32 cos  + 10 k π  − 32 isen  + 10 k π  .
(A) N > 50. (D) N = 1.  3   3 
(B) N = zero. (E) N > 2 e N < 10.  2 
93

(C) N = 2. 20 (ITA 96) O valor da potência   é:


 1+ i 
14 Resolva as equações: −1 + i
( )
93
(A) . (D) 2 i.
2
a. | z | −2 z = 3 − 4 i
b. | z | + z = 3 + 4 i 1+ i
( )
93
(B) 2 +i.
. (E)
c. z 2 + |z|= 0 2
−1 − i
15 (ITA 87) Seja S a coleção de todos os números complexos z, que são (C) .
2
raízes da equação |z|–z=1+2i, em que i é a unidade imaginária. Então,
podemos garantir que:
21 (AFA 01) Considere o polinômio P(z) = z 2 − 2 z + 2 iw , w complexo.
3  1  Se P(3 + 2 i ) = 1 + 10 i , em que i = −1 , então uma forma trigonométrica
S =  + 3i  .
(A) S =  − 2 i  . (D)
2  4  de w é:
1 1 
S = {1 + 2 k π; k = 1, 2, 3} .
(B) S =  + 2 i , − − 2 i  . (E)  π π
2 2  (A) 2  cos + isen  .
 4 4
1 
(C) S =  + 4 k π;k = 1, 2, 3  . 3π 3π
2  (B) 2  cos + isen  .
 4 4 
16 (ITA 89) O produto dos números complexos z = x + yi, que têm
módulo igual a 2 e se encontram sobre a reta y = 2x – 1 contida no  7π 5π 
(C) 2  cos + isen  .
plano complexo, é igual a:  4 4 
 7π 5π 
6 8 (D) 2  cos + isen  .
(A) − i.  4 4 
5 5
22 (ITA 94) Considere as afirmações:
4 2
(B) − i.
58 58 I. (cos θ + isenθ)10 = cos(10θ) + isen(10θ) , para todo θ real
(C) − − i . II. (5 i ) / ( 2 + i ) = 1 + 2 i .
5 5
(D) 2 + 2i. III. (1 − i )4 = −4 .
(E) não existe nenhum complexo que pertença à reta y = 2x – 1 e cujo IV. Se z 2 = (z)2 , então z é real ou imaginário puro.
módulo seja 2 . 4 3
V. O polinômio x + x − x − 1 possui apenas raízes reais.

IME-ITA 119
Matemática II – Assunto 2

Podemos concluir: + 2( z)−-n 9seja


A sequência de todos os valores de n naturais, ztais = 3um
i imaginário
puro é uma progressão:
(A) Todas são verdadeiras.
(B) Apenas quatro são verdadeiras. (A) aritmética com primeiro termo igual a 2 e razão 8.
(C) Apenas três são verdadeiras. (B) geométrica com primeiro termo igual a 2 e razão 2.
(D) Apenas duas são verdadeiras. (C) aritmética com primeiro termo igual a 2 e razão 4.
(E) Apenas uma é verdadeira. (D) geométrica com primeiro termo igual a 2 e razão 1.

n
23 Seja z = i + 3 . Escrever, na forma a + bi , o complexo z 15 .  3 1 
31 (EN 01) O valor do menor inteiro positivo n tal que  + i  seja

24 Utilizando as fórmulas de De Moivre, mostre que imaginário puro, com coeficiente negativo é:  2 2 
cos( 3α) = 4 cos3 α − 3 cos α
 3
(A) 3.
 sen(3α) = 3 sen α − 4 sen α (B) 5.
(C) 6.
25 (D) 9.
a. Desenhe e calcule o ângulo formado pelos complexos i – 2 e 3 + i.
ix
i −2 32 Admitindo a fórmula de Euler e = cisx , (e = 2,71...):
b. Determine o argumento de .
3+i
26 (ITA 80) Seja z um número complexo de módulo 1 e de argumento θ. (A) Calcule e2≠i ;

1 (B) Calcule e 4 ;
i
Se n é um número inteiro positivo z n + n é igual a:
z e xi + e− xi e xi − e− xi
(C) Prove que cos x = e senx = ;
(A) cos(nθ). ≠i 2 2i
(B) 2 cos(nθ). (D) Calcule e 2 e conclua que um valor real correspondente a i i é e−π / 2 .
(C) sen(nθ).
(D) 2sen(nθ).  π π
(E) sen(nθ)+ cos(nθ). 33 (ITA 92) Sabe-se que 2  cos + isen  é uma raiz quíntupla de
 20 20 
w − 16 2 i
w. Seja S o conjunto de todas as raízes de z − 2 z +
4 2
27 (ITA 97) Considere os números complexos z = 2 + i 2 e = 0 . Um
2 subconjunto de S é: 8 2
w6 + 3 z4 + 4i
w = 1 + 3 i . Sendo m = , então m vale:
z 2 + w 3 + 6 − 2i
 1/ 4  7π 7π  1/ 4  π π 
(A) 34. (A) 2  cos + isen  ; 2  cos + isen   .
  8 8   8 8 
(B) 26.
(C) 16.   9π π  5π 5π  
(B) 21/ 4  cos + isen  ; 21/ 4  cos + isen   .
(D) 4.   8 8  8 8 
(E) 1.
 7π 7π π π 
(C) 21/ 4  cos + isen  ; 21/ 4  cos + isen   .
28 (ITA 97) Considere um hexágono regular centrado em z0 = i. Represente   4 4   4 4 
por z1,z2,..., z6 os seus vértices, quando percorridos no sentido anti-horário.   7π 7π  π π 

Se z1 = 1, então 2z3 é igual a: (D) 21/ 4  cos + isen  ; 21/ 4  cos + isen   .
  8 8   4 4 
(A) 2 + 4i. (E) n.d.a.
(B) ( 3 − 1) + ( 3 + 3)i .
34 Determine o lugar geométrico das imagens dos complexos z tais que:
(C) 6 + ( 2 + 2)i .
(D) ( 2 3 − 1) + ( 2 3 + 3)i . (A) |z| = 1
(
(E) 2 + 6 + 2 i . ) (B) |z + i| < 1

29 (ITA 93) Seja a o módulo do número complexo ( 2 − 2 3 i )10 . Então, o 35 Determine os possíveis valores complexos de 3
−1 .
valor de x que verifica a igualdade (4a)x = a é:
36 Determine as raízes da equação 2x2 + 4x + 2ix + 3 = 0.
(A) 10/11.
(B) –2. 37 (ITA 87) A soma de todas as raízes da equação z3 – 1 = 0 é:
(C) 5/8.
(D) 3/8. (A) 1.
(E) 1/5. (B) 2.
(C) zero.
1 i (D) −2 2i .
z + 2 z −o 9conjugado
30 (AFA 01) Seja = 3i do número complexo z = + .
2 2 (E) 2 + 3 i .

120 Vol. 2
Números complexos

38 (ITA 88) Seja a equação z4 – a – bi = 0, em que a e b são reais não  x 3 + 3 xy 2 = 1


nulos. Sobre as raízes desta equação, podemos afirmar que: 06 (ITA 98) Sejam x e y números reais tais que  2 3
. Então,
3 x y − y = 1
(A) uma delas é um imaginário puro. o número complexo z = x + yi é tal que z3 e |z| valem, respectivamente:
(B) os seus módulos formam uma progressão aritmética de razão
(|a+bi|)1/4 (A) 1 – i e 6 2 .
(C) o seu produto é um imaginário puro. (B) 1 + i e 6 2 .
(D) cada uma tem argumento igual a [arg (a + bi) ] / 4. (C) i e 1.
(E) a sua soma é zero. (D) – i e 1.
39 (ITA 98) Considere, no plano complexo, um polígono regular cujos (E) 1 + i e 3 2 .
vértices são as soluções da equação z6 = 1. A área deste polígono, em
unidades de área, é igual a: 07 (ITA 89) O valor da expressão |1 − z |2 + |1 + z |2 , sendo z um número
complexo, é:

(A) 3 . (D) 3 3 .
(A) 5, se | z |″ 1 .
(B) 5. (E) 2π.2 (B) 4, se | z |= 1 .
(C) π. (C) 0, se Im(z) = 0.
(D) 2, para todo z.
40 (EN 97) As soluções da equação (z – 1 + i)4 = 1pertencem à curva: (E) 3, se Re(z) = 0.
(A) x2 – x + y2 + y = 0. 08
(B) x2 + y2 – 2x + 2y + 1 = 0.
(C) x2 + y2 – 2x – 2y + 1 = 0.
a. Mostre que |a + b|2 + |a − b|2 = 2(|a|2 + |b|2 ), para todos a e b
(D) x2 + y2 = 1.
complexos.
(E) x2 – x + y2 – y = 0.
b. A partir disso, deduza um teorema geométrico.
c. Deduza também uma fórmula para a mediana de um triângulo em função
EXERCÍCIOS NÍVEL 2
de seus lados.
01 (ITA 88 e IME CG) O número natural n tal que ( 2 i )n + (1 + i )2 n = −16 i ,
em que i é a unidade imaginária do conjunto dos números complexos, vale: 09 Prove que, se |1+ i z| = |1 – i z|, então z é real.

(A) n = 6. 10 (ITA 81) O conjunto A definido por A = { z ∈ C;( z − i )( z − i ) = 4}


(B) n = 3. representa no plano complexo:
(C) n = 7.
(D) n = 4. (A) uma elipse cujos focos se encontram nos pontos i e –i.
(E) não existe n nessas condições. (B) uma circunferência de centro no ponto (0,1) e raio 2.
(C) uma circunferência de centro no ponto (0,0) e raio 4.
az + b (D) um par de retas que se encontram no ponto (1,1).
02 Considere uma função da forma f ( z ) = , tal que f(i) = 2i + 3
cz + d (E) nenhuma das anteriores.
e f(3) = 2. Determine f sabendo que a, b, c, d são números reais.
z − 12 5 z−4
11 Determine os números complexos z tais que = e = 1.
03 Seja p(x) um polinômio com coeficientes reais. Mostre que p(x) = p(x), z − 8i 3 z −8
para todo x. Conclua que se a+bi é raiz de p(x), então seu conjugado a – bi
12 (ITA 97) Seja S o conjunto dos números complexos que satisfazem
também é.
simultaneamente as equações:

(ad− bc)(z − z) | z − 3 i | = 3 e | z + 1| = | z − 2 − i |
04 Sejam a, b, c e d reais. Prove que w − w = , em que
az + b | cz + d |2
w= . O produto de todos os elementos de S é igual a:
cz + d
Conclua que, se ad – bc > 0, então as partes imaginárias de z e w têm o (A) −2 + i 3 . (D) −3 + 3i .
mesmo sinal. −2 + 2i .
(B) 2 2 + 3 i 3 . (E)
(C) 3 3 − 2 i 3 .
05 (ITA 91) Seja w = a + bi com b ↑ 0 e a, b, c,∈ ℜ . O conjunto dos
números complexos z que verificam a equação wz + wz + c = 0 , descreve: 13 Demonstre que, para todos os complexos z1, z2, z3, tem-se que:

(A) um par de retas paralelas. a. | z1 + z2 |2 + | z1 + z3 |2 + | z2 + z3 |2 =| z1 |2 + | z2 |2 + | z3 |2 + | z1 + z2 + z3 |2 ;


(B) uma circunferência.
| z1 + z2 | + | z1 + z3 | + | z2 + z3 | =| z1 | + | z2 | + | z3 |2 + | z1 + z2 + z3 |2 ;
2 2 2 2 2
(C) uma elipse.
a b. |1 + z1 z 2 |2 + | z1 − z2 |2 = (1+ | z1 |2 )(1+ | z2 |2 )
(D) uma reta com coeficiente angular m = . c. |1 − z1 z 2 |2 − | z1 − z2 |2 = (1− | z1 |2 )(1− | z2 |2 )
(E) n.d.a. b

IME-ITA 121
Matemática II – Assunto 2

14 (ITA 89) Considerando que a imagem da função arcsen é o intervalo 22 (ITA 78) O lugar geométrico, no plano complexo, representado pela
 π π  1 + xi  equação z z − z 0 z − z 0 z + k = 0 , em que k é um número real positivo e
 − 2 , 2  e i = −1 , podemos garantir que arcsen  1 − xi  está definida: |z20|>k, é:
   

(A) apenas para x = 0 e vale . a) uma hipérbole com centro z0.
2 b) uma elipse com um dos focos em z0.
≠ c) uma circunferência com centro em z0.
(B) para todo x ∈ ℜ e vale . d) uma parábola com vértice em z0.
2
e) n.d.a.
(C) apenas para x ∈ ℜ tal que |x| < 1 e seu valor depende do valor de x.
(D) apenas para x ∈ ℜ tal que x ≥1 e seu valor é π 23 (ITA 86) No conjunto C os números complexos, seja a tal que |a| < 1.
(E) apenas para x ∈ ℜ tal que x ≤ – 1 e seu valor depende do valor de x. z−a
O lugar geométrico dos pontos z ∈ C que satisfazem a igualdade = 1 é:
2 1 − az
15 Os complexos de módulo 1 tais que |z 2 + z |= 1 determinam um
(A) uma circunferência de centro na origem e raio 1.
polígono no plano complexo.
(B) uma hipérbole.
(C) uma elipse de semieixo maior ou igual a 1.
a. Qual é o gênero desse polígono? Ele é regular?
(D) uma parábola.
b. Quanto vale a área do polígono?
(E) formado por duas retas concorrentes.
16 (ITA 99) Sejam ak e bk números reais com k=1, 2, ..., 6. Os números
24 Sejam x1 e x2 as raízes da equação x2 – x + 1 = 0 . Calcule:
complexos zk = ak + ibk são tais que |zk| = 2 e bk ≥ 0, para todo k=1,
1
2,..., 6. Se (a1, a2, ..., a6) é uma progressão aritmética de − e soma 9, a. x12011 + x22011
então z é igual a: 5 b. x12010 + x22010
3
c. x1n + x2n, n inteiro.
−3 3 73
(A) 2i. (D) + i. n n
5 5  −1 + i 3   −1 − i 3 
25 Determine todos os inteiros n tais que   +   = 2
8 6 4 2 2 17 . .  2   2 
(B) + i . (E) + i
5 5 5 5
1
(C) 3+i. 26 Seja x um número complexo tal x + = 2cosα .
x
17 Determine o lugar geométrico das imagens dos complexos z tais que a. Resolva a equação do 2o grau correspondente e ache x.
|2z – 1 + i| ≤ 4. 1
b. Mostre que x n + n = 2cos( nα) .
x
18 Determine os valores máximo e mínimo de |z+i|, quando |z – 2|=1. 27 Supondo 2 π < θ < 4 π, determine o argumento principal do complexo
1 + cisθ
19 Entre os números complexos z que satisfazem a condição |z – 25i| .
≤ 15, determine o de menor argumento. 1 + cis( −θ)

20 (ITA 81) Sejam a e k constantes reais, sendo a > 0 e 0 < k < 1. De 3


28 Resolva a equação z = z .
todos os números complexos z que satisfazem a relação |z – ai| ≤ ak,
qual é o de menor argumento? 1 3 3 n +1
29 Seja z = − −
3m
i . Para m, n e p naturais, calcule z + z + z 3 p+ 2 .
2 2
2 2
(
(A) z = ak 1 − k + ia 1 − k . ) θ iθ
2
( 2
(B) z = k 1 − k − ia 1 − k . ) iθ

30 Mostre que valem as seguintes fatorações : e − 1 = 2 isen ·e 2 e
2
θ
(C) z = k 1 − k 2 − i 1 − k 2 . e i θ + 1 = 2 cos · e2.
2
(D) z = − k 1 − k 2 − ia(1 − k 2 ) . 31 (ITA 91) Se z = cos t + isent , em que 0 < t < 2π, , então podemos
(E) z = a + ki . 1+ z
afirmar que w = é dado por:
1− z
21 (ITA 95) Seja z um número complexo satisfazendo Re(z) > 0 e
(z + i)2 + |z + i|2 = 6 . Se n é o menor natural para o qual zn é um número t
(A) i cot .
imaginário puro, então n é igual a: 2
t
(A) 1. (B) itg .
2
(B) 2.
(C) 3. (C) i cot t .
(D) 4. (D) i tg t .
(E) 5. (E) n.d.a.

122 Vol. 2
Números complexos

32 (ITA 82) Considere as famílias de curvas do plano complexo, definida 37 (AFA 03) Analise as alternativas e marque a correta:
 1
por Re   = c, em que z é um complexo não nulo e c é uma constante (A) Dado o complexo z = m + mi , em que m ∈ ℜ * e i é a unidade
 z
real positiva. imaginária, pode-se dizer que o afixo de ( z )2 é, em relação à origem,
simétrico do afixo ( −2 m2 , 0) .
Para cada c, temos uma: (B) No plano de Argand-Gauss dos complexos z, tais que | z − 1|≤ 1 , são
representados pelos pontos do círculo de centro (0, 1) e raio unitário.
(A) circunferência com centro no eixo real e raio igual a c
(C) Se n ∈  e i é a unidade imaginária, então ( i n +1 + i n )8 é um número
(B) circunferência com centro no eixo real e raio igual a 1/c.
real maior do que zero.
(C) circunferência tangente ao eixo real e raio igual a 1/2c.
(D) S e z = a + bi(a ∈ ℜ*,b ∈ ℜ e i é a unidade imaginária) é u m
(D) circunferência tangente ao eixo imaginário e raio igual a 1/2c.
(E) circunferência com centro na origem do plano complexo e raio igual a complexo, então z − z é um imaginário puro.
1/c.
2
38 (AFA 02) Considere no campo complexo uma curva tal que Im   ≥ k ,
33 (ITA 83 - Adaptado) Considerando um número complexo z tal que  z
z2 ≠ em que z é um complexo não nulo. Se k = 2, tem-se sua representação
tem argumento igual a e log2 (z + z + 2) = 3. Nestas condições,
zi 4
gráfica dada pelo:
considerando Im z > 0, podemos afirmar que:

3
(A) círculo de raio 1 / 4 e tangente ao eixo real.
 z−z  1 1 (B) círculo de raio 1 / 2 e tangente ao eixo imaginário.
(A) não existe In   z  = 103 (1 + i) .
 . (D)
(C) conjunto de pontos do plano complexo exterior ao círculo de raio 1 / 2
 i   
e centro (– 1 / 2, 0).
4
 z−z  1
3
1 (D)círculo de raio 1/2 e tangente ao eixo real.
(B) z + In   = −324 . (E)   =− (1 + i ) .
 i  z
  108
39 (ITA 90) Considere as equações z3 = i e z2 + (2 + i) z + 2i = 0, em
(C) z + 2 z é um número real. que z é complexo. Seja S1 o conjunto das raízes da primeira equação e S2
o da segunda. Então:
34 (ITA 90) A igualdade 1+ | z |=|1 + z | , em que z ∈ C , é satisfeita:
(A) S1 ∩ S2 é vazio.
(A) para todo z ∈C tal que Re z = 0 e Im z < 0 .
(B) S1 ∩ S2 ⊂ ℜ .
(B) para todo z ∈C tal que Re z ≥ 0 e Im z = 0 .
(C) S1 possui apenas dois elementos distintos.
(C) para todo z ∈C tal que |z| = 1.
(D) S1 ∩ S2 é unitário
(D) para todo z ∈C tal que Im z = 0 .
(E) S1 ∩ S2 possui 2 elementos
(E) para todo z ∈C tal que | z |< 1.
40 (ITA 92) Considere o número complexo z = a + 2i cujo argumento
35 (ITA 99) O conjunto de todos os números complexos z, z ↑ 0 , que
 π
satisfazem a igualdade | z + 1 + i |=|| z | − |1 + i || é: está no intervalo  0,  . Sendo S o conjunto dos valores reais de a para os
 2
 5π  quais z6 é um número real, podemos afirmar que o produto dos elementos
(A)  z ∈ C : arg z = + 2 k π, k ∈ Z  .
 4  de S vale:
 π 
(B)  z ∈ C : arg z = + 2 k π, k ∈ Z  . (A) 4.
 4  (B) 4 i 3 .
 π  (C) 8.
(C)  z ∈ C :| z |= 1 e arg z = + k π, k ∈ Z  .
 6  (D) 8 i 3 .
π (E) n.d.a..
 
(D)  z ∈ C :| z |= 2 e arg z = + 2 k π, k ∈ Z  .
 4  0 1 2
41 Se ξ é raiz n-ésima da unidade, calcule ξ + ξ + ξ +...+ ξ .
n–1

 π 
(E)  z ∈ C : arg z = + k π, k ∈ Z  .
 4  EXERCÍCIOS NÍVEL 3

36 (ITA 95) Sejam z1 e z2 números complexos com |z1|=|z2|=4. Se 1 1


é uma raiz da equação z1 z6 + z2 z3 – 8 = 0, então a soma das raízes reais 01 Seja z um número complexo. Se z + é um número real, então
podemos afirmar: z
é igual a:

(A) – 1. (A) z ≠ 0 e Re(z) ≥ 0 .


(B) – 1 + 21/2. (B) Im(z) = 0 ou |z| = 1.
(C) 1 – 2 1/3. (C) z é necessariamente um número real.
(D) 1 + 3 1/2. (D) z2 = –1.
(E) – 1+3 1/2. (E) n.d.a..

IME-ITA 123
Matemática II – Assunto 2

02 Determine o valor do produtório: 12 Determine as somas:

2014  1 + i 2k   n  n  n
P= ∏k =0
1 +


2 
 , em que i = −1

a. cos x +   cos 2 x +   cos 3 x + ... +   cos(( n + 1) x );
1
  2
   n
 n  n  n
b. senx +   sen2 x +   sen3 x + ... +   sen(( n + 1) x ).
03 O par (z1, z2) de números complexos é chamado de “parceiro” se existe  1  2  n
 z12 + z22 = az1 z2
um número real a tal que  . Prove que, para todo n natural, 13 Quantas soluções possui a equação z n−1 = i · z (n é natural)?
 a ∈ [ −2, 2]
se (z1, z2) é “parceiro”, então ( z1n , z2n ) também é. 14 Seja ξ uma raiz n-ésima da unidade diferente de 1. Prove que:
 n, se k não é múltiplo de n
04 Considere os conjuntos: 1 + ξ k + ξ2 k + ξ3 k + ... + ξ( n −1)k  .
0, se k não é múltiplo de n
 x   y  15 Considere a equação z5 = 1. Resolva essa equação fatorando (e usando
A = ( x , y ) : x 2 − y 2 = 2 2
, B = ( x , y ) : 2 xy + 2 = 3
 x +y   x +y 2
 1
uma substituição w = z + ). Em seguida, resolva utilizando as fórmulas
{
C = ( x , y ) : x 3 − 3 xy 2 + 3 y = 1} { }
e D = ( x, y ) : 3 x 2 y − 3 x − y 3 = 0 .
z
de De Moivre para, então, determinar o valor de cos 36°.
Prove quue A ∩ B = C ∩ D.
16 Resolva a equação (z + 1)n=(z – 1)n, em que z ∈ C e n ∈ N.
 z1 + z2 + z3 = 0
05 Sejam z1, z2, z3 complexos tais que  . Prove que 17 Sejam A, B e C os afixos dos complexos a, b e c no plano complexo.
z12 + z22 + z32 = 0 . | z1 |=| z2 |=| z3 |= 1 b−a
Mostre que o ângulo BAC é reto se e somente se é imaginário puro.
c−a
z −1 18 Dado um triângulo ABC, constroem os quadrados ABDE e ACFG,
06 Determine os valores máximo e mínimo de quando |z| = 3
z +1 exteriores ao triângulo. Mostre que CE é perpendicular a BG e que CE = BG.
Sugestão: Observando a figura no plano de Argand-Gauss, você precisa
1 1 1 mostrar que C − E = ± i ·( G − B) .
07 Prove que se Re(z) > 1, então − < .
z 2 2
19 Um turista faz um passeio por uma cidade em algumas etapas. Cada
08 Determine o maior e o menor valores possíveis para |z|, dado que
etapa consiste em 3 segmentos de tamanho de 100m separados por giros
1
z + = 1. de 60 graus no sentido horário. Entre o último segmento de uma etapa e o
z primeiro da etapa seguinte, o turista gira de 60 graus no sentido anti-horário.
1 Após 2014 etapas, a que distância do ponto inicial estará o turista?
09 Seja z um complexo não nulo tal que z 3 + 3 ≤ 2. Prove que
z 20 Um antigo mapa dava instruções para localizar um tesouro enterrado
1
z + ≤ 2. em certa ilha...
2
10 Sendo x, y, z ângulos em [0,2π), resolva o sistema “Ande da palmeira até a entrada da caverna . Lá chegando, vire 90º à
 3 direita e caminhe o mesmo número de passos. No fim desse trajeto coloque
cos x + cos y + cos z = 2 uma marca e retorne à palmeira. Agora, caminhe em direção à pedra. Lá
 . chegando, vire 90º à esquerda e caminhe o mesmo número de passos que
senx + seny + senz = 3 3 foram dados da palmeira à pedra. Coloque uma marca no fim desse trajeto.
 2 O tesouro está no ponto médio das duas marcas.”
11 Determine os valores dos somatórios abaixo: Quando chegamos a ilha, a palmeira não existia mais. Como fazer para
achar o tesouro?
n
a. S = ∑ sen( a + kr )
k =0

n
b. C = ∑ cos( a + kr )
k =0

124 Vol. 2
Probabilidade A ssunto
3
Matemática III

1. Introdução Assim, estamos calculando que fração do espaço amostral um


subconjunto representa.
A Teoria das Probabilidades tem por objetivo facilitar a tomada de Uma vez que o espaço amostral é o conjunto de todos os casos
decisões em experimentos não determinísticos, ou seja, em situações nas possíveis, e os eventos são os casos que queremos analisar, tem-se:
quais não se sabe o resultado final. Teve início com os jogos de cartas,
dados e roleta, uma vez que estes jogos, também chamados de jogos de
azar, são exemplos clássicos em que não se pode prever o resultado final. # A casos favoráveis
P( A) = =
# Ω casos possíveis
O presente assunto tem por objetivo apresentar o conceito de
probabilidade e suas propriedades, além de mostrar os principais exemplos
de sua aplicação. – Propriedades:
Além disso, serão apresentados conceitos mais avançados como a I. ∀ A ⊆ Ω; 0 ≤ P (A) ≤ 1
probabilidade condicional e a probabilidade em espaços contínuos. II. P (Ω) = 1
III. P (∅) = 0
IV. P (A ∪ B) = P (A) + P (B) – P (A ∩ B)
2. Probabilidade de Laplace V. P (AC) = 1 – P (A)
O conceito de probabilidade está associado à frequência com que um
dado evento deve ocorrer se realizarmos um mesmo experimento certa Ex.:
quantidade de vezes. Uma vez que estamos tratando de experimentos I. Probabilidade de cair cara exatamente uma vez lançando-se uma
aleatórios, ou seja, que não necessariamente possuem o mesmo resultado, moeda três vezes:
a probabilidade deve determinar apenas o que ocorre na média dos eventos. #A: (K, C, C); (C, K, C); (C, C, K) três possibilidades.
Por exemplo, mesmo que não se saiba nenhum conceito formal sobre #Ω: cada moeda tem duas possibilidades: 23 = 8.
probabilidade, é razoável pensarmos que se jogarmos um dado para o 3
alto “muitas” vezes, em um sexto destas jogadas cada um dos números P( A) =
8
deve aparecer.

2.1 Espaço amostral (W) e evento II. Probabilidade de retirar duas cartas de mesmo símbolo em um baralho
de 52 cartas:
Chamamos de espaço amostral o conjunto de todos os resultados
#A:
possíveis de um dado experimento aleatório.
Escolha do símbolo: 13 possibilidades (A, 2, 3, ..., 10, J, Q, K)
Ex. 1: Espaço amostral associado ao lançamento de um dado. Escolha dos naipes desse símbolo: C4,2 = 6 possibilidades.
Ω = {1, 2, 3, 4, 5, 6} Total: 13 · 6 = 78 casos favoráveis
Ex. 2: Espaço amostral associado ao lançamento de duas moedas. 52! 52 ⋅ 51
C52,2 = = = 26 ⋅ 51
Ω = {(K, K), (K, C), (C, C), (C, K)} 50 ! ⋅ 2! 2
13 ⋅ 6 1
Além disso, chamamos de evento qualquer subconjunto de um espaço P( A) = =
26 ⋅ 51 17
amostral associado a um experimento aleatório.
Por exemplo, no espaço amostral associado ao lançamento de um
III. Probabilidade de cair soma menor ou igual a 15 no lançamento de 3
dado, podemos olhar para os lançamentos em que aparece um número
dados:
par: {2, 4, 6}.
No caso dos subconjuntos unitários de um espaço amostral, dizemos P (A) = 1 – P (AC)
que o evento é um evento elementar.
#AC:
2.2. Probabilidade em espaços finitos e (6, 6, 6): uma possibilidade;
propriedades (5, 6, 6) e suas permutações: três possibilidades;
Se um experimento aleatório obedece as seguintes restrições: (4, 6, 6) e suas permutações: três possibilidades;
(5, 5, 6) e suas permutações: três possibilidades;
I. O espaço amostral Ω é finito, # Ω =n; Total: 10 casos.
II. Os eventos elementares são equiprováveis;
# Ω: cada dado lançado tem 6 possibilidades: 63 = 216.
III. Todo evento A é a união de m eventos elementares com m ≤ n.
10 206 103
#A P( A) = 1 − = =
Definimos a probabilidade de LaPlace por P( A) = . 216 216 108
#Ω

IME-ITA 125
Matemática III – Assunto 3

3. Eventos mutuamente exclusivos EXERCÍCIOS RESOLVIDOS


e eventos independentes 01 (UFSCAR) Dois dados usuais e não viciados são lançados. Sabe-se
Dizemos que dois eventos são mutuamente exclusivos, ou disjuntos, que os números observados são ímpares. Então a probabilidade de
se não existe interseção entre eles, ou seja, se P (A ∪ B) = P (A) + P (B). que a soma deles seja 8 é:
Em geral, se tomarmos qualquer partição do espaço amostral, ou seja,
2
se tomarmos uma união de conjuntos disjuntos (Ai ∩ Aj=∅) que cobre o (A) .
36
 n 
espaço amostral   Ai = Ω  teremos: P (A1) + P (A2) + ... + P (An) = 1. 1
 i =1  (B) .
6
Finalmente, dizemos que dois eventos são independentes quando a
ocorrência de um não influência a ocorrência do outro. Nesse caso, temos: 2
(C) .
P (A ∩ B) = P (A) · P (B). 9
Veja que isso funciona como o Princípio Multiplicativo em Combinatória, 1
(D) .
uma vez que para eventos sucessivos e independentes iremos multiplicar 4
as probabilidades.
(E) 2 .
18
EXERCÍCIOS RESOLVIDOS

01 Uma moeda viciada é cunhada de tal forma que é quatro vezes Solução: Letra C.
mais provável cair cara que coroa. A probabilidade de cair cara ou Como os números são ímpares, o novo espaço amostral é dado por:
coroa nessa moeda é: Ω = {(1,5); (5,1); (1,3); (3,1); (3,5); (5,3); (1,1); (3,3); (5,5);}.
Solução: P (K) = 4 · P (C) e P (K) + P (C) = 1, logo: Dentro deste espaço amostral, temos apenas dois pares ordenados
com soma 8.
4 1
=P( K ) = e P( C) .
5 5
02 (VUNESP) Dois jogadores, A e B, vão lançar um par de dados.
02 Um dado é fabricado de tal modo que a probabilidade de cair um Eles combinam que, se a soma dos números dos dados for 5, A
número é diretamente proporcional a esse número. Nesse caso a ganha, e se essa soma for 8, B é quem ganha. Os dados são lançados.
probabilidade de cair o número 4 no dado é: Sabe-se que A não ganhou. Qual a probabilidade de B ter ganhado?
Solução: Seja x a probabilidade de cair 1 no dado, temos: x + 2x +
1
3x + 4x + 5x + 6x = 1 ∴ x = , donde a probabilidade de cair 4 (A) 10 .
4 21 36
no dado será: . 5
21 (B) .
32
03 Jogando-se uma moeda para o alto 5 vezes, qual a probabilidade
de cair cara exatamente duas vezes? (C) 5 .
36
Solução: Vejamos qual a probabilidade de cair a sequência CCCKK. 5
(D) .
Como cada lançamento de moeda é um evento independente, 35
1 (E) Não se pode calcular sem saber os números sorteados.
podemos multiplicar as probabilidades 5 .
2
5! Solução: Letra B.
Além disso, temos = 10 sequências equiprováveis, logo:
3 !2! Seja A o evento em que a soma dos dados é 5 e B em que a soma
10 5
= . é 8, temos:
25 16 Resultados com soma 5: (4, 1); (3, 2); (2, 3); (1, 4).
4 32
Assim, a probabilidade de A não ganhar é: P(A C ) = 1 − = .
36 36
4. Probabilidade condicional Além disso, temos os seguintes casos com soma 8: (6, 2); (5, 3);
(4, 4); (3, 5); (2, 6).
Chamamos de probabilidade condicional a probabilidade de ocorrer 5
um evento A dado que um evento B já ocorreu. Deste modo, P (B) = donde:
36
Uma vez que o evento B já ocorreu, existe uma restrição no espaço P( B ∩ AC ) P( B) 5
amostral no cálculo da probabilidade, ou seja, devemos considerar como P( B| AC ) = = = .
P( AC ) P( AC ) 32
novo espaço amostral o conjunto B.
Assim,
#( A ∩ B)
#( A ∩ B) # Ω ⇒ P( A| B) = P( A ∩ B)
P( A| B) = =
#B #B P( B)
#Ω

126 Vol. 2
Probabilidade

5. Probabilidade em espaço contínuo


Em alguns problemas de probabilidade pode ser impossível a contagem II. Considere uma circunferência de raio R. Se pegarmos dois pontos da
de elementos, uma vez que o conjunto pode não ser discreto. Quando circunferência, qual a probabilidade de que a distância entre esses dois
temos um espaço amostral não enumerável e contínuo, dizemos que o pontos seja maior que R?
cálculo de probabilidade se dá sobre um espaço contínuo.
Para determinar a probabilidade de um evento em um espaço 60° 60°
contínuo, devemos usar a mesma ideia apresentada em espaços discretos,
R R
comparando casos favoráveis e casos possíveis, porém, dessa vez,
devemos comparar duas coisas de mesma dimensão, por exemplo: áreas,
volumes ou comprimentos.

Ex.:
I. Considere um quadrado Q1 e a circunferência C inscrita nele. Seja Q2
um quadrado inscrito nessa circunferência, determine a probabilidade
de ao escolhermos um ponto de Q1 tomarmos um ponto que também
pertence a Q2.
Solução: Considere o primeiro ponto fixo na circunferência (a probabilidade
Solução: Seja L o lado do quadrado Q 1, o raio da circunferência
independe da escolha desse ponto).
L
inscrita será R = . Nesse caso, para determinar o lado de Q2, temos: Agora como o outro ponto deve distar R do primeiro, teremos uma
2
L 2 corda que será lado de um hexágono regular inscrito, ou seja, teremos um
l 2 = 2R = L ⇒ l = . arco de 60º. Como podemos ter essa corda para ambos os lados, o arco é
2
de 120º. Qualquer ponto fora desse arco tem distância maior que R, assim:
l 2 L2 / 2 1
Assim: P( A=
) = = 240o 2
L2 L2 2 =
P ( A) =
360o 3

EXERCÍCIOS RESOLVIDOS
28 2
01 Três dados honestos de 6 faces são lançados. Qual é a probabilidade Portanto, a probabilidade é igual a = .
de o produto dos valores obtidos ser par? 210 15

Solução: O aluno desatento pode achar que, por haver duas opções 03 Uma caixa marrom tem x bolas brancas e 3 bolas pretas, enquanto
1 1 1 1 uma caixa cinza tem 1 bola branca e x bolas pretas. Determine x sabendo
(par ou ímpar), a probabilidade é igual a .⋅ No
⋅ entanto,
= isso está que, ao escolher uma bola de cada caixa, a probabilidade de se obterem
2 2 2 8
duas bolas pretas é de 40%.
completamente errado. Veja que o produto de três números ser par

depende de pelo menos um deles ser par. Problemas com ‘pelo menos’, Solução: A probabilidade de obter bola preta na caixa marrom é
em algumas ocasiões, têm solução muito mais simples através do seu 3 x
. Já na caixa cinza, essa probabilidade é de . Escrevendo
complementar. Como o produto só é ímpar quando os três resultados são x +3 x +1
ímpares, a probabilidade de isso 2 3 x 2
40%= , temos × = , o que nos leva à equação do 2o
1 1 1 1 5 x +3 x +1 5
acontecer é igual a ⋅ ⋅ = Portanto, a probabilidade de o produto 3
2 2 2 8 grau 2x2 – 7x + 6 = 0, que tem raízes x = 2, x = . Como x é inteiro
2
ser par é igual a 1 − 1 = 7 . (número de bolas), devemos ter x = 2.
8 8
02 Uma turma da NASA tem 10 alunos, dos quais 2 são irmãos. Se 04 Em uma sala com 40 alunos, 25 gostam de alface e 20 gostam de
uma tripulação de 4 pessoas será escolhida na turma, determine a jiló e sabe-se que todos os alunos gostam de alface ou jiló. Um aluno
probabilidade de os 2 irmãos serem escolhidos. é escolhido ao acaso. Sabendo que ele gosta de alface, determine a
probabilidade de ele também gostar de jiló.
Solução: O espaço amostral deste problema é o conjunto de
Solução:
todos os quartetos que podem ser formados na turma, que tem Estamos diante de um problema de probabilidade condicional, pois é dada
 10  10 ! uma informação sobre o aluno escolhido (ele gosta de alface). Sejam A o
 = = 210 elementos. Portanto, o denominador é igual a conjunto dos que gostam de alface e B o conjunto dos que gostam de jiló.
 4  4 ! ⋅ 6!
Pelo princípio da inclusão-exclusão, temos que
210. Para o em umerador, vamos calcular o número de quartetos que
n (A ∪ B) = n(A) + n(B) – n (A ∩ B); logo, 40 = 25 + 20 – n (A ∩ B)
8 8! ⇒ n (A ∩ B) = 5.
contêm os 2 irmãos. Veja que são   = = 28 , pois, supondo
 2  2! ⋅ 6 !
que os 2 irmãos estão no quarteto, das outras 8 pessoas precisamos Queremos a probabilidade P (B | A), que é igual a:
n( A ∩ B) 5 1
escolher 2 para os acompanharem. = = .
n( A) 25 5

IME-ITA 127
Matemática III – Assunto 3

EXERCÍCIOS NÍVEL 1 09 (AFA – 94) Uma urna contém 2 peças boas e 5 defeituosas. Se 3
peças forem retiradas aleatoriamente, sem reposição, qual a probabilidade
01 Dois dados são jogados simultaneamente. Calcule a probabilidade de
de serem 2 (duas) boas e 1 (uma) defeituosa?
que a soma dos números mostrados na face de cima seja 7.
33
02 (AFA – 89) Dois dados são lançados simultaneamente. Qual a (A) 1 . (C) .
probabilidade da soma ser menor do que 4? 12 68
33
(B) 3 . (D) .
1 17 64
(A) 1 . (C) .
6 12 10 (AFA-1999) A probabilidade de observarmos um número na face
1 1 superior de um dado viciado é diretamente proporcional a esse número.
(B) . (D) .
8 16 Ao lançarmos esse dado, a probabilidade de ocorrer um número par é:

03 Dois dados são jogados simultaneamente. Calcule a probabilidade de 1 4


(A) . (C) .
que o máximo seja maior ou igual a 3. 2 7
13
04 Uma caixa contém 20 peças em boas condições. Uma amostra de (B) 11 . (D) .
10 peças é extraída. Calcule a probabilidade de que ao menos uma peça 21 21
na amostra seja defeituosa.
11 (AFA-05) Dentro de uma caixa há nove etiquetas. Cada etiqueta
05 (AFA – 90) Com os dígitos 1, 2, 3, 4 e 5 são formados números de recebe um número de 01 a 09, sem repetir nenhum. Retira-se três delas,
4 algarismos distintos. Um deles é escolhido ao acaso. A probabilidade uma a uma, sem reposição. A probabilidade de que os três números
desse número ser par é: correspondentes às etiquetas retiradas sejam, nesta ordem, ÍMPAR – PAR
– ÍMPAR ou PAR – ÍMPAR – PAR é de:

(A) 1 . 20
3 (A) 1 . (C) .
28 81
(B) 2 . (B)
5 5
. (D) .
5 18 36
3 12 (EN-1991) Lançam-se simultaneamente cinco dados honestos. Qual
(C) .
5 a probabilidade de serem obtidos, nesta jogada, uma trinca e um par (isto
é, um resultado do tipo AAABB com B ≠ A)?
(D) 2 .
3 125
(A) 5 . (D) .
(E) n.r.a. 1296 324
125
06 Uma moeda foi cunhada de tal forma que é quatro vezes mais provável (B) 5 . (E)
3888 648
dar cara do que coroa. Calcule as probabilidades de cara e coroa.
25
(C) .
07 (AFA - 09) No lançamento de um dado viciado, a face 6 ocorre com 648
o dobro da probabilidade da face 1, e as outras faces ocorrem com a 13 Para a Copa do Mundo, 32 países são divididos em oito grupos, com
probabilidade esperada em um dado não viciado de 6 faces e numeradas quatro países cada um. Supondo que a escolha do grupo de cada país é
de 1 a 6. feita ao acaso, calcule a probabilidade de que dois países determinados
Dessa forma, a probabilidade de ocorrer a face 1 nesse dado viciado é: A e B se encontrem no mesmo grupo. (Na realidade a escolha não é feita
de forma completamente aleatória.)
(A) 1 .
9 14 Cinco pessoas são escolhidas aleatoriamente. Qual a probabilidade
de haver alguma coincidência de signos zodiacais?
(B) 2 .
3
15 Os jogadores X e Y lançam cada um deles um dado. Qual a
1 probabilidade de X obter um número de pontos maior ou igual a Y?
(C) .
3
16 (AFA-1995) Uma urna contém bolas enumeradas de 1 a 9. Sorteiam-se,
(D) 2 . com reposição, duas bolas. A probabilidade de o número da segunda bola
9 ser estritamente menor que o da primeira é:
08 Um torneio é disputado por 4 times A, B, C e D. É três vezes mais
provável que A vença do que B, é duas vezes mais provável que B vença 10 5
(A) . (C) .
do que C e é três vezes mais provável que C vença do que D. Quais as 27 9
probabilidades de ganhar para cada um dos times? 4 8.
(B) . (D)
9 9

128 Vol. 2
Probabilidade

17 Um prédio de três andares com dois apartamentos por andar tem EXERCÍCIOS NÍVEL 2
exatamente três apartamentos ocupados. Qual a probabilidade de haver
01 (OBM 01 – 1 F – N3) Uma rifa foi organizada entre os 30 alunos
a
exatamente um apartamento ocupado por andar?
da turma do Pedro. Para tal, 30 bolinhas enumeradas de 1 a 30 foram
18 Sete lâmpadas de neon estão dispostas formando um oito no visor de colocadas em uma urna. Uma delas foi, então, retirada da urna. No entanto,
uma máquina de calcular. Acendem-se ao acaso quatro dessas lâmpadas. a bola caiu no chão e se perdeu e uma segunda bola teve que ser sorteada
Qual a probabilidade de se formar um quatro? entre as 29 restantes. Qual a probabilidade de que o número de Pedro tenha
sido o sorteado desta segunda vez?
19 (OBM 02 – 1aF – N3) Duas pessoas vão disputar uma partida de par
ou ímpar. Elas não gostam do zero e, assim, cada uma coloca 1, 2, 3, 4 (A) 1/29. (D) 1/60.
ou 5 dedos com igual probabilidade. (B) 1/30. (E) 2/31.
A probabilidade de que a pessoa que escolheu par ganhe é: (C) 1/31.

(A) 1/2. (D) 12/25. 02 Escolhem-se ao acaso duas peças de um dominó. Qual a probabilidade
(B) 2/5. (E) 13/25. de elas possuírem um número comum?
(C) 3/5.
03 Uma urna contém 4 bolas brancas, 4 bolas pretas e 4 bolas vermelhas.
20 Uma moeda equilibrada (probabilidade de cara = probabilidade de
Sacam-se 6 bolas dessa urna. Determine a probabilidade de serem sacadas
coroa = 1 / 2) é jogada n vezes. Calcule a probabilidade de se obterem
2 bolas de cada cor:
exatamente k caras, 0 ≤ k ≤ n.

21 Há oito carros estacionados em doze vagas consecutivas. Qual a a. supondo a extração com reposição;
probabilidade de as vagas vazias serem consecutivas? b. supondo a extração sem reposição.

22 Um número entre 1 e 200 é escolhido aleatoriamente. Calcule a 04 Em uma caixa há sete bolas brancas e três bolas pretas. Sacam-se uma
probabilidade de que seja divisível por 5 ou por 7. a uma as bolas dessas caixas até que todas as pretas sejam encontradas.
Qual a probabilidade de o número de extrações ser igual a k (3 ≤ k ≤ 10)?
23 O extraterrestre X possui 6 dedos em cada mão, enquanto o Y possui
4. Na disputa de uma partida de “par ou ímpar”, X escolhe par. Determine 05 Tem-se n urnas. Bolas são colocadas ao acaso nas urnas, uma de
a probabilidade de X vencer, sabendo que X joga cada um dos valores 0, cada vez, até que alguma urna recebe duas bolas. Qual é a probabilidade
1, 2, 3, 4, 5 e 6 com igual probabilidade e Y joga cada um dos valores 0, de colocarmos exatamente p bolas nas urnas?
1, 2, 3 e 4 com igual probabilidade.
06 A probabilidade de uma mulher ter câncer de mama é de 1%. Se uma
24 (OBM04 – 1aF – N3) Dois cubos têm faces pintadas de ocre ou magenta. mulher tem câncer de mama, a probabilidade de apresentar um exame
O primeiro cubo tem cinco faces ocre e uma face magenta. Quando os positivo é de 60%. Entretanto, se uma mulher não tem câncer de mama, a
dois cubos são lançados, a probabilidade de as faces viradas para cima chance de apresentar um exame positivo é de 7%. Qual é a probabilidade
dos dois cubos serem da mesma cor (sim, ocre e magenta são cores!) de uma mulher com exame positivo ter câncer de mama?
é 1/2. Quantas faces ocre tem o segundo cubo?
07 Há apenas dois modos, mutuamente excludentes, de Genésio ir
(A) 1. (D) 4.
para Genebra participar de um congresso: ou de navio ou de avião. A
(B) 2. (E) 5.
probabilidade de Genésio ir de navio é de 40% e de ir de avião é de 60%.
(C) 3.
Se ele for de navio, a probabilidade de chegar ao congresso com dois
dias de atraso é de 8,5%. Se ele for de avião, a probabilidade de chegar
25 Escolhe-se ao acaso um número entre 1 e 50. Se o número é primo,
ao congresso com dois dias de atraso é de 1%. Sabe-se que Genésio
qual a probabilidade de que seja ímpar?
chegou com dois dias de atraso para participar do congresso em Genebra.
Determine a probabilidade de Genésio ter ido de avião ao congresso.
26 Em uma cidade, 10% das pessoas possuem carro importado. Dez
pessoas dessa cidade são selecionadas ao acaso e com reposição.
08 Em uma certa cidade existem 10.000 bicicletas, que irão receber um número
Determine a probabilidade de que exatamente 7 das pessoas selecionadas
de licença de 1 a 10.000 (duas bicicletas não podem receber o mesmo número).
possuam carro importado.
Determine a probabilidade de que a primeira bicicleta vista por uma pessoa
andando pelas ruas não contenha o dígito 8 em seu número de licença.
27 (OBM06 – 1aF – N3) Uma colônia de amebas tem inicialmente uma
ameba amarela e uma ameba vermelha. Todo dia, uma única ameba se
09 Uma caixa branca contém 5 bolas verdes e 3 azuis, e uma caixa preta
divide em duas amebas idênticas. Cada ameba na colônia tem a mesma
contém 3 bolas verdes e 2 azuis. Pretende-se retirar uma bola de uma das
probabilidade de se dividir, não importando sua idade ou cor. Qual é a
caixas. Para tanto, 2 dados são atirados. Se a soma resultante dos dois
probabilidade de que, após 2006 dias, a colônia tenha exatamente uma
dados for menor que 4, retira-se uma bola da caixa branca. Nos demais
ameba amarela?
casos, retira-se uma bola da caixa preta. Qual é a probabilidade de se
retirar uma bola verde?
1 (D) 1
(A)
22006 2006 ⋅ 2007 10 Em um armário há n pares de sapatos. Retiram-se ao acaso m pés
1 2006 de sapato desse armário. Calcule a probabilidade:
(B) (E)
2006 2007
a. que saia pelo menos um par;
(C) 1
b. que saia exatamente um par.
2007

IME-ITA 129
Matemática III – Assunto 3

11 Seja P uma probabilidade sobre os eventos de um espaço amostral W. EXERCÍCIOS NÍVEL 3


Sejam A e B eventos tais que P(A) = 2/3 e P(B) = 4/9. Prove que:
01 Em um programa de auditório, o convidado deve escolher uma dentre
a. P(A ∪ B) ≥ 2/3. três portas. Atrás de uma das portas há um carro e atrás de cada uma
b. 2/9 ≤ P(A ∩ Bc) ≤ 5/9. das outras duas há um bode. O convidado ganhará o que estiver atrás da
c. 1/9 ≤ P(A ∩ B) ≤ 4/9. porta; devemos supor neste problema que o convidado prefere ganhar o
carro. O procedimento para escolha da porta é o seguinte: o convidado
12 Existem 1.001 bolas vermelhas e 1.001 bolas pretas em uma caixa. escolhe inicialmente, em caráter provisório, uma das três portas. O
Seja P1 a probabilidade de que duas bolas retiradas aleatoriamente da apresentador do programa, que sabe o que há atrás de cada porta, abre
caixa sejam da mesma cor e seja P2 a probabilidade de que sejam de neste momento uma das outras duas portas, sempre revelando um dos dois
cores diferentes. O valor de |P1 – P2| é: bodes. O convidado agora tem a opção de ficar com a primeira porta que
ele escolheu ou trocar pela outra porta fechada. Com uma boa estratégia,
(A) 0. (D) 2/2001. que probabilidade tem o convidado de ganhar o carro?
(B) 1/2002. (E) 1/1000.
(C) 1/2001. 02 Resolva uma outra versão do exercício anterior, agora com 4 portas,
sendo 3 bodes e 1 carro.
13 Qual a probabilidade de obtermos soma 12 lançando três dados?
03 Um móvel tem três gavetas iguais. Em uma gaveta há duas bolas
14 Considere um quadrado ABCD de lado L. Escolhendo-se aleatoriamente brancas, em outra há duas bolas pretas, e na terceira há uma bola branca
um ponto P em seu interior, determine a probabilidade de que o ângulo e outra preta. Abrimos uma gaveta ao acaso e tiramos uma bola ao acaso
APB seja menor do que 90º. sem olhar a segunda bola que está na gaveta. A bola que tiramos é branca.
Qual é a probabilidade de que a segunda bola que ficou sozinha na gaveta
15 Um ponto P é selecionado aleatoriamente no interior do pentágono seja também branca?
de vértices A(0, 2), B(4,0), C(2p + 1,0), D(2p + 1,4) e E(0,4). Qual é a
^
probabilidade de que o ângulo A PB seja obtuso? 04 (OBMU – 13) Quatro feijões mexicanos estão nos vértices de um
quadrado, inicialmente um feijão em cada vértice. A cada segundo, cada
1 3 feijão pula aleatoriamente para um vértice vizinho, com probabilidade 1/2
(A) . (D) .
5 8 para cada vértice. Calcule a probabilidade de, após 2013 segundos, haver
exatamente um feijão em cada vértice.
1 1
(B) . (E) .
4 2 05 (Desafio PUC 09) Zé Roberto e Humberto disputam um jogo. Eles
5 jogam um dado comum até sair duas vezes consecutivas o mesmo número.
(C) .
16 Se este número a aparecer repetido for par, ganha Zé Roberto; se for ímpar,
ganha Humberto.
16 Um ponto M é selecionado ao acaso no interior de um círculo de raio 2 Eles começam a partida: o primeiro número sorteado é 1, o segundo é 4,
e centro O. Em seguida, constrói-se um quadrado, também centrado em O, o terceiro é 2. Qual é, neste momento, a probabilidade de que Zé Roberto
que tem M como ponto médio de um de seus lados. Calcule a probabilidade ganhe?
de que o quadrado assim construído esteja inteiramente contido no círculo
C. 06 No programa de auditório Toto Bola, o apresentador Ciço Magallanes
dispõe de duas caixas idênticas. Um voluntário da plateia é chamado a
17 (OBM 13 – 1 F – N3) Uma potência perfeita é um número inteiro da
a
participar da seguinte brincadeira: ele recebe dez bolas verdes e dez
forma ab, a e b inteiros, b > 1. Seja ƒ(n) a maior potência perfeita que não bolas vermelhas e as distribui nas duas caixas, sem que o apresentador
excede n. Por exemplo, ƒ (7) = 4, ƒ (8) = 8 e ƒ (99) = 81. Sorteando veja, de modo que em cada caixa haja pelo menos uma bola. Em seguida,
ao acaso um número inteiro k com 1 ≤ k≤ 100, qual a probabilidade de o apresentador escolhe uma das caixas e retira uma bola. Se a bola for
ƒ (k) ser um quadrado perfeito? VERDE, o voluntário ganha um carro. Se for VERMELHA, ele ganha uma
banana. A máxima probabilidade que o voluntário tem de ganhar um
(A) 64%. (D) 90%. m
carro é igual a , em que m e n são inteiros positivos primos entre si.
(B) 72%. (E) 96%. n
(C) 81%.
Determine o valor de m + n.
18 (OMERJ) Considere uma turma com n alunos (2 deles são Miguel
e Rodrigo). Considere todas as comissões de p alunos que podem ser 07 Um quadrado de lado 3 é dividido em 9 quadrados de lado unitário,
formadas nessa turma. Uma dessas comissões é escolhida ao acaso. formando um quadriculado. Cada quadrado unitário é pintado de azul ou
Sabendo-se que Miguel é um aluno dessa comissão, qual é a probabilidade vermelho. Cada cor tem probabilidade 1/2 de ser escolhida e a cor de cada
de Rodrigo também estar na mesma comissão? quadrado é escolhida independentemente das demais. Qual a probabilidade
de obtermos, após colorirmos todos os quadrados unitários, um quadrado
19 (IIT JEE) Uma pessoa pode ir ao trabalho de carro, moto, ônibus ou de lado 2 pintado inteiramente de uma mesma cor?
1 3 2 1
trem com probabilidades: , , e respectivamente. A probabilidade
7 7 7 7 08 (AIME-01) Os números 1, 2, 3, 4, 5, 6, 7 e 8 são escritos nas faces
de ele chegar atrasado no trabalho, se for de carro, moto, ônibus ou trem de um octaedro. Determine a probabilidade de não existirem dois números
2 1 4 1 consecutivos em faces que possuem uma aresta em comum.
é: , , e respectivamente. Sabendo que ele chegou no escritório Obs: Considere 1 e 8 números consecutivos.
9 9 9 9
a tempo, qual a probabilidade dele ter ido de carro?

130 Vol. 2
Binômio de Newton A ssunto
4
Matemática III

1. Introdução Seja p o número de parênteses em que escolhemos o b (0 ≤ p ≤ n),


 n
Quando estudamos no 8o ano (antiga 7a série), nos é apresentado de nos demais n – p parênteses escolheremos o a. Temos   escolhas
forma mais rigorosa o conceito de expressões algébricas. Neste momento  p
deixamos de olhar apenas para expressões numéricas e tratamos também desse tipo, e como esse é o formato de qualquer termo do binômio,
de expressões com incógnitas. chamaremos este de termo geral, denotado por Tp+1.
Após aprender a solução de alguns tipos de equação, aprendemos
também algumas operações com variáveis, como a distributiva e o  n
agrupamento, e finalmente, os produtos notáveis. Vemos então a expansão Tp +1 =   a n − p b p
das expressões (a+b)2 e (a+b)3 , e uma pergunta que normalmente acaba  p
aparecendo é: o que aconteceria se aumentássemos esses expoentes?
Veremos a resposta para essa pergunta deduzindo uma fórmula geral Tp+1 denota o termo da posição p + 1 no desenvolvimento do binômio
para a expressão (a+b)n, com n natural. Além disso, veremos que a expansão em potências decrescentes de a. Repare que o + 1 aparece uma vez que
de (a+b)n tem relação direta com o desenvolvimento de (a+b)n–1, o que nos devemos começar com p = 0.
traz a construção do triângulo de Pascal. Assim, como o Binômio de Newton é uma multiplicação de vários fatores,
Esse “triângulo” basicamente é formado por combinações, que aqui devemos somar todas as multiplicações presentes na distributiva, logo:
denotaremos de números binomiais, e através dele é possível visualizar
diversas relações entre esses números como o Teorema das Linhas, das n  n  n− p p
( a + b)n = ∑  a b
Colunas e das Diagonais que serão vistos neste capítulo. p = 0 p

2. O binômio de Newton 3. O triângulo de Pascal


(distributiva inteligente) Analisemos novamente a expressão (a+b)n para n pequeno:
Vejamos o desenvolvimento da expressão (a+b) para n = 2 e n = 3.
n
(a+b)0 = 1, a + b ≠ 0;
(a+b)1 = a + b;
(a+b)2 = (a+b)(a+b) = a2 +2ab + b2
(a+b)2 = a2 + 2ab + b2
(a+b)3 = (a+b)(a+b)(a+b) = a3 +3a2b +3ab2 + b3
(a+b)3 = a3 + 3a3b + 3ab2 + b3
(a+b)4 = a4 + 4a3b + 6a2b2 + 4ab3 + b4
Em geral no desenvolvimento de (a+b)3 quando fazemos a distributiva
Repare que as extremidades sempre possuem coeficiente 1, uma vez
pensamos em fazer (a+b)2 (a+b). Não poderíamos fazer a distributiva
direta? Ou seja, com os três parênteses de uma só vez, ao invés de multiplicar  n  n
que   =   = 1.
os dois primeiros e depois multiplicar pelo terceiro? Lembrando que a  n 0
distributiva é a soma de todas as multiplicações possíveis com os elementos
desses parênteses, tentemos desenvolver (a+b)3 de forma inteligente: Além disso, veja que existe uma relação entre os coeficientes de uma
linha e outra, por exemplo, se somarmos dois coeficientes consecutivos
Em cada parêntese devemos escolher a ou b durante a distributiva, como
de (a+b)3 obteremos os coeficientes de (a+b)4: 1 + 3 = 4; 3 + 3 = 6;
temos três termos, podemos escolher o b três, duas, uma ou nenhuma vez.
3 + 1 = 4. A pergunta é se isso realmente vale independente do valor de n.
– Se escolhermos o b três vezes temos uma única opção: bbb=b3;
Uma vez que estamos interessados apenas nas possíveis relações
– Escolhendo o b duas vezes temos três opções: bba = bab = abb = ab2; entre coeficientes, para facilitar a visualização, omitiremos as variáveis a
– Escolhendo o b uma vez, três opções: aab = baa = aba = a2b; e b, montando assim um triângulo com as combinações presentes nos
– Não escolhendo o b em nenhum parêntese: aaa=b3.  n
binômios, chamadas aqui de números binomiais   .
Como devemos somar todos os produtos possíveis:  p
(a+b)3 = a3 + 3a2b + 3ab2 + b3 1
C00
Repare que os coeficientes da expressão acima estão associados 1 1
C10 C11 →
ao número de escolhas possíveis nos parênteses então, por exemplo, o 1 2 1
termo ab2 aparece multiplicado por 3, uma vez que temos três parênteses C02 C12 C22
1 3 3 1
3
e queremos escolher o b em dois deles:   = 3. -------------- ----------------
 2 Cn0 C1n Cn2 ... Cpn ... Cnn
Uma vez entendido isso, podemos deduzir uma expressão geral para
(a+b)n = (a+b)(a+b)... (a+b), podendo na distributiva escolher o b de  n
Repare então que na combinação   o n representa a linha do
zero a n vezes.  p
elemento e o p a coluna, contando a linha e a coluna zero.

IME-ITA 131
Matemática III – Assunto 4

3.1. Relação de Stifel 3.3. Teorema das linhas


Como vimos anteriormente, a primeira motivação para montarmos o Outra propriedade interessante é o que ocorre quando somamos todos
Triângulo de Pascal foi a análise de que, nos primeiros casos, a soma de os termos da uma linha do Triângulo de Pascal:
dois termos consecutivos gera o termo imediatamente abaixo. De fato isso 1
vale independente do expoente n do binômio, ou seja, 1 1
1 2 1 → 1 + 2 + 1 = 22
 n   n   n + 1
 + =  1 3 3 1 → 1 + 3 + 3 + 1 = 26
 p   p + 1  p + 1
Repare que ao somarmos os termos da linha n (contando a linha zero)
sempre obtemos como resultado uma potência de 2, ou seja, 2n.
Essa relação é de fundamental importância uma vez que é através
Assim devemos ter:
dela que se dá toda construção do Triângulo de Pascal, já que ela permite
construir uma linha a partir da anterior.
1  n  n  n  n
  +   +   + ... +   = 2
n

0 1
      2  n
1 1
1 2 + 1
Demonstração (1): algébrica
↓ n
 n
Considere o binômio de Newton: ( a + b)n = ∑   a n − p b p , agora
1 3 3 1
n p=0  p 
tome a = b = 1: ∑   = 2n.
n
1 4 6 4 1
p=0  p 
−−−−−−−−−−−−
Demonstração (1): algébrica
Demonstração (2): argumento combinatório
 n  n  n! n! Lembre que os coeficientes do binômio de Newton servem para
 + = + =
 p   p + 1 p!( n − p ) ! ( p + 1)!( n − p − 1) ! ver quantas vezes cada parcela aparece assim, se somarmos todos os
n !( p + 1 + n − p) ( n + 1)!  n + 1 coeficientes, ou seja, todas as combinações da linha n do Triângulo de
= =  Pascal, devemos ter o total de termos obtidos na distributiva de (a+b)n.
( p + 1)!( n − p ) ! ( p + 1)!( n − p ) !  p + 1
Como na distributiva temos que fazer basicamente a escolha entre dois
elementos possíveis em todos os parênteses, a ou b, devemos ter 2n elementos.
Demonstração (2): argumentos combinatórios 3.4. Teorema das colunas
Considere o conjunto {1, 2, 3, ..., n + 1} dos quais queremos escolher Repare através dos primeiros termos do Triângulo de Pascal que se
p + 1 elementos. De quantos modos isso pode ser feito? somarmos os elementos de uma coluna, começando no primeiro termo
 n + 1 desta coluna, até um ponto qualquer, essa soma será equivalente ao número
Resp:   imediatamente na diagonal inferior a direita da última parcela.
 p + 1 1
Por outro lado, na nossa escolha o elemento n + 1 pode entrar ou não, assim: 1 1
1 2 1
1o Caso: n + 1 entra. Aqui ainda devemos escolher p elementos no
1 3 3 1
 n
conjunto {1, 2, 3, .., n}, donde temos   possibilidades. 1 4 6 4 1
 p 1 5 10 10 5 1
2o Caso: n + 1 não entra. Nesse caso ainda faltam escolher p +1
−−−−−−−−−−−−
De modo geral, têm-se:
 n   n   n + 1
 +
elementos no conjunto {1, 2, 3, ..., n}, logo temos =
escolhas.
 
 p   p + 1  p + 1  p   p + 1  p + 2   p + n   p + n + 1
Como dividimos o problema em casos devemos somar as respostas, e como  + +  + ... +  = 
 p  p   p   p   p+1 
ambas as soluções se referem ao mesmo problema, elas devem ser iguais.

3.2. Combinações complementares Demonstração: por indução em n,


Outra propriedade que á facilmente notada no triângulo é que elementos  p + 1
 p
equidistantes das extremidades são sempre iguais: 1o Passo: n = 0,   = 1 =   , ok!
1
p
   p + 1
1 1 2o Passo: Supondo válido para n,
 p   p + 1  p + 2   p + n   p + n + 1  p + n + 1  p + n + 1  p + n + 2 
1 2 1  + +  + ... +  + =  + = 
 p  p   p   p   p   p+1   p   p+1 
1 3 3 1
 p   p + 1  p + 2   p + n   p + n + 1  p + n + 1  p + n + 1  p + n + 2 
  +n  + n   + − ...
− +−− − −−+ − =  + = 
 p
De fato  p   p , para todo
 p   p+1   p
p n e p naturais.   p+1 
  = 
 p  n − p
De fato, na primeira igualdade usamos a hipótese de indução e na
segunda a Relação de Stifel. Assim, podemos perceber que quando a
relação vale para n também vale para n + 1, fechando a indução.

132 Vol. 2
Binômio de Newton

3.5. Teorema das diagonais EXERCÍCIOS NÍVEL 1


Podemos reparar através dos primeiros termos que se somarmos 6 ⋅ 12 ⋅ 18 ⋅ ... ⋅ 300
01 Determine n tal que = 6 n.
os elementos de uma diagonal do Triângulo de Pascal, começando no 1⋅ 2 ⋅ ... ⋅ 50
primeiro termo, obteremos um valor igual ao termo imediatamente abaixo
02 (EN-1983) O menor valor natural de n para o qual se tem
da última parcela:
n! 1
1 < é:
2 ⋅ 4 ⋅ 6...( 2 n) 402
1 1
1 2 1
(A) 6
1 3 3 1 (B) 1600
1 4 6 4 1 (C) 40
1 5 10 10 5 1 (D) 11
−−−−−−−−−−−− (E) 9
 n   n + 1  n + 2   n + p   n + p + 1 ( n + 1)!− n !
Generalizando:   +  +  + ... +  =  03 (EN-1998) Se an = então a1997 é:
0
   1   2   p   p  n 2 [( n − 1)!+ n !]
 n + p + 1  n + p   n + p  1997
Demonstração
 n + p + 1 =pela Relação
 n + p  +  nde+Stifel:
p  (A)
p = p + p−1 1996
 n + pp + 1  n +p p   np+− 1p 
 n + p   =n +  p − 1 +  n + p − 1 (B) 1
 p p− 1 +  np+− p1 − 1
 n + pp  =  n +
 p − 1  =  p − 1  +  p − 2  1998
 n + p   n + p − 1  n + p 2− 1
 np+− 1p −=1 pn−+1 p −+2 p −n +  2 (C) 1998!
 
+  np+− 1p − 1 = pn−+1 p − 2 +p −n + 2 pp− − 2  (D) 1997
+  p − 2  =  p − 2  +  p − 3  (E) 1
 n + p−2p − 1  n + p
p−2 − 2   n + −
p − 3 2
p
+     =     +     
p−2 p−2 p−3
 n + 2  n + 1  n +1      04 (ITA-1996) Dadas as afirmações:
 n + 2 = n + 1 +  n + 1   
 1  =  1  +  0   n  n  n  n   n
I.   +   +   + ... +   +   = 2 , n∈N
n
 n +1 2  =  n 1+ 1 +  n 0+ 1  0   1  2   n − 1  n 
 1   1   0 
       n  n 
 II.   =   , n ∈ N, k = 1, 2, 3 ..., n
k n− k
 n + p + 1  n + p   n + p − 1  n + 1  n 
 n + p + 1 =  n + p  +  n + p − 1 + +  n + 1 +  n  III. Existem mais possibilidades de escolher 44 números diferentes entre
 n + pp + 1 =  n +p p  +  n +p −p 1− 1 + +  n 1+ 1 +  0n  os números inteiros de 1 a 50 do que escolher 6 números diferentes entre
 p   p   p−1   1  + 0
 = + + +   n + 1  n  os números inteiros de 1 a 50.
 pBasta  ver que p é uma  somap − 1 telescópica
  1e que
  0   =  
 0  0
Conclui-se que:
De fato o que fizemos aqui é equivalente a fazer indução em p, e
também poderia ser feito na dedução do Teorema das Colunas. (A) Todas são verdadeiras.
(B) Apenas as afirmações I e II são verdadeiras.
4. Polinômio de Leibiniz (C) Apenas I é verdadeira.
(D) Apenas II é verdadeira.
Quando aprendemos produtos notáveis, é comum vermos não só as (E) Apenas II e III são verdadeiras.
expressões para (a+b)2 e (a+b)3, mas também para (a+b+c)2. Se foi
possível generalizar a expansão do binômio para qualquer expoente, é
 n  n  n n  n
natural pensarmos que também podemos aumentar o número de termos. 05 Calcule   −   +   −  + ( −1)   .
0 1
      2  n
Chamaremos a expressão (x1+x2+x +...+ xk)n de Polinômio de
06 Se n é um número natural não nulo, então
Leibiniz e deduziremos uma fórmula para ela, utilizando a mesma ideia
 2 n + 1  2 n + 1  2 n + 1  2 n + 1  2 n + 1
que foi usada no Binômio de Newton.  + +  + ... +  +  é igual a:
 0   1   2   n −1   n 
Nesse caso, iremos escolher x1 em a1 parênteses, x2 em a1 parênteses,
..., xk em ak parênteses de modo que a1+a2+...+ ak= n. Assim o termo
(A) 22n
geral do polinômio de Leibiniz será igual a:
(B) 22n+1
(C) 22n – 1
T = Cna1 ⋅ Cna2− a1 ⋅ ... ⋅ Cnak− a1 − ... − ak −1 ⋅ x1a1 x 2a2 ⋅ ... ⋅ x kak e abrindo as combinações:
(D) 2n
n! (E) 2n + 1
T= ⋅ x1a1 x 2a2 ... x kak
a1 ! a2 ! ... ak ! 1
C20 C2 C20
07 Calcule o valor da soma: S = C20
0
− + 202 −  + 20 .
E o polinômio de Leibiniz será um somatório de parcelas com esse 2 2 220
formato obedecendo à restrição a1+a2+...+ ak= n.
 n  n  n
08 Calcule   +   +   + ... (soma sobre os índices pares)
0  2 4

IME-ITA 133
Matemática III – Assunto 4

 n  n  n  n  n  n
09 Calcule   + 2   + 4   + 8   + 16   + ... 21 O símbolo   indica a combinação de n objetos k a k. O valor de
0  1  2 3 4  k  20  20  3 k 20 20
  2 
k
 
x2 – y2 quando x = 4 20 ∑     e y = 520 ∑     é igual a:
 n  n  n  n k =0  k   4  k =0  k   5 
10 Calcule   + 4   + 16   + 64   + ...
0
  2
  4
  6 (A) 0. (D) –25.
11 Na expansão em potências decrescentes de (x+y)n, a diferença entre (B) –1. (E) –125.
o terceiro coeficiente e o segundo coeficiente é 54. Determine o valor de n. (C) –5.

k k −1
12 Prove que kCn = nCn −1 . n
 n n m  m m
∑ ( −1)   7 + ∑   2 = 64 , é válida
k
22 (ITA – 92) A igualdade
para: k =0 k k =0  j 
13 Seja n natural tal que C93 + C94 + C105 + C116 + C127 = C13n . Determine os
possíveis valores de n. (A) quaisquer que sejam n e m naturais positivos.
(B) Qualquer que seja n natural positivo e m = 3
14 (EN-1990) O coeficiente x2 no desenvolvimento de (x3+3x2+3x+1)12 é: (C) n = 13 e m = 6
(A) 1260. (D) 230. (D) n ímpar e m par
(B) 630. (E) 115. (E) n.d.a
(C) 315.
23 (UERJ-Específica) Em uma barraca de frutas, as laranjas são
15 Qual é a soma dos coeficientes do desenvolvimento de (x – 2x ) ?
3 2 15 arrumadas em camadas retangulares, obedecendo à seguinte disposição:
uma camada de duas laranjas encaixa-se sobre uma camada de seis;
16 (ITA-2001) Sabendo que é de 1024 a soma dos coeficientes do essa camada de seis encaixa-se sobre outra de doze; e assim por diante,
polinômio em x e y, obtido pelo desenvolvimento do binômio (x + y)n, conforme a ilustração abaixo.
temos que o número de arranjos sem repetição de m elementos, tomados
2 a 2, é:

(A) 80. (D) 100.


(B) 90. (E) 60.
(C) 70.

17 (EN-1981) A soma dos coeficientes dos termos de ordem ímpar do


n
desenvolvimento de  x 3 − 1  é 216. O coeficiente do termo do 2o
 x
grau deste desenvolvimento é: (Disponível em: <http://chiquinho.org/gabteste012AB20101tri.pdf>.)

(A) –136. (D) 680. Sabe-se que a soma dos elementos de uma coluna do triângulo de Pascal
(B) 136. (E) –2380.
pode ser calculada pela fórmula Cpp + Cpp+1 + Cpp+ 2 + ... + Cnp = Cnp++11, na
(C) –17.
Cpp + naturais,
qual n e p são números Cpp+1 + Cpp+n2 ≥+ ... Cnp++11,
p e+ Cnp =correspondem ao número
18 Determine o termo independente de x no desenvolvimento de de combinações simples de n elementos tomados p a q.
10 Com base nessas instruções, calcule:
 2 1
 x + x3  2 2 2 2
  . a. a soma C2 + C3 + C4 + ... + C18 .
b. o número total de laranjas que compõem quinze camadas.
19 (AFA-2001) O termo independente de x no desenvolvimento de
7 EXERCÍCIOS NÍVEL 2
 4 1  é:
 x + x3  01 Calcule: CR + CRn1 + CRn2 +  + CRnp .
0
  n

(A) 4. (C) 21. 02 Qual é o valor da soma: S = 1·2·3 + 2·3·4 + 3·4·5 + ... + 50·51·52?
(B) 10. (D) 35.
03 Considere o desenvolvimento de (x + a)n ordenado do modo usual, isto
20 Determine a condição que o inteiro m deve satisfazer para que exista é, segundo as potências decrescentes de x. Calcule a soma dos termos
m de ordem par desse desenvolvimento.
termo independente de x no desenvolvimento de  x 4 − 1  .
 x8 
04 Calcule :
(A) m deve ser múltiplo de 5
(B) m deve ser múltiplo de 3 n

(C) m deve ser múltiplo de 7 (A) ∑ kC x


k=
k
n
k

n0
(D) m deve ser múltiplo de 11
(B) ∑ kC x k k

(E) m deve ser múltiplo de 4 k=


n0
n

(C) ∑ kC k
n
k =0

134 Vol. 2
Binômio de Newton

n
Cnk 11 Deter mine o coeficiente de x 6 no desenvolvimento de
05 Calcule o valor de ∑ k +1. 
3
1  2 1 
3

k =0
 2x + x2  ⋅  x + 2x  .
   
06 Qual é o valor da soma: S = 12 + 22 + ... + n2 ?
12 Para quantos valores de n, com n variando de 1 a 1000, a expressão
n
07 (ITA – 90) Sejam os números reais a, x, onde a está no primeiro quadrante  2
1
8  x + x 2  possui termo independente de x?
  
e x é não nulo. Se no desenvolvimento de ( cos α ) x + ( sen α )  , o termo
 x  p
independente de x vale 35 , então o valor de a é: 13 Prove que se 0<k<p é inteiro, onde p é primo, então   é múltiplo
8 de p. k

(A)
6 14 Prove a identidade:
 
(B) ≠ n 2k
 n  x   1  x
3 ∑    tg  1 + 2
k
k 
= sec2 n + sec n x

(C)

k =0  k   2 

1 − tg 2 x
2 ( ( )) 

2

12
≠  n  n  n  n
(D) 15 Determine o valor da soma:   −   +   −   + ...
4
0  2 4 6 
(E) n.d.a
 n  n  n
16 Determine o valor da soma:   +   +   + ...
0  4 8 
( )
m
08 Escreva o desenvolvimento do binômio tg 3 x − csc6 x , onde m é
um número inteiro maior que zero, em termos de potências inteiras de sen x 17 (ITA–95) Para cada n ∈ N, temos que:
e cos x. Para determinados valores do expoente, este desenvolvimento  4n   4n   4n 
1 −   +   − ... −   + 1 é igual a:
possuirá uma parcela P, que não conterá a função sen x. Seja m o menor  2 4   4n − 2
valor para o qual isto ocorre. Então P = –64/9 quando x for igual a:
(A) (–1)n . 22n
π (B) 22n
(A) x = + 2 k π, k ∈ Z .
3 (C) (–1)n . 2n
π (D) (–1)n+1 . 22n
(B) x = ± + k π, k ∈ Z . (E) (–1)n+1 . 2n
3
π
(C) x = + k π, k ∈ Z . 18 Calcule:
4 2 2
 n  n  n  n   n  n  n  n 
(D) x = ± π + 2 k π, k ∈ Z .    −   +   −   + ...  +    −   +   −   + ... 
3 0 2 4
        6 1 3
         5 7  .
(E) não existe x satisfazendo a igualdade desejada. 65
19 Determine o termo máximo do desenvolvimento de  1 + 1 
 3
09 Sejam m, n naturais primos entre si. Sabendo que no desenvolvimento  
de (mx+n)2011 os coeficientes de x2 e x3 são iguais, determine os valores 20 Qual é o valor da soma: S=2 · 12+5 · 22 + 8 · 32 +...+ (3n – 1) · n2.
de m e n.
21 Calcule o valor da soma: S = Cn0 − 2 Cn1 + 3 Cn2 −  + ( −1)n ( n + 1) Cnn .
10
 3 a2 2 m 
10 (ITA-94) No desenvolvimento de A =  +  , a razão entre 22 Determine o coeficiente de x17 no desenvolvimento de (1 + x5 + x7)20.
 2 3 
a parcela contendo o fator a14m3 e a parcela contendo o fator a16m2 é igual 23 Determine o coeficiente de x4 no desenvolvimento de (x2 – x + 2)6.
9
a . Se a e m são números reais positivos tais que A=(m2+4)5, então:
n!
16
24 Calcule ∑
a+ b+ c= n a! b! c!
0 ≤ a, b , c ≤ n
2
(A) a ⋅ m = EXERCÍCIOS NÍVEL 3
3
01 Eduardo e Mônica estão disputando uma série de partidas de peteca.
(B) a ⋅ m = 1 Em cada partida, a probabilidade de Eduardo vencer é 0,6 e a de Mônica
3 vencer é 0,4. Seja P a probabilidade de Eduardo vencer uma quantidade
5 par de partidas nas 10 primeiras partidas. Determine se P < 0,5 ou P =
(C) a + m = 0,5 ou P > 0,5
2
(D) a+m=5
02 Calcule o valor de: S = Cn0 − Cn1 + Cn2 −  + ( −1) p Cnp .
5
(E) a − m =
2

IME-ITA 135
Matemática III – Assunto 4

03 05 Mostre que não há 4 termos consecutivos (numa mesma linha) do


a. Demonstre a fórmula de Euler: triângulo de Pascal em progressão aritmética.
Cm0 Chp + Cm1 Chp −1 + Cm2 Chp − 2 +  + CmpCh0 = Cmp + h.
06 Para cada inteiro positivo n considere an, bn e cn os coeficientes
b. Demonstre a fórmula de Lagrange:
(1 + )
n
inteiros da expansão de 3
2+34 , ou seja, considere:
( Cn0 )2 + ( Cn1 )2 + ( Cn2 )2 +  + ( Cnn )2 = C2nn .
( 4)
n
an + bn 3 2 + 3 4 = 1 + 3 2 + 3
n
c. Calcule: ∑ C C , ( n ≤ m) .
k
n
k
m

n n
 n
 an , se n é múltiplo de 3.
 3

k =0
Mostre que: 2 3
  ak =  bn 2, se n deixa resto 2 na diviisão por 3.
3 n −1
 6n  k k
k =0    3
∑ ( −1)
k
04 Calcule a soma  3 . cn 4 , se n deixa resto 1na divisão por 3.
k =0  2 k + 1
RASCUNHO

136 Vol. 2
Geometria analítica: ponto e reta A ssunto
2
Matemática IV

Introdução 1.2 Operações básicas entre pontos


A geometria analítica surgiu com o objetivo de estabelecer uma As operações de adição, subtração e multiplicação dos números reais
relação entre a geometria sintética (que estudamos em Matemática V) e são estendidas para pares ordenados (isto é, pontos) da seguinte forma:
a álgebra (que estudamos, principalmente, em Matemática II). Esta =
Adição de pontos: Dados A (=
a1, a2 ), B ( b1, b2 ) , define-se:
representação algébrica é a forma como computadores conseguem
A + B = ( a1 + b1, a2 + b2 )
modelar problemas geométricos e efetuar simulações de problemas
estruturais (por exemplo, medir a resistência de uma ponte quando Multiplicação por um escalar: Dados A = ( a1, a2 ), t ∈ ¡ , define-se:
sujeita ao peso de múltiplos carros). Em provas, além de a geometria
t ⋅ A = ( ta1, ta2 )
analítica ser impor tante para a solução de problemas deste assunto,
ela pode fornecer uma alternativa para a abordagem de questões de =
Subtração de pontos: Dados A (=
a1, a2 ), B ( b1, b2 ) , define-se:
geometria plana.
B − A = ( b1 − a1, b2 − a2 )
Nesta seção, estudaremos as retas (equações de grau 1 em
duas variáveis). Nas seções subsequentes, estudaremos o círculo e
as cônicas (equações de grau 2 em duas variáveis) e, por último, os Exs.: (1, 2) + ( 2, 3) = (3, 5) , k ⋅ (1, 2) = ( k , 2 k ) , ( 2, 3) − (1, 2) = (11
,)
elementos básicos da geometria espacial, incluindo o plano (equação
de grau 1 em três variáveis). Nota:
 A diferença B – A entre os pontos A e B representa um vetor
Os seus objetivos nesta seção incluem entender a representação AB , orientado de A para B.
algébrica do ponto, saber encontrar o ponto que divide um segmento em
uma razão dada, calcular área de polígonos a par tir de seus vér tices, Obs.: Essas operações de adição e subtração claramente obedecem às
achar a equação de uma reta rapidamente (este é o seu principal propriedades comutativas e associativas e, junto com a multiplicação por
objetivo!), identificar condições de paralelismo e perpendicularismo escalar, obedecem à propriedade distributiva.
entre retas, memorizar a fórmula de distância ponto-reta e, finalmente,
modelar problemas de geometria algebricamente.
1.3 Distância entre pontos
=
Dados os pontos A (= a1, a2 ) e B ( b1, b2 ) , podemos calcular a
distância entre eles pela fórmula:
1. Pontos no plano
d = ( a1 − b1)2 + ( a2 − b2 )2
1.1 Representação algébrica de um ponto
Demonstração: Basta aplicar o teorema de Pitágoras na figura abaixo.
no plano
Para representar uma situação geométrica plana algebricamente,
inicialmente traçamos um par de retas orientadas perpendiculares,
denominadas eixos x e y. Chamamos as quatro regiões definidas por esse
desenho de 1o, 2o, 3o e 4º quadrantes, conforme figura abaixo.

Obs.: Trocar a ordem dos pontos A e B na fórmula acima não modifica o


resultado, pois x 2 = ( − x )2 .

1.4 Divisão de um segmento


em uma razão dada
Um ponto no 1o quadrante que esteja situado a uma distância x p do
eixo y e y p do eixo x é representado algebricamente pelo par ordenado Um problema comum da geometria é encontrar um ponto P, dentro de
AP
( x p , y p ) . Nos outros quadrantes, a representação também é feita pela um segmento AB, de tal forma que a razão seja igual a uma constante
distância do ponto aos eixos, mas utilizamos um número negativo na AB
pré-definida k. Esse ponto tem coordenadas dadas por:
primeira coordenada se o ponto estiver à esquerda do eixo y e um número
negativo na segunda coordenada se o ponto estiver abaixo do eixo x.
AP
P = (1 − k ) ⋅ A + k ⋅ B , em que k =
AB

IME-ITA 137
Matemática IV – Assunto 2

Demonstração: A partir das semelhanças na figura, temos: EXERCÍCIOS RESOLVIDOS


y 01 Considere o triângulo formado pelos vér tices A(1,1),
B” B(b1, b2) B(2, 3) e C(3, 2). Sendo X o ponto que divide AB em segmentos
proporcionais a 2 e 3, AX < XB, determine o comprimento da ceviana
P” x CX.

Solução:
B
B’(b1, a2)
x 3k
AP′ AP p −a
∆APP′ ~ ∆ABB′ ⇒ = ⇒ 1 1 = k ⇒ p1 = (1 − k ) a1 + kb1 x
AB′ AB b1 − a1 C
AP " AP p −a 2k
∆APP " ~ ∆ABB " ⇒ = ⇒ 2 2 = k ⇒ p2 = (1 − k ) a2 + kb2
AB " AB b2 − a2
A
Obs. 1: Se k estiver entre 0 e 1, P pertence ao segmento AB; se k > 1, então Pelo resultado 1.4. (divisão em uma razão dada), temos:
P está no prolongamento de AB a partir de B; se k < 0, então P pertence 3 A + 2B  3 2 3 2  7 9
ao prolongamento de AB a partir de A. X= =  ⋅ 1 + ⋅ 2, ⋅ 1 + ⋅ 3  =  ,  .
5  5 5 5 5  5 5
Obs. 2: Também vale a recíproca desse resultado, isto é, se Pelo resultado 1.3. (distância entre pontos), temos:
P ' = (1 − k ) ⋅ A + k ⋅ B para k entre 0 e 1, então P ' pertence ao segmento 7  9
2
 65
2
65
AB (pois P ' = P ). CX 2 =  − 3  +  − 2  = , log o CX = .
5  5  25 5
AP m
Corolário: Se P divide AB na razão m:n (isto é, = ), então
n ⋅ A PB
+ m ⋅ Bn 02 Dados dois pontos A(1,2) e B(2, 3), determine as coordenadas
AP m n
k= = , 1− k = e, portanto: P = . do ponto X que é simétrico de A em relação a B.
AB m + n m+n m+ n

Solução:
1.5 Ponto médio, baricentro e X
caracterização de paralelogramos
Ponto médio de AB B
A+ B
M=
2
1 A
Demonstração: Basta tomar k = em 1.4.
2
X é simétrico de A em relação a B se, e somente se, B é ponto médio
Baricentro G de um triângulo ABC de AX:
A+ B+C A+ X
G= B= ⇒ X = 2B − A
3 2
Demonstração: Basta provar que o ponto G definido acima está nas Substituindo, temos: X = ( 2 ⋅ 2 − 1, 2 ⋅ 3 − 2) = (3, 4) .
três medianas.
1 2 B+C
Como G = ⋅ A+ ⋅ , temos que G está na mediana por A
3 3 2 2. Área de polígonos
1
(pela obs. 2 de 1.4, com k =
3
). 2.1 Área de um triângulo
Sendo A( a1, a2 ), B( b1, b2 ), C( c1, c2 ) os vértices de um triângulo, sua
Analogamente, G está nas medianas por B e por C, pois:
área é dada por:
1 2 A+C 1 2 A+ B
G= ⋅B+ ⋅ = ⋅C + ⋅ . a1 a2 1
3 3 2 3 3 2 1
S= ⋅ ∆ , em que ∆ = b1 b2 1
2
c1 c2 1
Condição para que A, B, C, D formem, nesta
ordem, um paralelogramo Obs.: ∆ é positivo se, e somente se, os vértices A, B, C estiverem no
sentido anti-horário.
A+C = B+ D
Demonstração: Esta condição é equivalente a dizer que as diagonais
de ABCD se cortam ao meio.

138 Vol. 2
Geometria analítica: ponto e reta

Demonstração: Com base na figura abaixo para o caso em que A, B, 3. Retas no plano
C estão no sentido anti-horário:
3.1 Coeficiente angular
=
Dados dois pontos A (=x a , y a ), B ( x b , y b ) , com x a ↑ x b , define-se
o coeficiente angular m como a tangente do ângulo que a reta AB forma
com o eixo x. Algebricamente, tem-se:

y b − y a ∆y
mAB = tanθ = =
x b − x a ∆x

Demonstração: Para mostrar que as definições algébricas e geométricas


coincidem, basta usar a definição de tangente na figura a seguir.

S = Sretângulo − S1 − S2 − S3

2S = 2 ( c1 − b1 ) ⋅ ( a2 − b2 ) − ( a1 − b1 ) ⋅ ( a2 − b2 ) − ( a2 − c2 ) ⋅ ( c1 − a1 ) −
( c1 − b1 ) ⋅ ( c2 − b2 )
Os termos da forma x1 x2 se cancelam, restando:
2S = a2 c1 − a2 b1 − c1b2 + a1b2 − a1c2 + b1c2 = ∆

2.2 Área de um polígono convexo


Sendo A( a1, a2 ), B( b1, b2 ), C( c1, c2 ), ..., P( p1, p2 ) os vértices de um
polígono, tomados no sentido anti-horário, sua área é dada por: 3.2 Equação rápida da reta
a1 a2 Dado um ponto A ( x a , y a ) , a reta que passa por esse ponto e faz
b1 b2 ângulo θ ≠ 90 com o eixo x é dada por:
1 c c2 1
S= ⋅ 1 = ⋅ (( a1b2 − a2 b1) + ( b1c2 − b2 c1) + ... + ( p1a2 − p2 a1))
2 ... ... 2 y = m ⋅ ( x − x a ) + y a , em que m = tanθ.
p1 p2
a1 a2 Essa equação pode também ser escrita como y = m ⋅ x + q
(denominada equação reduzida da reta), em que m e q são constantes reais.
Ideia da demonstração: Ligando cada vértice do polígono até a origem,
1 Demonstração: Um ponto P = ( x , y ) está na reta se, e somente se,
nota-se que cada termo da forma ⋅ ( a1b2 − a2 b1) representa a área de um o ângulo entre AP e o eixo x é igual a θ. Essa condição nada mais é que
2
triângulo formado pela origem e dois vértices do polígono.
y − ya
mAP = m, i.e., = m.
x − xa
EXERCÍCIOS RESOLVIDOS Obs.: Se θ = 90 (reta vertical), a equação é dada por x = x a . Em ambos
03 Considere um triângulo de vértices A = (1,2), B = (5,5) e os casos, podemos escrever y = m ⋅ x + q .
C = (– 3,1). Determine o comprimento da altura relativa ao vértice A. A equação rápida é muito útil quando precisamos achar uma
reta por dois pontos dados ou quando temos o coeficiente angular e
Solução: um ponto da reta. O segundo caso é a própria fórmula. No primeiro,
1 y − ya
A ideia é calcular a área. Lembre que a área do triângulo é igual a ∅ , calculamos primeiramente m = b e, em seguida, escrevemos a
2 xb − xa
em que: reta y = m ⋅ ( x − x a ) + y a .
1 2
5 5 =
Ex.: Encontrar a equação da reta que passa pelos pontos A (11=
, ), B ( 2, 4) :
∆= = 5 + 5 − 6 − 10 + 15 − 1 = 8 .
−3 1 ∆y 4 − 1
Temos m = = = 3 , logo y = 3 ⋅ ( x − 1) + 1 ou y = 3 x − 2 .
1 2 ∆x 2 − 1

Por tanto, a área é igual a 4. Por outro lado, a distância entre Se usássemos o ponto B na etapa final, o resultado seria o mesmo:
y = 3 ⋅ ( x − 2) + 4 ⇒ y = 3 x − 2
B e C é igual a ( 5 − ( −3 ) ) + ( 5 − 1) = 4 5 . Segue que
2 2

BC ⋅ hA 2 5
= 4, log o hA = .
2 5

IME-ITA 139
Matemática IV – Assunto 2

3.3 Condição de paralelismo, Equação paramétrica da reta


perpendicularismo e ângulo entre retas Em problemas de lugar geométrico, é muito útil escrever os pontos
da reta em termos de uma única variável t ∈ ¡. Neste caso, podemos
Paralelismo escrever a equação da reta como:
Duas retas r e s são paralelas se, e somente se, formam ângulos x = x0 + a1t
iguais com o eixo x, isto é:
y = y 0 + a2 t
r / / s ⇔ mr = ms
Em que t percorre o conjunto dos números reais, ( x0 , y 0 ) é um ponto
Perpendicularismo da reta e a1, a2 são constantes.
Duas retas r e s são perpendiculares se, e somente se: Demonstração: Eliminando t, temos:
r ⊥ s ⇔ mr ⋅ ms = −1 x − x0 y − y0
t= = ⇔ k2 x − k1 y − ( k2 x0 − k1 y 0 ) = 0
k1 k2
Demonstração: Como indicado na figura,
r ⊥ s ⇔ β = α + 90 ⇔ cot(β − α) = 0 Que tem a forma da equação geral
ax + by + c = 0 para: a = k2 , b = − k1, c = −( k2 x0 − k1 y0 ) .

Equação segmentar da reta


Sendo (p, 0) e (0, q), p · q ≠ 0, as coordenadas dos pontos de
interseção da reta com os eixos x e y, respectivamente, a equação da reta
pode ser escrita como:
x y
+ =1
p q
Demonstração: Partindo da equação rápida da reta:
q−0 x y
y= ⋅ x + q ⇔ qx + py = pq ⇔ + = 1
0− p p q
Pela fórmula da soma da tangente, a última condição é:
Equação normal da reta
1 + tan α ⋅ tan β
= 0 ⇔ mr ⋅ ms = −1. Escrevendo a equação segmentar em função dos parâmetros r e θ
tan β − tan α r r
da figura abaixo, temos p = e q= , logo a equação da reta
cosθ senθ
Ângulo entre retas pode ser escrita como:
No caso geral, o ângulo θ entre duas retas r e s, medido no sentido x cos θ + y sen θ = r
anti-horário de r para s, é tal que:
ms − mr
tanθ =
1 + ms ⋅ mr
Demonstração: Similar à anterior, partindo de tan θ = tan(β − α) .
Obs.: Se a orientação entre as retas s e r não for conhecida, deve-se
colocar um módulo no lado direito da fórmula de ângulo entre retas.

3.4 Equação geral da reta


Multiplicando-se a equação reduzida por uma constante qualquer,
obtemos a forma mais geral possível para a equação de uma reta:
Ax + By + C = 0
Conforme estudaremos mais adiante, os coeficientes A e B são as 3.6 Distância entre ponto e reta
coordenadas de um vetor n = ( A, B) perpendicular (normal) à reta. Dada
a equação geral de uma reta, para obter o seu coeficiente angular basta Sendo P = ( x p , y p ) um ponto e r : Ax + By + C = 0 uma reta, a
isolar o y e olhar para o coeficiente de x. distância de P até r é dada por:
ax p + by p + c
d=
3.5 Outras equações de reta a2 + b 2
Na maioria dos problemas de reta em analítica, você deve pensar nas Demonstração: O caso em que r é paralela a um dos eixos é simples.
equações 3.4. e 3.3. Entretanto, em alguns casos pode ser mais eficiente Nos demais casos, considere, como na figura, o triângulo XPY tal que
pensar em outras formas de se encontrar uma reta:  C   C
X =  − , 0  , Y =  0, −  são as interseções de r com os eixos:
 A   B

140 Vol. 2
Geometria analítica: ponto e reta

Nota: Se ax p + by p + c e c têm o mesmo sinal, então P e a origem O


P(xp, yp) estão do mesmo lado em relação à reta r; se ax p + by p + c e c têm sinais
d contrários, então P e O estão em lados opostos da reta r.
 C Ex.: Determine a distância entre o ponto P0(2,– 4) e a reta r: 3x – y
y  0, − 
 B r + 2 = 0.
| 3 ⋅ 2 − ( −4) + 2| 12 6 10
d(P0, r) = = = u.c.
 C  32 + ( −1)2 10 5
x  − ,0 
 A  3.7 Equação da bissetriz
Por um lado, tem-se
Bissetriz dadas duas retas
0 −C / B
As bissetrizes das retas r : ax + by + c = 0, s : a ' x + b ' y + c ' = 0 :
1 −C / A 0 1 Cy Cx C2
SXPY = ⋅ = ⋅− P− P− ax + by + c a' x + b' y + c'
2 xP yP 2 A B AB =±
2 2
0 −C / B a +b ( a ')2 + ( b ')2
Demonstração: Basta usar 3.6. e lembrar que as bissetrizes são o lugar
1 C geométrico dos pontos que equidistam das duas retas dadas.
S= ⋅ ⋅ Ax P + By P + C
2 AB Nota: Na prática, a melhor forma de diferenciar a bissetriz do ângulo
1 agudo e a do ângulo obtuso é fazer um bom desenho. Uma alternativa mais
Por outro lado, tem-se SXPY = ⋅ XY ⋅ d ( P, r ) . formal seria calcular o ângulo θ entre a bissetriz e uma das retas. Temos
2
C2 C2 C tanθ < 1 ⇔ 2θ < 90 ⇔ a bissetriz é do ângulo agudo.
Como XY = + = ⋅ A2 + B2 , obtém-se:
B2 A2 AB Bissetriz de um triângulo dados os três vértices
Neste caso, em vez de encontrar a equação dos lados do triângulo,
Ax P + By P + C é mais simples usar o teorema da bissetriz interna (ou externa).
d ( P, r ) =
A2 + B2 Primeiramente, descobrimos o ponto em que a bissetriz divide o lado
oposto e depois utilizamos o resultado 1.4. para encontrar o pé da bissetriz.

EXERCÍCIOS RESOLVIDOS

04 Sejam O = (0,0) , P = (1,0) e um ponto Q variável sobre a reta r Resolvendo o sistema resultante da interseção das duas retas,
de equação y = x + 3. Determine o lugar geométrico do baricentro do 57 7
encontramos x = − e y = − .
triângulo OPQ quando Q varia sobre r. 25 25

Solução: Se Q está na reta de equação y = x + 3, podemos escolher um 06 Qual o ângulo agudo entre as retas 2 x − y + 1 = 0 e x − 3 y + 3 = 0 ?
parâmetro t real variável e escrever Q = (t,t + 3). Sendo G o baricentro
x
O+P+Q  t +1 t +3 Solução: Isolando y em cada equação, temos y = 2 x + 1, y = + 1 . Logo,
do triângulo OPQ, temos G = , logo G =  , . 3
3  3 3  1
os coeficientes angulares das retas são m1 = 2 e m2 = .
Isso já nos dá G em seu formato paramétrico. Para finalizar o problema, 3
 t +1 Pela fórmula do ângulo entre retas (3.3.), o ângulo agudo procurado satisfaz
 x G = 3
precisamos ‘desparametrizar’ G. Fazendo  e eliminando t 1
2−
y = t + 3 3
 G tanθ = = 1, logo, θ = 45 .
3 1
do sistema, temos que 3 y G − 3 = 3 x G − 1 , ou seja, G se move sobre a 1+ 2 ⋅
3
2
reta de equação y = x + . 07 Determine as equações das bissetrizes dos ângulos formados pelas
3
retas de equações 3 x − 4 y + 1 = 0 e 5 x + 12 y + 3 = 0 .
05 Considere os pontos A = (1,2) , B = (5,5) e C = (– 3,1). Determine
o pé da altura traçada de C no triângulo ABC. Solução: Pelo resultado 3.7., temos que (x,y) está na bissetriz se, e
somente se,
5−2 3 3 x − 4 y + 1 5 x + 12 y + 3 3 x − 4 y + 1 5 x + 12 y + 3
Solução: SejararetaperpendicularaABquepassaporC.Como mAB = = , = , ou seja, = .
5 −1 4 5 13
3 + ( −4 )
2
4 2 2
5 + 12 2

temos (pela condição de perpendicularidade em 3.3.) que mr = − .


3
Tendo o coeficiente angular e um ponto de r, podemos usar a equação 3x − 4y + 1  5 x + 12 y + 3 
Com isso, temos duas possibilidades: = ± .
rápida da reta (3.2.) : 5  13 
4
y = mr ( x − x C ) + y C , que nos dá r : y = − x − 3 . Multiplicando cruzado, temos que as bissetrizes são 7 x − 56 y − 1 = 0
3 e 16 x + 2 y + 7 = 0 .
Da mesma forma, a equação de AB é y − y A = mAB ( x − x A ) , que nos
3 5
dá AB : y = x + .
4 4

IME-ITA 141
Matemática IV – Assunto 2

(A) A(0,0) e B(4,2). (C) A(–2,4) e B(2,0).


08 Mostre que, em um paralelogramo, a soma dos quadrados dos (B) A(0,0) e B(– 2,– 4). (D) A(–1,2) e B(4,2).
lados é igual à soma dos quadrados das diagonais.
11 (EFOMM) A área do quadrilátero de vértices A(0,1), B(1,0), C(3,2)
Solução: e D(2,4) é:
D=(a+b,c)
y
(A) 11/2. (D) 17/4.
(B) 13/2. (E) 19/4.
C=(b,c) (C) 15/4.
x
12 (EFOMM) A área da figura abaixo vale:
B=(a,O)

A=O(0,0)
D
2
Dado o paralelogramo ABCD, sempre podemos escolher um par de
eixos ortogonais de forma que a origem esteja no ponto A e o eixo
1 A
Ox sobre o lado AB.
Como ABCD é paralelogramo, temos A+D = B+C, logo podemos 3 4
escrever as coordenadas de A, B, C e D como indicado na figura. 1
Segue que: 2
AD2 + BC2 = ( a + b)2 + c2 + ( b − a)2 + c2 = 2( a2 + b2 + c2 ) –1
B
AC2 + CD2 + BD2 + AB2 = ( b2 + c2 ) + a2 + ( b2 + c2 ) + a2 –2
2 2 2 2 2 2 C
Logo, AC + CD + BD + AB = AD + BC .
(A) 3/2. (D) 6.
EXERCÍCIOS NÍVEL 1 (B) 5. (E) 9.
(C) 7/2.

01 Ache no eixo das ordenadas um ponto M, de tal forma que sua distância
13 (AFA) Dadas as retas de equações r : y = ax + b e r1 : y = a1 x + b1 ,
ao ponto N(– 8,13) seja igual a 17.
pode-se afirmar que:
02 Calcule a área de um triângulo equilátero cujos dois vértices são os (A) se a = a1 e b ↑ b1 , tem-se r paralela a r1.
pontos A(– 3,2) e B(1,6).
(B) se a = a1 e b = b1 , tem-se r ↑ r1 .
(C) se a ↑ a1 , tem-se r = r1 .
03 A área de um triângulo é S = 3 u.a. e dois de seus vértices são os
pontos A(3,1) e B(1,– 3), achando-se o centro de gravidade desse triângulo (D) se a ↑ a1 e b ↑ b1 , tem-se r paralela a r1.
sobre o eixo Ox. Determine as coordenadas do vértice C.
14 Seja PS a mediana do triângulo de vértices P(2, 2), Q(6, –1) e R(7,
04 Dados os vértices A(1,4); B(3,– 9) e C(– 5,2) de um triângulo, calcule 3). A equação de reta que passa por (1, –1) e é paralela a OS é dada por:
o comprimento da mediana traçada do vértice B.
(A) 2 x − 9 y − 7 = 0 .
(B) 2 x − 9 y − 11 = 0 .
05 Dados dois pontos A(3,– 1) e B(2,1); ache:
(C) 2 x + 9 y − 11 = 0 .
a. as coordenadas do ponto M simétrico do ponto A, em relação ao ponto B; (D) 2 x + 9 y + 7 = 0 .
b. as coordenadas do ponto N simétrico do ponto B, em relação ao ponto A.
15 Dados dois pontos M(2,2) e N(5,–2), ache sobre o eixo das abscissas
^ seja reto.
um ponto tal que o ângulo MPN
06 Dados três vértices A(3,– 5); B(5,– 3) e C(– 1,3) de um paralelogramo,
ache o quarto vértice D oposto ao vértice B.
16 O or tocentro do triângulo formado pelas retas xy = 0 e pela reta
07 Dados três vértices A(3,–7), B(5,–7), C(–2,5) de um paralelogramo x + y = 1 é:
ABCD cujo quarto vértice D é oposto a B, calcule o comprimento das
diagonais do paralelogramo.  1 1
(A)  ,  . (C) (0,0).
2 2
08 Dado um quadrilátero de vértices A(–2,14), B(4,–2), C(6,–2) e D(6,10),
ache o ponto de interseção de suas diagonais AC e BD.  1 1  1 1 .
(B)  ,  . (D)  4,4 
 3 3   
09 Determine a área do quadrilátero ABCD tal que A = (–2, 6); B = (–1, 8);
C = (0, 9); D = (–3, 7). 17 (AFA) O eixo das ordenadas, a reta y = 2 x − 1 e a reta s que é
perpendicular a r e passa pela origem determinam um polígono cujo valor
10 (AFA) Determine os pontos A na reta (r) 2x + y = 0 e B na reta (s) da área é:
x – y – 2 = 0 tal que P(2,1) seja ponto médio de AB.

142 Vol. 2
Geometria analítica: ponto e reta

1 5. 26 (AFA) Há dois pontos sobre a reta y = 2 que distam 4 unidades da


(A) . (C) reta 12y = 5x + 2. A soma das abcissas desses pontos é:
5 5
2 2 5 . (A) – 2.
(B) . (D)
5 5 (B) 6.
(C) 42/5.
18 Dados dois pontos P(2,3) e Q(–1,0), ache a equação da reta que passa (D) 44/5.
pelo ponto Q e é perpendicular ao segmento PQ.
27 (AFA) A distância entre o ponto de interseção das retas
19 Se o ponto P(x, y) é equidistante dos pontos A(a + b, a – b) e  x = t −2 x 1
B(a – b, a + b), então, necessariamente: r : 2x + 3y + 4 = 0 e s :  , t ∈  e a reta q : y = + é:
 y = 2t + 1 2 8

(A) ax = by . (C) bx = ay . 3 5
(A) 4 5 . (C) .
(B) P = ( a, b) x 2 − y 2 = 2( ax + by ) .
. (D) 10

(B) 3 7 . (D)
5 7 .
20 Se as coordenadas dos vértices de um triângulo são números 20 4
racionais, então qual dos pontos abaixo necessariamente tem coordenadas
racionais?
28 (ITA) Considere a reta r mediatriz do segmento cujos extremos são os
pontos em que a reta 2 x − 3 y + 7 = 0 intercepta os eixos coordenados.
(A) Baricentro. (C) Circuncentro.
(B) Incentro. (D) Ortocentro.  1 1
Então a distância do ponto  ,  à reta r é:
4 6
21 (ITA) Dadas as retas r 1: x + 2y – 5 = 0, r 2: x – y – 2 = 0,
5 3 2 3
r3: x – 2y – 1 = 0, podemos afirmar que: (A) . (D) .
2 7
(A) são 2 a 2 paralelas. 4 2
(B) r1 e r3 são paralelas. (B) . (E) .
13 13
(C) r1 é perpendicular a r3.
(D) r2 é perpendicular a r3. (C) 3 13 .
(E) as três retas são concorrentes em um mesmo ponto.
29 (ITA) A equação da reta bissetriz do ângulo agudo que a reta
22 Os vértices de um triângulo são os pontos A(3, 6); B(– 1, 3) e C(2, – 1). y = mx , m > 0 forma com o eixo x é:
Calcule o comprimento da altura traçada do vértice C.
1 + 1 + m2 −1 + 1 + m2
23 (EFOMM) Determine o coeficiente angular da reta cujas equações são (A) y = y=
x . (D) x.
dadas por x = 2t + 1, y = t + 2, t ∈ ¡. m m

1 − 1 + m2
(A) – 1. (D) 1/2. (B) y = x . (E) n.d.a.
m
(B) – 1/2. (E) 1.
(C) 2/5. −1 − 1 + m2
(C) y = x.
m
24 (AFA) Uma reta, que passa pelo primeiro quadrante, intercepta os
eixos cartesianos nos pontos A(k, 0) e B(0, k), determinando o triângulo EXERCÍCIOS NÍVEL 2
OAB com 8 unidades de área. Então, a equação geral dessa reta pode ser
escrita por: 01 Determine as coordenadas do ponto simétrico a P(a, b) com relação
à bissetriz do primeiro quadrante.
(A) x – y – 4 = 0.
(B) x + y – 4 = 0. 02 O lugar geométrico dos pontos P(x, y) que satisfazem
(D) x + y + 4 = 0.
max{| x |,| y |} = k , k ∈ ¡*+ é:
(D) x + y − 2 2 = 0 .
(A) uma circunferência.
25 (ITA) Dados os pontos A(0, 8), B(– 4, 0) e C(4, 0), sejam r e s (B) uma reta.
as retas tais que A, B ∈ r; B, C ∈ s . Considere P1 e P2 os pés das retas (C) um quadrado.
perpendiculares traçadas de P(5, 3) às retas r e s, respectivamente. Então, (D) um triângulo.
a equação da reta que passa por P1 e P2 é:
03 Os lados de um triângulo pertencem às retas x + 5y – 7 = 0; 3x – 2y
(A) y + x = 5 . (D) y+ x =2. – 4 = 0 e 7x + y +19 = 0. Calcule a área S desse triângulo.
(B) y + 2 x = 5 . (E) n.d.a.
(C) 3 y − x = 15 . 04 Dadas as equações de dois lados de um paralelogramo 8x + 3y +
1 = 0, 2x + y – 1 = 0 e a equação de uma de suas diagonais 3x + 2y
+ 3 = 0, ache as coordenadas de seus vértices.

IME-ITA 143
Matemática IV – Assunto 2

05 (ITA) As retas y = 0 e 4 x + 3 y + 7 = 0 são retas suportes das diagonais 16 (ITA) Num sistema de coordenadas cartesianas, duas retas r e s, com
de um paralelogramo. Sabendo que essas diagonais medem 4 cm e 6 cm, coeficientes angulares 2 e ½, respectivamente, se interceptam na origem
então, a área desse paralelogramo, em cm2, vale: O. Se B ∈ r,e C ∈ s são dois pontos no primeiro quadrante tais que o
36 48 segmento BC é perpendicular a r e a área do triângulo OBC é igual a 1,2,
(A) . (D) .
5 3 então a distância de B ao eixo das ordenadas vale:
27 48
(B) . (E) . 8 1
4 5 (A) . (D) .
5 5
44 4
(C) . (B) . (E) 1.
3 5
06 A área de um triângulo é S = 8 u.a.; dois de seus vértices são os pontos 2
A(1, – 2); B(2, 3) e o terceiro vértice C pertence à reta 2x + y – 2 = 0. Ache (C) .
as coordenadas do vértice C. 5
17 Dados dois vértices M1(– 10, 2) e M2(6, 4) de um triângulo cujas alturas
07 A área de um paralelogramo é S = 17 u.a.; dois dos vértices coincidem se intersectam no ponto N(5, 2), ache as coordenadas do terceiro vértice M3.
com os pontos A(2, 1) e B(5, – 3). Ache os dois outros vértices, sabendo que
o ponto de interseção das diagonais se encontra sobre o eixo das ordenadas. 18 O ponto P(– 2, 3) é comum a três retas. Uma delas (r1) é paralela ao
eixo das abscissas, outra (r2) contém o ponto de coordenadas (2, 1) e a
08 A área de um triângulo é S = 1,5 u.a.; dois de seus vértices são os terceira (r3) passa pela origem.
pontos A(2, – 3); B(3, – 2) e o baricentro desse triângulo pertence à
reta 3x – y – 8 = 0. Ache as coordenadas do vértice C. a. Mostre que dado qualquer ponto A em r1, é possível encontrar B e C,
em r2 e r3 respectivamente, de modo que B é o pé da mediana relativa
09 A área de um triângulo é de 4 unidades de superfície, sendo dois a P no triângulo PAC.
de seus vértices os pontos A(2,1) e B(3,– 2). Sabendo que o terceiro
vértice encontra-se sobre o eixo das abscissas, pode-se afirmar que suas b. Mostre que o ângulo que a reta CA forma com o eixo x é constante
coordenadas são: quando A varia e determine seu valor.

(A) (– 1/2, 0) ou (5, 0). (D) (– 1/3, 0) ou (4, 0). 19 Seja PQR um triângulo retângulo isósceles com ângulo reto em P(2, 1).
(B) (– 1/2, 0) ou (4, 0). (E) (– 1/5, 0) ou (3, 0) Se a equação da reta QR é 2 x + y = 3 , então a equação representando o
(C) (– 1/3, 0) ou (5, 0). par de retas PQ e PR é:

10 Ache um ponto Q simétrico do ponto P(– 5, 13) em relação à reta


(A) 3 x 2 − 3 y 2 + 8 xy + 20 x + 10 y + 25 = 0 .
2x – 3y – 3 = 0.
(B) 3 x 2 − 3 y 2 + 8 xy − 20 x − 10 y + 25 = 0 .
11 Dado um quadrilátero de vértices A(– 3, 12); B(3, – 4), C(5, – 4) e (C) 3 x 2 − 3 y 2 + 8 xy + 10 x + 15 y + 25 = 0 .
D(5, 8), determine a razão na qual sua diagonal AC divide a diagonal BD. (D) 3 x 2 − 3 y 2 − 8 xy − 10 x − 15 y − 20 = 0 .

12 Ache a projeção do ponto P(– 6, 4) sobre a reta 4x – y + 3 = 0. 20 Um raio luminoso parte do ponto M0(– 2, 3) sob um ângulo a com o
eixo Ox. Sabe-se que tana = 3. Atingido o eixo Ox, o raio é refletido. Ache
13 (AFA) Seja P(3, 1) o ponto médio do segmento AB, em que A é as equações das retas que representam os raios incidente e refletido.
interseção da reta t com a reta r : 3 x − y = 0 , e B é a interseção de t com
a reta s x + 5 y = 0 . O coeficiente angular de t é: 21 (AFA) A reta (s), simétrica de (r) x – y + 1 = 0 em relação à reta
(t) 2x + y + 4 = 0:
(A) negativo. (C) 5, pois t é perpendicular a s.
(B) par positivo. (D) nulo. (A) passa pela origem.
(B) forma um ângulo de 60 graus com (r).
14 A equação da reta perpendicular à reta ax + by + c = 0 passando pelo (C) tem – 1/5 como coeficiente angular.
ponto ( x0 , y 0 ) pode ser escrita como: (D) é paralela à reta de equação 7y – x + 7 = 0.

22 Ache a equação de uma reta que passe pelo ponto de interseção das
(A) Bx − Ay = Bx0 − Ay 0 . (D)
Ax + By = Ax0 + By 0 .
retas 3x + y – 5 = 0 e x – 2y + 10 = 0 e que esteja a uma distância
(B) Bx + Ay = Bx0 + Ay0 . (E) Bx − Ay = C . d = 5 u.c. do ponto C(– 1, – 2).
(C) Ax − By = Ax0 − By0 .
23 (ITA) Sendo r uma reta dada pela equação x − 2 y + 2 = 0 , então, a
15 Dadas duas retas concorrentes r1 : a1 x + b1 y + c1 = 0 e equação da reta s simétrica à reta r em relação ao eixo das abscissas é
r2 : a2 x + b 2 y + c2 = 0 , pode-se dizer que a equação descrita por:
α ⋅ ( a1 x + b1 y + c1) + β ⋅ ( a2 x + b 2 y + c2 ) = 0 , α,β ∈ ¡ sempre representa:
(A) x + 2 y = 0 . (D) x + 2y + 2 = 0 .
(A) uma reta paralela à reta r1. (B) 3 x − y + 3 = 0 . (E)
x − 2y − 2 = 0 .
(B) uma reta paralela à reta r2. (C) 2 x + 3 y + 1 = 0 .
(C) uma reta paralela à bissetriz do ângulo agudo das retas dadas.
(D) uma reta passando pela interseção das retas dadas.

144 Vol. 2
Geometria analítica: ponto e reta

24 Sobre os pontos P(x, y) que estão no interior do triângulo formado 04 (IIT) Considere um segmento AB de comprimento constante c, tal que
pelos pontos A(1, 3), B(5, 0) e C(– 1, 2), considere as afirmativas abaixo: A encontra-se sempre no eixo x e B no eixo y. Sendo OAPB um retângulo,
determine o lugar geométrico dos pés das perpendiculares traçadas de P
I. 3x + 2y ≥ 0 ao segmento AB.
II. 2 x + y − 13 ≥ 0
III. 2 x + −3 y − 12 ≤ 0 05 Dado um triângulo de vértices A(– 1, –1); B(3, 5) e C(– 1, – 2), ache
IV. −2 x + y ≥ 0 o ponto de interseção da bissetriz do ângulo externo do vértice A com o
prolongamento do lado BC.
O número de afirmativas corretas é:
06 Um raio luminoso se desloca segundo a reta x – 2y + 5 = 0. Após
(A) 0. (D) 3. ter alcançado a reta 3x – 2y + 7 = 0, o raio é refletido. Ache a equação
(B) 1. (E) 4. da reta que representa o raio refletido.
(C) 2.
07 Ache as equações dos lados de um triângulo ABC, conhecendo-se um
25 C alcule a distância entre as retas paralelas ax + by + c1 = 0 e dos vértices A(3, – 1), bem como as equações de uma bissetriz x – 4y + 10
ax + by + c2 = 0 . = 0 e de uma mediana 6x + 10y – 59 = 0.

26 (IME) Prove que a soma das distâncias de um ponto qualquer interior 08 Ache todos os vértices de um triângulo ABC, conhecendo-se um dos
a um triângulo equilátero aos lados é constante.2 vértices C(4,3), bem como as equações de uma bissetriz interna x + 7y + 5
= 0 e da mediana 4x +13y – 10 = 0.
27 Ache a equação da bissetriz do ângulo agudo formado pelas retas
3x + 4y – 5 = 0 e 5x –12y + 15 = 0. 09 Um reta L passando pela origem intersecta a reta x + y = 1 e x + y = 3
nos pontos P e Q respectivamente. Por P e Q, duas retas L1 e L2 são
28 Dado um triângulo de vértices A(2, – 2); B(3, – 5) e C(5, 7), ache a desenhadas, paralelas às retas 2 x − y = 5 e 3 x + y = 5 respectivamente.
equação da perpendicular traçada do vértice C à bissetriz do ângulo interno Determine o lugar geométrico da interseção de L1 e L2 quando L varia.
do vértice A.
10 Mostre que não existe triângulo equilátero tal que todos os vértices
29 Dado um triângulo de vértices A(3, – 5); B(– 3, 3) e C(– 1, – 2), calcule
tenham coordenadas inteiras.
o comprimento da bissetriz do ângulo interno do vértice A.
11 (IME) Considere um quadrado ABCD. Determine o ponto P do plano
30 Mostre que as retas da forma ( a + 2 b) ⋅ x + ( a + 3 b) ⋅ y = a + b, a, b ∈ ¡ 2 2 2 2
têm um ponto em comum e encontre esse ponto. que minimiza a soma PA + PB + PC + PD .

12 (IME) Sobre uma reta r são marcados os pontos A, B, C e D. São


EXERCÍCIOS NÍVEL 3 construídos os triângulos equiláteros ABE, BCF e CDG, de forma que os
pontos E e G encontrem-se do mesmo lado da reta r, enquanto o ponto F
01 (ITA) A área do polígono, situado no primeiro quadrante, que é encontra-se do lado oposto, conforme mostra a figura. Calcule a área do
delimitado pelos eixos coordenados e pelo conjunto triângulo formado pelos baricentros de ABE, BCF e CDG, em função dos
{( x , y ) ∈ ¡ 2 : 3 x 2 + 2 y 2 + 5 xy − 9 x − 8 y + 6 = 0} é igual a: comprimentos dos segmentos AB, BC e CD.

(A) 6 . (D) 3. E
5 10 .
(B) . (E) G
2 3
(C) 2 2 .

02 (ITA) Sejam r e s duas retas paralelas distando entre si 5 cm. Seja P A B C D


um ponto na região interior a essas retas, distando 4 cm de r. A área do
triângulo equilátero PQR, cujos vértices Q e R estão, respectivamente, F
sobre as retas r e s, é igual, em cm2, a:
13 (IME) As medianas BM e CN de um triângulo ABC se cortam em G.
(A) 3 15 . 12S
Demonstre que tan( BGC) = 2 , em que S é a área do triângulo
b + c 2 − 5 a2
(B) 7 3 .
ABC, AC = b, AB = c e BC = a.
(C) 5 6 .
15 14 Seja ABC um triângulo com AB = AC. Se D é o ponto médio de BC,
(D) .
2 E é o pé da perpendicular de D a AC e F é o ponto médio de DE, mostre
7 que AF é perpendicular a BE.
(E) ⋅ 15 .
2
15 E e F são pontos do lado AB, do triângulo obtusângulo ABC (C > 90°),
03 Ache a equação de uma reta que passa pelo ponto C(– 5, 4), tais que AE = EF = FB. D é ponto da reta BC tal que BC é perpendicular
sabendo-se que o comprimento do segmento compreendido entre as retas a ED. AD é perpendicular a CF. Os ângulos BDF e CFA medem x e 3x,
x + 2y + 1 = 0 e x + 2y – 1 = 0 é igual a 5. respectivamente. Calcule a razão DB/DC.

IME-ITA 145
Matemática IV – Assunto 2

RASCUNHO

146 Vol. 2
Linhas proporcionais A ssunto
6
Matemática V

Na primeira apostila, essencialmente vimos como comparar segmentos • OA x OB= OP2 = OQ2 = OP . OQ
e ângulos, por vezes calculando-os, embora mais por igualdade a outros já
OA
dados anteriormente. Nessa apostila, veremos efetivamente como calculá-­ = k2
• OB
-los, sem necessariamente comparar uns aos outros.
III. Se P e Q dividem harmonicamente AB na razão k, vale a fórmula:
Como dito no bloco sobre triângulos, a estrutura triangular nos permite 2k
determinar muitos elementos a partir de poucas informações dadas. Esta PQ = AB. 2 .
| k − 1|
apostila vem para corroborar isso e mostrar como exatamente podemos
usar tal ferramenta, entre várias outras, a nosso favor. 2 1 1
IV. Se P e Q dividem harmonicamente AB, vale a fórmula: = ±
Dado um segmento AB, se P está entre A e B, dizemos que P [se Q divide externamente]. PQ QA QB
divide internamente AB. Chamamos os segmentos PA e PB de aditivos
[PA + PB = AB]. Associado a essa divisão, dizemos que P divide AB na
razão PA:PB. Se Q é tal que está na reta AB, porém fora do segmento AB,
2. Teorema de Tales
dizemos que Q divide externamente AB. Chamamos QA e QB de segmentos Se várias retas paralelas entre si [feixe de paralelas] seccionam retas
subtrativos [|QA – QB|=AB]. Associado a essa divisão, dizemos que Q transversas, gerando segmentos, então eles são correspondentemente
divide AB na razão QA:QB. proporcionais entre si.

A P B
r

C D Q
x
x’
s
y y’
Dado um segmento AB e uma razão k real positiva, prova-se que
existem e são únicos os pontos P e Q que dividem interna e externamente, t
respectivamente, o segmento AB, na razão PA : PB = k, QA:QB = k [desde z z’
que k seja diferente de 1].
u
1. Divisão Harmônica
Dado um segmento AB e dois pontos, P e Q, que dividem internamente
e externamente, respectivamente, o segmento AB numa dada razão k, x y z
Na figura, r  s  t  u ⇒ = =
dizemos que P e Q dividem harmonicamente AB na razão k. x' y' z'
PA QA Recíproca do teorema de Tales
Ou seja, se P e Q dividem harmonicamente AB, vale que = .
PB QB
Se AA’//BB’ e dois pontos, P e P’, dividem AB e A’B’ respectivamente

A P M B PA QA na mesma razão, ou seja, PA = P ' A ' , então tem-se que PP’ é paralelo
Q = = k <1
PB QB a AA’ e a BB’. PB P ' B '

B B’
A M P B PA QA
= 1, ∃ Q | =1
PB QB
3x 3y
A M PB Q
P P’
PA QA 2x 2y
= = k > 1 [suficientemente grande]
PB QB
A A’
Veja que sempre o ponto da divisão externa está mais afastado de A
e B do que o da divisão interna.
3. Teoremas das bissetrizes
Propriedades da Divisão Harmônica – Interna: Seja ABC um triângulo, e AD a bissetriz interna. Então, D divide
I. Se P e Q dividem harmonicamente AB, então A e B dividem BC numa razão igual à dos lados. Precisamente, BD = AB .
harmonicamente PQ. CD AC
II. Se P e Q dividem harmonicamente AB na razão k, sendo O médio de
PQ, valem as duas:

IME-ITA 147
Matemática V – Assunto 6

A
Casos de semelhança
Existem três grupos de informações que são necessárias e suficientes
para determinarem que dois triângulos são semelhantes. Chamamos essas
informações de ‘casos de semelhança’. Vejamos:
D
B C – Ângulo Ângulo [AA]: Se dois triângulos têm dois pares de ângulos
iguais entre si, então os triângulos são semelhantes.
– Externa: Seja ABC um triângulo, e AE a bissetriz externa em A. Então, E
divide BC [externamente] numa razão igual à dos lados. Precisamente, – Lado Ângulo Lado [LAL de semelhança]: Se dois triângulos têm um
BE AB par de ângulos iguais, e os lados que os formam são proporcionais,
= . então os triângulos são semelhantes.
CE AC
F
– Lado Lado Lado [LLL de semelhança]: Se dois triângulos têm todos os
A
seus lados proporcionais entre si, então os triângulos são semelhantes.
A F
5
x=7
B C E 8
J

Observe que, como consequência, podemos dizer que D e E dividem D E x 11


harmonicamente BC numa razão igual à dos lados.
2
C
Círculo de Apolônio B G 14 H

Dado um segmento BC e uma razão k, o círculo de Apolônio sobre Veja que DE//BC, logo os triângulos Os lados de FJI são 5 e 8, e os de FGH
ABC e ADE são semelhantes [AA] são 10 e 16, e eles têm o ãngulo F em
PB
BC na razão k é o lugar geométrico dos pontos P do plano tais que =k. comum, logo FJI~FGH [caso LAL]
PC
Sejam X e Y os pontos que dividem BC harmonicamente na razão k.
Então, o círculo de Apolônio sobre BC na razão k é o círculo de diâmetro XY.
P EXERCÍCIOS RESOLVIDOS
01 Seja ABCD um quadrilátero convexo tal que
= 
BAC = e ACD
CBD   . Prove que BC2 + AD2 = AC2 .
ADB
B X C O Y Solução:
Este é um problema muito importante, pois exemplifica uma situação
extremamente comum em geometria: a semelhança invertida.
Seja P a interseção das diagonais AC e BD. Veja que os triângulos ABC
e BCP são semelhantes e o mesmo acontece com os triângulos ACD e
ADP (os triângulos têm os mesmos ângulos). Das duas semelhanças,
tiramos que:
4. Semelhança de triângulos BC AC
= ⇒ BC2 = AC ⋅ CP e
AD AC
= ⇒ AD2 = AC ⋅ AP.
Dizemos que dois triângulos são semelhantes quando eles CP BC AP AD
apresentam os mesmos ângulos internos e os lados opostos aos ângulos Somando as duas relações, temos que:
correspondentes são respectivamente proporcionais. Precisamente, vale: BC2 + AD2 = AC ⋅ ( CP + AP ) = AC2 .
∆ABC ~ ∆ DEF ⇔ A = D
, B  = F , AB = AC = BC .
 = E , C
DE DF EF
A
C’ B’ EXERCÍCIOS NÍVEL 1
a g b
Nos triângulos ao lado, tem-se que 01 Um segmento AB é tal que 7.AB = 3.CD. Qual será sua medida na
a 1
A’ AB AC BC unidade CD ?
= = =k
b g
C
A' B ' A' C ' B ' C ' 4
B
02 Calcule x para que os pontos da figura abaixo formem divisão
harmônica:
Se dois triângulos são semelhantes, então ângulos homólogos
[correspondentes pela semelhança] são sempre iguais, e segmentos 2x 2 3x
homólogos estão sempre na mesma razão de semelhança. Por exemplo, N A M
B
se dois triângulos são semelhantes, então o ângulo entre uma mediana e
uma bissetriz partindo do mesmo vértice é igual ao ângulo entre mediana
03 Os pontos M e N dividem harmonicamente o segmento AB de 42 cm
e bissetriz no vértice homólogo no outro triângulo, e as medianas estão
5
na mesma razão que as bissetrizes, que o perímetro, que o circunraio, etc. na razão . Calcule MN.
2

148 Vol. 2
Linhas proporcionais

04 Os pontos M e N dividem harmonicamente o segmento AB na razão 03 Os pontos A, M, B e N de uma reta formam uma divisão harmônica
3 . de razão MA = NA . Se J é o ponto médio de MN, determine a razão
2 JA . MB NB = k
Sabemos que os pontos A e B dividem o segmento harmonicamente. JB
Calcule a razão desta divisão.
04 Em um ∅ABC de lados AB = 12, AC = 8, BC = 10, a bissetriz interna
de B encontra a bissetriz AN externa de A no ponto F. Determine a razão
05 Os lados de um triângulo medem 6 cm, 9 cm e 10 cm. Calcule de
quanto se deve prolongar o lado maior para que ele encontre a bissetriz FN
.
externa traçada do ângulo oposto. FA
05 Um triângulo ABC tem 9 cm de perímetro e AB = 3 cm. Os pés das
06 Num triângulo retângulo, os ângulos agudos medem 30º e 60º. Qual a bissetrizes dos ângulos interno e externo no vértice C dão formação a um
razão entre os segmentos determinados sobre o lado oposto pela bissetriz segmento que mede 4 cm. Calcular AC e BC.
do ângulo de 60º?
AB 3
06 Em um ∅ABC , BC = a e = .Calcule o comprimento da altura
07 Considere os quadrados ABCD e ABEF da figura. Se FG = 12, AC 2
calcule HD. relativa ao lado a sabendo que ela é máxima.

E A D 07 Considere em um círculo de centro O um diâmetro AB. Prolongue uma


corda AP qualquer do círculo de um comprimento PQ = AP. Os segmentos
JQ
QO e BP cortam-se em J. Calcule a razão .
H JO
G
08 Em um ∅ABC , a bissetriz interna de A encontra BC em D e o círculo
circunscrito em E. Se AB = 8, AC = 6 e DE = 3, calcule o comprimento
F B C da bissetriz AD.

08 Em um ∅ABC , de lados AB = 9, AC = 12 e BC = 15, traça-se DE 09 É dada uma reta r e três círculos A, B e C, num mesmo semiplano
paralela a BC passando pelo baricentro do triângulo (D em AB e E em AC). gerado por r. Tais círculos são tangentes à reta r, e seus centros são
Determine o perímetro do ∅ADE . colineares. Além disso, A e B são tangentes externamente, bem como B
e C. Sabendo que os raios de A e C são 4 e 9, calcule o raio de B:
09 As bases de um trapézio medem 6 cm e 8 cm e a altura 5 cm. Quais
as alturas dos triângulos obtidos, prolongando-se os lados não paralelos? (A) 5. (D) 6.5.
(B) 5.5. (E) 7.
10 Determine o comprimento do lado do quadrado inscrito em um triângulo (C) 6.
de base 12 e altura 8, sabendo que um dos lados do quadrado está contido
nessa base do triângulo. 10 Num círculo de raio igual a 12 está inscrito um ∅ABC cujos lados AB
e AC medem 8 e 9, respectivamente. Determine a altura relativa ao lado
11 AD e BC são as bases de um trapézio ABCD, de medidas a e b (a > b). BC.
MN é um segmento paralelo às bases, com M e N internos aos lados AB
AM DN p 11 Um trapézio tem bases com medidas 2 e 3. Calcule a medida
e CD. Sabendo que = = , calcule a medida de MN. do segmento paralelo às bases, que contém o ponto de interseção
MB NC q
das diagonais.
12 Pelas extremidades do segmento AB = 32 cm levantam-se, no mesmo
semiplano, duas perpendiculares AC = 10 cm e BD = 6 cm. Pede-se AB 3
12 Em um ∅ABC , BC = a e = . Calcule o comprimento da altura
localizar um ponto E sobre AB tal que tenham o ângulo CED reto. AC 2
13 Os lados de um triângulo ABC medem AB=4 cm, AC=8 cm e relativa ao lado a, sabendo que ela é máxima.
BC = 6 cm. Uma tangente ao círculo circunscrito no vértice A intersecta
o prolongamento de BC no ponto P. Quanto mede o segmento PA? 13 Em um triângulo, os lados de medidas m e n são opostos aos ângulos
de 60° e 40°. O segmento da bissetriz do maior ângulo interno do triângulo
EXERCÍCIOS NÍVEL 2 é dado por:

01 Demonstre que dado um segmento AB e uma razão k, é único o ponto m+n m+n
(A) m . (D) n .
n m
M, interno a AB, tal que a razão AM vale k.
BM n m
Demonstre também que é único o ponto N, externo a AB, tal que a razão (B) m . (E) .
m+ n n
AN
vale k. m+n
BN (C) n .
m
02 Os pontos P e Q pertencem ao interior do segmento AB e estão de
2
um mesmo lado de seu ponto médio. P divide AB na razão e Q divide
3 14 No triângulo ABC, temos que I é seu incentro, e M e N são pontos
3
AB na razão . Se PQ = 2, determine a medida de AB. sobre AC e BC tais que BN.AB = BI², e AM.AB = AI². Prove que M, N e I
4 são colineares.

IME-ITA 149
Matemática V – Assunto 6

15 Os segmentos PA e PB são tangentes a um círculo nos pontos A e B, 03 Sejam ABC um triângulo, H seu ortocentro, O seu circuncentro, G seu
e Q é um ponto qualquer da circunferência. Se as distâncias de Q às retas baricentro e M o ponto médio de BC.
PA e PB são, respectivamente, 4 e 9, quanto vale a distância de Q a AB?
a. Prove que AH = 2OM.
(A) 4. (D) 7. b. Prove que H, O e G são colineares (reta de Euler).
(B) 5. (E) 8. GH
(C) 6. c. Determine a razão .
GO
EXERCÍCIOS NÍVEL 3 04 Seja ABC um triângulo qualquer, e AD, BE, CF cevianas internas a esse
AP BP CP
triângulo, que se intersectam em P. Prove que a soma + +
01 Em um triângulo ABC, de incentro I e exincentro J relativo a BC, AD BE CF
AP EP FP
demonstre que vale a relação: AI x AJ = AB x AC. é constante. [Dica: Prove que = + , considerando uma paralela
AD BE CF
a BC pelo ponto P.]
02 Um triângulo ABC tem lados em progressão aritmética. AB < BC < AC.
Sendo G o baricentro de ABC, I o seu incentro, e L o ponto em que AI
intersecta o círculo circunscrito a ABC, prove que IG//B e que I é ponto
médio de AL.
RASCUNHO

150 Vol. 2
Relações métricas no triângulo A ssunto
7
Matemática V

Com as ferramentas do bloco anterior, e algumas definições Para que essa definição alternativa esteja de acordo com as definições
convenientes que relacionam ângulos e segmentos, podemos deduzir algébricas, devemos estendê-la para os ângulos obtusos, da seguinte
importantes relações métricas em triângulos que permitirão calcular muitas forma:
informações a partir de poucas. α + β = 180º
senα = senβ
1. Relações métricas no triângulo cosα = − cosβ
retângulo – tanα = − tan β

Observe que, por Pitágoras, tem-se sen 2a+cos 2a=1[Relação


1.1 Teorema de Pitágoras fundamental]
Seja ABC um triângulo retângulo em A, AH altura relativa à hipotenusa,
Seguem algumas fórmulas úteis no estudo de trigonometria, a respeito
como na figura abaixo. Observe que o ângulo BÂH é igual ao ângulo agudo
de como calcular seno e cosseno de arco-soma e arco-diferença:
C, e CÂH é igual ao ângulo agudo B [por soma de ângulos internos]. A partir
disso, podemos observar triângulos BAH, ACH e BCA semelhantes, e, daí, sen( x + y ) = sen( x )cos( y ) + sen( y )cos( x )
deduzir relações métricas importantes no triângulo retângulo. Veja abaixo: sen( x − y ) = sen( x )cos( y ) − sen( y )cos( x )
cos( x + y ) = cos( x )cos( y ) − sen(x)sen(y)
A cos( x − y ) = cos( x )cos( y ) + sen(x)sen( y )

c b 3. Lei dos Senos


h
Em um triângulo ABC, cada lado é proporcional ao seno do ângulo
oposto. Além disso, o coeficiente de proporcionalidade é o diâmetro do
B H C círculo circunscrito.
n m
a A
A
BH AH
∆BAH ~ ∆ACH ⇒ = ⇒ h2 = mn D
AH CH A
AH AC
∆ACH ~ ∆BCA ⇒ = ⇒ ah = bc O a b c
BA BC = = = 2R
senα senβ senγ
CH AC
∆ACH ~ ∆BCA ⇒ = ⇒ b2 = ma B
CA BC
a C

Analogamente, deduzimos c2 = na. Somando as duas últimas relações,


o b t e m o s : b2 + c2 = ma + na = ( m + n)a = a ⋅ a = a2 ⇒ b2 + c2 = a2
[Teorema de Pitágoras] 3.1 Lei dos Senos para cálculo de cordas
Também é possível deduzir a seguinte relação a par tir daí: Em um círculo de raio R, se uma corda determina um arco de medida
1 1 1 α
= + . angular igual a a, então vale que a corda mede 2 R ⋅ sen . . Essa fórmula
h 2 b2 c2 2
é muito útil para calcular lados e diagonais de polígonos regulares, ou para
calcular os lados de um quadrilátero inscritível.
2. Trigonometria Básica
Por semelhança, se dois triângulos retângulos possuem ângulos
iguais, tem-se que as razões entre seus catetos e a hipotenusa são sempre
iguais. Dessa forma, podemos relacionar um ângulo agudo e as razões
ditas anteriormente, definindo as funções trigonométricas. O
C
b
b senα =
a A L B
a c
b cos α = a
a
α
senα b Na figura, L = 2 R ⋅ sen  
B a tan α = = 2
90º A cos α c
c

IME-ITA 151
Matemática V – Assunto 7

4. Lei dos Cossenos EXERCÍCIOS NÍVEL 1


Em um triângulo ABC, vale a seguinte relação entre os lados e 01 Considere um ∅ABC retângulo de catetos AB = 5a e AC = 4a. Pelo
ponto M, médio de AC, trace MN perpendicular a AC. Se N é exterior ao
um ângulo: a2 = b2 + c2 − 2 bc ⋅ cosA . Dados os lados de um triângulo, triângulo e se MN = a, calcule BN.
podemos então calcular cada ângulo através dessa fórmula.

A 02 Dois círculos de raios 8 e 6 são ortogonais. Determine o comprimento


da corda comum.
A

c 03 Considere um ponto P no interior de um quadrado de lado a, de forma


b que tenha mesma distância a dois vértices consecutivos e ao lado oposto
a esse vértice. Se d é a distância comum, calcule d.
B C
a 04 O triângulo ABC é retângulo em A. Sabendo-se que a hipotenusa BC
é igual a 25 cm e o cateto AB mede 20 cm, pede-se calcular:
Na figura, a2 = b2 + c2 − 2 bc ⋅ cos A
a. O cateto AC, a altura AH e os segmentos BH e CH determinados pela
referida altura.
4.1 Síntese de Clairault b. Toma-se HN paralelo a AB. Calcule HN e os segmentos AN e NC
determinados pela altura HN sobre AC.
Serve para testar se um triângulo é acutângulo, retângulo ou c. Toma-se HM, bissetriz do ângulo BHA. Calcule os segmentos
obtusângulo. Se os lados de um triângulo são a>b>c, temos então: determinados sobre AB.
a2 > b2 + c2 ⇒ A > 90º (obtusângulo)
05 ABC é um triângulo retângulo de hipotenusa BC e altura AH. Seja P
a2 = b2 + c2 ⇒ A = 90º (retângulo) um ponto do mesmo semiplano de A em relação à reta suporte de BC.
a2 < b2 + c2 ⇒ A < 90º (acutângulo) Os ângulos HPC e ABC  são iguais a 15°. Se o segmento PH é o maior
possível, pode-se afirmar que PH é igual a:
5. Relação de Stewart (A) AC. (D) HC .
Seja AD uma ceviana interna em um triângulo ABC. A relação de (B) AB. 2
Stewart determina o cálculo da ceviana AD em função dos lados do (C) BC . (E) AH.
triângulo. 2
A 06 Prove que a equação 2 x 2 − 2 x + 1 = 0 admite sempre raízes reais,
a h b
se a e b forem catetos de um triângulo retângulo e h a altura relativa
b c à hipotenusa.
x
07 Determine o raio de um círculo inscrito num setor circular de 60° e
C m D n B raio 6 dm.
a
b 2
x 2
c2 08 Em um triângulo retângulo ABC, com ângulo reto em C, pontos D e E
− + = 1, ou equivalente, b2 n + c2 m = x 2 a + mna. dividem o lado BC em três partes iguais. Prove que se BC = 3AC, então
m⋅ a m⋅ n n⋅ a  + ADC
 + ABC = 90º.
AEC

EXERCÍCIOS RESOLVIDOS abc


09 O produto dos senos dos ângulos de um triângulo é k ⋅ , em que
01 Em um triângulo retângulo de catetos b e c e altura relativa à R3
hipotenusa h, prove que 1 = 1 + 1 . . a, b e c são os lados e R é o raio do círculo circunscrito. Calcule k.
h 2 b2 c2
Solução: 10 Considere o triângulo ABC de lados AB = AC = 6 e BC = 4. Seja M
bc
Lembrando que ah=bc, temos que h = , por tanto, o prolongamento do lado AC tal que MC = 1 . Calcule BM.
2 2 2 a
1 a b +c 1 1 MA 3
= = 2 2 = 2+ 2.
h 2 b2 c2 bc b c . 11 Calcule x na figura.

02 Em um triângulo ABC qualquer, demonstre que senA < senB + x+3


x x+1
senC.
Solução: x 2x
Sejam a, b, c os lados e R o circunraio. Pela desigualdade triangular,
temos que a < b + c. Utilizando a lei dos senos, segue que 2RsenA
< 2RsenB + 2RsenC, o que finaliza o problema. 12 Seja ABC um triângulo de lados a, b e c. Calcule a mediana relativa
ao vértice A.

152 Vol. 2
Relações métricas no triângulo

13 Seja ABC um triângulo de lados a, b e c. Calcule a altura relativa ao EXERCÍCIOS NÍVEL 2


vértice A.
01 No quadrilátero ABCD, B=D  = 90° . Traçamos por C paralelas CE
14 Seja ABC um triângulo de lados a, b e c. Calcule a bissetriz interna
relativa ao vértice A. e CF aos lados AB e AD, respectivamente. Se AF = 8, FB = 3, AE = 4 e
ED = 6, determine a medida da diagonal AC.
  
15 Seja ABC um triângulo de lados a, b e c. Calcule cos A + cos B + cos C . 02 Calcule o comprimento do segmento que une os pontos médios das
a b c bases AB e CD de um trapézio, conhecendo seus lados: AB = 14, BC
16 São dados dois círculos concêntricos. De um ponto P variável do =7, CD = 4 e DA = 5.
círculo exterior traçam-se PA e PB, sendo A e B extremos de um diâmetro
do círculo interior. Prove que PA2 + PB2 é constante. 03 Considere um quadrado Q de lado a e cinco círculos de mesmo raio r,
interiores a Q, dos quais um é concêntrico com Q e tangente exteriormente
17 Em um triângulo as medianas são ma = 6 cm, mb = 8 cm e mc = 12 cm. aos quatro outros, e cada um destes tangencia dois lados consecutivos
Calcule os lados. de Q. Calcule r.

18 Considere um triângulo equilátero T, de lado a, e três círculos internos 04 Determine a medida do lado de um quadrado ABCD sabendo que M
a T, tangentes exteriormente entre si, e a cada dois dos lados do triângulo. é ponto médio de AB, CP é perpendicular a MD e MP = 3.
r
Sendo r o raio dos círculos, a razão vale:
a 05 Uma reta passando pelo vértice A de um quadrado ABCD intersecta
o lado CD em E e a reta BC em F. Prove que 1 1 1
(A) 1 ( 3 − 1) (D) 3 2
+ 2 = 2.
AE AF AB
4 4
06 Seja ABCD um retângulo. Prove que para um ponto P qualquer:
(B) 1 ( 3 − 1) (E) 5
PA2 + PC2 = PB2 + PD2.
2 2
(C) 1 ( 5 − 1) 07 Seja ABCD um retângulo de centro O. Prove que, se um ponto P varia
2 sobre um círculo de centro O, a soma PA2 + PB2 + PC2 + PD2 é constante.

19 Em um triângulo ABC, em que AB=6 e AC=8, uma ceviana interna 08 Sendo M e N os pontos que dividem em três segmentos congruentes a
AP mede 4. Sabendo-se que PC=3 · PB, temos que a medida de BC é: 2
hipotenusa BC de um triângulo ABC, prove que: AM 2 + AN 2 + MN 2 = BC2 .
3
(A) 13. (D) 10.
(B) 12. (E) 9. 09 Dado um segmento AB, prove que se P varia numa reta perpendicular
(C) 11. a AB, a quantidade PA²–PB² é constante.

20 Considere o losango de lado 5 cm, em que uma das diagonais mede 10 A partir de P, interno a ABC, traçam-se perpendiculares aos lados BC,
8 cm. A altura do losango mede: CA e AB, cujos pés são D, E e F. Sabendo que BD = 8, DC = 14, CE =13,
AF = 12, e BF = 6, então calcule AE.
(A) 3,6 cm. (D) 5,2 cm.
(B) 4,0 cm. (E) 6,0 cm. 11 Considere um triângulo ABC tal que AB = 20, AC = 21 e BC = 29.
(C) 4,8 cm. Sejam D e E pontos do segmento BC tais que BD = 8 e CE = 9. Determine
 .
o ângulo DAE
21 Dois círculos secantes têm raios 10 cm e 17 cm. Sabendo que a corda
comum a eles mede 16 cm, qual é a distância entre os seus centros? 12 Dois lados consecutivos de um paralelogramo têm por medidas a e
b, e uma das diagonais tem por medida c. Determine a medida da outra
(A) 18 cm. (D) 24 cm.
diagonal.
(B) 20 cm. (E) 25 cm.
(C) 21 cm.
13 Sendo AD a bissetriz interna do ângulo  do ∅ABC, prove que:
AD2 = AB . AC – BD . DC.
22 Seja ABCD um quadrilátero de diagonais perpendiculares. Prove que
AB²+CD²=BC²+AD².
14 No triângulo ABC, AM é mediana, e a razão dos ângulos BÂM e MÂC
23 (ITA-88) Por um ponto A de uma circunferência traça-se o segmento é 1 . Prolonga-se AM até D, de forma que M está entre A e D, e BD é
AA’ perpendicular a um diâmetro desta circunferência. Sabendo-se que 2 AD
perpendicular a BA. Prove que AC = .
o ponto A’ determina no diâmetro segmentos de 4 cm e 9 cm, podemos 2
afirmar que a medida do segmento AA’é:
15 Seja ABCDEF um hexágono convexo inscrito numa circunferência de
raio r. Dado que AB = BC = CD = 2 e DE = EF = FA = 1, determine r.
(A) 4 cm. (D) 6 cm.
(B) 12 cm. (E) 13 cm. 16 Considere AOB um quadrante de círculo de centro O e raio R. Toma-
(C) 13 cm. -se um semicírculo de diâmetro OA interno ao quadrante. Calcule o raio do
círculo tangente ao semicírculo, a OB e ao arco AB do quadrante.

IME-ITA 153
Matemática V – Assunto 7

17 (AFA-00) Uma corda de comprimento a define em uma circunferência EXERCÍCIOS NÍVEL 3


π
de raio 2a um arco θ, 0 ≤ θ < . Nessa mesma circunferência, o arco
2 01 No retângulo ABCD, o lado BC é igual a 2AB. O ponto P está sobre
2θ é definido por uma corda de comprimento: AP 3
o lado AB e = . Traça-se a reta PS com S no interior de ABCD e
PB 4
(A) a 11 . (C) a 15 .
C pertence a PS. Marcam-se, ainda, M em AD e N em BC de modo que
4 4
BN
a 13 a 15 . MPNS seja um losango. O valor de é:
(B) . (D) AM
3 2 3
(A) . (D) 5
7 .
18 (EN-84) As medidas dos lados de um triângulo ABC são 3 números 11
inteiros e consecutivos e o ângulo maior, A, é o dobro do menor, C. Os (B) 3 . (E)
7
lados deste triângulo são: 11 .
11
(C) 5 .
(A) 2, 3 e 4. (D) 4, 5 e 6. 7
(B) 3, 4 e 5. (E) 5, 6 e 7.
(C) 8, 9 e 10.
02 ABC é um triângulo retângulo em A. Considere o círculo que tangencia
os lados AB e AC, e também o círculo circunscrito a ABC. Calcule o raio
19 (EN-88) O ponto B pertence ao segmento AC , dista 2 cm do ponto A desse círculo, sabendo que AB = 6 e AC = 8.
e dista 1 cm do ponto C. O raio de um círculo que tangencia exteriormente
os círculos de diâmetro AB e BC e tangencia internamente o círculo de
diâmetro AC é:

4
(A) 1 cm. (D) cm.
3 6
(B) 2 cm. (E)
5
cm.
5 11
(C) 3 cm.
7
RASCUNHO

154 Vol. 2
Colinearidade e concorrência A ssunto
8
Matemática V

Metricamente, existem dois critérios muito úteis e recorrentes em


geometria plana para determinar se três pontos são colineares, ou,  3
igualmente, determinar se três cevianas de um triângulo são concorrentes: 03 Num triângulo ABC, os lados medem AB = , AC = CB = .
2
os teoremas de Menelaus e Ceva. Sendo H o pé da altura de A, M médio de AH e X a interseção das retas
Os dois envolvem calcular também razões em que retas bissectam CM e AB, calcule a razão AX : XB.
segmentos, então podem ser úteis para contas envolvendo divisão de
segmentos, bem como, mais pra frente, para calcular razões de áreas 04 O lado AB de um quadrado ABCD é prolongado de um segmento
e determinar a posição de pontos sobre segmentos, mesmo no espaço. BP = 2 · AB. Com M, ponto médio de CD, traça-se BM, que corta AC
em Q. Sabendo que PQ corta BC em R, calcule a razão CR : RB.
1. Colinearidade de pontos – 05 (Gergonne) Seja ABC um triângulo, e D, E e F os pontos de tangência
Teorema de Menelaus do incírculo de ABC com BC, AC e AB, respectivamente. Prove que as
cevianas AD, BE e CF são concorrentes.
Sejam M, N e P pontos sobre os lados de um triângulo – dois
sobre os lados e um no prolongamento do terceiro lado, ou os três nos
06 No triângulo ABC, AM é uma mediana, P é um ponto dela, e as cevianas
prolongamentos dos lados–. Então os pontos M, N e P são colineares se,
BX e CY são concorrentes no ponto P. Prove que XY//BC.
AM BP CN
e somente se, vale a relação de Menelaus: ⋅ ⋅ = 1.
MB PC NA 07 Num triângulo ABC, AD é bissetriz interna, e I é incentro. Calcule a
A razão AI : ID em função dos lados do triângulo ABC.

M 08 Do vértice C do ângulo reto do triângulo ABC, a altura CK é traçada, e


no triângulo ACK traça-se a bissetriz CE. A reta que passa por B paralela a
N CE encontra CK no ponto F. Calcule a razão em que EF divide o segmento
AC.
B C P 09 Prove que os pés das bissetrizes internas a partir de dois vértices
de um triângulo escaleno e o pé de uma externa pelo terceiro vértice são
2. Concorrência de retas – colineares.
Teorema de Ceva 10 (Menelaus para quadriláteros) Seja r uma reta que corta os lados (ou
prolongamentos) AB, BC, CD e DA de um quadrilátero convexo ABCD nos
Sejam AD, BE e CF cevianas de um triângulo – as três internas ou duas MA NB PC QD
pontos M, N, P e Q, respectivamente. Prove que ⋅ ⋅ ⋅ = 1.
externas e uma interna–. Então as retas AD, BE e CF são concorrentes se, MB NC PD QA
AF BD CE
e somente se, vale a relação de Ceva: ⋅ ⋅ = 1.
FB DC EA EXERCÍCIOS NÍVEL 2
A 01 Demonstre (via Ceva) que, num triângulo, são concorrentes:

a. as medianas;
F b. as bissetrizes internas;
E c. as alturas;
d. duas bissetrizes externas e uma interna.
D
B C 02 Prove que, num triângulo escaleno, os pés das bissetrizes externas
são colineares.
EXERCÍCIOS NÍVEL 1
03 Seja ABCD um quadrilátero convexo. Seja E a interseção das retas
que contêm os lados AD e BC. Seja F a interseção das retas que contêm
01 Os pontos E e D pertencem aos lados AB e BC de um DABC e são os lados AB e CD. Prove que as diagonais AC e BD intersectam a reta EF
AE 1 CD 1 em pontos conjugados harmônicos em relação a E e F.
tais que = e = . Sendo F o ponto de concurso de AD e CE,
EB 3 DB 2
EF AF 04 Prove que os três pares de tangentes externas comuns a três círculos,
então calcule o valor de + .
FC FD tomadas duas a duas, cortam-se em três pontos colineares.
02 No triângulo retângulo ABC, P e Q estão sobre BC e AC, respectivamente,
05 Seja ABC um triângulo escaleno, e seja AD uma ceviana tal que os
tais que CP = CQ = 2. Pelo ponto de interseção R de AP e BQ, uma reta é
triângulos ADB e ADC tenham o mesmo perímetro. Construa analogamente
desenhada passando também por C e cortando AB em S. O prolongamento
as cevianas BE e CF. Prove que as cevianas AD, BE e CF são concorrentes.
de PQ corta AB em T. Se a hipotenusa AB = 10 e AC = 8, encontre TS.

IME-ITA 155
Matemática V – Assunto 8

RASCUNHO

156 Vol. 2
Relações métricas no círculo A ssunto
9
Matemática V

Neste assunto, veremos propriedades métricas a respeito de cordas


A
em um círculo, que induzirão a uma definição de ‘potência de ponto’. Entre
outros motivos puramente métricos, tal definição pode ser usada até para
B
detectar a existência de quadriláteros inscritíveis. P
D
1. Potência de ponto
C
Seja P um ponto interno a um círculo de centro O, e AB e CD cordas
quaisquer que se intersectam em P. Observe que os triângulos PAD e PCB
são semelhantes [ângulos iguais pela ideia do arco capaz], logo vale que
PA:PC :: PD:PB, e, portanto, vale que PA · PB = PC · PD. Isso independe Na figura, PA · PB = PC · PD
da escolha das cordas AB e CD, logo podemos deduzir que o produto
PA · PB é constante, se variarmos a corda AB. A
Como o produto PA · PB é constante, tomando o diâmetro que passa
por P, temos que PA · PB = (R+d) · (R-d) = R² – d², sendo R o raio do B
círculo, e d = OP, a distância do ponto P ao centro do círculo. P
O D
C d–r
r r
C
B
P
A
Na figura, PA · PB = (d – r) · (d + r) = d² – r²

Observe que nas duas situações a quantidade d² – R² é extremamente


D relacionada com o produto PA · PB associado. Chamaremos a quantidade
d² – r² de potência do ponto P em relação ao círculo Γ, sendo d a distância
de P ao centro de Γ, e r o raio de Γ.
Na figura, PA · PB = PC · PD Da definição, podemos ver se um ponto é interno, externo ou se
pertence a um círculo:
C
B I. se d2 – r2 = PotΓP < 0, então P é interno a Γ;
r–d
II. se d2 – r2 = PotΓP = 0, então P pertence a Γ;
P
III. se d2 – r2 = PotΓP < 0, então P é externo a Γ.
A d
O Observe também que, para o caso em que P é externo a Γ, temos
que se PT é segmento tangente a Γ, então PT2 = d2 – r2 = PotΓP. Logo, o
r cálculo de segmentos tangentes está relacionado com cálculo de potência
de ponto.
D Dada uma circunferência Γ de raio r, o lugar geométrico dos pontos
que têm potência igual a k é um círculo concêntrico a Γ, de raio k + r² .
Na figura, PA · PB = (r+d) · (r – d) = r² – d²
Em particular, o lugar geométrico dos pontos que têm tangente a Γ igual
a um certo L é o círculo concêntrico a Γ de raio L² + r ² .
Agora, consideremos P externo a um círculo de centro O, e PAB e
Como visto anteriormente, podemos estabelecer mais um critério para
PCD retas secantes ao círculo em A, B, C e D. Observe que os triângulos
observar quadriláteros inscritíveis: seja ABCD um quadrilátero convexo,
PAD e PCB são semelhantes, de novo, e que vale PA:PC :: PD · PB, logo
P a interseção das diagonais AC e BD. Se PA · PC = PB · PD, então o
PA · PB = PC · PD. Isso independe das retas secantes escolhidas, logo
quadrilátero ABCD é inscritível.
podemos dizer que o produto PA · PB é também constante, se variarmos
a secante PAB. Outro jeito de identificar quadriláteros inscritíveis é: seja ABCD um
quadrilátero convexo, e P a interseção dos lados AB e CD. Se PA · PB = PC ·
Como o produto PA · PB é constante, então, temos duas coisas:
PD, então o quadrilátero ABCD é inscritível.
se escolhermos a reta tangente PT ao círculo em T, temos que
PA · PB = PT · PT = PT²; e também, se escolhermos a reta diametral PO, Essas afirmativas são verdadeiras porque, a partir das premissas,
temos que PA · PB = (d + R)(d – R) = d² – R², sendo R o raio do círculo podemos reconhecer triângulos semelhantes, logo ângulos iguais. A partir
e d = OP, a distância de P ao centro O do círculo. deles, notamos que os quadriláteros são inscritíveis.

IME-ITA 157
Matemática V – Assunto 9

2. Eixo radical
Dados dois círculos Γ e Γ’, chamamos de eixo radical de Γ e Γ’ o lugar
geométrico dos pontos P tais que PotΓP = PotΓ’P. Se os círculos não forem
concêntricos, temos que o eixo radical é uma reta perpendicular à reta O M H O’
que passa pelos centros dos círculos. A posição do eixo radical depende
da posição dos centros dos círculos e das medidas dos raios. Em alguns
casos, situar o eixo radical é complicado, mas em alguns outros o eixo
radical está numa posição bem simples, como veremos abaixo.

Veja que os pontos médios dos segmentos tangentes comuns [internos


e externos] são todos colineares. Isso, porque, se TT’ é segmento tangente
comum, e M é médio de TT’, então PotΓM = MT2 = MT’2 = PotΓ’M. Como
os pontos que têm igual potência estão no eixo radical, que é uma reta,
então esses pontos médios são colineares. Além do mais, ligando-os,
construímos o eixo radical.

Se os círculos são secantes em dois pontos A e B, então seu eixo


radical é a reta AB. Veja que, se P pertence a AB, então a potência de P
em relação aos círculos é igual a PA · PB, para os dois círculos (– PA · PB,
caso P esteja dentro do segmento AB, que é a corda comum aos círculos). C O’

Na figura, os pontos médios dos segmentos tangentes.


Eles são colineares, estão no eixo radical dos círculos.

B
3. Relações métricas nos
quadriláteros inscritíveis –
Se os círculos são tangentes, interna ou externamente, em T, então Teoremas de Ptolomeu e Hiparco
seu eixo radical é a reta tangente comum em T. Veja que, se P pertence a
essa reta, então a potência de P em relação aos círculos é igual a PT², o Teorema de Ptolomeu
quadrado da tangente aos círculos, para os dois círculos. Seja ABCD um quadrilátero inscritível de lados a, b, c, d e diagonais p,
q. Vale a relação pq = ac + bd. [O produto das diagonais é igual à soma
P dos produtos dos lados opostos].

Teorema de Hiparco
Seja ABCD um quadrilátero inscritível, como descrito antes. Vale a
p ab + cd
T relação: = .
q ad + bc
D
C
c

Se os círculos não se cortam, então a visualização do eixo radical fica d


b
um pouco mais complicada. Sejam R e r os raios dos círculos, com R>r, e
sejam O e O’, nessa ordem, seus centros. Então o eixo radical é uma reta q p
R² − r ² A B
perpendicular a OO’ que dista do ponto médio do segmento OO’,
2 · OO '
a
do lado mais próximo do círculo de menor raio. Veja o esquema a seguir:

158 Vol. 2
Relações métricas no círculo

EXERCÍCIOS RESOLVIDOS 03 As cordas AB e CD de um círculo são perpendiculares e cortam-se


em I. Se AI = 4, IB = 6 e CI = 3, calcule o diâmetro desse círculo.
01 Em um círculo de 10 2 de diâmetro temos duas cordas medindo
2 e 10. Ache a corda do arco soma dos arcos das cordas anteriores. 04 É dado um triângulo isósceles ABC (AB = AC), inscrito em um círculo,
e um ponto M do prolongamento da base BC do triângulo. Prove que:
Solução: MA2 = AB2 – MB · MC
Sejam as cordas AB = 2 e AC = 10. Tomando o diâmetro AD =
10 2, tem-se que os triângulos ABD e ACD são retângulos, portanto, 05 Os segmentos das tangentes traçadas de P a dois círculos distintos
por Pitágoras, vale que BD = 14 e CD = 10. No quadrilátero BACD, não concêntricos são congruentes. Determine o lugar geométrico de P.
AD e BC são as diagonais [para que BC seja a corda do arco soma].
Por Ptolomeu, tem-se que: BC · 10 2 = 2 · 10 + 10 · 14, logo 06 Um círculo de centro O é circunscrito a um triângulo ABC, obtusângulo
BC = 8 2 . em C. Traça-se o círculo de diâmetro OC, que intersecta AB em D e D’.
AD = 3 e DB = 4. Encontre a medida de DC.

EXERCÍCIOS NÍVEL 1 07 O quadrilátero ABCD é inscritível em um círculo de diâmetro AD = 4


cm. Sabendo que AB = BC = 1 cm, calcule CD.
01 Seja P um ponto exterior a um círculo de centro O e raio r e tal
08 Do vértice A de um triângulo ABC, traçam-se a mediana AM e a bissetriz
que OP = r 3. Traça-se por P a secante PAB ao círculo. Se PA = r. interna AD. O círculo que passa por A, D e M corta AB em E e AC em F.
Determine AB. Sendo BE = 9 cm, o valor de FC é:

02 Em um ∆ABC, a ceviana AD encontra o círculo circunscrito em E. Se (A) 4 cm. (D) 6 cm.


AB = 5, AC = 4, BC = 6 e BD = 4. Determine DE: (B) 4,5 cm. (E) 10 cm.
(C) 9 cm.
03 Em um círculo, as cordas AB e CD são perpendiculares e cortam-se
em I. Traça-se por I um perpendicular a AD que corta o círculo em E e G e EXERCÍCIOS NÍVEL 3
AD em F (F entre I e G). Se AF = 4, FD = 9 e FG = 5, então, determine EI.
01 O ângulo entre as tangentes traçadas de P ao círculo A é o mesmo
04 Calcule a menor diagonal do quadrilátero inscritível ABCD cujos lados
ângulo formado pelas tangentes traçadas desse ponto ao círculo B.
AB, BC, CD e DA medem, respectivamente, 1, 2, 2 e 3.
Determine o lugar geométrico de P.
05 Seja ABC um triângulo tal que os lados AB, BC e AC são respectivamente
02 Prove que, se uma secante a dois círculos ortogonais passa pelo centro
proporcionais a 6, 3 e 4. Considere o círculo circunscrito ao triângulo, e seja
de um deles, os quatro pontos de interseção formam uma divisão harmônica.
P um ponto do menor arco AC. Se PA = 2a, PB = 3a, qual o valor de PC?
03 Considere três círculos 1, 2 e 3. Sejam A e B as interseções de 1 e 2;
06 ABCD é um paralelogramo tal que AC = 8 e BD = 4. A circunferência
C e D as interseções de 1 e 3; E e F as interseções de 2 e 3. Prove que os
de círculo que passa pelos pontos A, B e C intersecta o prolongamento de
segmentos AB, CD e EF são concorrentes.
BD no ponto P. O comprimento do segmento DP é:
04 Considere um ponto P no exterior de uma circunferência w. As
EXERCÍCIOS NÍVEL 2
duas retas tangentes a w partindo de P intersectam w nos pontos A e B.
Sendo M o ponto médio de AP e N a interseção de BM com w, prove que
01 Dois círculos de raios 3 e 4 são ortogonais. Calcule a distância de um PN = 2MN.
ponto P à reta que contém os centros sabendo que ele possui potência
igual a 16 em relação aos dois círculos. 05 Seja ABC um triângulo, I seu incentro e O seu circuncentro. Sendo
r o raio do círculo inscrito, R o raio do círculo circunscrito, prove que
02 Em um quadrilátero inscritível ABCD, AD = DC. Se as diagonais desse OI² = R² – 2Rr.
quadrilátero cortam-se em I e se AI = 6, CI = 4 e BI = 8, então, determine
o maior lado desse quadrilátero.

RASCUNHO

IME-ITA 159
Áreas de figuras planas A ssunto
10
Matemática V

Neste assunto, mais ferramentas métricas surgirão: como medir áreas 4. Área de um quadrilátero / Área
de superfícies. É claro que muitos problemas falam exclusivamente de
calcular áreas, mas é necessário observar e entender como usar áreas do losango
pode ser bom para calcular comprimentos, inclusive. Medir áreas será
Se um quadrilátero tem diagonais D e d, formando ângulo a, então a
muito útil para deduzir razões de segmentos, bem como a medida deles.
D⋅d
área dele é dada por S = senα.
2
1. Definição e propriedades Se um quadrilátero tem diagonais perpendiculares, então a área é
Área é uma função de medida que associa a cada subconjunto do D⋅d D⋅d
S= . A área de um losango de diagonais D,d vale S = .
plano um certo número real positivo, com as seguintes propriedades: 2 2
I. se duas figuras são congruentes, então são equivalentes [possuem a D
mesma área];
II. se duas [ou mais] figuras são disjuntas [não possuem interseção], A
então a área da união é a soma das áreas de cada figura;
a
III. se duas figuras são semelhantes em uma razão k, a razão das suas d
áreas é k².
D
B C
2. Área do retângulo / Área do
paralelogramo D⋅d
S= senα
2
Se um retângulo tem lados B, H, então a área dele é S = B · H. Em
particular, a área de um quadrado de lado L é igual a S = L².
Se um paralelogramo tem lado a e altura relativa a esse lado igual a 5. Área do trapézio
h, então a área é S = a · h. A área de um trapézio de bases B, b, e altura h é o produto da base
média pela altura.
D C H G L M K
( B + b)
S= ⋅h
h 2
S= b · h S= L2 D S= a · h
A B E F I N J
b L a 6. Área de alguns polígonos
regulares
3. Área do triângulo A ideia aqui não é dar um formulário, mas uma maneira esperta para
Valem as fórmulas abaixo: deduzir a partir do raio do círculo circunscrito ao polígono dado. Se n é o
a ⋅ ha b ⋅ hb c ⋅ hc gênero do polígono, então:
S= = =
2 2 2 R2  360° 
S = n⋅ sen   . Basta unir o centro aos vértices do polígono,
b⋅c 2  n 
S= ⋅ senα
2 quebrando-o em vários triângulos isósceles de lados R, e ângulo principal
a⋅ b⋅c igual ao externo do polígono.
S= [R = circunraio]
4R
S = p( p − a)( p − b)( p − c) [p = semiperímetro] F E
S = p ⋅ r = ( p − a)ra = ( p − b)rb = ( p − c)rc , em que r = inraio, ra = exinraio
relativo ao lado a. G D
A

a a
H C
c b
ha

A B
B D C R2
Na figura, S = 8 ⋅ sen45 = 2 R 2 2
a 2

160 Vol. 2
Áreas de figuras planas

7. Área do círculo, do setor 8. Fórmula de Bramagupta


circular e do
Para um quadrilátero de lados a,b,c,d e dois ângulos opostos a, b, vale
a seguinte fórmula de área:
segmento circular
α+β
S = ( p − a)( p − b)( p − c)( p − d ) − abcd ⋅ cos2  .
A área de um círculo de raio R é S = πR 2 .  2 
Caso o quadrilátero seja inscritível, a fórmula se reduz para
Para calcular a área de um setor circular, basta fazer uma regra de três
α S = ( p − a)( p − b)( p − c)( p − d ) .
com o ângulo que o define: S = πR 2 . [se a for medida em graus]
360º
Para calcular a área de um segmento circular, basta fazer a área 9. A ideia do “colado”
do setor que o define menos a área do triângulo formado pelos raios
Ssegm = Ssetor − S∆ . Seja um triângulo ABC e AD uma ceviana interna qualquer. Então, a
ceviana divide o triângulo em duas regiões cuja razão entre áreas vale a
razão entre as bases. Veja na figura:
A

O
S1 S2
R D
C B m D n C
S1 m
Na figura, =
S2 n

EXERCÍCIOS RESOLVIDOS

01 Em um triângulo ABC de lados a, b, c e área S, prove que 03 Considere um triângulo ABC tal que AB = 13, BC = 14, AC = 15.
b 2 + c 2 − a2
cot A = . a. Determine a altura relativa ao lado BC.
4S b. Determine o raio do círculo inscrito.
c. Dado um ponto P em seu interior que dista 2 de AB e 3 de BC,
Solução: determine a distância de P ao lado AC.
Pela lei dos cossenos, temos (*) 2 bc ⋅ cos A = b2 + c2 − a2 . Sabemos,
1 Solução:
também, que (**) bc ⋅ senA = S . Dividindo (*) por (**), segue o
2 Todos os itens podem ser resolvidos usando conceitos de áreas.
resultado.
Inicialmente, utilizamos o radical de Heron para determinar a área.
13 + 14 + 15
02 Considere um triângulo ABC e um ponto P em BC tal que P AB = α Observando que o semiperímetro é igual a p = = 21 ,
2
e P AC = β . Calcule o comprimento da ceviana AP em função dos lados temos que S = 21⋅ 8 ⋅ 7 ⋅ 6 = 84 .
AB e AC e dos ângulos dados. a ⋅ ha
a. 1Sendo ha a altura relativa a BC, temos que S = , o que nos
14 ⋅ ha 2
Solução: dá 84 = 2 ⇒ ha = 12 .
A i d e i a é u s a r á r e a s . Fa z e n d o =
AB c=
, AC b e AP = x e
b. Lembrando que S = pr, temos que 84 = 21r ⇒ r = 4 .
considerando que área (ABC) = área(ABP) + área(ACP), temos
1 1 1
que bc ⋅ sen( α + β) = cx ⋅ senα + bx ⋅ senβ , o que nos leva a c. Seja x a distância de P ao lado AC. Essas distâncias de P aos lados
2 2 2 são alturas de triângulos, portanto, a ideia é usar áreas. Usando que
bc ⋅ sen( α + β) 13 ⋅ 2 14 ⋅ 3 15 ⋅ x
x= . SABC = SABP + SBCP + SACP , temos 84 = + + , logo
c ⋅ senα + b ⋅ senβ 2 2 2
20
x= .
Observação importante: Com essa ideia, é possível obter uma fórmula 3
muito útil para o cálculo do comprimento de uma bissetriz interna em
04 Em um triângulo qualquer de inraio r e circunraio R, prove que R ≥ 2 r .
A
um triângulo (é o caso α = β = ).
2
Solução:
A A
bc ⋅ 2sen cos Para obtermos expressões para os raios em função dos lados do triângulo,
bc ⋅ senA 2 2 2 bc A
bissetriz A = = ⇒ bissetriz A = ⋅ cos . devemos usar as fórmulas de área. Sejam a, b, c os lados e S a área do
A A b + c 2
( b + c) ⋅ sen ( b + c) ⋅ sen S abc
2 2 triângulo. É sabido que r = e R = . Portanto, temos R abcp .
p 4S =
r 4S2

IME-ITA 161
Matemática V – Assunto 10

12 Divida a área de um círculo de raio R em “n” partes equivalentes por


Utilizando o radical de Heron, temos que R abcp (*). meio de círculos concêntricos de raios r1, r2 , ..., ri , ..., rn −1 . Estabeleça o
=
r 4 p( p − a)( p − b)( p − c) valor de ri em função de R, n e i.
p − a = x
 13 O centro de um círculo de raio r = π coincide com o centro de
Agora, façamos uma mudança de variáveis:  p − b = y . Somando as
p − c = z um quadrado. Calcule o lado do quadrado sabendo-se que a porção do
 quadrado exterior ao círculo possui área igual à porção do círculo exterior
a = y + z ao quadrado.

equações duas a duas, temos que  b = x + z . Substituindo em (*),
c = x + y 14 Calcule a área do quadrilátero ABCD inscritível cujos lados medem:

temos que R ( y + z )( x + z )( x + y ) Utilizando a desigualdade das AB = 2, BC = 3, CD = 4 e DA = 7.
= .
r 4 xyz 15 As cevianas internas AP e BQ de um triângulo ABC se interceptam em
 y + z ≥ 2 yz um ponto K. Sabendo-se que a área do triângulo KAB é 80 m2, do triângulo
 KPB é 10 m2 e do triângulo KAQ é 160 m2, a área do triângulo ABC, em
médias, temos  x + z ≥ 2 xz , logo R 2 yz ⋅ 2 xz ⋅ 2 xy
≥ = 2, metros quadrados, é:
 r 4 xyz
 x + y ≥ 2 xy
(A) 270.
o que finaliza a demonstração. (B) 330.
(C) 360.
(D) 420.
EXERCÍCIOS NÍVEL 1 (E) 520.

01 Dois círculos de centros A e B e raios R e 4R são tangentes 16 ABCD é um quadrilátero cujas diagonais se intersectam no ponto I.
exteriormente. Uma reta é tangente em C e D aos dois círculos. Determine Sabendo que as áreas de AIB, BIC e CID são, respectivamente, 2 cm2,
a área do quadrilátero ABCD. 4 cm2 e 6 cm2, qual a área, em cm², do quadrilátero ABCD?

(A) 12.
02 Considere um paralelogramo ABCD de lados AB = 12 e BC = 4 3. (B) 14.
Se um dos ângulos desse paralelogramo mede 60º, calcule a área do (C) 15.
losango inscrito de forma que uma diagonal seja formada pelos pontos (D) 16.
médios dos lados AD e BC. (E) 18.

03 Determine a razão entre as áreas dos quadrados inscrito e circunscrito 17 As medianas BM = 8 cm e CN = 12 cm de um triângulo ABC são
ao mesmo círculo. perpendiculares entre si. Calcule a área do triângulo ABC, em cm2:

04 Um ângulo de um losango mede 60°. Qual a razão da área desse (A) 96.
losango para a área de um quadrado de mesmo perímetro? (B) 48.
(C) 64.
05 No quadrilátero qualquer ABCD, P é médio de AD e M é médio de BC. (D) 108.
Se a área de ABCD é 18, determine a área do quadrilátero APCM. (E) 72.

06 Considere um triângulo equilátero DEF inscrito em um triângulo 18 O círculo inscrito ao triângulo retângulo ABC tangencia a hipotenusa BC
equilátero ABC de modo que os lados de DEF sejam respectivamente no ponto P. Sabendo que BP = 4 cm, CP = 9 cm, calcule a área do triângulo
perpendiculares aos lados de ABC. Determine a área do triângulo DEF. ABC.

07 Dado um triângulo de altura h, considere duas paralelas à base que o 19 Duas circunferências de raio R são tangentes entre si, e tangentes
dividam em três partes equivalentes. Calcule, em função de h, as distâncias internamente a uma outra de raio 3R. Calcule a menor das duas áreas
destas retas ao vértice do triângulo. limitadas por arcos das três circunferências.

08 Sejam ABCD um trapézio de bases AB e CD e O o ponto de interseção 20 Dado o segmento AB=x, calcule a área da lúnula determinada pelos
de suas diagonais. Prove que os triângulos ADO e BCO têm áreas iguais. arcos capazes de 30° e 45° sobre o segmento AB.

09 Calcule a área do trapézio de bases 25 e 4 e lados não paralelos 17 e 10. 21 (AFA-99) Considere um triângulo equilátero, um quadrado e um
hexágono regular, todos com o mesmo perímetro. Sejam AT, AQ e AH as
10 Em um triângulo ABC os lados são medidos por três números inteiros áreas do triângulo, do quadrado e do hexágono, respectivamente. Então,
e consecutivos. O número que mede a área é o mesmo que mede o pode-se afirmar que:
semiperímetro. Calcule as alturas do DABC.
(A) AT < AQ < AH. (C) AT < AQ e AQ > AH.
11 Um triângulo ABC tem lados AB = 13, BC = 14 e AC = 15. Um ponto (B) AT = AQ = AH. (D) AT < AQ e AQ = AH.
P no interior do triângulo dista 3 de AB e 6 de BC. Calcule a distância deste
ponto AC.

162 Vol. 2
Áreas de figuras planas

EXERCÍCIOS NÍVEL 2 12 Um dos lados de um quadrilátero simples mede 4 cm. Um lado


consecutivo a este é perpendicular e mede 6 cm. O lado oposto ao primeiro
01 Calcule os lados de um triângulo sabendo-se que suas alturas medem mede 3 2 cm e forma com o segundo um ângulo de 135°. Calcule a área
3 cm, 4 cm e 2,4 cm. do quadrilátero.

02 Em um trapézio isósceles de bases 10 e 6, as diagonais são 13 São dados dois círculos de raios 4 cm e 9 cm, tangentes externamente
perpendiculares aos lados oblíquos às bases. Determine a área entre si. Traçam-se as duas tangentes comuns externas a eles, obtendo-se
desse trapézio. quatro pontos de tangência com essas retas. Calcule a área do trapézio
cujos vértices são esses pontos.
03 Considere duas cordas de um semicírculo de raio 6 que determinam
neste semicírculo arcos de 60º e 120º. Calcule a área da figura limitada 14 Dados dois círculos de raios 4 cm e 6 cm e cuja distância, entre
por essas cordas e pelo semicírculo. os centros, é de 10 2 cm, determine a área do triângulo formado por
uma tangente comum exterior aos dois círculos e pelas duas tangentes
04 Calcule em função das bases a e b de um trapézio, o comprimento comuns interiores.
do segmento das paralelas às bases que divide o trapézio em dois
outros equivalentes. 15 Um triângulo é dividido em 6 triângulos menores por cevianas
concorrentes em um ponto. São S1 , S2 , S3 , S4 , S5 , S6 as áreas desses
05 Calcule a razão entre as áreas dos triângulos AMN e ABC, na triângulos menores, no sentido horário. Prove que S1 S3 S5 = S2 S4 S6.
figura abaixo.
16 Sejam ABCD um trapézio de bases AB e CD e O a interseção de suas
A
diagonais. Se as áreas dos triângulos ABO e CDO são iguais a S1 e S2,
y
N respectivamente, determine a área do trapézio.

M 17 Considere um triângulo de área S, inraio r e raios dos círculos


4y
ex-inscritos iguais a ra, rb, rc.

1 1 1 1
B C a. Prove que = + + .
r ra rb rc
x 5x b. Prove que S = rra rb rc .
06 No triângulo ABC, AB = c, BC = a, CA = b. Uma reta corta AB em
F, BC em D e o prolongamento de AC em E. O triângulo tem área igual a 18 Observe a figura a seguir:
a b H
36. Se CD = e CE = , calcule a área do triângulo BDF.
3 3 I
D E C
07 Um triângulo equilátero ABC tem 60 cm de perímetro. Prolonga-se a G
base BC e sobre esse prolongamento toma-se CS = 12 cm. Une-se o
ponto S ao ponto médio (M) do lado AB. A interseção de AC e MS é G.
Calcule a área do quadrilátero BCGM. F

08 Seja ABC um triângulo de área 1. Sejam D, E e F pontos em seu interior A B


tais que:
– D é ponto médio de CE; A figura acima apresenta um quadrado ABCD de lado 2. Sabe-se que E
– E é ponto médio de BF; e F são os pontos médios dos lados DC e CB, respectivamente. Além
– F é ponto médio de AD. disso, EFGH também é um quadrado e I está sobre o lado GH, de modo
GH
Determine a área do triângulo DEF. que GI = . Qual é a área do triângulo BCI?
4
7
09 Um triângulo acutângulo ABC está inscrito em um círculo. Sendo AM, (A)
BN e CP diâmetros, prove que a área do hexágono APBMCN é o dobro da 8
área do triângulo ABC. 6
(B)
7
10 Se um dos lados não paralelos de um trapézio mede 12 cm e dista 6 5
(C)
cm do meio do outro lado não paralelo, determine a área do trapézio. 6
4
11 Dados 3 pontos consecutivos A, B e C sobre uma reta r, traçam-se (D)
5
três semicírculos de diâmetros AB, AC, BC do mesmo lado da reta.
3
Determine a área do triângulo formado pelos pontos de máxima elevação (E)
dos três semicírculos , sabendo-se que o segmento BF (F sobre o maior 4
semicírculo), perpendicular a reta r, mede 6 cm.

IME-ITA 163
Matemática V – Assunto 10

19 EXERCÍCIOS NÍVEL 3
E
01 Dado um triângulo ABC, um ponto W em seu interior é chamado de
ponto de Brocard se W AB = W BC  = WCA = α . Nesse caso, prove que
A 30º B cot α = cot A + cot B + cot C (e veja que só há 2 pontos de Brocard para
O F
cada triângulo).

02 Considere o quadrilátero convexo ABCD tal que as retas BC e AD se


cruzem em E. Sendo G e H médios de AC e BD, prove que a área de EGH
é um quarto da área de ABCD.

D C 03 Sobre o lado AB de um quadrilátero ABCD tomam-se os pontos A’ e


Na figura acima, ABCD é um quadrado de área 104 e o ponto O é o centro B’, e sobre o lado CD os pontos C’ e D’, de forma que AA’=BB’= p.AB,
do semicírculo de diâmetro AB. A área do triângulo AEF é: e CC’=DD’=p.CD, com p<0,5. Prove que SA ' B ' C ' D ' .
= 1 − 2 p.
SABCD
(A) 2(3 3 + 3).
(B) 6( 4 3 − 3). 04 Sejam AD, BE e CF cevianas bissetrizes internas em um triângulo
[ DEF ] 2 abc
(C) 5( 4 3 − 6) . ABC. Prove que = , sendo a, b, c os lados
[ ABC] ( a + b)( b + c)( c + a)
(D) 3( 4 3 − 3).
do triângulo ABC.
(E) 8( 4 3 − 3).

RASCUNHO

164 Vol. 2
Gabarito V olume
2
Matemática I 03 p = 0 ou p = 30. 12 11
04 Letra E. 13 Letra E.
05 Letra D. 14 Letra D.
Assunto 4
06 Letra B. 15 Letra D.
Exercícios Nível 1 07 Letra C. 16 Letra E.
01 Letra C. 39 Letra E. 08 Letra C. 17 m > 4/3
02 Letra E. 40 Letra C. 09 11/9 < m < 3 18 m = – 4
p
03 Letra D. 41 ≥ 1 + 2
h 19 –1/2 < p < ½
04 Letra E. 42 Letra C. −2 − 15 −2 + 15
20 m < ou < m<1
05 Letra A. 43 S = {–9/2, 3} 2 2
06 Letra A. 44 todo x real 21 1760 m.
07 x2 – 6x + 4 = 0 45 2 < x < 3 22 S = {1}
08 n = 4m + 1 46 1 ≤ x < 2 23 λ < –1 ou λ > 1
09 p = 1, q qualquer 47 Letra A.
24 x1 = 0; x 2 = − 3 21 a; x3 = 3 21 a
10 – 48 x ≥ 1 14 14
3 1
11 m = − , n = − 49 Letra E. 25 x1 = – 2; x2 = –1; x3 = 7
10 5
26 Letra C.
12 Letra D. 50 Letra E. 27 Letra C.
13 Letra B. 51 Letra D. 28 Letra A.
14 Letra D. 52 Letra E. 29
15 Letra A. 53 Letra D. a. 3;
16 Letra A. 54 F – F – F – V
1, se m > 0
17 Letra D. 55 S = {–7, 7} 
18 Letra C. 56 x = – 2 ou x = 6 2, se − 1 < m ≤ 0
 4
19 Letra B. 57 Letra B. 
b. f ( m ) =  1
20 Letra D. 58 Letra C. 1, se m = − 4
21 Letra B. 59 18/5 
22 Letra C. 60 x|x|  1
0 , se m < − 4
23 Letra C. 61 Letra E.
24 Letra C. 62 Letra B.
30
25 (–∞, –2) ∪ (2, ∞) 63 Letra D.
2 3 a. ∅
26 Letra D. 64 < x < b. ( 21; 28 ]
3 2
c. [0, +∞)
27 m = –1 65 x < 2 13
d. ( − ∞; − ] ∪ [3; +∞)
28 –3 < m < 73 66 –1 ≤ x ≤ 4 6
1
29 2 < m < 4 67 e. R
2 4n − 1
31 x1 = 0; x 2 = n
5 4 +1
30 Letra E. 68 32 S = {4}
6 33
31 Letra D. 69 3 a.
2 y
32 Letra E. 70 1. m=1
33 Letra A. 71 Letra B.
34 Letra E. 72 Letra A.
35 Letra D. 73 Letra B. 2 1
m=
36 Letra D. 74 Letra A. 4
37 Letra C. 75 Letra A. 1
38 Letra B. 76 Letra D.
–2 –1 0 1 2 x
Exercícios Nível 2
01 Letra B. 10 m ≤ 7
02 k = 0 ou k = 6. 11 110

IME-ITA 365
Gabarito

3 5  −1 ± 3 i 
b. − ; 0 e
2 2 b. 
 ;
 2 
c. m = 0 → 2 raízes distintas 07 2
1
0 < m < → 4 raízes distintas 08 i
2 09 x= 1 – 3i; y = – 1 + 2i
1 10 {(i, 1 – i)}
m= → 3 raízes distintas
2 11 Letra C.
1 12 Letra C.
m > 2 raízes distintas
2 13 Letra B.
14
34 a = 2, b = – 6, c = – 8
 −6 + 57 
35 Letra B. a.  + 2i  ;
36  3 
 2
a.  0, 
 5  7 
b. − + 4i  ;
b. ( −∞; −1 − 3 ] ∪ [−1 + 3 ; +∞)  6 
c. [ 3 ; 2) c. {0,i, − i}
2 15 Letra A.
d. (– 5; – 2) ∪ (2; 3) ∪ (3; 5) 16 Letra A.
e. (– ∞; 3) 17 ± (2 + i)
f. (– ∞; – 4) ∪ (– 2; 1) ∪ (3; +∞) 18 Letra B.
g. (0; +∞) 19 Letra A.
 7 5  20 Letra A.
h.  −∞,  ∪  , +∞ 
 4 2  21 Letra D.
22 Letra B.
37 –
23 215 i
Exercícios Nível 3
24 Use que (cisα)3=cis(3α).
01 – 13 –
25
02 – 14 –
a. 135°;
03 − 1 ≤ x < 45 , x ≠ 0 15 x = 0, y = 1
2 8 b. 135°;
26 Letra B.
04 4 2 − 2 16 6
27 Letra A.
05 –1 < m ≤ 2 17 {(2,1), (1,2}
28 Letra B.
06 – 18 –
29 Letra A.
07 – 19 –
30 Letra C.
08 – 20 x = 1996
31 Letra D.
{
09 1 ± 5 ; 2 ± 6 } 21 1 32
10 – 22 – a. 1; c. –;
11 – 23 96
2
12 – b. (1 + i ); d. i;
2
24
33 Letra A.
a. c + d – a – b;
34
b. c – a;
a. Circunferência de raio 1 e centro na origem;
25 1001000
b. interior da circunferência (disco) de raio 1 e centro em –i.
 1 ± 3 i 
35 −1; .
Matemática II  2 

Assunto 4 36 
−1 + i −3 − 3 i 
, .
Exercícios Nível 1  2 2 
01 37 Letra C.
1 3
a. =x = ,y x = −2, y = 8
; b. 38 Letra E.
02 2 4 4
39 Letra D.
03 – 40 Letra B.
04 z + z ∈R
05 Letra E. Exercícios Nível 2
06 01 Letra B.
a.{2 –i, –i};

366 Vol. 2
Gabarito

3z − 1 Exercícios Nível 3
02 f (z) = 01 Letra B.
z +1 2014
02 (1 − 2 − 2−2 )(1 + i )
03 -
03 -
04 -
04 -
05 Letra D. 05 -
06 Letra B. 1
07 Letra B. 06 máx = 2, mín =
07 - 2
08
a. -; 08 Interprete geometricamente e use a desigualdade traiangular.
3
b. “Num paralelogramo, a soma dos quadrados dos comprimentos das  1 3 1  1
09 Use que  z +  = z + 3 + 3  z +  e a desigualdade triangular.
diagonais é igual à soma dos quadrados dos lados.”;  z z  z
1 π
c. ma = 2(b2 + c2 ) − a 2 ; 10 x = y = x = (analise a soma dos cis)
2 2
11
09 Eleve ao quarado e use que |z|2 = z z (n+ 1)r (n+ 1)r
sen sen
10 Letra B. 2  nr  2 sen  a + nr  ;
a. S = sen  a +  b. C = 
11 6 + 8i e 6 + 17i;
sen
r  2  sen
r  2 
12 Letra D. 2 2
13 Use várias vezes que |z|2 = z z .
12
14 Letra B.
x  n+2 x  n+2
15 a. 2n cos n cos  2n cos n sen 
 x ; b.  x;
a. Octógono não regular; b. 1 + 3 u.a. 2  2  2  2 
16 Letra B. 13 n + 1, para n ≥ 2. Para n = 0 e n = 1, as respostas são 0 e 1,
1− i respectivamente.
17 Interior e bordo da circunferência de raio 2 e centro em ;
2 14 Soma de P.G.
18 mín = 5 − 1, máx = 5 + 1;
19 12 + 16i 5 +1
15 cos36° =
20 Letra A. 4
21 Letra C.  kπ 
22 Letra C. 16 − i cot ; k = 1, 2,..., n − 1
 n 
23 Letra A.
24 17 -
a. 1; 18 -
b. 2; 19 200 3m
n≠ 20 “Saindo da pedra, ande em direção à caverna até a metade do caminho
c. 2 cos ;
3 e vire 90° à direita; então, ande a mesma quantidade de passos andada
anteriormente.”
25 n deve ser múltiplo de 3.
26
a. cis(±α); Matemática III
b -;
27 θ – 2π Assunto 3
28 {0;±1;±i} Exercícios Nível 1
29 0 1
 iθ − iθ  iθ 01 .
30 e i θ ± 1 =  e 2 ± e 2  e 2 6
31 Letra A.   02 Letra C.
32 Letra D.
33 Letra E.
34 Letra B. 03 8 .
9
35 Letra A.
36 Letra C. 04 0.
37 Letra C. 05 Letra B.
38 Letra D.
39 Letra D. 4 1
06 e .
40 Letra A. 5 5
0, se ξ ≠ 1
41  07 Letra A.
 n, se ξ = 1
3 1
08 P(wA) = 18 , P(wB) = 6 , P(wC) = , P(wD) = .
28 28 28 28

IME-ITA 367
Gabarito

09 — 10
10 Letra C.  2n   n  m  n − 1  m−2
11 Letra D.   − ⋅2 n⋅ ⋅2
m m m − 2
12 Letra C. (A)     (B) 
 2n   2n 
   
13 3 .  m  m
31 11
12 Letra C.
14 89 . 25
144 13
216
15 21 .
36 14 1 − π
4
16 Letra B.
15 Letra C.
8
17 . 1
20 16 .
2
18 1 .
35 17 Letra D.
19 Letra E. C
18 n - 2, p - 2
 n Cn - 1, p - 1
20   ÷ ( 2n )
k 1
19
7
1
21 .
55 Exercícios Nível 3
2
01
63 3
22 .
200 1
02
3
18 1
23 (> ).
35 2 03 2
3
24 Letra C.
1
04
4
25 14 . 4
15 . (0,1)7 . (0,9)3. 05
26 120 7
06 33.
27 Letra C.
95
Exercícios Nível 2 07
01 Letra B. 256
7 1
02 . 08
18 84
03 Assunto 4
18 Exercícios Nível 1
(A) 10 . (B) .
81 77 01 50.
02 Letra D.
 k − 1 03 Letra B.
04  2 
04 Letra B.
120 05 0.
06 Letra A.
05 n( n − 1)( n − 2)...(pn − p + 2)( p − 1)
n 07 1
220
06 0,0795. 08 n · 2n–1
07 15%. 09 3n
08 0,6561. n
289 10 3 + 1
09 2
480
11 12.
12
13 6 e 7.

368 Vol. 2
Gabarito

14 Letra B. b. –
15 –1. c. Cnn+ m
16 Letra B. 04 –
17 32. 05 –
18 210 06 –
19 Letra D.
20 Letra B. Matemática IV
21 Letra A.
22 Letra B. Assunto 2
23
Exercícios Nível 1
a. 969; 01 M1(0, 28) e M2(0, – 2).
b. 1360.
02 8 3 u.a.
03 (5,2) ou (2,2).
Exercícios Nível 2 04 13.
01 Cnp+ p 05
02 1756950. a. M(1, 3). b. N(4, – 3).
06 D(– 3, 1).
n
03 ( x + a) − ( x − a)
n
07 13 e 15.
9
2 08  , 1
04 2 
a. (1 + x)n; 09 12 u.a.
b. n · x · (1 + x)n–1; 10 Letra A.
c. n · 2 n–1 11 Letra A.
12 Letra d.
2n +1 − 1
05 13 Letra A.
n+1 14 Letra D.
06 n + 1)( 2 n + 1)
n( 15 P1(1, 0) e P2(6, 0).
6 16 Letra A.
07 Letra D. 17 Letra A.
08 Letra D. 18 x + y + 1 = 0.
19 Letra C.
09 n = 2009, m = 3. 20 Letra A.
10 Letra C. 21 Letra E.
11 24. 22 5.
12 333. 23 Letra D.
13 – 24 Letra B.
14 – 25 Letra A.
15 – 26 Letra D.
16 – 27 Letra C.
17 Letra A. 28 Letra b.
18 – 29 Letra D.
Exercícios Nível 2
1 01 (b, a).
19 C65
16

316 02 Letra C.
2
03 17 u.a.
20 ( n + 1)n(9 n + 5 n − 2)
12 04 (1, – 3); (– 2, 5); (5, – 9) e (8, – 17).
05 Letra E.
21 0. 06 C1(– 1, 4) ou C2(25/7, – 36/7).
22 3420. 07 C1(– 2, 12). D1(–5, 16) ou C2(– 2, 2/3). D2(– 5, 14/3).
23 780. 08 C1(1, – 1) ou C2(– 2, – 10).
24 3n. 09 Letra C.
10 Q(11, – 11).
Exercícios Nível 3 11 1/4.
01 Maior.
02 12  − 2 , 43  .
a. ( −1) p Cnp−1  17 17 
13 Letra A.
b. 14 Letra A.
03 15 Letra D.
a. – 16 Letra B.
17 (6, – 6).

IME-ITA 369
Gabarito

18
a. – b. arctan 3/2.
19 Letra B. 06 6 a 13 –
5
20 3x – y + 9 = 0 e 3x + y + 9 = 0.
07 2:1. 14 Letra C.
21 Letra D.
22 3x – 4y + 20 = 0 e 4x + 3y –15 = 0. Assunto 7
23 Letra D.
Exercícios Nível 1
24 Letra C. (I e III são verdadeiras).
01 2 a 10 .
02 9.6.
c1 − c2
25 d = . 5a
a2 + b 2 03 d =
26 – 04 8
27 32x – 4y + 5 = 0. a. AC = 15 cm, AH = 12 cm, BH = 16 cm, CH = 9 cm.
28 x – 5 = 0. b. HN = 7.2 cm, AN = 9.6 cm, NC = 5.4 cm.
29 14 2 / 3 . 80 60
c. cm e cm .
30 P = (2, – 1). 7 7
05 Letra A.
Exercícios Nível 3 06 –
01 Letra B. 07 2 dm.
02 Letra B. 08 –
03 3x + 4y – 1 = 0 e 7x + 24y – 61 = 0.
2 2 2 09 1
04 x 3 + y 3 = c 3 8
10 33 .
(
05 (3 + 2 13 / (1 − 2 13 ),(5 + 4 13 ) / (1 − 2 13 ) )
06 29x – 2y + 33 = 0. 11 A situação é impossível, pois x = 1 gera contradição por desigualdade
07 2x + 9y – 65 = 0; 6x – 7y – 25 = 0 e 18x + 13y – 41 = 0. triangular.
08 (4, 3); (9, – 2); (– 12, 1). 2 b 2 + 2c 2 − a 2
09 reta x – 3y + 5 = 0 12 ma = .
2
10 –
11 P deve ser o centro do quadrado. 13 ha = 2 p( p − a)( p − b)( p − c) .
a
12 S = ( a + b)( b + c) 3 . 2
12 14 lα = bcp( p − a) .
13 – b+c
2 2 2
14 – 15 a + b + c .
15 – 2 abc
16 –
17 –
Matemática V 18 Letra A.
19 Letra B.
Assunto 6 20 Letra C.
Exercícios Nível 1 21 Letra C.
01 12 u 08 24. 22 –
7 23 Letra B.
02 x = 2. 09 15 cm e 20 cm.
03 40 cm. 10 4.8. Exercícios Nível 2
aq + bp 01 8 2 . 11 45°.
04 1:5. 11 MN = .
q+p
05 20 cm. 12 AE = 2 cm ou AE = 30 cm. 02 2 3 . 12 2 a2 + 2 b2 − c2 .
06 1:2. 13 4 cm. 03
a
2
(
2 − 1 . ) 13 –
07 8.
04 2 5 . 14 –
Exercícios Nível 2 05 – 15 7 .
−3 + 201 3
01 – 08 AD =
2 06 – R
16 .
02 70 cm. 09 Letra C. 4
03 k². 10 3. 07 – 17 Letra C.
04 5:2. 11 2.4. 08 – 18 Letra D.
05 4 cm e 2 cm. 12 Letra B. 09 – 19 Letra C.
10 145 .

370 Vol. 2
Gabarito

Exercícios Nível 3 Assunto 10


01 Letra B.
Exercícios Nível 1
02 r = 4. i
01 10R². 12 ri = R
n
Assunto 8
02 36. 13 p.
Exercícios Nível 1
03 1:2. 14 2 30
01 3 06 –
2 04 3 15 Letra C.
2
02 24. 07 AB + AC = b + c 05 9. 16 Letra C.
BC a
03 1:3. 08 1:1. 06 1 SABC 17 Letra C.
04 3:4. 09 – 3
05 – 10 – 07 h 3 e h 6 18 36 cm².
3 3
Exercícios Nível 2 08 – 19 5 π R 2 − R 2 3
01 – 04 – 6
02 – 05 – 09 116. 20 –
03 – 10 2.4, 3 e 4. 21 Letra A.
11 3.
Assunto 9
Exercícios Nível 1 Exercícios Nível 2
01 3 cm, 4 cm, 5 cm. 11 9 cm².
01 r. 04 7 . 02 32. 12 27 cm².
02 8 11 03 ≠ R
2
05 a. 13 1728 cm2
11 6 13
03 1.2. 06 6.
a2 + b 2
04 . 14 24 cm².
Exercícios Nível 2 2
05 3:25. 15 –
01 21 ⋅ 76 06 6 2
( ).
2

02 2 33 07 2 3 06 8. 16 S1 + S2

03 5 5 08 3.5. 07 700 3 17 –
04 – 09 Letra C. 11
1
08 . 18 Letra E.
05 Eixo radical dos círculos.
7
Exercícios Nível 3 09 – 19 Letra D.
01 Círculo de Apolônio sobre o segmento formado pelos centros, e de 10 72 cm².
razão igual à razão dos raios.
02 –
03 –
04 –
05 –

ANOTAÇÕES

____________________________________________________________________________________________________________________

____________________________________________________________________________________________________________________

____________________________________________________________________________________________________________________

____________________________________________________________________________________________________________________

____________________________________________________________________________________________________________________

____________________________________________________________________________________________________________________

____________________________________________________________________________________________________________________

____________________________________________________________________________________________________________________

IME-ITA 371
Exponencial e logaritmo A ssunto
5
Matemática I

1. Função exponencial 5
4,5

1.1 Definição 4

Na apostila de Álgebra Básica, foram definidas potências de expoentes 3,5


naturais, inteiros e racionais. Além disso, também foi vista uma maneira 3
natural de se definir uma potência de expoente irracional (aproximando 2,5
cada irracional por uma sequência de racionais). Sendo assim, para cada
2
a ≠ 1 positivo, fica bem definida uma função f:  → + (+ é o conjunto
1,5
dos reais positivos) dada por f(x) = ax (consideramos a ≠ 1, pois se a = 1, 0
teríamos uma função bastante trivial). Tal função herda propriedades vistas 1
na apostila de Álgebra Básica. Temos para x, y ∈ : 0,5
0
ax
I. ax ⋅ ay = ax + y e y = a x − y –2,5 –2 –1,5 –1 –0,5
0
0,5 1 1,5 2 2,5 3 3,5 4 4,5 5 5,5 6 6,5
II. (ax)y = axy a –0,5

Veremos agora dois exemplos de funções exponenciais e seus


gráficos:
1.2 Crescimento e decrescimento
I. f1(x) = 2x: Nos exemplos vistos acima, pudemos ter alguma ideia de como as
funções exponenciais se comportam com relação ao crescimento. De
fato, a intuição prevalece.
x –2 –1 0 1 2 Teorema 1 (Crescimento e decrescimento)
I. Se a > 1, a função f(x) = ax é crescente, ou seja, f(x) > f(y) ⇔ x > y.
1 1 II. Se a < 1, a função f(x) = ax é decrescente, ou seja, f(x) > f(y) ⇔ x < y.
f1(x) = 2x 1 2 4
4 2
Demonstração:
Faremos I e então II seguirá de forma análoga.
4,5
Veja que f(x) > f(y) ⇔ ax > ay ⇔ ax – y > 1. Provaremos então que
4
a > 1 ⇔ r > 0 e isso implicará f(x) > f(y) ⇔ x – y > 0 ⇔ x > y.
r

3,5
Para demonstrar que ar > 1 ⇔ r > 0, faremos isso para r inteiro,
3 racional e o caso r irracional seguirá pela teoria de limites de sequências
2,5 (que está fora do nosso escopo).
2
Caso 1:
1,5
1
r = n ∈ :
Queremos provar que an > 1 ⇔ n > 0. Temos duas partes a
0,5
demonstrar:
0
–4,5 –4 –3,5 –3 –2,5 –2 –1,5 –1 –0,5 0 0,5 1 1,5 2 2,5 3
Parte 1: n > 0 ⇒ an > 1:
n
x Como a = a ⋅ a… ⋅ a é o produto de n termos maiores que 1, segue
 1 
II. f2 ( x ) =   : n vezes
2 que an > 1 (poderíamos usar indução para sermos mais formais, mas
não é necessário).
x –2 –1 0 1 2
Parte 2: an > 1 ⇒ n > 0:
x 1 1
 1 Faremos aqui a contrapositiva, ou seja, provaremos que se n ≤ 0,
f2 ( x ) =   4 2 1 2 4 1
2 n
então an ≤ 1. Para isso, veja que a = − n e que, como – n ≤ 0, temos
a
1
que a− n ≥ 1 ⇒ − n ≤ 1 , como queríamos.
a
Caso 2:

IME-ITA 89
Matemática I – Assunto 5

p Teorema 2 (Igualdade dos expoentes)


r= ∈  , p, q, ∈ :
q Se a ≠ 1 e ax = ay, então temos que x = y.
Mais uma vez, temos duas partes a demonstrar: Demonstração:
p
p A demonstração é imediata a partir do fato de que a função exponencial
q
Parte 1: >0⇒ a >1 é monótona (crescente ou decrescente) e, portanto, injetiva.
q
Para vermos algumas técnicas, vejamos exemplos de equações
Podemos supor que p, q são inteiros positivos.
exponenciais:
q 1
 1
Como q é inteiro positivo e  a q  = a > 1 , segue que a q > 1 . Logo, Ex. 1: Resolva a equação 32x + 1 ⋅ 27x – 1 = 92x + 5.
p  
 1 p  
 a  = a > 1, como queríamos. (Usamos algumas vezes o resultado
q q Solução: A ideia é escrever todas as potências em uma mesma
  base. Como 9 = 3 2 e 27 = 3 3, escreveremos tudo na base 3:
  32x + 1 ⋅ (33)x – 1 = (33)2x + 5 ⇔ 32x + 1 ⋅ 33x – 3 = 34x + 10. Assim, temos que
do caso 1). 35x – 2 = 34x +10 ⇔ 5x – 2 = 4x + 10 ⇔ x = 12.
p
p Logo, o conjunto solução é {12}.
Parte 2: a q > 1 ⇒ >0:
q
p Ex. 2: Resolva a equação 5x – 2 + 5x + 1 = 505.
Provaremos a contrapositiva, ou seja, provaremos que se ″ 0 , então
p q
a q ″ 1 . Podemos supor que p ≤ 0 e q > 0. Assim, pelo mesmo argumento Solução: Aqui podemos colocar no lado esquerdo 5x – 2 em evidência e obter
1 p 5x – 2(1 – 52 + 53) = 505 ⇔ 5x – 2 ⋅ 101 = 505 ⇔ 5x – 2 = 51 ⇔ x – 2 = 1
q
 1 p
⇔ x = 3.
da parte 1, segue que a > 1 e, usando o caso 1, como  a q  = a q e
p   Logo, o conjunto solução é {3}.
 
p ≤ 0, segue que a q ″ 1 .
Ex. 3: Sendo x real não negativo resolva a equação xx = 1.
2
– 4x + 3
Isso conclui a prova nos dois casos.

1.3 Gráfico Solução: Inicialmente, devemos verificar se x = 0 e x = 1 são soluções da


equação dada. Testando tais valores, vemos que x = 1 satisfaz a equação,
Vimos dois exemplos de gráficos em 1.1 e a partir de 1.2, podemos pois 10 = 1. Supondo agora x ≠ 0, x ≠ 1, temos que xx – 4x + 3 = x0 ⇔ x – 4x
2 2

concluir as seguintes propriedades: + 3 = 0. Daqui, concluímos que x = 3, já que estamos supondo x ≠ 1.


I. O gráfico está sempre acima do eixo x, pois ax > 0 para todo x real. Logo, o conjunto solução é S = {1,3}.
II. O gráfico corta o eixo y no ponto (0,1).
III. O gráfico é de uma das formas a seguir:
Ex. 4: Resolva a equação 4x – 6 ⋅ 2x + 8 = 0.

Solução: Aqui podemos fazer uma troca de variáveis bastante útil:


2x = t. Assim, a equação dada se torna uma equação do segundo grau:
t 2 − 6t + 8 ⇔ t = 2 ∨ t = 4 . Daí, temos que 2x = 21 ou 2x = 22 e então o
conjunto solução é S = {1,2}.

1.5 Inequações exponenciais


Muitas inequações exponenciais se reduzem à seguinte inequação mais
simples (via técnicas algébricas adquiridas na apostila de álgebra básica):
(a > 1) Teorema 3 (Desigualdade entre os expoentes)
I. Se a > 1 e ax > ay, então x > y.
II. Se a < 1 e ax > ay, então x < y.

Demonstração: A demonstração é imediata a partir do fato de que a


função exponencial é crescente se a > 1 e decrescente se a < 1.
Para vermos algumas técnicas, vejamos exemplos de inequações
exponenciais:

1
( )
2 x −7
x
Ex. 1: Resolva a inequação 3 > .
27
(a < 1)
Solução: Coloquemos tudo na base 3: 3x(2x – 7) > 3–3. Como 3 > 1
1.4 Equações exponenciais (sempre tenha muito cuidado nesta passagem!), temos que x(2x – 7) > – 3
Muitas equações exponenciais se reduzem à seguinte equação mais  1
simples (via técnicas algébricas adquiridas na apostila de álgebra básica): ⇔ 2x2 – 7x + 3> 0 e então o conjunto solução é S =  −∞,  ∪ ( 3, +∞ ) .
 2

90 Vol. 3
Exponencial e logaritmo

Ex. 2: Resolva a inequação x2x2 – 9x + 4 < 1 nos reais não negativos.


f3(x) = log2x 2 1 0 –1 –2
Solução: Primeiramente, devemos verificar se x = 0 e x = 1 são soluções.
Após uma simples verificação, vemos que apenas x = 0 é solução. Agora,
dividiremos o problema em dois casos: 3,5

Caso 1: 3

0 < x < 1: 2,5


A
Podemos reescrever a inequação como x2x2 – 9x + 4 < x0. Agora, devemos 2
ficar MUITO atentos, pois 0 < x < 1. Dessa forma, temos que 2x2 – 9x + 4 > 0 1,5
1 B
(MUITA atenção aqui mais uma vez!) e então x < ou x > 4. Fazendo a 1
2 1 0,5
interseção com 0 < x < 1, temos que a solução deste caso é 0 < x < . 0 C
2
0 0,5 1 1,5 2 2,5 3 3,5 4 4,5 5 5,5 6 6,5 7 7,5 8
Caso 2: –0,5

x > 1: –1 D

Mais uma vez, podemos reescrever a inequação x <x.


2x2 – 9x + 4 0 –1,5
Entretanto, neste caso, a base da potência é maior do que 1 e então –2 E
1
podemos concluir que 2x2 – 9x + 4 < 0, o que nos dá < x < 4 . Fazendo –2,5
2
a interseção com x > 1, temos que a solução deste caso é 1 < x < 4.
Juntando a solução x = 0 e os casos 1 e 2, temos que o conjunto II. f4 ( x ) = log 1 x :
 1 2
solução é S = 0,  ∪ (1, 4 ) .
 2 1 1
x 4 2 1
2. Função logarítmica 2 4

f4 ( x ) = log 1 x
2.1 Contexto histórico –2 –1 0 1 2
2
Os logaritmos foram introduzidos no começo do século XVII por
John Napier basicamente para simplificar contas de multiplicação
3,5
(as contas de multiplicação eram transformadas em contas de adição,
substancialmente mais simples). Com o passar do tempo, os logaritmos 3
foram rapidamente usados por navegadores, cientistas, engenheiros e 2,5
vários outros para simplificar suas contas. 2 E

2.2 Definição 1,5


1 D
A ideia dos logaritmos é ser a operação reversa da potenciação.
Por exemplo, como 2 elevado à potência 3 é 8, segue que o logaritmo de 0,5
0 C
8 na base 2 é 3, e escrevemos log2 8= 3.
0 0,5 1 1,5 2 2,5 3 3,5 4 4,5 5 5,5 6 6,5 7 7,5 8
Formalmente, a função logarítmica g:  → , definida por
+
–0,5
g(x) = logax(a > 0, a ≠ 1) , é a inversa da função exponencial f:  → +, –1 B
dada por f(x) = ax. O real a é chamado de base do logaritmo e x é dito –1,5
logaritmando.
–2 A
Em palavras mais simples, temos o seguinte: logax = y ⇔ ay = x.
–2,5
Exs.:
I. log5 3125 = 5, pois 3125 = 55.
−3 CUIDADO!
 1
II. log 1 8 = −3 , pois 8 =   . Lembre-se sempre de que a base do logaritmo é um real positivo
2 2
diferente de 1 e o logaritmando é sempre positivo.
1 1
III. log3 = −4 , pois = 3−4 .
81 81
2.2 Crescimento e decrescimento
Veremos dois exemplos de funções logarítmicas e seus gráficos:
Nos exemplos vistos, pudemos ter alguma ideia de como as funções
I. f3(x) = log2x:
logarítmicas se comportam com relação ao crescimento. De fato, a intuição
mais uma vez prevalece.
1 1
x 4 2 1 Teorema 4 (Crescimento e decrescimento)
2 4
I. Se a > 1, a função g(x) = logax é crescente, ou seja, g(x) > g(y) ⇔
x > y.

IME-ITA 91
Matemática I – Assunto 5

II. Se a < 1, a função g(x) = logax é decrescente, ou seja, g(x) > g(y) b
⇔ x < y. III. (Logaritmo da divisão) log a = log a b − log a c
c
Demonstração: Demonstração:
Segue do fato de o logaritmo ser a inversa da função exponencial. Fica Análogo a II.
como exercício demonstrar o seguinte resultado:
“Sejam f, g duas funções tais que g é a inversa de f. Então, f é crescente IV. (“Regra do peteleco” – este é apenas um nome para facilitar a
(decrescente) se, e somente se, g é crescente (decrescente).” memorização) logaxr = rlogax, para r ∈ .

2.3 Gráfico Demonstração:


Vimos dois exemplos de gráficos em 2.1 e, a partir de 2.2, podemos Fazendo logaxr = k e logax = l, temos que x = al ⇒ xr = (al)r = alr.
concluir as seguintes propriedades: Como ak = xr, segue que ak = alr ⇒ k =lr e então logaxr = rlogax.
I. O gráfico está todo à direita do eixo y, pois a função logarítmica só
está definida nos reais positivos. V. (“Regra do peteleco invertido” – este é apenas um nome para facilitar
II. O gráfico corta o eixo x no ponto (1,0) 1
a memorização) log ar x = log a x , se r ∈ *.
III. É simétrico ao gráfico da função f(x) = ax com relação à reta y = x r
(já que o logaritmo é a inversa da exponencial).
Demonstração:
IV. O gráfico é de uma das formas a seguir:
Análogo a IV.

VI. (“Mudança de base”) Em muitos problemas, queremos mudar as bases


dos logaritmos para uma base mais conveniente. Para isso, usamos
log x
o seguinte: log a x = b .
log b a
Demonstração:
Sejam logax = k, logbx = l, logba = m. Então x = ak, x = bl, a = bm.
Daí, x = ak = (bm)k = bmk e então bmk = bl ⇒ mk = l. Com isso, segue
log x
que log a x = b .
log b a

Obs.: Uma maneira muito útil de se usar a mudança de base é


(a > 1) 1
log a b = .
log b a
Comentário: Essa é uma das propriedades mais úteis de logaritmos.

2.5 Equações logarítmicas


Muitas equações logarítmicas se reduzem à seguinte equação mais
simples (usando as propriedades vistas e as técnicas adquiridas na apostila
de álgebra básica). Devemos sempre nos lembrar das restrições sobre o
logaritmando e a base e testar as soluções encontradas no fim.
Teorema 5 (Igualdade dos expoentes)
Se logax = logay, então x = y.

Demonstração:
(a < 1) A demonstração é imediata a partir do fato de que a função logarítmica
é monótona (crescente ou decrescente) e, portanto, injetiva.
2.4 Propriedades Para vermos algumas técnicas, vejamos exemplos de equações
I. (Definição) alogab = b logarítmicas.
Demonstração:
Fazendo logab = k, temos que ak = b ⇒ alogab = b. Ex. 1: Resolva a equação (log2x)2 – log2x = 2.
II. (Logaritmo do produto) logabc = logab + logac
Solução: Fazendo t = log2x, temos que t2 – t – 2 = 0 e, portanto,
Demonstração: 1
t = 2 ou t = –1. Assim, x = 4 ou x = e o conjunto solução é
Fazendo logabc = k, logab = l e logac = m, temos que b = al e c = am. 2
1 
Logo, bc = al + m. Por outro lado, bc = ak e então ak = al + m ⇒ S =  , 4 .
k = l + m ⇒ logabc = logab + logac. 2 

92 Vol. 3
Exponencial e logaritmo

Ex. 2: Resolva a equação logx(2x + 3) = 2. Juntando os casos, temos que o conjunto solução é
 3 − 5 1  3 + 5 
Solução: Verificaremos as restrições no fim. Temos que x2 = 2x + 3 e S= , ∪ , +∞ .
 2 2   2 
então x = 3 ou x = –1. Veja que x = –1 não pode ser solução, pois a base    
deve ser positiva. Assim, o conjunto solução é S = {3}.
2.7 Outras definições
2.6 Inequações logarítmicas I. Antilogaritmo: antiloga x = ax
Muitas inequações logarítmicas se reduzem à seguinte inequação
mais simples (usando as propriedades vistas e as técnicas adquiridas na Ex.: antilog2 3 = 23 = 8
apostila de álgebra básica). Devemos sempre nos lembrar das restrições
sobre o logaritmando e a base. II. Cologaritmo:
cologa x = –logax
Teorema 6 (Desigualdade entre os expoentes)
I. Se a > 1 e logax > logay, então x > y.
Ex.: colog2 8 = –log28 = – 3
II. Se a < 1 e logax > logay, então x < y.
III. Logaritmo decimal: É o logaritmo cuja base é 10. Para representá-lo,
Demonstração: simplesmente omitimos a base: log x. Assim, log100 = 2, por exemplo,
A demonstração é imediata a partir do fato de que a função logarítmica pois 100 = 102.
é crescente se a > 1 e decrescente se a < 1.
Para vermos algumas técnicas, vejamos exemplos de inequações IV. Logaritmo natural: É o logaritmo cuja base é e = 2,7182717284...,
exponenciais: um número irracional chamado número de Euler ou constante de
Napier. Tal número pode ser definido de algumas maneiras, que serão
Ex. 1: Resolva a inequação log3x2 – 3log3x + 2 > 0. vistas mais a fundo na apostila de Limites. Uma maneira de defini-lo é
+∞

∑ k ! = 0! + 1! + 2! + 3! + 4 ! + . A representação do logaritmo
1 1 1 1 1 1
e=
Solução: A restrição dos logaritmos é x > 0. Fazendo log3x = t, temos
k =0
que t2 – 3t + 2 > 0 e então t < 1 ou t > 2. Logo, log3x < 1 = log33 ⇒
natural é lnx.
x < 3 ou log3x > 2 = log39 ⇒ x > 9. Intersectando com a restrição,
temos que o conjunto solução é S = (0,3) ∪ (9, +∞). V. Característica (parte inteira) e mantissa (parte fracionária): Dado um
número x > 0, consideremos seu logaritmo decimal logx. Definimos
Ex. 2: Resolva a inequação logx(2x2 – 5x + 2) > 1. a característica (parte inteira) de logx, representada por log x  ,
como sendo o maior inteiro que não ultrapassa logx. A mantissa
Solução: Fazendo a restrição, temos x > 0, x ≠ 1 e 2x2 – 5x + 2 > 0, o (parte fracionária) de logx, representada por {logx}, é definida como
1 1 sendo {log x} = log x − log x  .
que nos dá x < ou x > 2. Juntando as restrições, temos: 0 < x <
2 2
ou x > 2. Temos agora que dividir o problema em dois casos, de acordo Ex.: log15 = 1,176... e então log15  = 1 e {log15} = 0,176...
com a base ser ou não maior que 1: Comentário: A característica e a mantissa montam as famosas
tábuas de logaritmos, usadas para efetuar as complicadas multiplicações
Caso 1: mencionadas no início do texto.
0 < x < 1:
2.8 Número de algarismos de um inteiro
Aqui temos logx(2x2 – 5x + 2) > 1 = logxx e como a base é
menor que 1, temos que 2x2 – 5x + 2 < x ⇔ x2 – 3x + 1 < 0 e então Teorema 7 (Número de algarismos em função do log)
3− 5 3+ 5. O número de algarismos de um inteiro x é dado por log x  + 1.
<x<
2 2
Demonstração:
Intersectando com a restrição e com 0 < x < 1, temos que Seja n o número de algarismos de x. Assim, podemos escrever que
3− 5 1 10n – 1 ≤ x < 10n ⇔ n – 1 ≤ logx < n e isto quer dizer precisamente que
<x< .
2 2 log x  = n − 1 ⇔ n = log x  + 1, como queríamos.
Comentário: Essa fórmula é útil para saber o número de algarismos
Caso 2:
de potências, quando conhecemos os logaritmos das bases.
x > 1:
Temos mais uma vez logx(2x2 – 5x + 2) > 1 = logxx e como a base Ex.: Sabendo que log2 = 0,301, calcule o número de algarismos de 220.
é maior que 1, segue que 2x2 – 5x + 2 > x ⇔ x2 – 3x + 1 > 0 e então Solução: Calculemos log220: usando a “regra do peteleco”, temos que
3− 5 3+ 5
x< ou x > . log220 = 20log2 = 6,02. Logo log220  = 6 e o número de algarismos
2 2 buscado é 6 + 1 = 7.
3+ 5
Intersectando com a restrição e com x > 1, segue que x > .
2

IME-ITA 93
Matemática I – Assunto 5

EXERCÍCIOS RESOLVIDOS

01 Resolva a equação 23x – 4 = 4x + 1. 05 Resolva a equação 2x = 3x + 1.

Solução: A ideia nesse tipo de problema é ‘igualar’ as bases: 23x – 4 = Solução: Como as bases não são potências de um mesmo inteiro, não
= (22)x + 1 = 22x + 2. Daí, temos 3x – 4 = 2x + 2, logo x = 6. podemos seguir a mesma ideia do exercício resolvido 1. Com variável no
expoente em uma situação dessas, a ideia é ‘tirar log’ dos dois lados. Isso
02 Resolva a inequação 25x – 23 ⋅ 5x – 50 < 0. pode ser feito em qualquer base, mas é conveniente escolher uma das
bases que já aparece no problema: log32x = log33x + 1 ⇒ x ⋅ log32 = x + 1
Solução: Fazendo 5x = t (t > 0), temos t2 – 23t – 50 < 0. As raízes da 1
⇒x= .
expressão quadrática são 25 e –2 e, como a concavidade é voltada para log3 2 − 1
cima, tem-se que –2 < t < 25. No entanto, repare que t é positivo (pois 2
É possível simplificar a resposta, pois log3 2 − 1 = log3 2 − log3 3 = log3
é potência de positivo), logo, 0 < t < 25. Voltando à variável original, 3
temos 0 < 5x < 25, o que nos dá x < 2. . Daí, segue que x = log 2 3 (repare que foi utilizada a fórmula de
3
03 Dado que log2 ≈ 0,3010, dê aproximações para log5 e log6,25. 1
mudança de base no formato log a b = )
log b a
Solução: A base dos logaritmos é 10 (pois não aparece). Logo,
06 Resolva a equação log2(x + 1) + log2(x – 1) = log23 .
10
log 5 = log = log 10 − log 2 ⇒ log5 = 1 – log2 ⇒log5 ≈ 1 – 0,3010 Solução: Usando a regra do produto, temos log2(x2 – 1) = log23.
2 Igualando os logaritmandos, temos x2 = 4, ou seja, x = ±2. No entanto,
= 0,6990. veja que não podemos ter valores de x menores do que 1 (pois aparece
625 x – 1 dentro de um logaritmo).
Para o outro log, veja que log 6, 25 = log = log 625 − log 100 ⇒
100 ∴ S = {2}.
log6,25 = log54 – 2 ⇒ log6,25 = 4log5 – 2.
Substituindo a aproximação anterior, segue que log6,25 = 0,796. Obs.: Neste problema, as bases já eram originalmente todas iguais. Em
algumas outras situações, teremos logaritmos em bases diferentes.
b a
04 Prove que alogc = blogc sempre que as expressões estão bem definidas. A 1ª coisa a ser feita, em geral, é utilizar a fórmula de mudança de base
para ‘igualar’ as bases de todos os logaritmos.
Solução: Basta ‘tirar log’ na base c dos dois lados, escreva você mesmo.

EXERCÍCIOS NÍVEL 2

01 Resolva as equações e inequações exponenciais abaixo: 04 (ITA) A soma de todos os valores de x que satisfazem à identidade
1
x2 – 15
x− 4
a. 3 = 9x 9 2 − 1− x
= −1 é:
3
b. 3 +3
x–1 1–x
= 10 ⋅ 3 –1

1 (A) 0.
1/ x
c. 3 < (B) 1.
9
(C) 2.
d. 3|x| > 9 (D) 3.
e. 2x – 3 + 2x – 1 + 2x + 1 + 2x + 3 = 42,5 (E) n.d.a.
1
x2 +
81 2
f. 3 x2 = 1
05 (ITA) Seja f ( x ) = e x − 4 , em que x ∈ . Um subconjunto de  tal
x+
3 x
que f : D →  é uma função injetora é:

02 Resolva as equações exponenciais abaixo: (A) D = {x ∈  | x ≥ 2 e x ≤ –2}


(B) D = {x ∈  | x ≥ 2 ou x ≤ –2}
2
a. 2x – 2 ⋅ x = 23x – 6 (C) D = 
0, 2 x −0,5 (D) D = {x ∈  | –2 ≤ x ≤ 2}
b. = 5 ⋅ 0, 04 x −1 (E) D = {x ∈  | x ≥ 2}
5
c. 4x + 2x + 1 – 24 = 0 06 (ITA) Dada a equação 32x + 52x – 15x = 0, podemos afirmar que:
d. 6 ⋅ 32x – 13 ⋅ 6x + 6 ⋅ 22x = 0
e. 3x = 5x (A) não existe x real que a satisfaça.
(B) x = log35 é uma solução desta equação.
f. x 2x – 3
= 1 (x > 0) (C) x = log53 é uma solução desta equação.
(D) x = log315 é uma solução desta equação.
03 Para que valores de m a equação: 81x – m ⋅ 9x + 2m – 3 = 0 admite (E) x = 3log515 é uma solução desta equação.
solução única?

94 Vol. 3
Exponencial e logaritmo

07 (ITA) Seja a um número real com 0 < a < 1. Então, os valores reais 17 Resolva a inequação: log3(x + 2) – log3 x > 1.
de x para os quais a2x – (a + a2)ax + a3 < 0 são:
18 Resolva a inequação: logx – 1 (2 – 3x) < 0.
(A) a2 < x < a. (D) a< x < a .
(B) x < 1 ou x > 2. (E) 0 < x < 4. 19 Resolva a inequação: log3x – log2 x > 1.
(C) 1 < x < 2.
20 Resolva as equações:
1+ e x
08 (ITA) Dadas as funções f ( x ) = , x ∈  – {0}, e
1− e x 2
a. 3x – 4 = 52x
g(x) = xsenx, x ∈ , podemos afirmar que: b. 51 + 2x + 61 + x = 30 + 150x

(A) ambas são pares. 21 Resolva a equação: 4x + 6x = 9x.


(B) f é par e g é ímpar.
(C) f é ímpar e g é par. 22 Sejam a e b dois números reais positivos e diferentes de 1. Qual a
(D) f não é par nem ímpar e g é par. relação entre a e b para que a equação x2 – x(logba) + 2logab = 0 tenha
(E) ambas são ímpares. duas raízes reais e iguais?
−32 x
09 (EN) O domínio da função y = é:
 1
x 23 Simplifique logx x1 ⋅ logx x2 ... logx xn – 1.
 3  − 243
2 3 n
(A) (–∞, – 5)  
(B) (–∞, 5) 24 Calcule −log n [log n
n n n
n ].
(C) (–5, +∞)
(D) (5, +∞) d) 25 Determine os valores reais de x que satisfazem:
(E) (–5, 5)
a. log3logx2logx2 x > 0.
4

10 (EFOMM) Calcule o valor de x na expressão 23x + 5 – 23x + 1 =


b. log2x ⋅ log32x + log3x ⋅ log23x ≥ 0.
33x + 5 – 33x + 4 – 142 ⋅ 33x:
xx...
26 Resolva a equação xx = 2 (x > 0).
(A) –1/2.
(B) –1/3.
27 Resolva (logx)logx = logx.
(C) 0.
(D) 1/3.
28 Determine os valores de k para os quais a equação 2x + 2–x = 2k
(E) 1/2.
admite solução real.
11 (AFA) Determine as soluções da equação 3 ⋅ 9x + 7 ⋅ 3x – 10 = 0.
29 Para quais valores reais de x vale a relação log(x2) = 2 logx?
12 Resolva a equação: log3(x2 – 8x) = 2.
4 3
30 (EN) Se logαx = n e logαy = 5n, então logα x y é igual a:
13 Resolva a equação log3 log4 (x + 1) = 2.
(A) n/4. (D) 3n.
 1 (B) 2n. (E) 5n/4.
14 Ache os valores de x para que exista log x  x −  . (C) 3n/4.
 2 
15 Resolva as equações logarítmicas abaixo: 1
31 (EN) Seja x a solução da equação log7 x + 1 + log7 x − 1 = log7 3.
2
a. log3(x2 – 3x – 5) = log3(7 – 2x) 1
O valor de z = log2 2 + log x 128 é:
b. log(x + 4) + log(2x + 3) = log(1 – 2x) 64
2
( )
c. log2 x − 1 = log 1 ( x − 1) (A) 4. (D) 1.
2 (B) 3. (E) 0.
d. logx + 4(x2 – 1) = logx + 4(5 – x) (C) 2.
7
e. log2 x + log x + 1 = log 2 + log 4 + log 8
log
x 32 (EFOMM) Sendo log2 = p e log3 = q, o valor de
log 6 + log 9
10 é igual a:
2 3
f. log0,5 x x − 14 log16 x x + 40 log4 x x =0 3p . p+q .
(A) (D)
g. x 1 – logx
= 0,01 q + 2p 2q
6p . 6p + q .
h. logx(3 ⋅ xlog5x + 4) = 2log5x (B) (E)
1 p + 3q 2p
1
i. log5 (5 x + 125) = log5 6 + 1 + 3p .
2x (C)
p+q
16 Resolva a equação: 2log2(x – 1) – log2(x + 1) = 3.

IME-ITA 95
Matemática I – Assunto 5

40 (ITA-82) O conjunto verdade da desigualdade


( a + b)
2

33 (EFOMM) Sendo a + b = 70ab, o valor de log5


2 2
em função  
de m = log52 e n = log53 é:
ab ( )
log2  log 1 x 2 − 2 x + 1  < 0 é:
 
 4 
(A) n + m. (D) 2n + 2m.
(B) 2n + m. (E) 2n + 3m.  1  3   1  3 
(C) 3n + m.
(A)  0,  ∪  , 2  (D)  −∞, 2  ∪  2 , +∞ 
 2   2     
3 
34 (AFA) Qual o valor da soma dos 7 primeiros termos da progressão (B) ( −2, 0 ) ∪  , 2  (E) o conjunto vazio.
geométrica log1/2 1/4, log1/2, 1/16, ...? 2 
 1 3
(A) 1/4. (C)  , 
2 2
(B) 1/2.
(C) 128. 41 (ITA) Os valores de a e k reais que tornam verdadeira a expressão
(D) 254. log k
log a 2 a + 2 a ⋅ log2a 2 a = (log a 2 a )(log a 3 ) são:
log6 a k
35 (AFA) Uma das soluções da equação
( x − 1) é:
2
1 1 2
− log ( x + 1) = log + log (A) a = e qualquer valor de k, k > 0.
2 ( x − 1)
3
x +1 2
(B) a = 2 e qualquer valor de k, k > 0, k ≠ 1.
(A) 1. 2
(C) a = e qualquer valor de k, k > 0, k ≠ 1.
(B) 2. 2
(C) 3. (D) quaisquer valores de a e k com k ≠ 6a.
(D) 4. 1
(E) qualquer valor de a positivo com a ≠ 1 e a ↑ , e qualquer valor
6
36 (ITA) Determine o conjunto solução da inequação positivo de k.

42 (ITA) Sejam os números reais x > 0, a > b > 1. Os três números


 (
log 1 log4 x 2 − 5  > 0
 ) reais x , x log a b ,log a ( bx ) são, nesta ordem, os três primeiros termos
3
de uma progressão geométrica infinita. A soma S desta progressão vale:

afirmar que:
(
37 (ITA) Com respeito à função g ( x ) = ln sen x + 1 + sen x , podemos
2
) 2x
(A) S = .
1 − log a b
(A) está definida apenas para x ≥ 0.
x +1
(B) é uma função que não é par nem ímpar. (B) S = .
(C) é uma função par. log b
1− a
(D) é uma função ímpar. 2
(E) n.d.a. 1
2 (C) S = .
 1  1 − log a b
38 (ITA) As raízes reais da equação 2 1 + log 2 (10 )  =  
são:
 x   log x −1
 ( ) 
 (D) n.d.a.

1 1
(A) 10 e 10 . (D) e . 43 (ITA) Acrescentando 16 unidades a um número, seu logaritmo na base
10 10 3 aumenta de 2 unidades. Esse número é:
1
(B) 10 e . (E) n.d.a.
10 (A) 5. (D) 4.
1 (B) 8. (E) 3.
(C) e 10 .
10 (C) 2.

39 (ITA) Seja a1, a2, ..., an (ai > 0, i = 1,2, ..., n) uma progressão 44 (ITA) Se x e y são números reais tais que
( ) ( )
4
geométrica de razão r. Seja também f : + →  uma função definida por ln  y 2 + 1 ⋅ e x  − ln y 2 + 1 = x − 3 ,
 
f(x) = log(qxp), em que p e q são reais positivos. Nessas condições, f(a1), então:
f(a2), ..., f(an) é:
(A) y = 1 + e − 1 . (D) y = 2 e − 1 .
(A) uma progressão geométrica de razão log(qrp)
e −1 .
(B) uma progressão geométrica de razão plogr (B) y = 10 − e − 1 . (E) y =
(C) uma progressão aritmética de razão logq + ploga1 2
(D) uma progressão aritmética de razão logq + plogr (C) y = ± e − 1 .
(E) uma progressão aritmética de razão plogr

96 Vol. 3
Exponencial e logaritmo

45 (ITA) Sejam f, g funções reais de variável real definidas por 51 Calcule: log2log3antilog3log1,52,25.
1
( )
f ( x ) = ln x 2 − x e g ( x ) =
1− x
. Então o domínio de f  g é:
 x + y = 20
2 2
52 Resolva o sistema  .
(A) ]0, e[ (D) ]–1, 1[ log x + colog y = log 2
(B) ]0, 1[ (E) ]1, +∞[
53 Determine as características de:
(C) ]e, e + 1[
a. log316.
( )
46 (ITA) Considere A ( x ) = log 1 2 x 2 + 4 x + 3 , ∀x ∈ . Então, temos: b. log 2 5.
2
3
(A) A(x) > 1, para algum x ∈ , x > 1.
(B) A(x) = 1, para algum x ∈ . 54 Determine, sabendo que 0,3010 < log2 < 0,3011, quantos dígitos
(C) A(x) < 1, apenas para x ∈  tal que 0 < x < 1. tem 2300.
(D) A(x) > 1, para cada x ∈  tal que 0 < x < 1.
(E) A(x) < 1, para cada x ∈ . EXERCÍCIOS NÍVEL 2
01 Determine as soluções positivas dos sistemas abaixo (x e y são as
47 (ITA) Seja f = log2(x2 – 1), ∀x ∈ , x < –1. A lei que define a inversa variáveis):
de f é:
 y
x = y
x
y
− 1 − 2 y , ∀y ∈ , y ≤ 0 .
(A) 1 + 2 , ∀y ∈ . (D) a. 
p q
( p ≠ q ), ( p, q > 0 )
x = y

y
1 + 1 − 2 y , ∀y ∈ , y ≤ 0 .
(B) − 1 + 2 , ∀y ∈ . (E) 1
 y
x = 2
y
(C) 1 − 1 + 2 , ∀y ∈ . b.  y
 x5 = y3

48 (ITA) Em uma progressão geométrica de razão q, sabe-se que:  y 1
x = y2
I. o produto do logaritmo natural do primeiro termo a1 pelo logaritmo 
c. 
natural da razão é 24. y x = 1
II. a soma do logaritmo natural do segundo termo com o logaritmo natural  x
do terceiro termo é 26.
02 (ITA) Considere as funções f : * → , g :  →  e h : * → 
Se lnq é um número inteiro então o termo geral an vale: 1
x+ 81
definidas por: f ( x ) = 3 x , g(x) = x2 e h ( x ) = . O conjunto dos
(A) e .
6n – 2
(D) e .
6n + 2 x
(B) e4 + 6n. (E) n.d.a. valores de x em * tais que ( f  g )( x ) = ( h  f )( x ) é um subconjunto de:
(C) e24n.
(A) [0, 3] (D) [–2, 2]
49 (ITA) O conjunto dos números reais que verificam a inequação (B) [3, 7] (E) n.d.a.
3logx + log(2x + 3)3 ≤ 3 log2 é dado por: (C) [–6, 1]

1 03 (ITA) Um acidente de carro foi presenciado por 1/65 da população de


(A) (0, +∞) (D)  ,1 Votuporanga (SP). O número de pessoas que souberam do acontecimento
2  B
t horas após é dado por f ( t ) = , em que B é a população da
(B) [1, 3] (E) n.d.a. 1 + Ce− kt
cidade. Sabe-se que 1/9 da população soube do acidente 3 horas após,
 1
(C)  0, 
 2 então o tempo que passou até que 1/5 da população soubesse da notícia
foi de:
50 (ITA) O domínio da função f(x) = log(2x2 – 3x + 1)(3x2 – 5x + 2) é:
(A) 4 horas.
 1  3   3  (B) 5 horas.
(A) ( −∞, 0 ) ∪  0,  ∪  1,  ∪  , +∞  (C) 5 horas e 24 min.
 2  2 2 
(D) 5 horas e 30 min.
 1  5   5  (E) 6 horas.
(B)  −∞,  ∪  1,  ∪  , +∞ 
 2  2 2 
1 1 2 3 3 04 Mostre que log32 + log23 > 2.
(C)  −∞,  ∪  ,  ∪  1,  ∪  , +∞ 
 2 2 3  2 2  05 Determine a soma das raízes reais da equação 100x – k ⋅ 10x + 0,1 = 0,
(D) (–∞, 0) ∪ (1, +∞) assumindo que esta tem 2 raízes distintas.

(E) n.d.a.

IME-ITA 97
Matemática I – Assunto 5

06 Resolva os sistemas abaixo: e x , se x ≤ 0



252 x + 252 y = 30 12 (ITA) Seja f :  →  definida por: f ( x ) =  x 2 − 1, se 0 < x < 1
a.  ln x , se x ≥ 1
25
x+y
=5 5 

 x y Se D é um subconjunto não vazio de  tal que f : D →  é injetora, então


2 3 = 12 podemos ter:
b.  y x
2 3 = 18

(A) D =  e f(D) = [–1, +∞)

x
y −2
=4 (B) D = (–∞, 1] ∪ (e, +∞] e f(D) = (–1, +∞)
c. 
 x 2 y −3
= 64 (C) D = [0, +∞) e f(D) = (–1, +∞)

(D) D = [0, e] e f(D) = [–1, 1]
 x log y x ⋅ y = x 5 / 2 (E) n.d.a.
d. 
a x − a− x
log4 y ⋅ log y ( y − 3 x ) = 1 13 (ITA) Sejam a ∈ , a > 1 e f :  →  definida por f ( x ) = .
A função inversa de f é dada por: 2
07 Resolva as equações :

2 2
( 2
)
(A) log a x − x − 1 , para x > 1.
a. 3 log16 ( x + 1 + x ) + log2( x + 1 − x ) = log16 ( 4 x + 1) − 1 / 2
(B) log ( − x + x + 1), para x ∈ .
2
23+ x 3 x 2 + 11x + 6 a
b. x log2 − x 2 log 1 ( 2 + 3 x ) = x 2 − 4 + 2 log
(C) log ( x + x + 1) , para x ∈ .
2
10 2
10 2
a
2 2 2 2
c. x log6 (5 x − 2 x − 3) − x log 1 (5 x − 2 x − 3) = x + x
(D) log ( − x + x − 1) , para x < –1.
2
6
a
08 (ITA) Seja f :  →  uma função que satisfaz à seguinte propriedade:
(E) n.d.a.

( )
2
f(x + y) = f(x) + f(y)∀x, y ∈ . Se g ( x ) = f  log10 x 2 + 1  ,
  14 (ITA) Considere uma progressão geométrica de razão inteira q > 1.
então podemos afirmar que:
Sabe-se que a1an = 243, logqPn = 20 e logqan = 6,em que an é o enésimo
(A) o domínio de g é  e g(0) = f(1). termo da progressão geométrica e Pn é o produto dos n primeiros termos.
(B) g não está definida para os reais negativos e g(x) = 2f(log10(x2 + 1)), Então a soma dos n primeiros termos é igual a:
para x ≥ 0.
(C) g(0) = 0 e g(x) = 2f(log10(x2 + 1)), ∀x ∈ . 39 − 1 .
(A)
(D) g(0) = f(0) e g é injetora. 6
(E) g(0) = –1.
310 − 1 .
(B)
09 (ITA) Supondo m uma constante real, 0 < m < 1, encontre 6
todos os números reais x que satisfazem a inequação 8

 x  2  (C) 3 − 1 .
( )
log m x 4 + m4 ≥ 2 + log m  
 2 m 
+ m2 .
 9
6

(D) 3 − 1 .
1 1 3
10 (ITA) Sobre a expressão M = + , em que 2 < x < 3,
log2 x log5 x
(E) n.d.a.
qual das afirmações abaixo está correta?
15 (ITA) Sejam x e y reais positivos, ambos diferentes de 1, satisfazendo
(A) 1 ≤ M ≤ 2.
(B) 2 < M < 4.  y 1
(C) 4 ≤ M ≤ 5. x = y2

(D) 5 < M < 7. o sistema:  . Então o conjunto {x, y} está contido
(E) 7 ≤ M ≤ 10. log x + log y = log 1
 x
11 (ITA) Considere o desenvolvimento (x + y)10 = A1x10 + A2x9y + ... , no intervalo:
em que x e y são números reais. A oitava parcela do lado direito é igual
405 (A) [2, 5]
(logk 2) , para algum k > 1, x = 2 log2 k e y = logk 2 . Neste
3
a (B) (0, 4)
2 log k 2 2 log2 k
caso: (C) [–1, 2]
(D) [4, 8)
(A) k2 = 2. (D) k3 = 7. (E) [5, +∞)
(B) k2 = 3. (E) k3 = 5.
(C) k3 = 2.

98 Vol. 3
Exponencial e logaritmo

1 18 Resolva as inequações:
16 (ITA) Se x é um número real positivo, com x ≠ 1 e x ↑ , satisfazendo
3 x +2 x
2 + log3 x log x ( x + 2 ) a. log 3 ( 2 − 4 ) ≥ −2
− = log x ( x + 2 ) , então x pertence ao intervalo l, 3
log( x + 2) x 1 + log3 x
em que: log x ( x + 2) > 2
b.  2 4 x −6
( x − 8 x + 13) <1
 1  3
(A) I =  0,  (D) I =  1,  log a x + log a y x+y
 9  2 19 Determine os valores de a reais, tais que: ≤ log a
2 2
 1 3  para todos x e y reais positivos. (Interprete geometricamente.)
(B) I =  0,  (E) I =  ,2 
 3 2 
1  20 Resolva a inequação: logx(4x – x2) > 1.
(C) I =  ,1
2 
21 Mostre que existem a e b irracionais, tais que ab é racional. (Sugestão:
2 2
17 (ITA) Determine o conjunto solução da inequação Calcule ( 2 ) )

logx[(1 – x)x] < logx[(1 + x)x2] 22 As representações decimais de 22000 e 52000 são colocadas uma ao
lado da outra. Determine o número de algarismos que foram escritos.

EXERCÍCIOS NÍVEL 3
05 Usando o exercício anterior, mostre que para todo p natural :
01 Resolva as inequações:
1 1 1
1+ + + ... + > ln ( p + 1)
a. 2x ≥ 11 – x. 2 3 p
x
3 7
b.   + ≥ 2 x
5 5 06 Dado que 0,3010 < log2 < 0,3011, determine quantos dígitos tem
2300 + 5300.
02 Resolva log(20 – x) = (logx)3. x
x x x
07 Determine todas as soluções reais da equação 2 + 3 + 5 = 160 3
03 Determine o número de soluções reais da equação .

1 08 Determine o número de soluções reais positivas da solução senx = logx.


x 3 − 7 x 2 + 17 x − 9 = .
3
log x 09 Quantas soluções possui a equação x2 = 2x?
5
04 Sabe-se que a área determinada pelas curvas y = 1/x, x = 1, x = t 10 Se a, b, c são reais positivos maiores que 1, determine o valor mínimo
e y = 0 é dada por H(t) = ln(t), para t > 1. Mostre que: c b c a b a
de log a b c + log b a c + logc a b .
3
t −1 t2 − 1 abc
a. para t > 1, tem-se < ln ( t ) < ; z
t 2t 11 Para x, y e z positivos, determine o mínimo de x + y − ( xy ) 4 .
z z

1  1 1
b. para todo n ≥ 1, tem-se < ln  1 +  < ;
n+1  n n
n n +1
 1  1
c. conclua que  1 + < e <  1+
 n   n 

RASCUNHO

IME-ITA 99
Matemática I – Assunto 5

RASCUNHO

100 Vol. 3
Polinômios A ssunto
3
Matemática II

1. Polinômio em x Portanto, dividindo por xn, temos que A0+


A1 A A
+ 2 + ... + n = 0,
x x2 xn
1.1 Conceito para todo x ≠ 0.
Chama-se polinômio em x, e denota-se por P(x), toda expressão Como esta última equação é válida para todo x ≠ 0, podemos fazer
algébrica da forma P(x) = A0xm + A1xm – 1 + A2xm – 2 + ... + Am, ou seja: x → + ∞(ou seja, tomar x arbitrariamente grande) e teremos A0 = 0,
m
uma contradição.
P( x ) = ∑ Ak x m− k em que os coeficientes A , A , A , ..., A
0 1 2 m
são números
k =0 Por isso, todos os coeficientes devem ser nulos.
complexos e os expoentes da variável x são inteiros não negativos. O
grau do polinômio será dado pelo maior expoente da variável x em uma
parcela não nula.
3. Identidade de polinômios

Ex.: P(x) = x4 + 5x2 – (2 + 3 )x + 2 + 3i (grau 4)


3.1 Conceito
Dois polinômios P(x) e Q(x) são idênticos, quando P(x) = Q(x), para
Q(x) = 0x4 + 5x2 – (2 + 3 )x + 2 + 3i (grau 2)
todo x.
Obs.: Se tivermos num polinômio mais de uma variável, podemos pensá-lo Utilizamos o símbolo P(x) ≡ Q(x) neste caso.
como de uma variável, desde que se escolha uma como ordenatriz.
3.2 Teorema
Ex.: P(x, y, z) = 10x4y2z – 5x3y4z2 + 2x2yz – 2xyz + 5
"Se P e Q são polinômios, então P(x) ≡ Q(x), se e somente se os
1.2 Propriedades do grau coeficientes correspondentes de P e Q são iguais. Em particular, os graus
de P e Q devem ser iguais."
Dados P(x) e Q(x), tem-se:
Demonstração: Considerando o polinômio U(x) = P(x) – Q(x),
I. grau[P(x) · Q(x)] = grau[P(x)] + grau[Q(x)] aplicamos o teorema 2.2 e está finalizado.
II. grau[P(x) + Q(x)] ≤ max{grau[P(x)], grau[Q(x)]}

1.3 Raiz 4. Divisão de polinômios


Dizemos que a é raiz do polinômio P, se P(a) = 0. Dados polinômios P(x) e D(x), existem (e são únicos) polinômios Q(x)
 P ( x ) = D ( x ) Q ( x ) + R ( x )
e R(x) tais que: 
grauR ( x ) < grau D ( x )
Ex.: Veja que 1 é raiz do polinômio P(x) = x3 + x2 + x – 3, pois P(1) = 0.

1.4 Operações Obs. 1: Existe um método para encontrar os polinômios Q e R, conhecidos


como quociente e resto, respectivamente. Isso será feito em sala de aula,
As operações de multiplicação, soma e diferença são feitas da maneira
já que uma explicação por escrito mais confundiria do que elucidaria
habitual. A única operação diferente, de fato, é a divisão de polinômios.
qualquer coisa.

2. Polinômio identicamente nulo Obs. 2: Quando R(x) = 0, para todo x, dizemos que a divisão é exata. Veja
que a condição grau R(x) < grau D(x) ainda é verdadeira, se considerarmos
2.1 Conceito o grau do polinômio identicamente nulo como –∞.
Um polinômio P(x) é identicamente nulo, quando P(x) = 0, x.
Pela definição de grau dada, o polinômio identicamente nulo não 5. Teorema do resto
possuiria grau. No entanto, é comum dizer que seu grau é igual a – ∞. "O resto da divisão de P(x) por (x – a) é igual a P(a)."
Veja que, fazendo isso, as propriedades de grau continuam válidas inclusive
Demonstração:Dividindo P(x) por (x – a), encontramos um quociente
para o polinômio identicamente nulo.
Q(x) e um resto R. Veja que R é número (não tem x), pois seu grau deve ser
menor que o grau de (x – a), que é 1. Daí, temos que P(x) ≡ (x – a)Q(x) + R.
2.2 Teorema Fazendo x = a, temos o resultado: R = P(a).
"Um polinômio P(x) é identicamente nulo, se e somente se todos os
seus coeficientes são nulos."
Demonstração: Esta demonstração deve ser omitida numa 1a leitura,
6. Fatoração
por sua sofisticação.
6.1 Teorema
Faremos por contradição. Considere um polinômio que seja sempre nulo,
mas que não possua todos os coeficientes nulos. Daí, existe um grau para "Se o número complexo a é raiz de um polinômio P, então P(x) é
este polinômio, ou seja, um expoente máximo dentre os existentes. Assim, divisível por (x – a)."
escrevemos A0xn + A1xn–1 + ... + An–1x + An = 0, para todo x(A0 ≠ 0).
Obs.: Isso é consequência imediata do teorema do resto.

IME-ITA 101
Matemática II – Assunto 3

7. Algoritmo de Briot-Ruffini 9. Grau × quantidade de raízes


m
I. Se P é um polinômio de grau ≥ 1, então #raízes (P) = grau (P);
II. Se P é um polinômio tal que #raízes (P) > grau (P), então P ≡ 0 (mais
Consideremos um polinômio P(x) do grau m, ou seja, P( x ) = ∑ Ak x m− k
precisamente, se um polinômio se anula numa quantidade de valores
k =0
e façamos a divisão por x – a. Então P(x) ≡ Q(x) · (x – a) + R, em que Q(x) maior que o seu grau, então este polinômio é identicamente nulo);
m −1 III. se P(x) = Q(x) para uma quantidade de valores de x maior que os
= ∑ Bk x m−1− k ; da identidade acima, obtemos as seguintes igualdades: graus de P e Q, então P e Q são idênticos (P(x) ≡ Q(x)).
k =0
Demonstrações: I decorre imediatamente do item anterior, a forma
 B0 = A0 fatorada. Por conta disso, veja que o único polinômio que possui mais
 raízes do que grau é o identicamente nulo, com infinitas raízes. Portanto, II
 B1 = A1 + a.B0 é verdade. Para o item III, basta considerar o polinômio P(x) – Q(X).
 B2 = A2 + aB1

 10. Teorema das raízes conjugadas
B = Am −1 + aBm − 2
 m −1 Se um polinômio de coeficientes reais admite uma raiz complexa
 R = Am + aBm −1
da forma a + bi (a e b reais), então também admite a raiz complexa
conjugada a – bi.
Essas igualdades nos permitem efetuar a divisão de P(x) por x – a – —
Demonstração:Basta ver que p(x ) = p(x) para polinômios com
através do dispositivo denominado algoritmo de Briot-Ruffini, ou seja:
coeficientes reais.

A0 A1 A2 ... Am–1 Am Obs.: Há um resultado similar para raízes irracionais da forma a + b d , para
a B0 B1 B2 ... Bm-1 R a, b, d racionais e d irracional, e a demonstração é basicamente a mesma.
O teorema é: "Se um polinômio de coeficientes racionais admite uma raiz
Ex.: 2x3 – 5x2 + 3x – 4 ÷ x – 2 complexa da forma a + b d , com a, b, d racionais e d irracional, então
também possui a raiz a – b d ."
2 –5 3 –4
2 2 –1 1 –2 11. Relações de Girard
Considere um polinômio, de grau n maior ou igual a 1, P(x) = A0xn+
Q(x) = 2x2 – x + 1 e R = –2
A1x + ... + An e suas n raízes x1, x2, ..., xn distintas ou não.
n–1

Ex.: x4 – 3x2 + 5x – 2 ÷ x + 1 Definamos as chamadas somas simétricas de x1, x2, ..., xn:

• Soma ⇒ σ1 = x1 + x2 + ... + xn
1 0 –3 5 –2
• Soma dos produtos 2 a 2 ⇒ σ2 = x1x2 + x1x3 + ... + xn–1xn
–1 1 –1 –2 7 –9 • Soma dos produtos 3 a 3 ⇒ σ3 = x1x2x3 + x1x2x4 + ... + xn–2xn–1xn
...
Q(x) = x3 – x2 + –2x + 7 e R = –9 • Soma dos produtos n a n, ou seja, produto ⇒ σn = x1x2...xn

8. Teorema fundamental A1 A A
As relações de Girard afirmam que: σ1 = − , σ 2 = 2 , σ3 = − 3 ,
da álgebra A0 A0 A0
A
Todo polinômio complexo de grau maior ou igual a 1 possui pelo ..., σ k = ( −1) k Ak , ..., σ n = ( −1) n n .
menos uma raiz complexa. A0 A0
A demonstração desse teorema envolve conceitos muito mais Demonstração:Parece difícil, porque a notação pode confundir um
avançados e não a faremos. pouco o aluno menos experiente. No entanto, a demonstração é bastante
A partir disso, veja que, se P é um polinômio de grau n maior ou simples.
igual a 1 tal que a1 é uma raiz, pela fatoração vista no item 6, temos que Dadas as raízes de P, temos sua forma fatorada P(x) = A0(x – x1)
P(x) = (x – a1)P1(x) e esse polinômio P1 possui grau n – 1. Agora, repetimos (x – x2)...(x – xn). Agora, vejamos qual é o coeficiente de xn – k. Vendo a
a mesma ideia. Se P1 ainda tiver grau maior ou igual a 1, P1 teria alguma forma fatorada como uma grande distributiva, note que, para gerarmos
raiz a2 e, por isso, poderíamos escrever P(x) = (x – a1)(x – a2)P2(x). xn – k, precisamos em n – k parêntesis escolher x e, nos outros k, escolher
Prosseguindo desta forma, os polinômios assim construídos P1, P2, ... têm as raízes. Cada parcela dessas contém um produto de k raízes. Portanto,
graus decrescentes e, num certo momento, o grau de algum P desses será se fizermos todas as possibilidades, teremos a soma dos produtos k a
nulo. Portanto, podemos escrever P(x) = A(x – a1)(x – a2) · ... · (x – an). z das raízes, ou seja, σk. Além disso, na distributiva, há um sinal que
Veja que a1, a2, ..., an são as raízes e que A é o coeficiente líder de está sendo multiplicado k vezes. Por isso, também há um (–1)k. Então,
P, ou seja, o coeficiente do termo de maior grau do polinômio. Esta é a o coeficiente de xn – k por um lado é A0 = (–1)kσk e, por outro lado, é Ak.
chamada forma fatorada de P. A
Então, σk = (–1)k k .
A0

102 Vol. 3
Polinômios

Ainda assim, a demonstração pode ter parecido difícil para os não Temos as seguintes propriedades:
iniciados. Para termos uma situação mais simples, consideremos o caso de
grau 3. Fazendo a distributiva em P(x) = A0(x – x1)(x – x2)(x – x3), chegamos 13.3.1 Equação recíproca de primeira espécie
a P(x) = A0(x3 – (x1 + x2 + x3) x2 + (x1x2 + x1x3 + x2x3) x – x1x2x3).
Igualando os coeficientes aos de P(x) = A0x3+ A1x2 + A2x + A3, chegamos • grau ímpar: sempre terá o –1 como raiz;
às relações de Girard para o grau 3, as mais utilizadas: • grau par: nada podemos afirmar.
 A1
 x1 + x2 + x3 = −
 A0 13.3.2 Equação recíproca de segunda espécie
 A2
 x1x2 + x1x3 + x2 x3 = • grau ímpar: sempre terá o 1 como raiz;
 A0
• grau par: sempre terá o 1 e o – 1 como raízes.
 A
 x1x2 x3 = − 3 13.4 Resolução de equação recíproca
 A0
Para entender o método, veja o exercício resolvido número 6.

12. Teorema da raiz racional


14. Raízes múltiplas
Considere o polinômio P(x) = A0xn + A1xn–1 + ... + An, de coeficientes
p
inteiros. Se esse polinômio possui uma raiz racional (p e q inteiros, na 14.1 Definição
q
forma irredutível), então: Um número r é dito raiz de multiplicidade m de um polinômio P, se
existir um polinômio Q tal que P(x) = (x – r)mQ(x), em que r não é raiz de Q.
I. p é divisor de An; Se r não é raiz de P, dizemos que r tem multiplicidade 0. Além disso,
II. q é divisor de Ao. raízes de multiplicidade 1, 2 e 3 são chamadas de raízes simples, duplas
e triplas, respectivamente.
Caso particular: Se A0 = 1, então qualquer raiz racional da equação
é inteira (e testamos divisores de A0). 14.2 Teorema
Demonstração: Exercício 18, nível 2. Um número r é raiz de multiplicidade m de um polinômio P, se e
somente se P(r) = P’(r) = P”(r) = ... = P(m – 1)(r) = 0, em que P’ denota
13. Polinômios recíprocos a primeira derivada do polinômio, P’’ denota a segunda derivada e, em
geral, P(k) denota a k-ésima derivada do polinômio.
13.1 Definição
15. Transformadas aditiva,
Um polinômio de grau n é dito recíproco, se P(x) = ±xnP  1  . Como
x multiplicativa, simétrica e
consequência disso, se a é raiz de um polinômio recíproco, 1 também é. recíproca
a
13.2 Classificação É possível transformar uma equação polinomial P(x) = 0 (equação
primitiva) em uma outra equação polinomial Q(y) = 0 (equação
transformada) de modo que as raízes y relacionem-se com as raízes de x
I. Se P(x) = xnP  1  , temos um polinômio recíproco de primeira
x através da função y = ϕ(x) (função transformatriz).
espécie. Neste tipo de polinômio, os coeficientes das parcelas
15.1 Transformada aditiva
equidistantes dos extremos são iguais, quando ordenados segundo
as potências decrescentes da variável. As raízes da nova equação são obtidas somando k unidades às raízes
de uma equação original. Se P é um polinômio, o polinômio Q(x) = P(x – k)
Ex.: P(x) = 12x4 – 56x3 + 89x2 – 56x + 12 possui para raízes as raízes de P aumentadas de k unidades.

II. Se P(x) = –xnP  1  , temos um polinômio recíproco de segunda Ex.: Considere a equação polinomial x3 + 5x2 + 4x – 8 = 0. Determine a
x equação polinomial cujas raízes sejam obtidas somando duas unidades
espécie. Neste tipo de polinômio, os coeficientes das parcelas às raízes da equação dada.
equidistantes dos extremos são simétricos, quando ordenados
segundo as potências decrescentes da variável. Solução: A ideia é trocar x por x – 2.

III. P(x) = –2x5 – 5x4 – 11x3 + 11x2 + 5x + 2. Q(x) = (x – 2)3+ 5(x – 2)2 + 4(x – 2) – 8 = x3 – x2 – x – 4
Obs.: Caso P possua grau par, seu termo central deve ser nulo.
Seja Q(x) ≡ P(x –2). Portanto, trocando x → x – 2, tem-se Q(x + 2) ≡
13.3 Propriedades P(x). Daí, é fácil ver que, se a é raiz de P, então a + 2 é raiz de Q.
Como 1 e –1 são as únicas raízes de uma equação recíproca que não Portanto, o polinômio procurado é:
precisam vir acompanhadas de outra (em pares), já que 1 é o inverso de 1
e –1 é o inverso de –1 , é claro que uma equação recíproca de grau ímpar Q(x) = (x – 2)3+ 5(x – 2)2 + 4(x – 2) – 8 = x3 – x2 – x – 4.
precisa ter 1 ou –1 como raiz.

IME-ITA 103
Matemática II – Assunto 3

15.2 Transformada multiplicativa


As raízes da nova equação são obtidas multiplicando por k unidades 1 5 1 –2 1
as raízes de uma equação original. Se P é um polinômio, o polinômio 2 7 15 28 57
x
Q(x) = P   possui para raízes as raízes de P multiplicadas por k 2 9 33 94
k
  2 11 55
unidades.
2 13
Ex.: Considere a equação polinomial x3 + 5x2 + 4x – 8 = 0. Determine a 2
equação polinomial cujas raízes sejam os dobros das raízes da equação
dada. Logo: P(x) = 57 + 94 · (x – 2) + 55 · (x – 2)2 + 13 · (x – 2)3 + (x – 2)4

x 3 x 2 x 16.2 Utilização da fórmula de Taylor


Solução: Estamos interessados em   + 5   + 4   – 8 = 0 ⇔
3 3 3 Se na fórmula do item anterior:
x3 + 15x2 + 36x – 216 = 0. P(x) = R0 + R1(x – a) + R2(x – a)2 + ... + Rn(x – a)n
assumindo x = a, encontraremos P(a) = R 0; se derivarmos,
Obs.: A transformada simétrica é um caso particular da transformada obteremos:
multiplicativa, quando k = –1.
P’(x) = R1 + 2 · R2(x – a) + 3 · R3(x – a)2 + 4 · R4(x – a)3 + ... + n ·
Rn(x – a)n – 1
15.3. Transformada recíproca assumindo x = a, encontraremos P’(a) = R1; derivando novamente,
resulta:
As raízes da nova equação são os inversos das raízes da equação
P”(x) = 2 · R2 + 2 · 3 · R3(x – a) + 3 · 4 · R4(x – a)2 + 4 · 5 · R5(x – a)3
original. Se P é um polinômio, o polinômio Q(x) = xnP  1  possui para +... + (n – 1) · n · Rn(x – a)n – 1
x
raízes os inversos das raízes de P.  
assumindo x = a, P”(a) = 2 · R2, analogamente:
Ex.: Considere a equação polinomial x3+ 5x2 + 4x – 8 = 0. Determine a
equação polinomial cujas raízes sejam os inversos das raízes da equação P”’(a) = 2 · 3 · R3 , ... , P(n)(a) = 2 · 3 · 4 · ... · n · Rn, ou seja:
dada.
P( ) ( a ) P( ) ( a )
4 n
P " ( a)
R0 = P(a), R1 = P’(a), R2 = , R3 = , ..., Rn = , logo:
  1 3  1
2
 1  2! 4! n!
Solução: Queremos encontrar x    + 5   + 4   − 8  = 0 ⇔
3
P( ) ( a )
n
 x 
 x x 
 P " ( a)
P(x) = P(a) + P’(a)(x – a) + (x – a)2 + ... + (x – a)n =
–8x3 + 4x2 + 5x + 1 = 0. (k) 2! n!
n P ( a)
∑ k!
(x – a)k, que é a fórmula de Taylor para polinômios.
16. Desenvolvimento em k =0

potências de x – a Ex.: Desenvolver P(x) = x4 + 5x3 + x2 – 2x + 1 em potências de x – 2.


P(x) = x4 + 5x3 + x2 – 2x + 1 → P(2) = 57
P’(x) = 4x3 + 15x2 + 2x – 2 → P’(2) = 94
16.1 Utilização do algoritmo de P”(x) = 12x2 + 30x + 2 → P”(2) = 110
Briot-Ruffini P”’(x) = 24x + 30 → P”’(2) = 78
Consideremos o polinômio, de grau n, P(x). Dividindo esse polinômio P(4)(x) = 24 → P(4)(2) = 24
por x – a, encontramos um quociente Q1(x) de grau n – 1 e um resto R0.
Dividindo Q1(x) por x – a, encontramos um quociente do grau n – 2 e um Logo: P(x) = 57 + 94 (x – 2) + 55 (x – 2)2 + 13 (x – 2)3 + (x – 2)4.
resto R1, e assim sucessivamente. Temos:
17. Decomposição de uma função
P(x) = (x – a)Q1(x) + R0
Q1(x) = (x – a)Q2(x) + R1 racional em uma soma de
Q2(x) = (x – a)Q3(x) + R2 frações parciais
...
Qn–1(x) = (x – a)Qn(x) + Rn – 1 17.1 Frações parciais
Qn(x) = (x – a)·0 + Rn
(observe que Qn(x) é do grau 0) A A
Dá-se o nome de frações parciais a frações do tipo ,
x − a ( x − a)n
, Ax + B , Ax + B , em que A, B, a, p, q  , n  , n ≥
Multiplicando-se a segunda igualdade por x – a, a terceira por (x – a)2,
( x 2 + px + q )
2 n
etc., e somando membro a membro, resulta: P(x) = R0 + R1(x – a) + x + px + q
R2(x – a)2 + ... + Rn(x – a)n, fórmula que permite desenvolver P(x) em
2 e x2 + px + q é um trinômio irredutível (D < 0).
potências de x – a.

Ex.: Desenvolver P(x) = x4 + 5x3 + x2 – 2x + 1 em potências de x – 2.

104 Vol. 3
Polinômios

17.2 Função racional I. A cada fator simples (x – a) no denominador corresponde uma fração
A
P( x)
É toda função f tal que ƒ(x) = , em que P e Q são polinômios parcial do tipo x − a;
Q( x )
II. A cada fator repetido (x – a)n no denominador correspondem n frações
e o denominador não é identicamente nulo. A1 A2 An
parciais do tipo , , ..., ;
17.3 Teorema x − a ( x − a )2 ( a)n
x −
III. a cada fator irredutível simples no denominador do tipo x2 + px + q
Toda função racional com grau do numerador menor do que o grau Ax + B
corresponde uma fração parcial do tipo ;
do denominador pode ser decomposta de maneira única numa soma de 2
x + px + q
frações parciais. Se o grau do numerador for maior do que o grau do IV. a cada fator irredutível repetido do tipo (x 2 + px + q) n no
denominador, dividiremos os polinômios e, assim, obteremos um polinômio A1x + B1
mais uma nova função racional que recai no caso anterior. denominador correspondem n frações parciais do tipo
2
A2 x + B2 An x + Bn x + px + q
, , ...,
( x 2 + px + q ) ( x 2 + px + q )
2 n
Ex.: x3 − 2 =x–1+ −1
x2 + x + 1 x2 + x + 1 Ex.:
3x4 − x2 + 5 A B C D
≡ + 2+ + +
( x 2 + 1)
2 x + 1 ( x + 1)2
17.4 Método para decomposição de x 2 ( x + 1)
3 x x
uma fração racional em uma E Fx + G Hx + I
soma de frações parciais + +
( x 2 + 1)
3 2 2
( x + 1) x +1
Deve-se fatorar ao máximo o denominador no conjunto dos números
reais e, então, observar as seguintes regras:

EXERCÍCIOS RESOLVIDOS
01 Mostre que se um polinômio P é uma função par, então, todos os 04 Dado que x = 2 é uma raiz do polinômio x3 – x2 – 4, determine as
coeficientes de expoentes ímpares são nulos. (Há um resultado similar outras.
para polinômios ímpares.)
Solução:
Solução: Seja P(x) = a0 + a1x + a2x2 + ... + an–1xn–1 + anxn. Se P é
função par, então P(–x) ≡ P(x). Daí, a0 + a1x+ a2x2 + ... + an–1xn–1 + Como x = 2 é raiz, então x – 2 é um fator. Portanto, basta dividir x3 – x2
anxn ≡ a0 – a1x + a2x2 – ... + (–1)n – 1an – 1xn – 1 + (–1)nanxn. Igualando os – 4 por x – 2. Isso pode ser feito utilizando o algoritmo de Briot-Ruffini
coeficientes correspondentes, temos que a1 = –a1, a3 = –a3, a5 = –a5, ..., ou completando as parcelas: x3 – x2 – 4 = x3 – 2x2 + x2 – 2x + 2x – 4
o que nos dá que todos os coeficientes de expoentes ímpares são nulos. = x2(x – 2) + x(x – 2) + 2(x – 2) = (x – 2)(x2 + x + 2).
−1 ± 7 i
Por isso, as outras raízes vêm de x2 + x + 2 e são iguais a x = .
02 Qual é o resto de P(x) = 4x9 + 7x8 + 4x3 + 3 por x + 1? 2
05 Considere o polinômio P(x) = x3 – mx2 + 4x + 1. Se P possui três
Solução: Pelo teorema do resto, o resto por x – (–1) é igual a P(–1) = – raízes reais em P.G., determine o valor de m.
4+ 7– 4 + 3 = 2.
Solução:
03 Determine todos os polinômios P tais que P(x + 1) – P(x) ≡ x2
e P(0) = 0. Sejam a , a, aq as raízes. Usando as relações de Girard, temos que o
q
Solução: Antes de qualquer coisa, determinaremos o grau de P produto das raízes é a3 = –1 e, como a é real, temos que a = –1. Daí,
(esta ideia é muito útil: olhar para o grau!). Para isso, observe que como –1 é raiz, segue que P(–1) = –1 – m – 4 + 1 = 0, o que nos
se P tem grau n, então P(x+1) – P(x) possui grau n – 1 (tente dá m = –4.
provar isso usando binômio de Newton!). Assim, o polinômio com
o qual estamos trabalhando possui grau 3. Sabendo que P(0) = 06 Resolva a equação recíproca 72x4 – 6x3 – 181x2 – 6x + 72 = 0.
0, podemos escrever P(x) = Ax 3 + Bx 2 + Cx. Então, teremos
A(x + 1)3 + B(x + 1)2 + C(x + 1) – Ax3 – Bx2 – Cx ≡ x2, ou seja, Solução:
3Ax2 +(3A +2B)x + A + B + C ≡ x2. Comparando os coeficientes,
Esta é uma equação recíproca de primeira espécie (os coeficientes
teremos 3A = 1, 3A + 2B = 0 e A + B + C = 0 . Ao resolver o sistema,
equidistantes do termo médio são iguais). A ideia para resolver esse tipo
1 1 1
chegamos em A = , B = − e C = . Portanto, o polinômio é de equação é primeiro verificar através das propriedades se 1 ou –1 são
3 2 6 raízes para simplificar a equação. Neste caso, não há como fazer uso
x3 x2 x
destas propriedades. Feita esta etapa, a ideia é dividir a equação dada
P(x) = – + .
3 2 6  2 1   1
Note que esse polinômio é útil para encontrar a soma dos quadrados por x2, obtendo 72  x + 2  – 6  x +  – 181 = 0 . Agora, fazendo
dos n primeiros naturais.  x   x

IME-ITA 105
Matemática II – Assunto 3

1 1 Solução: A ideia é expressar a5 em função de potências menores de a


x+ = t, temos que x2 + 2 = t2 – 2 , donde nossa equação fica (e o mesmo para b e c).
x x
72t2 – 144 – 6t –181 = 0 ⇔ 72t2 – 6t – 325 = 0 , cujas raízes são Como a é raiz do polinômio, temos que a3 – 2a2 – 3a – 4 = 0; logo, a3
25 13 = 2a2 + 3a + 4. Multiplicando por a, temos a4 = 2a3 + 3a2 + 4a =
4 3 3 2
e . Desta forma, os possíveis valores para x são − , − , , . 2(2a2 + 3a + 4) + 3a2 + 4a = 7a2 + 10a + 8.
12 6 3 4 2 3 Novamente, multiplicando por a: a5 = 7a3 + 10a2 + 8a = 7(2a2 + 3a
07 Resolva o sistema abaixo: + 4) + 10a2 + 8a = 24a2 + 29a + 28.
Usando os mesmos argumentos para b e c, chegamos a:
8 a + 4 b + 2c + d = 0
27 a + 9 b + 3c + d = 0  a5 = 24 a2 + 29 a + 28
 
 5 2
 b = 24 b + 29 b + 28
729 a + 81b + 9c + d = 0  5
1000 a + 100 b + 10c + d = 0 2
c = 24 c + 29c + 28

a5 − b5
Solução: Considere P(x) = ax3 + bx2 + cx + d. O enunciado nos dá Daí, a5 – b5 = 24(a2 – b2) + 29(a – b) e = 24(a + b) + 29.
a−b
que P(2) = P(3) = P(9) = P(10), ou seja, 2, 3, 9 e 10 são raízes de P.
Como o polinômio P tem grau no máximo igual a 3, veja que #raízes(P) a5 − b5 b5 − c5 c5 − a5
Portanto, temos que + + = [24(a + b) +
> grau(P). Portanto, P é o polinômio identicamente nulo e (a, b, c, d) a−b b−c c−a
= (0, 0, 0, 0). 29] + [24(b+ c) + 29] + [24(c + a) + 29] = 48(a + b + c) + 87.
Podemos ver, pelas relações de Girard, que a + b + c = 2. Então,
08 Se a, b e c são as raízes do polinômio x3 – 2x2 – 3x – 4, determine
a expressão pedida é igual a 48 · 2 + 87 = 183.
a5 − b5 b5 − c5 c5 − a5
o valor numérico de + + .
a−b b−c c−a

EXERCÍCIOS NÍVEL 1

01 Escrever o trinômio 2x4 – 2x2 + 25 sob a forma (x2 + m)2 + (x2 + n2). 09 (ITA-82) Os valores de a ,b ,g que tornam o polinômio P(x) = 4x5 +
2x4 – 2x3 + ax2 + bx + g divisível por Q(x) = 2x3 + x2 – 2x + 1 satisfazem
02 Determine a e b para x4 – 3x2 – 2x + 1 ≡ (x2 – ax + 1)(x2 – b) as desigualdades:

03 Determine a e b, em 5x2 – 19x + 18 = (x – 2)(x – 3) + a(x – 1)(x – 3) (A) a > b > g


+ b(x – 1)(x – 2) (B) a > g > b
(C) b > a > g
04 Dado o polinômio ax(x + 10) + bx(x + 1) + cx(x + 4) + 5a – b + (D) b > g > a
c, determinar a, b e c de modo que o polinômio seja identicamente nulo. (E) g > a > b

05 Determine a relação entre a, b e c na identidade: a(x + 5y – 3z) + 10 (ITA-91) Seja S o conjunto de todas as raízes da equação 12x3 – 16x2
b(2x – 2y + 6z) + c(7x + 11y + 3z) = 0. – 3x + 4 = 0. Podemos afirmar que:

06 Um polinômio P(x) = ax3 + bx2 + cx + d é tal que P(–2) = –2, P(1) (A) S ⊂ ]–1,0[ ∪ ]0,1[ ∪ ]1,2[
= –3, P(2) = 2, P(–1) = 3. Temos que: (B) S ⊂ ]–2, –1[ ∪ ]0,1[ ∪ ]3,4[
(C) S ⊂ [0,4]
(A) b = 0. (D) S ⊂ ]–2, –1[ ∪ ]1,2[ ∪ ]3,4[
(B) b = 1. (E) n.d.a.
(C) b = 2.
(D) b = 3. 11 (ITA-96) Considere o polinômio P(z) = z6 + 2z5 + 6z4 + 12z3 + 8z2
(E) n.d.a. + 16Ex.: .

ax + b (A) Apenas uma raiz é real.


07 Achar a condição necessária e suficiente para que a fração
seja independente de x. cx +d (B) Apenas duas raízes são reais e distintas.
(C) Apenas duas raízes são reais e iguais.
08 (ITA-95) A divisão de um polinômio P(x) por x2 – x resulta no quociente (D) Quatro raízes são reais, sendo duas a duas distintas.
6x2 + 5x + 3 e resto –7x. O resto da divisão de P(x) por 2x + 1 é: (E) Quatro raízes são reais, sendo apenas duas iguais.

(A) 1. (D) 4. 12 O polinômio P(x) = x4 – 4x3 + ax2 + bx + c é divisível por (x + 1)3.


(B) 2. (E) 5. Qual o resto da divisão de P(x) por x – 2?
(C) 3.
13 (CN) Qual é o resto de 2x2010 – 5x2 – 13x + 7 por x2 + x + 1?

106 Vol. 3
Polinômios

14 Encontre o resto da divisão de x2006 + 2x + 87 por x3 – 1. 22 Determine o valor de k para que o polinômio P(x) seja divisível por
D(x), nos casos abaixo:
15 (ITA-97) Sejam P1(x), P2(x), P3(x) polinômios na variável real x de
graus n1, n2, n3, respectivamente, com n1 > n2 > n3. Sabe–se que P1(x) (A) P(x) = x4 – 2kx2 + 3x – 1; D(x) = x + 2
e P2(x) são divisíveis por P3(x). Seja R(x) o resto da divisão de P1(x) por (B) P(x) = x4 – 5x2 + 4x + k; D(x) = 2x + 1
P2(x). Considere as afirmações: (C) P(x) = 4x2 – 6x + 2k; D(x) = 2x – 1
(D) P(x) = 2x3 – kx2 + 7x – 6; D(x) = 2x + 3
I. R(x) é divisível por P3(x);
1 23 Calcule m sabendo que o resto da divisão de 4x3 – 3x2 + mx + p por
II. P1(x)− P2(x) é divisível por P3(x);
2 2x – 1 é p.
III. P1(x) · R(x) é divisível por (P3(x))2.
24 Determine p e q para que x3 + 2x2 + px + q seja divisível por x2 – 1.
Então:
25 (ITA-67) O polinômio P(x) dividido por x + 2 dá resto 1, por x + 1 dá
(A) Apenas I e II são verdadeiras. resto –1 e por x – 1 dá resto 1. Qual o resto da divisão de P(x) por (x +
(B) Apenas II é verdadeira. 2)(x + 1)(x – 1)?
(C) Apenas I e III são verdadeiras.
(D) Todas as afirmações são verdadeiras. (A) x2 – x + 1. (D) x2 – x – 1.
(E) Todas as afirmações são falsas. (B) x – 1. (E) n.d.a.
(C) x2 + x + 1.
16 Sendo 8 e 6 os restos das divisões de P(x) por x – 5 e x – 3,
respectivamente, pede-se determinar o resto da divisão de P(x) por (x – 5) 26 (PUC) Determine p e q tais que x4 + px2 + q seja divisível por
(x – 3). x2 + 2x + 5.

17 (ITA-87) Considere Q(x) e R(x), respectivamente, o quociente e o 27 Encontre todos os polinômios P(x) tais que P(2 – x) + 2P(x) =
resto da divisão de um polinômio A(x) pelo trinômio B(x) = –x2 + 5x – 6. x3 + 2x + 2.
Admita que o grau de A(x) é quatro e que os restos da divisão de A(x) por
x + 1 e x – 2 são, respectivamente, 3 e –1. Supondo também que Q(x) 28 Qual é o maior valor de n para o qual o polinômio (x – 1)n divide o
seja divisível por x + 1, obtenha R(x). polinômio x2006 – 1?

18 (ITA-82) Sabendo-se que o polinômio P(x) = ax3 + bx2 + 2x – 2 é 29 O polinômio P(x), quando dividido por x2 + 1, deixa resto x + 1, quando
divisível por (x + 1) e por (x – 2), podemos afirmar que: dividido por x + 1 deixa resto 2 e é divisível por x – 1. Qual é o resto da
divisão de P(x) por x4 – 1?
(A) a e b têm sinais opostos e são inteiros.
(B) a e b têm o mesmo sinal e são inteiros. 30 Para a e b distintos, prove que (a + b – x)m + xm – am – bm é divisível
(C) a e b têm sinais opostos e são racionais não inteiros. por (x – a)(x – b).
(D) a e b têm o mesmo sinal e são racionais não inteiros.
(E) somente a é inteiro. 31 Forme uma equação do 3o grau de coeficientes reais, sabendo que
uma raiz é 2, e a outra 2 + 3i.
19 (ITA-67) Um polinômio P(x) dividido por x – 1 dá resto 3. O quociente
desta divisão é dividido por x – 2 dando resto 2. O resto da divisão de P(x) 32 O polinômio P(x) do 5o grau e de coeficientes reais admite 0, –1, i
por (x – 1)(x – 2) será? como raízes, sendo 0 raiz dupla. Determine P(x) sabendo que P(2) = 120.

(A) 3x + 2. (D) 4 – x. 33 (ITA-00) Sendo 1 e 1 + 2i raízes da equação x3 + ax2 + bx + c =


(B) 3x – 1. (E) n.d.a. 0, em que a, b, c são números reais, então:
(C) 2x + 1.
(A) b + c = 4.
20 (ITA-88) Sejam A(x) e B(x) polinômios de grau maior que um e admita que (B) b + c = 3.
existam polinômios C(x) e D(x) tais que a igualdade a seguir se verifica: A(x) (C) b + c = 2.
· C(x) + B(x) · D(x) = 1,  x ∈ . Prove que A(x) não é divisível por B(x). (D) b + c = 1.
(D) b + c = 0.
21 (ITA-86) Sejam a, b e c números reais que nesta ordem formam uma
progressão aritmética de soma 12. Sabendo-se que os restos das divisões 34 (ITA-84) Sabendo-se que z1 = i, z2, z3 são as raízes da equação
de x10 + 8x8 + ax5 + bx3 + cx por x – 2 e x + 2 são iguais, então a razão z3 + az2 + bz + c = 0, onde a, b, c são reais não nulos, podemos
desta progressão aritmética é: afirmar que:

(A) 1. (A) z1, z2, z3 são imaginários puros.


(B) 28/5. (B) z2 e z3 são reais.
(C) 37/5. (C) z1z2z3 = c.
(D) 44/15. (D) z1 + z2 + z3 = a.
(E) –3. (E) pelo menos uma das raízes é real.

IME-ITA 107
Matemática II – Assunto 3

35 (ITA-88) Se P(x) e Q(x) são polinômios com coeficientes reais, de 45 Determine p e q na equação 20x3 – 8px2 + 5px + 5q = 0, sabendo que
graus 2 e 4 respectivamente, tais que P(i) = 0 e Q(i) = 0, então podemos duas de suas raízes são iguais e que a terceira raiz é a soma das duas primeiras.
afirmar que:
46 Resolver a equação do terceiro grau x3 – 6x2 + 11x – 6 = 0, sabendo
(A) P(x) é divisível por x + 1. que uma das raízes é igual à soma das duas outras.
(B) P(x) é divisível por x – 1.
(C) P(x) · Q(x) é divisível por x4 + 2x2 + 1. 47 Resolver a equação x3 – 4x2 + x + 6 = 0, sabendo que possui duas
(D) P(x) e Q(x) são primos entre si. raízes cuja razão é 3/2.
(E) Q(x) não é divisível por P(x).
48 Dada a equação x3 – 7x + λ = 0, determinar λ de modo que uma das
36 Achar as raízes da equação x3 – 9x2 + 23x – 15 = 0, sabendo que raízes seja o dobro da outra. Resolver a equação.
estão em P.A.
49 Calcule os coeficientes p e q da equação x2 + px + q = 0, sabendo-se
37 Resolver a equação x3 – 5x2 + 8x – 4 = 0, sabendo que uma das que suas raízes aumentadas de 1 são as raízes de x2 – p2x + pq = 0.
raízes é o dobro da outra.
50 Determine a e b tais que P(x) = (ax + b)(x5 + 1) – (5x + 1) é divisível
38 Dada a equação x – 2ax + 8x + a – 6 = 0, determine o valor de a,
3 2
por (x2 + 1).
sabendo que a soma das raízes é igual ao seu produto.
51 Resolva a equação 2x4 + 5x3 + x2 + 5x + 2 = 0.
39 Calcule k e as raízes a, b, c da equação x – 7x + k = 0 de sorte que
3

a raiz c seja igual à metade de b. x3 + 4


a bx + c
52 (ITA-94) A identidade = 1+ + é válida para
x3 + 1 x + 1 x2 − x + 1
40 (ITA-91) Os valores de m, de modo que a equação x3 – 6x2 – m2x +
todo x real diferente de –1. Então, a + b + c é igual a:
30 = 0 tenha duas de suas raízes somando um, são:
(A) 5. (D) 2.
(A) 0. (D) 2 e –2.
(B) 4. (E) 1.
(B) 3 e 3. (E) n.d.a.
(C) 3.
(C) 1 e –1.
6 − 5x
A B C
41 (ITA-94) As raízes da equação de coeficientes reais x3 + ax2 + bx + 53 (ITA-77) Se + ≡
+ , onde a, b, c são
3 x − 2
a x − b x −c
c = 0 são inteiros positivos consecutivos. A soma dos quadrados dessas x − 5x + 6x
raízes é igual a 14. Então, a2 + b2 + c2 é igual a: raízes da equação x3 – 5x2 + 6x = 0, então:

(A) 190. (D) 193. (A) A = – 2; B = –1; C = 0. (D) A = 5; B = 2; C = 1.


(B) 191. (E) 194. (B) A = 2; B = 4; C = 1. (E) n.d.a.
(C) 192. (C) A = –2; B = – 1; C = 0.

42 (ITA-81) Considere a equação x3 + px2 + qx + r = 0 de coeficientes 54 Decompor as frações abaixo, numa soma de frações parciais:
reais, cujas raízes estão em progressão geométrica. Qual das relações é
verdadeira? 2x + 3 2x3 + x + 3
(A) . (D) .
3 2
x + x − 2x ( x 2 + 1)2
(A) p2 = rq
x3 + 1 3 2
(B) 2p + r = q (B) . (E) 3 x − 7 x + 2 x − 7 .
(C) 3p2 = r2q x( x − 1) 3 2 2
( x − 1) ( x + x + 1)
(D) p3 = rq3
4
(E) q3 = rp3 (C) .
x3 + 4 x
43 (ITA-78) Se a, b, c são raízes da equação x3 – rx + 20 = 0, onde r é
um número real, podemos afirmar que o valor de a3 + b3 + c3 é: 55 (ITA-91) Considere as afirmações abaixo:

(A) –60. I. A equação 3x4 – 10x3 + 10x – 3 = 0 só possui raízes reais;


(B) 62 + r. II. Toda equação recíproca admite um número par de raízes;
(C) 62 + r2. III. As raízes da equação x3 + 4x2 – 4x – 16 = 0 são exatamente o dobro
(D) 62 + r3. das raízes da equação x3 + 2x2 – x – 2 = 0.
(E) 62 – r.
Então:
44 (ITA-87) Multiplicando-se por 2 as raízes da equação x3 – 2x2 + 2x – 1
= 0, vamos obter raízes da seguinte equação: (A) apenas I é verdadeira.
(B) apenas II é falsa.
(A) 2y3 – 6y2 + 6y – 4 = 0. (D) y3 – 8y2 + 8y + 8 = 0. (C) apenas III é verdadeira.
(B) y3 – 4y2 + 8y – 8 = 0. (E) 4y3 – 4y2 – 4y – 8 = 0. (D) todas são verdadeiras.
(C) 8y3 – 8y2 + 4y – 1 = 0. (E) n.d.a.

108 Vol. 3
Polinômios

EXERCÍCIOS NÍVEL 2 09 (ITA-95) Sabendo que 4 + i 2 e 5 são raízes do polinômio 2x5 –


22x4 + 74x3 + 2x2 – 420x + 540, então a soma dos quadrados de todas
01 (IME) Determine os polinômios do 4o grau tais que P(x) = P(1 – x). as raízes reais é:

02 (ITA-98 e AFA-01) Seja P(x) um polinômio de grau 4 com coeficientes (A) 17. (D) 23.
reais. Na divisão de P(x) por x – 2 obtém-se um quociente Q(x) e resto (B) 19. (E) 35.
igual a 26. Na divisão de P(x) por x2 + x – 1 obtém-se um quociente H(x) (C) 21.
e resto 8x – 5. Sabe–se que Q(0) = 13 e Q(1) = 26. Então, H(2) + H(3)
é igual a: 10 (ITA-94) Seja P(x) um polinômio de grau 5, com coeficientes reais,
admitindo 2 e i como raízes. Se P(1)P(–1) < 0, então o número de raízes
(A) 16. (D) –28. reais de P(x) pertencentes ao intervalo ]–1, 1[ é:
(B) zero. (E) 1.
(C) –47. (A) 0. (D) 3.
(B) 1. (E) 4.
03 (ITA-99) Seja P(x) um polinômio de grau 3 tal que P(x) = P(x + 2) – (C) 2.
x2 – 2 para todo x real. Se –2 é uma raiz de P(x), então o produto de todas
as raízes de P(x) é: 11 Determine a e b tais que as equações x3 + ax2 + 11x + 6 = 0 e x3
+ bx2 + 14x + 8 = 0 possuam duas raízes em comum.
(A) 36. (D) –18.
(B) 18. (E) 1. P 3
12 (ITA-93) Considere a equação de coeficientes reais x5 + mx4 + 2 x
(C) –36. m
– 316x + 688x + P = 0, m ≠ 0, para a qual 1 + 3i é raiz. Sabendo-se
2

04 (ITA-91) Na divisão de P(x) = a5x5 + 2x4 + a4x3 + 8x2 – 32x + a3 que a equação admite mais de uma raiz real e que suas raízes reais formam
por x – 1, obteve-se o quociente Q(x) = b4x4 + b3x3 + b2x2 + b1x + b0 e uma progressão geométrica de razão inteira q cujo produto é igual a 64,
P
o resto –6. Sabe-se que (b4, b3, b2, b1) é uma progressão geométrica de podemos afirmar que é:
razão q > 0 e q ≠ 1. Podemos afirmar: m
(A) 20. (D) 120.
(A) b3 + a3 = 10. (D) b4 + b1 = 16. (B) 30. (E) 160.
(B) b4 + a4 = 6. (E) n.d.a. (C) 40.
(C) b3 + b0 = 12.
13 (ITA-90) Seja P(x) = 16x5 – 78x4 + ... + ax – 5 um polinômio de
05 (ITA-77) Se P(x) é um polinômio do 5 grau que satisfaz as condições
o
coeficientes reais tal que a equação P(x) = 0 admite mais do que uma
1 = P(1) = P(2) = P(3) = P(4) = P(5) e P(6) = 0, então temos: raiz real e ainda, a + bi é uma raiz complexa desta equação com ab ≠
1
0. Sabendo–se que é a razão da progressão geométrica formada pelas
(A) P(0) = 4. (D) P(0) = 2. a
(B) P(0) = 3. (E) n.d.a. 7
raízes reais de P(x) = 0 e que a soma destas raízes reais vale , enquanto
(C) P(0) = 9. 8
1
que o produto é , o valor de a é:
06 Um polinômio P(x) do quarto grau é divisível por sua derivada segunda 6
P"(x) = x2 – 4. Determine P(x).
(A) 32. (D) 11.
07 Dadas as equações: (B) 56. (E) 0.
(C) 71.
I. x4 – 16x3 + 89x2 – 206x + 168 = 0
II. x4 – 16x3 + 91x2 – 216x + 180 = 0 14 Calcular as raízes da equação x4 – 12x3 + 47x2 –72x + 36 = 0,
III. x4 – mx3 + nx2 – 462x + 432 = 0 sabendo que o produto de duas das suas raízes é igual ao produto das
outras duas raízes.
determinar:
15 Determinar m de modo que a equação x4 – mx2 + 8x – 3 = 0 tenha
a. as raízes comuns das equações I e II; uma raiz tripla e calcular as raízes dessa equação.
b. os valores de m e n, sabendo que III admite as raízes determinadas
no item (a). 16 Resolver a equação x4 + 3x3 + 3x2 – 2 = 0, sabendo que a soma de
duas de suas raízes é –1.
08 (ITA-97) Sejam a1, a2, a3, a4 números reais, formando, nesta ordem,
uma progressão geométrica crescente, com a1 ≠ 0. Sejam x1, x2, x3 as 17 Mostre que toda equação polinomial de grau ímpar (de coeficientes
raízes da equação a1x3 + a2x2 + a3x + a4 = 0. Se x1 = 2i, então: reais) possui uma raiz real.

(A) x1 + x2 + x3 = –2. 18 Considere a equação anxn + an – 1xn – 1 + ... + a1x + a0 = 0. Mostre que
(B) x1 + x2 + x3 = 1. se essa equação possui uma raiz racional p/q (na forma irredutível), então
(C) x12 + x22 + x32 = 4. p é divisor de a0 e q é divisor de an (Obs: Este é um método poderosíssimo,
(D) x1x2x3 = 8. para achar possíveis raízes racionais de equações algébricas). Conclua
(E) x1x2 + x2x3 + x3x1 = 5. que se an=1, então qualquer raiz racional da equação é inteira.

IME-ITA 109
Matemática II – Assunto 3

19 Para m e n naturais, mostre que se n m não é inteiro, então é irracional. 27 (ITA-93) Sabendo-se que a equação de coeficientes reais x6 – (a + b
(Sugestão: Use o exercício anterior.) + c)x5 + 6x4 + (a – 2b)x3 – 3cx2 + 6x – 1 = 0 é recíproca de segunda
espécie, então o número de raízes reais desta equação é:
20 Resolva nos reais as seguintes equações algébricas, com o auxílio do
exercício 18, nível 2: (A) 0. (D) 4.
(B) 2. (E) 6.
a. 2x4 – 5x3 + 2x2 + 32x – 16 = 0 (C) 3.
b. 20x3 – 27x2 + 4x + 3 = 0
28 (ITA-85) Como ax4 + bx3 + 5x + 3 = 0 é recíproca e tem o 1 como
21 (ITA-00) Sendo I um intervalo de números reais com extremidades raiz, o produto das raízes reais desta equação é:
a e b com a < b, o número real b – a é chamado de comprimento de I.
Considere a inequação 6x4 – 5x3 – 7x2 + 4x < 0. A soma dos comprimentos (A) 2. (D) 3.
dos intervalos nos quais ela é verdadeira é igual a: (B) –1. (E) 4.
(C) 1.
3 11 .
(A) . (D)
4 6 29 Determinar a relação que deve existir entre os coeficientes da equação
3 7 x3 + px + q = 0, para que tenha duas raízes iguais.
(B) . (E) .
2 6
30 Calcular m de modo que a equação x3 + mx – 2 = 0 tenha uma raiz
7
(C) . dupla e calcular as raízes desta equação.
2
22 (ITA-77) Seja R o corpo dos números reais. Em relação à equação 31 A equação x3 – 11x2 + 29x – 7 = 0 possui uma raiz da forma u +
5x3 – 15x2 – 15x – 20 = 0, x ∈ , podemos afirmar que: 3. Determine todas as suas raízes.

(A) não tem solução inteira. 32 Pode um polinômio inteiro em x ser nulo para todo valor de x no
(B) tem somente uma solução. intervalo [a, b] exceto num ponto c desse intervalo?
(C) tem somente duas soluções distintas.
(D) tem três soluções distintas. 33 Considere os polinômios P(x) = a0x4 + a1x3 + a2x2 + a3x + a4 tais
(E) n.d.a. que P(2) = P(3) = P(4) = P(r) = 0, onde r ∉ {2, 3, 4}. Se não há outras
raízes, temos, necessariamente, que:
23 Sejam a e b constantes reais. Sobre a equação x4 – (a + b)x3 + (ab
+ 2)x2 – (a + b)x + 1 = 0, podemos afirmar que: (A) a0 > 4. (D) a0 > 0.
(B) a0 < 0. (E) n.d.a.
(A) não possui raiz real se a < b < –3. (C) a0 ≠ 0.
(B) não possui raiz real se a > b > 3.
(C) todas as raízes são reais se |a| ≥ 2 e |b| ≥ 2. 34 Seja P um polinômio tal que P(x) = ax3 + bx2 + cx + d para todo x
(D) possui pelo menos uma raiz real se –1 < a ≤ b < 1. real, onde a, b, c, d são reais. Se P(x) = 0 para todo x do conjunto {1, 2,
(E) n.d.a. 3, 4, 5}, temos que:

24 (IME) (A) P(6) = a + 1. (D) P(6) = d.


a. Mostre que, se P(x) = a0 + a1x + a2x2 + a1x3 + a0x4, então existe um (B) P(6) = a + 2. (E) n.d.a.
polinômio G(x) do 2o grau , tal que P(x) = x2 · G(x + x –1). (C) P(6) = a + 3.
b. Determine todas as raízes do polinômio P(x) = 1 + 4x + 5x2 + 4x3 + x4
35 (IME) Mostre que se a equação x3 + px + q = 0 possui três raízes
25 (ITA 97) Seja S o conjunto de todas as raízes da equação 2x6 – 4x5 + reais e distintas, então p < 0.
4x – 2 = 0. Sobre os elementos de S, podemos afirmar que:
36 (IME) Dada a equação x4 + 4x3 – 4cx +4d = 0, determine a, b, c,
(A) todos são números reais. d, sabendo que ela possui uma raiz dupla da forma a + b 3 (a, b, c, d
(B) 4 são números reais positivos. racionais).
(C) 4 não são números reais.
(D) 3 são números reais positivos e 2 não são reais. 37 Mostre que os polinômios P(x) = x4 – x3 + x2 + 2x – 6 e Q(x) = x4
(E) 3 são números reais negativos. + x3 + 3x2 + 4x + 6 possuem um par de raízes complexas comuns e
determine-as.
26 (ITA-99) A equação polinomial P(x) = 0 de coeficientes reais e grau
105 38 Desenvolva P(x) = 2x5 – 13x2 + 4 em potências de 1 – x.
6 é recíproca de 2a espécie e admite i como raiz. Se P(2) = − e
8
255 39 Prove que x4 + x3 + x2 + x + 1 divide x44 + x33 + x22 + x11 + 1.
P(–2) = , então a soma de todas as raízes de P(x) é igual a:
8
(A) 10. (D) 2. 40 Resolva a equação:
(B) 8. (E) 1.
(C) 6. 4z11 + 4z10 – 21z9 – 21z8 + 17z7 + 17z6 + 17z5 + 17z4 – 21z3 – 21z2
+ 4z + 4 = 0.

110 Vol. 3
Polinômios

EXERCÍCIOS NÍVEL 3

01 (Putnam) Determine todos os polinômios P(x) tais que P(1)=1 e 11 (ITA-98) Seja a um número real tal que o polinômio P(x) = x6 + 2x5
P(x²+1) = (P(x))² +1 para todo x real. + ax4 – ax2 – 2x – 1 admite apenas raízes reais. Então:

02 Determinar um polinômio inteiro em x, que verifique a identidade: (A) a ∈ [2, +∞[. (D) a ∈ [–2, –1[.
P(x + 2) – 2P(x + 1) + P(x) ≡ x. (B) a ∈ [–1, 1]. (E) a ∈ ]1, 2[.
(C) a ∈ [– ∞, –7].
03 Considere que as raízes m, n e p da equação x3 + ax + b = 0 sejam
racionais. Prove que as raízes de mx2 + nx + p = 0 também são racionais. x2 x3 xn
12 (IME-CG) Mostre que o polinômio 1 + x + + + ... + +
2! 3! n!
04 Seja f um polinômio mônico (ou seja, de coeficiente líder igual a 1) e ... não possui raízes múltiplas.
de coeficientes inteiros tal que existem 4 inteiros distintos a, b, c e d tais
que ƒ(a) = ƒ(b) = ƒ(c) = ƒ(d) = 12. Prove que não existe k inteiro tal 13 (IME) Para que valores de p a equação x4 + px + 3 = 0 tem raiz
que ƒ(k) = 25. dupla? Determine, em cada caso, as raízes da equação.

05 14 Determine o maior valor de k inteiro para o qual (x – 1)k divide


a. Transforme, por uma substituição de variável, uma equação geral do x2n + 1 – (2n + 1)xn + 1 + (2n + 1)xn – 1.
terceiro grau numa equação do terceiro grau na qual o coeficiente em
x­2 é igual a zero. k
15 Seja P um polinômio de grau n de maneira que P(k) = , para k
b. Resolva a equação x3 + px + q = 0. (Sugestão: Faça x = a + b e = 0, 1, ..., n. Encontre P(n + 1). k +1
tente calcular a e b.)
16 Se P é um polinômio recíproco de grau ímpar, prove que –1 é raiz de
06 (IME–adaptada) Seja P(x) = anxn + ... + a1x + a0 um polinômio de P e, então, que P(x) = (x + 1)Q(x), onde Q é um polinômio recíproco de
coeficientes inteiros. Mostre que se a0 e P(1) são ímpares, então P(x) não grau par.
possui raízes inteiras.
17 Para quais inteiros a o polinômio x2 – x + a é um fator de x13 + x +
07 Sejam A, B e C as raízes da equação x(x – 2)(3x – 7) = 2, tais que 90?
A ≤ B ≤ C.
18 Determine todos os polinômios P não identicamente nulos tais que
a. Mostre que A ∈ (0,1), B ∈ (1,2) e C ∈ (2,3). P(3x – 2) = 81P(x) para todo x real. (Sugestão: Primeiramente, determine
b. Calcule arctan A + arctan B + arctan C. o grau de P.)

08 Qual a condição para que a equação de coeficientes reais x3 + px + 19 (OMERJ) Encontre todos os possíveis polinômios não constantes P tais
q = 0 admita raízes complexas de módulo igual a a? que, para todo x real, vale a relação P(2x)P(–2x)P(x2) = P(–4x2)(x2 – 4)2.

09 Existe algum polinômio P(x) com coeficientes inteiros tal que P(3) = 20 Sejam a e b inteiros distintos. Mostre que o polinômio (x – a)2(x – b)2+
4 e P(9) = 9? 1 não pode ser escrito como produto de dois polinômios de coeficientes
inteiros e de grau menor que 4.
10 (USAMO) Sejam a, b e c inteiros distintos e P um polinômio com
coeficientes inteiros. Mostre que as condições P(a) = b, P(b) = c e P(c)
= a não podem ser satisfeitas simultaneamente.
RASCUNHO

IME-ITA 111
Matemática II – Assunto 3

RASCUNHO

112 Vol. 3
Matrizes e determinantes A ssunto
5
Matemática III

1. Introdução 1, se i > j



II. Considerando A = (aij)3×3, tal que ij 0, se i = j teremos:
a =
Diversos são os exemplos do dia a dia nos quais temos que ler dados −1, se i < j

em formato de tabela, onde as informações são distribuídas através de
linhas e colunas. A primeira parte desse capítulo trata dessas tabelas, as  a11 a12 a13  0 −1 −1
quais chamaremos de matrizes.    
A3 x 3 =  a21 a22 a23  =  1 0 −1
Uma vez definido o conjunto das matrizes, definiremos operações com  a31 a32 a33   1 1 0 
os seus elementos. Estudaremos as operações de adição, multiplicação por
escalar e multiplicação entre matrizes. Além disso, trataremos o conceito de
matriz inversa, mostrando exemplos de matrizes inversíveis e não inversíveis. Dizemos ainda que duas matrizes de mesma ordem são iguais se os
A segunda parte do capítulo aborda os determinantes. Veremos que elementos correspondentes (elementos que ocupam a mesma posição
toda matriz quadrada está associada a um número escalar, e veremos como nas matrizes) são todos iguais.
calcular esse número independente do tamanho da matriz. Uma vez definida uma matriz de ordem m por n, chamaremos de
Para isso, definiremos de forma algébrica o determinante de uma matriz Mm×n(R) o conjunto de todas as matrizes com essa ordem e entradas
quadrada, mostrando como a definição se aplica a matrizes 2×2 e 3×3. Depois (elementos) reais e de Mm×n (C) as matrizes com entradas complexas.
veremos as principais propriedades que derivam da definição e os teoremas de
Jacobi e LaPlace, muito necessários para o cálculo de matrizes n×n. 2.2 Operações algébricas
É através do determinante que pode-se determinar se uma matriz A
Adição
é inversível ou não.
Dadas duas matrizes de mesma ordem, m×n, definimos a soma sendo
uma matriz m×n obtida através da soma dos termos correspondentes, ou
2. Matrizes seja, se Am×n = (aij) e Bm×n = (bij), temos: A + B = (aij + bij)m×n.

2.1 Definições e conceitos iniciais


 2 3 4  4 0 2 6 3 6
Uma matriz m×n (lê-se m por n) é uma tabela de elementos distribuídos Ex.: A =  e B=  → A+ B =  
 − 1 3 5   3 − 1 4  2 2 9
em m linhas e n colunas. As dimensões de uma matriz definem a sua ordem,
ou seja, nesse caso, podemos dizer que ela é de ordem m por n. Multiplicação por escalar
Normalmente representamos uma matriz por letra maiúscula, colocando Dada uma matriz A e um escalar α, chamaremos de αA uma matriz
o número de linhas e de colunas como índices (o número de linhas sempre de mesma ordem que A obtida pelo produto de todos os elementos de A
vem primeiro). Exemplo: A3x2 é uma matriz com três linhas e duas colunas. por α, ou seja, se A =(aij)m×n, têm-se αA =(αaij)m×n.
Chamamos ainda de aij o elemento da linha i, e da coluna j da matriz.
Nesse caso, também podemos representar uma matriz m×n por (aij)m×n.  2 3 4  4 6 8
Ex.: A =   → 2A =  
 − 1 3 5   −2 6 10 
Ex.:
Obs.: Chamaremos o produto (–1) · A de –A, uma vez que esta matriz é o
 a11 a12  inverso aditivo de A. Assim, definimos a diferença de matrizes de mesma
  ordem por: A – B = A + (–B)
A3 x 2 =  a21 a22  = ( aij )3 x 2
 a31 a32  Propriedades
Sejam A, B e C matrizes de mesma ordem; α e β escalares, têm-se:
Obs.: Os elementos de uma matriz podem estar entre parênteses ou colchetes.
De modo geral, uma matriz fica bem definida se soubermos determinar I. (Comutativa da adição) A + B = B + A;
cada aij em função de i e j, seja por meio de uma expressão ou por meio II. (Associativa da adição)(A + B) + C = A + (B + C);
de uma sentença. III. (Existe elemento neutro da adição) Seja 0m×n uma matriz com todas
as entradas nulas (chamada de matriz nula), têm-se:
Ex.: ∀A; 0 + A = A + 0 = A;
I. Seja A = (aij)2x2 com aij = (–1)i+j · i · j. Como ficaria essa matriz? IV. (Existe inverso aditivo) ∀A, ∃(–A) | A + (–A) = (–A) + A = 0;
V. (Distributiva por escalar em relação a matrizes):
a11 = (–1)1+1 · 1 · 1 = 1; a12 = (–1)1+2 · 1 · 2 = –2; α(A + B) = αA + αB;
VI. (Distributiva por matriz em relação a escalares):
a21 = (–1)2+1 · 2 · 1 = –2; a22 = (–1)2+2 · 2 · 2 = 4; (α + β)A = αA + βA;
VII. (Associativa da multiplicação por escalar): (αβ)A = α(βA);
 1 −2 VIII. (Existe elemento neutro na multiplicação por escalar): 1 · A = A.
A2 x 2 =  
 −2 4  Todas as propriedades são consequências diretas da definição de
soma e multiplicação por escalar.

IME-ITA 113
Matemática III – Assunto 5

2.3 Produto de matrizes Vejamos o caso 2×2.


Definimos o produto escalar (ou produto interno) de dois vetores pela
soma dos produtos das coordenadas correspondentes, ou seja, se u, v ex1 + fx 2 = y1  ay1 + by 2 = z1
Sejam os sistemas:  ,
∈ Rn tem-se:  gx1 + hx 2 = y 2 cy1 + dy 2 = z2

u = (u1, u2, u3,..., un) e v = (v1, v2, v3,..., vn) ⇒ que podem ser escritos como:
uv = u1v1 + u2v2 + u3v3 + ... + unvn.
 e f   x1   y1   a b   y1   z1 
    =   ( BX = Y ),     =   ( AY = Z )
Agora pense em uma matriz m×n sendo um conjunto de m vetores de Rn,  g h   x2   y2   c d   y 2   z2 
ou seja, considere que cada linha da matriz é um vetor de dimensão n. Nesse
caso, como ficaria o produto da matriz Am×n por um vetor de dimensão n? Como representar z1 e z2 em função de x1 e x2?
Iremos considerar esse vetor como um vetor coluna, ou seja, uma
matriz de dimensão n×1, assim queremos calcular:  a( ex1 + fx 2 ) + b ( gx1 + hx 2 ) = z1 ( ae + bg) x1 + ( af + bh) x 2 = z1
 ⇒
 a11 a12 ... a1n   u1   ( 1
c ex + fx 2) + d ( 1 2 ) 2 ( ce + dg) x1 + ( cf + dh) x2 = z2
gx + hx = z
   
 a21 a22 ... a2 n  .  u2  = ?
       que pode ser escrito na forma:
   
 am1 am 2 … amn   un 
 ae + bg af + bh   x1   z1 
    =   ( CX = Z )
 ce + dg cf + dh   x 2   z2 
Uma vez que cada linha da matriz pode ser vista como um vetor de
dimensão n, podemos calcular o produto escalar de cada linha pelo vetor
u = (u1, u2, u3,..., un), de modo que teremos m produtos diferentes. Esses Porém, AX = Y e BY = Z, em que B · (AX) = Z. Seria interessante
resultados formarão um vetor de dimensão m, assim: então definir o produto BA de modo que B · (AX) = (BA) · X = Z = CX;
para isso devemos ter BA = C, ou seja,
 a11 a12 ... a1n   u1   v1 
       a b   e f   ae + bg af + bh 
 a21 a22 ... a2 n  .  u2  =  v 2   ⋅ = 
          c d   g h   ce + dg cf + dh 
     
 am1 am 2 … amn   un  v m 
O que é coerente com a definição mostrada anteriormente.
em que:
n
Exs.:
v i = ∑ aik uk = ai 1u1 + ai 2 u2 + ai 3 u3 + ... + ain un para todo i ∈{1, 2,..., m}. 1.
k =1

A vantagem de definirmos o produto de matrizes por um vetor dessa


1 2  1 3
forma é poder tratar sistemas lineares como produto de matrizes por A=  e B = 
vetores. 3 4  2 4 
 
Finalmente, se quisermos multiplicar duas matrizes, podemos pensar
que ambas são conjuntos de vetores, assim como fizemos no caso anterior.
Ao multiplicar a matriz A pela matriz B, multiplicaremos A por cada
um dos vetores que formam B; logo, temos a restrição que o número de  1× 1 + 2 × 2 1× 3 + 2 × 4   5 11
colunas de A deve ser o mesmo número de linhas de B (para existir os AB =  = .
3 × 1 + 4 × 2 3 × 3 + 4 × 4  11 25 
produtos escalares).
Resumindo, para obter a primeira coluna do resultado, multiplica-se 3 2 − 3 
2 3 5   
A pela primeira coluna de B (como anteriormente); para obter a 2a coluna, 2. 2 · A =   e B = 2 3 1
 4 − 1 2 2 x 3 0 5
multiplica-se A pela 2a coluna de B, e assim sucessivamente. Deste modo,
 2 3 x 3
a matriz AB terá o mesmo número de linhas de A e o mesmo número de
colunas de B. c11 = 2 · 3 + 3 · 2 + 5 · 0 = 12, c21 = 4 · 3 + (–1) · 2 + 2 · 0 = 10,
De modo geral, se A = (aij) é uma matriz m×n e B = (bij) é uma matriz c12 = 2 · 2 + 3 · 3 + 5 · 5 = 38, c22 = 4 · 2 + (–1) · 3 + 2 · 5 = 15,
n c13 = 2 · (–3) + 3 · 1 + 5 · 2 = 7, c23 = 4 · (–3) + (–1) · 1 + 2 · 2 = –9,
n × p então AB = C tal que C = (cij) é uma matriz m×p cij = ∑ aik ⋅ bkj , ou seja,
k =1  12 38 7 
Logo: A· B =  
o termo da linha i, coluna j de C é obtido pelo produto escalar da linha i  10 15 − 9 2×3
de A com a coluna j de B.

Obs.: Um dos motivos do produto das matrizes ser definido dessa forma Propriedades
está associado à resolução de sistemas lineares através de matrizes. Sejam A, B e C matrizes; α e β escalares; tem-se:

114 Vol. 3
Matrizes e determinantes

I. Am×p; Bp×n → (αA)(βB) = αβ(A · B) Ex.:


II. (associativa)Am×p; Bp×q; Cq×n → A(BC) = (AB)C 1 3 2 
 
III. (distributiva pela esquerda)Am×p; Bp×n; Cp×n → A(B + C) = AB + AC A = 0 1 4 
5 3 2
IV. (distributiva pela direita)Ap×n; Bm×p; Cm×p → (B + C) · A = B · A + C · A

Atenção: Dadas duas matrizes quaisquer, pode-se ter AB ≠ BA (não vale Diagonal secundária Diagonal principal
à comutativa).
Matriz nula
Podemos ter um produto existindo e o outro não; ambos existindo, porém,
com ordens diferentes; ou os dois com mesma dimensão, mas com Matriz em que todos os elementos são nulos.
entradas diferentes.
A demonstração dessas propriedades foge ao escopo do assunto. Ex.:
0 0 0 
0 2× 3 =  
2.4 Matriz transposta 0 0 0 
Dada uma matriz de ordem m×n, definimos sua transposta sendo
uma matriz de ordem n×m obtida pela inversão de papéis das linhas e Matriz triangular
colunas, ou seja, as linhas passam a ser colunas, assim como as colunas Matriz quadrada cujos elementos de uma das bandas da diagonal
passam a ser linhas. principal são todos nulos.
Assim, se Am×n = (aij), tem-se AT = (aji).
Ex.:
2 4  1 0 0 
2 3 5  t    
Ex.: A =   → A =  3 − 1 2 −1 0 
 4 − 1 2 2 x 3 5 2  3 5 2 
  3× 2  

Propriedades Obs.: Se os zeros estão na banda superior, dizemos que a matriz é


Sejam A = (aij), B = (bij) matrizes e α um escalar, tem-se: triangular superior, se os zeros estão na banda inferior, dizemos que a
matriz é triangular inferior.
I. (At)t = A
II. (αA)t = αAt Matriz diagonal
III. Am×n; Bm×n → (A + B)t = At + Bt Matriz quadrada cujos elementos das duas bandas da diagonal principal
IV. Am×n; Bn×p → (A · B)t = Bt · At são todos nulos.

Atenção: Na propriedade (IV) é importante lembrar a ordem, uma vez Ex.:


que, Bt At ≠ At Bt. 1 0 0 
Dem (IV): Primeiro, veja que as dimensões são coerentes, já que a ordem  
de (AB)t e de Bt At é p×m. 0 − 1 0 
0 0 2 
Basta então provar que os elementos correspondentes são iguais.  
Considere o elemento da linha i, coluna j de (AB)t. Este elemento pertence
à linha j, coluna i de AB; logo, é o produto da linha j de A pela coluna i de B. Matriz identidade
Porém, a coluna i de B é a linha i de Bt, e a linha j de A é a coluna j de Matriz diagonal na qual os elementos da diagonal principal são iguais
At, donde estamos calculando o produto da linha i de Bt com a coluna j de a 1. Se a matriz for de ordem n, escreve-se In.
At, que é o elemento da linha i, coluna j de Bt At.
Ex.:
2.5 Matrizes notáveis e seus elementos 1 0 0 
 
Matriz quadrada I3 =  0 1 0
0 0 1 
Se uma matriz tem o mesmo número de linhas e de colunas, ela 
é denominada matriz quadrada (m = n). Dizemos que a matriz tem n2
elementos ou que é de ordem n. Nesse caso, é comum colocar apenas Matriz simétrica
uma dimensão da matriz como índice. Uma matriz quadrada A é dita simétrica se aij = aji para todos i e j.
Em toda matriz quadrada de ordem n, tem-se: Isso é equivalente a At = A.

I. Diagonal Principal: diagonal formada pelos elementos aij, com i = j. Ex.:


Chamamos os elementos dessa diagonal de elementos principais. 1 2 3 
II. Diagonal Secundária: diagonal formada pelos elementos a ij com  
i + j = n + 1. Chamamos os elementos dessa diagonal de 2 −1 5 
3 5 0 
elementos secundários.  
III. Elementos Conjugados: são aqueles que apresentam posições
simétricas em relação à diagonal principal. Os elementos principais
são autoconjugados.

IME-ITA 115
Matemática III – Assunto 5

Matriz hemissimétrica (ou antissimétrica) Pelo que foi exposto acima, dizemos que B é a matriz inversa de A se
Uma matriz A é dita hemissimétrica se aij = – aji para todos i e j. Isso AB = BA = In. Nesse caso, pode-se representar a matriz inversa por A–1.
é equivalente a At = – A. Veja que, fazendo i = j em aij = aji, obtemos aii É importante lembrar que nem toda matriz possui inversa. Por exemplo,
= 0, ou seja, os elementos da diagonal principal devem ser nulos.  1 2
tentemos achar a matriz inversa de A =   . Nesse caso, queremos
 a b 2 4 
Ex.: determinar uma matriz   , tal que:
0 − 2 − 3 c d 
 
2 0 5   a b   1 2  1 0  a + 2b = 1
3 − 5 0    = ⇒
  c d  2 4  0 1 2 a + 4 b = 0

2.6 Traço Absurdo!


Seja A uma matriz quadrada; chamamos de traço a soma dos Mais a frente, quando for introduzido o conceito de determinantes,
n será visto um jeito de caracterizarmos as matrizes que possuem inversa.
elementos da diagonal principal, ou seja, se An = (aij)n×n, então trA = ∑ akk .
k =1
Quando uma matriz possui inversa, dizemos que esta é inversível,
regular, ou não singular. Caso não possua inversa, dizemos que ela é não
Propriedades inversível ou singular.
Sejam A e B matrizes quadradas de ordem n, tem-se:
Teorema: Seja A uma matriz quadrada de ordem n; então a inversa de A,
I. tr(A) = tr(At) caso exista, é única.
II. tr(k · A) = k · tr(A), em que k é escalar. Dem.: sejam B e C inversas de A, tem-se:
III. tr(A + B) = tr(A) + tr(B) B = B · I = B · (A · C) = (B · A) · C = I · C = C
IV. tr(AB) = tr(BA)
Propriedades
As três primeiras propriedades são consequências diretas da definição Sejam A e B matrizes quadradas não singulares de ordem n, e
de traço. Analisemos a propriedade (IV): α ≠ 0 um escalar, tem-se:

Vejamos a diagonal principal de AB: I. (A–1)–1 = A


II. (At)–1 = (A–1)t
c11 = a11b11 + a12 b21 + a13 b31 + ... + a1n bn1 1 −1
III. (αA)–1 = A
 α
c22 = a21b12 + a22 b22 + a23 b32 + ... + a2 n bn 2
 IV. (AB)–1 = B–1 · A–1
c33 = a31b13 + a32 b23 + a33 b33 + ... + a3 n bn 3
        
 Atenção: Na propriedade (IV) é importante lembrar a ordem, uma vez
cnn = an1b1n + an 2 b2 n + an 3 b3 n + ... + ann bnn
que B–1 · A–1 ≠ A–1 · B–1.

As propriedades (I), (III) e (IV) são consequências diretas da definição


A soma de todas as linhas é o traço de AB, porém, na direita podemos de matriz inversa. Vejamos a propriedade (II):
somar por linhas ou por colunas. Repare que a soma dos elementos da
primeira coluna pode ser vista como o produto da primeira linha de B com Para termos (At)–1 = (A–1)t devemos ter At · (A–1)t = I. De fato isso
a primeira coluna de A, ou seja, o primeiro elemento da diagonal principal é verdade, pela propriedade (IV) da transposta ((XY)t = Yt · Xt); assim:
de BA, assim como as demais colunas, logo, essa soma também é o At · (A–1)t = (A–1 · A)t = It = I
traço de BA.
3. Determinantes
Obs.: Na propriedade (IV), as matrizes não precisam ser quadradas, basta
que os produtos AB e BA o sejam. 3.1 Definições e conceitos iniciais
2.7 Matriz inversa Motivação
Em qualquer conjunto definimos o inverso por uma operação como Tentemos resolver o seguinte sistema linear:
sendo o elemento que aplicado àquela operação é levado no elemento  a11 x1 + a12 x 2 = b1
neutro. Por exemplo, no conjunto dos números reais, o inverso multiplicativo 
 a21 x1 + a22 x 2 = b2
1 1
de x é ( x ↑ 0), uma vez que o produto x ⋅ = 1 (elemento neutro da
x x Multiplicando a primeira equação por a 22, a segunda por a12 e
multiplicação nos reais). subtraindo, tem-se: (a11a22 – a12a21)x1 = b1a22 – b2a12.
Considere Mn(C) o conjunto das matrizes quadradas de ordem n com Usando a mesma ideia para obter x2, encontra-se (a11a22 – a12a21)x2 =
entradas complexas. Repare que esse conjunto possui elemento neutro b2a11 – b1a21 de modo que se a11a22 – a12a21 ≠ 0, o sistema terá solução.
para operação de multiplicação, uma vez que AI = IA= A, em que I =
In é a matriz identidade. Deste modo, I é considerado o elemento neutro Vejamos agora um sistema 3×3:
da multiplicação.

116 Vol. 3
Matrizes e determinantes

 a11 x1 + a12 x 2 + a13 x3 = b1 Classes de uma permutação


 Dizemos que uma permutação é de classe par (ímpar) se o número
 a21 x1 + a22 x 2 + a23 x3 = b2
a x + a x + a x = b de inversões em relação à principal for par (ímpar). Nesse caso, como a
 31 1 32 2 33 3 3
Permutação Principal apresenta zero inversão ela é de classe par.
Ao resolver o sistema encontram-se as seguintes equações:
( a11a22 a33 + a12 a23 a31 + a13 a21a32 − a13 a22 a31 − a11a23 a32 − a12 a21a33 ) x1 = k1 Teorema 1: Uma permutação muda de classe quando se troca a posição
 de dois elementos consecutivos.
( a11a22 a33 + a12 a23 a31 + a13 a21a32 − a13 a22 a31 − a11a23 a32 − a12 a21a33 ) x 2 = k2 Dem.: De fato, considere um conjunto A com n elementos, e uma
 Permutação Fundamental de seus elementos.
( a11a22 a33 + a12 a23 a31 + a13 a21a32 − a13 a22 a31 − a11a23 a32 − a12 a21a33 ) x3 = k3
Seja p uma permutação simples com k inversões: p = (a1, a2,... ai,
em que k1, k2 e k3 são constantes. ai+1,..., an); troquemos os elementos ai e ai+1. Essa troca não altera a
posição desses dois elementos em relação aos demais.
Deste modo, se a11a22a33 + a12a23a31 + a13a21a32 – a13a22a31 – a11a23a32
Se os dois formavam uma inversão em p, com essa troca deixaram
– a12a21a33 ≠ 0, o sistema terá solução.
de formar, tendo a nova permutação k – 1 inversões. Se os dois não
Repare que em ambos os casos o denominador depende apenas dos formavam uma inversão em p, agora passaram a formar, donde temos k
coeficientes do sistema, e uma vez que já vimos que um sistema pode ser + 1 inversões. Em ambos os casos mudamos a paridade das inversões.
escrito na forma AX = B, estamos dizendo que o sistema tem solução de
acordo com uma relação que envolve as entradas da matriz A. Teorema 2: O número de permutações de classe par de um conjunto de
Devemos, então, tentar enxergar como essa relação aparece em n elementos, n > 1, é igual ao número de permutações de classe ímpar.
matrizes maiores, e para isso devemos conjecturar algo através dos Dem.: Considere P0 o conjunto das permutações de classe par e P1 o
casos pequenos. conjunto das permutações de classe ímpar.
Nos casos 2×2 e 3×3 temos a soma de todos os produtos possíveis Seja f: P0 → P1 a função que leva uma permutação de classe par em
com os elementos da matriz (existindo, em cada parcela, exatamente um uma de classe ímpar através da inversão das posições dos dois primeiros
termo de cada linha e de cada coluna), tendo metade deles sinal positivo elementos (essa troca muda a classe pelo Teorema 1).
e metade deles sinal negativo. Veja que f é uma bijeção, uma vez que toda permutação ímpar pode
O sinal de cada uma das parcelas parece estar associado à posição ser obtida por uma par através dessa troca (sobrejetora) e permutações
dos elementos que está “pegando” na matriz, ou seja, a permutação de pares diferentes irão gerar permutações ímpares diferentes (injetora). Logo,
seus elementos. P0 e P1 têm a mesma quantidade de elementos.
Associaremos então cada matriz a um escalar, do mesmo modo que Teorema 3: (Teorema de Bezout): Uma permutação muda de classe se
ocorreu nos casos particulares, e veremos no próximo assunto que de fato trocarmos dois quaisquer de seus elementos.
isso garante a existência de solução para sistemas lineares. Dem.: De fato, considere um conjunto A com n elementos, e uma
A função que fará essa associação é conhecida como determinante. Permutação Fundamental de seus elementos.
Seja p uma permutação simples com k inversões: p = (a1, a2,... ai,
Permutações e inversões
ai+1,..., an) troquemos os elementos ai e aj. Essa troca não altera a posição
Uma permutação dos elementos de uma sequência é um rearranjo de desses elementos em relação aos que não estão entre eles.
seus elementos em alguma ordem sem omissões ou repetições. Deste
Entre os elementos ai e aj temos j – i elementos. Se ki e kj são,
modo, um conjunto de n elementos possui n! permutações simples.
respectivamente, os números de inversões, antes da troca, que ai e aj
Consideremos uma delas como referência, a qual denominaremos fazem com esses elementos, então existem j – i – ki não invertidos com
Permutação Fundamental (ou Principal). Neste caso, dizemos que dois ai e j – i – kj não invertidos com aj.
elementos de uma permutação formam uma inversão quando estão dispostos
Após a troca, os invertidos deixam de ser invertidos e vice-versa,
em ordem diferente daquela em que estão na Permutação Fundamental.
ou seja, passamos a ter entre esses elementos: (j – i – ki) + (j – i – kj)
Ex.: Quantas inversões apresenta a permutação 312 em relação a 123
± 1 inversões (–1 se ai ou aj estiverem invertidos e +1 caso contrário).
tomada como principal?
Assim, a diferença no número de inversões antes e depois da troca
312 → 132 → 123; 2 inversões.
desses elementos é: |2(j – i – ki – kj) ± 1|, que é ímpar.

Ex.: Quantas inversões apresenta a permutação cadb em relação a


abcd tomada como principal? Termo principal e deduzido
cadb → acdb → acbd → abcd; 3 inversões. Chamamos de termo principal associado a uma matriz quadrada
de ordem n o produto dos elementos principais dessa matriz, ou seja,
Tp = a11 · a22 · a33 · ... · ann.
Obs.: A permutação fundamental não apresenta inversões.
Fixados no termo principal os índices representativos das linhas,
Chamamos ainda de Permutação Inversa aquela que possui todos os chamaremos termo deduzido da matriz qualquer dos produtos da
elementos em ordem inversa da ordem em que figuram na Permutação forma (–1)k a1α · a2β · ... · anλ, em que (α, β, ... , λ) indica uma das n!
Fundamental. permutações com os índices representativos das colunas e k é o número
de inversões dessa permutação em relação à fundamental considerada
Nesse caso, o número de inversões será máximo, uma vez que
como (1, 2, 3, ... , n).
todos os pares de elementos formam uma inversão e será dado por
n( n − 1)
Cn2 = .
2

IME-ITA 117
Matemática III – Assunto 5

Obs.: De fato, reparem que em ambos, estamos calculando a soma de todos


I. Incluindo o termo principal, teremos numa matriz de ordem n, n! termos os produtos possíveis, formados por um único elemento de cada linha e
deduzidos. um único elemento de cada coluna, com o sinal variando de acordo com
II. Como, pelo Teorema 2, n!/2 permutações são de classe par e n!/2 são o número de inversões dos índices das colunas.
de classe ímpar, teremos uma quantidade igual de termos multiplicados
por (–1) e por (+1). 3.3 Propriedades
III. Todos os termos deduzidos são produtos de (–1)k por n fatores. I. O determinante de uma matriz é igual ao determinante de sua matriz
IV. Cada termo terá um e somente um elemento de cada linha ou coluna. transposta.
Basta ver que todos os produtos calculados no determinante de
3.2 Definição de determinante uma matriz são os mesmos produtos calculados no determinante da
O determinante associado a uma matriz quadrada de ordem n é a soma transposta.
algébrica de seus n! termos deduzidos, ou seja, Obs.: Decorre desse teorema que qualquer propriedade relativa às
linhas é válida para colunas e vice-versa.
           , em que |σ| representa a classe da

permutação σ. II. É nulo todo determinante que contém uma fila nula.
Com efeito, cada termo do determinante contém um elemento dessa
Representa-se por:
fila, logo, terá um fator nulo.
a11 a12  a1n
a21 a22  a2 n III. Multiplicando-se (dividindo-se) todos os elementos de uma fila por um
∆= número, o determinante fica multiplicado (dividido) por esse número.

Com efeito, como em cada termo do determinante aparece um,
an1 an 2  ann
e só um, elemento da fila considerada, todos os termos ficarão
multiplicados (divididos) pelo número e, consequentemente, o
Obs.: O determinante de uma matriz cujos elementos são números inteiros determinante fica multiplicado (dividido) por esse número.
é um número inteiro, por ser a soma algébrica de produtos de números
inteiros. IV. Sendo k um escalar e A uma matriz n×n, então, det (k · A) = kn · det A.
Através da definição, podem-se obter regras práticas para o cálculo Com efeito, em k · A, cada linha de A fica multiplicada por k. Pela
de determinantes 2×2 e 3×3: propriedade (III), segue que o determinante fica multiplicado por

 ⋅ 
⋅ 
⋅  =  .
 
Determinante de 2a ordem
Como consequência, toda matriz antissimétrica de ordem ímpar tem
O determinante de uma matriz de 2a ordem é a diferença entre o determinante nulo. De fato, se A é uma matriz antissimétrica, então
produto dos elementos da diagonal principal e o produto dos elementos A = – At. Aplicando det dos dois lados e usando as propriedades (I) e
da diagonal secundária. (IV): det A = det (–At) = (–1)n det At = – det A ⇒ det A = 0.
Ou seja:
– + V. Um determinante muda de sinal quando se troca a posição de duas
a a filas paralelas.
 a        
a a Com efeito, uma troca no determinante ∆ = |aij| da posição de duas
filas paralelas implica que cada um dos termos do desenvolvimento
de ∆, supostos ordenados em relação aos índices de linha, terá uma
Determinante de 3a ordem (regra de Sarrus) troca de dois elementos na permutação dos índices das colunas, e pelo
teorema 3, cada um dos termos do desenvolvimento troca de sinal.
Repete-se, após a 3a coluna, a 1a e a 2a, respectivamente (o mesmo
pode ser feito com as linhas).
VI. Um determinante que possui duas filas paralelas iguais é nulo.
Somam-se os produtos dos três elementos da diagonal principal e Com efeito, trocando a posição dessas duas filas, o determinante
das diagonais paralelas a ela. não se altera, e pela propriedade anterior muda de sinal, logo,
Subtraem-se os produtos dos três elementos da diagonal secundária ∆ = –∆ → ∆ = 0.
e das paralelas a ela.
Somam-se algebricamente os resultados obtidos. VII. Um determinante que possui duas filas paralelas proporcionais é nulo.
Com efeito, seja ∆= |aij|um determinante em que os elementos de
a11 a12 a13 a11 a12
uma fila são os produtos do fator k pelos elementos correspondentes de
∆ = a21 a22 a23 a21 a22 = outra fila paralela. Se k ≠ 0, podemos dividir a fila considerada por k e
a31 a32 a33 a31 a32 ∆ = k · ∆’, em que ∆’ tem duas filas paralelas iguais e, portanto, é nulo.
– – – + + +
= a11a22a33 + a12a23a31 + a13a21a32 – a13a22a31 – a11a23a32 – a12a21a33 VIII. Um determinante em que são nulos todos os elementos de uma das
bandas da diagonal principal reduz-se ao seu termo principal.
Ex.: a11 0 0  0
    a21 a22 0  0
∆ =      =      +     +      −     −      −     = Com efeito, seja ∆ = a31 a32 a33  0
    
=  +  −  −  =  an1 an 2 an 3  ann

118 Vol. 3
Matrizes e determinantes

Como em cada termo deve figurar um e apenas um elemento de Dem.: De fato, assim como no caso 3×3, basta lembrarmos que na
cada linha e um e apenas um elemento de cada coluna e como na definição de determinantes aparecem todos os produtos possíveis com
primeira linha há um único elemento não nulo a11, só não se anularão os elementos da matriz, tendo cada parcela exatamente um elemento de
no desenvolvimento de ∆ os termos em que figura o fator a11. Na cada linha e um elemento de cada coluna.
segunda linha há dois elementos não nulos, a21 e a22, mas como a21 Assim fixada uma linha i, por exemplo, todas as parcelas terão algum
não pode figurar em termo que contenha a11, todos os termos não elemento aij dessa linha, j ∈ {1, 2, 3, ..., n}.
nulos do desenvolvimento de ∆ contêm o fator a11 · a22. Prosseguindo, Se tivéssemos i = j = 1, seria fácil perceber que colocando a11 em
obtemos: ∆ = a11 · a22 · ... · ann. evidência, teríamos o a11 multiplicando o det sem a 1a linha e a 1a coluna.
IX. Um determinante em que são nulos todos os elementos de uma das No caso de um elemento aij qualquer, temos um problema com o
bandas da diagonal secundária reduz-se ao produto de (–1)C pelo n,2 número de inversões, já que os produtos são os mesmos que aparecem
produto dos elementos secundários. no determinante tirando a linha i e a coluna j, mas os sinais podem ser
Com efeito, com raciocínio análogo ao anterior, verifica-se que o diferentes.
único termo não nulo de ∆ é o formado pelos elementos secundários. Como é mais fácil de visualizar o que ocorre quando se tira a 1a linha
T = (–1)k a1n . a2,n–1 · ... · an1. Nesse caso, k é o número de inversões e a 1a coluna, o que seria necessário para levar a linha i e a coluna j para
da permutação (n, n – 1, ... , 2, 1) que é, com já sabemos, Cn,2. essas posições?
Se trocássemos a linha i e a coluna j com cada uma das linhas
3.4 Cofator e Teorema de LaPlace e colunas anteriores, levaríamos essas filas para posições iniciais e
Cofator manteríamos as demais na mesma ordem da matriz original (que é o que
ocorre com o cofator); assim seriam necessárias (i – 1) + (j – 1) trocas.
Definimos o cofator de um elemento aij de uma matriz A através do
seguinte modo: Em cada troca de filas alteramos o sinal, em que teremos (–1)i + j + 2
= (–1)i + j.
Inicialmente, traçamos uma reta vertical e outra horizontal por aij,
riscando alguns elementos da matriz. Assim, colocando aij em evidência, este ficará multiplicado por (–1)
i+j
e pelo determinante da matriz sem a linha i, coluna j, ou seja, pelo
Calculamos o determinante ∆ij da matriz constituída dos termos não
cofator de aij.
riscados, matriz esta chamada de menor complementar de aij.
O cofator de aij é dado por Aij = (–1)i+j ∙ ∆ij. 3.5 Teorema das filas e Teorema de Jacobi
Ex.: O cofator do elemento da 2a linha e 3a coluna do determinante Teorema das filas
1 2 5 Um determinante em que os elementos de uma fila são compostos
2+3 1 2 por somas de p parcelas é igual a uma soma de p determinantes, obtidos
∆ = −3 2 2 é: A23 = ( −1) · = ( −1) · (1 · 4 − 2 · 1) = −2.
1 4 do determinante dado, tomando-se no lugar da fila composta as primeiras,
1 4 3 segundas, etc., parcelas e conservando todas as outras filas.
Dem.: Seja D = det(aij)n·nem que os elementos da linha i são da forma:
Sabendo o conceito de cofator, vejamos, por exemplo, o que acontece
quando desenvolvemos um determinante 3×3: ai1 = a1 + b1 + ... + l1
a11 a12 a13 ai2 = a2 + b2 + ... + l2
∆ = a21 a22 a23 = .......
a31 a32 a33 ain = an + bn + ... + ln

= a11a22 a33 + a12 a23 a31 + a13 a21a32 − a13 a22 a31 − a11a23 a32 − a12 a21a33 = Desenvolvendo ∆ segundo os elementos da i-ésima linha, tem-se:
= a11( a22 a33 − a23 a32 ) − a12 (aa21a33 − a23 a31) + a13 ( a21a32 − a22 a31) =
∆ = (a1 + b1 + ... + l1)Ai1 + (a2 + b2 + ... + l2)Ai2 + ... + (an + bn +
a a23 a a a a ... + ln)Ain =
= a11 22 − a12 21 23 + a13 21 22 = a11 A11 + a12 A12 + a13 A13
a32 a33 a31 a33 a31 a32 (a1Ai1 + a2Ai2 + ... + anAin) + ( 1Ai1 + b2Ai2 + ... + bnAin) + ... +
(l1Ai1 + l2Ai2 + ... + lnAin);

De fato, o que fizemos com os elementos da primeira linha, poderíamos e pelo Teorema de LaPlace, temos:
ter feito com os elementos de qualquer outra linha ou coluna, de modo
que, para calcular o determinante 3×3 pode-se escolher qualquer linha ou a11  a1n a11  a1n a11  a1n
coluna e somar os produtos das entradas dessa fila pelos seus respectivos   
cofatores. ∆ = a1  an + b1  bn + ... + l1  ln
Na verdade, esse resultado não vale apenas para determinantes 3 × 3;   
seja A uma matriz n × n, vale o seguinte resultado:
an1  ann an1  ann an1  ann
Teorema de LaPlace
Um determinante sempre é igual à soma dos produtos dos elementos Ex.:
de uma fila pelos seus respectivos cofatores: 1 2 5 1 2 5 0 2 5
n n
2 + x2 1 −2 = 2 1 −2 + x 2 1 −1
∆ = ∑ aij . Aij ou ∆ = ∑ aij ⋅ Aij
j =1 i =1 5 + x3 3 x2 5 3 x2 x3 3 x2

IME-ITA 119
Matemática III – Assunto 5

Teorema de Jacobi Ex.:


Um determinante não se altera quando soma-se a uma fila, 1 2 4 2
7 − 6 5 − 12 6 − 6 1 −7 0
combinações lineares de filas paralelas. 3 7 5 6
= 10 − 2 −4 − 4 5 − 2 = 8 −8 3 =
Dem.: Seja ∆ = det (aij)n×n, formemos o determinante ∆’ em que todas 1 10 −4 5
as linhas são iguais às de ∆, exceto a i-ésima, que é obtida através da 8 − 6 2 − 12 3 − 6 2 −10 −3
3 8 2 3
i-ésima linha somada com uma combinação linear das demais, ou seja,
−8 + 56 3 − 0 48 3
= = = −144 − 12 = −156
Li ' = Li + λ1L1 + λ2 L2 + ... + λi −1Li −1 + λi +1Li +1 + ... + λn Ln , e m q u e L j −10 + 14 −3 − 0 4 −3
representa a linha j.
Pelo teorema das filas, podemos “quebrar” ∆’ em vários determinantes
em que a linha i ficará com cada uma das parcelas da linha Li’. Nesse Obs.:
caso, todos os determinantes terão filas proporcionais, exceto um que I. Se na matriz M, a11 ≠ 1 e existir algum outro elemento igual a 1,
será exatamente igual ao original, ou seja, teremos: ∆’ = ∆. podemos, através de troca de filas paralelas, transformar M em uma
outra matriz que tenha a11= 1.
3.6 Abaixamento de ordem de um II. Se não existir em M nenhum elemento igual a 1, podemos, usando o teorema
de Jacobi, obter uma nova matriz M’ que tenha um elemento igual a 1.
determinante (regra de Chió)
Como consequência do teorema de Jacobi, veremos agora um 3.7 Teorema de Binet
processo bastante prático para reduzirmos em uma unidade a ordem de um Se A e B são matrizes quadradas então det(AB) = det(A) · det(B).
determinante n ≥ 2, sem alterá-lo, e consequentemente facilitar seu cálculo. Dem.: Faremos apenas o caso 2×2; o caso n x n foge ao escopo do assunto.
Consideremos uma matriz M de ordem n ≥ 2, tal que a11= 1, isto é:  a b e f 
S e j a m A=  e B=  , nesse caso:
 c d   g h
 1 a12 a13 ..... a1n 
   ae + bg af + bh 
 a21 a22 a23 ..... a2 n  AB =  
M =  a31 a32 a33 ..... a3 n   ce + dg cf + dh 
 
..... ...... ..... ..... .....
a Em que: det( AB) = ( ae + bg)( cf + dh) − ( af + bh)( ce + dg) =
 n1 an 2 an 3 ..... ann 
= aecf + aedh + bgcf + bgdh − afce − afdg − bhce − bhdg =
Adicionemos à 2a coluna, a 1a multiplicada por – a12 = ( ad( eh − fg) − bc( eh − fg)) = ( ad − bc)( eh − fg) = det A · det B
Adicionemos à 3a coluna, a 1a multiplicada por – a13
....................................................................................
Adicionemos à j-ésima coluna, a 1a multiplicada por – a1j Uma consequência direta do teorema de Binet é que se A é uma matriz
.................................................................................... inversível, então:
Adicionemos à n-ésima coluna, a 1a multiplicada por – a1n AA–1 = I ⇒ det A–1 · det A = 1, de modo que det A ≠ 0 e:
Obteremos a matriz M’ tal que det M’ = det M.
1
det A−1 =
1 0 0 .......... 0 det A
a21 a22 − a21a12 a23 − a21a13
.......... a2n − a21a1n
det M' = a31 a32 − a31a12 a33 − a31a13 .......... a3n − a31a1n 3.8 Determinante de Vandermonde
..... .................. .................. .......... ................... Chama-se determinante de Vandermonde de base (a1, a2, ..., an–1, an)
ao determinante:
an1 an2 − an1a12 an3 − an1a13 .......... ann − an1a1n
1 1  1 1
Pelo teorema de LaPlace: a1 a2  an −1 an
V ( a1, a2 , ... , an −1, an ) = a12 a22  an2−1 an2
a22 − a21a12 a23 − a21a13 ............. a2n − a21a1n


a32 − a31a12 a33 − a31a13 ............. a3n − a31a1n
det M' = a n −1
a2n −1  ann−−11 ann −1
.................. .................. ............. .................. 1

an2 − an1a12 an3 − an1a13 ............. ann − an1a1n ( n −1) x ( n −1) Um determinante de Vandermonde é igual ao produto de todas as
diferenças obtidas subtraindo cada elemento a1, a2, ...,an–1,ande todos os
que o seguem, ou seja,
Isso pode ser resumido através da regra conhecida como regra de Chió:
1 1  1 1
I. Desde que M tenha a11 = 1, suprimimos a 1 linha e a 1 coluna de a a a1 a2  an −1 an
M. V ( a1, a2 , ... , an −1, an ) = a12 a22  an2−1 an2 =
II. De cada elemento restante na matriz subtraímos o produto dos
elementos que se encontram nas “extremidades das perpendiculares” 
n −1
traçadas do elemento considerado à 1a linha e à 1a coluna. a 1 a2n −1  ann−−11 ann −1
III. Com as diferenças obtidas, construímos uma matriz de ordem (n – 1) = ( a2 − a1)( a3 − a1)...( an − an −1)
cujo determinante é igual ao de M.
Dem.: A demonstração foge do escopo do assunto.

120 Vol. 3
Matrizes e determinantes

4. Matriz inversa Matrizes semelhantes


Duas matrizes A e B são ditas semelhantes quando existe uma matriz
Matriz inversa: inversível P tal que A = P–1 BP.
Como visto anteriormente dizemos que A é inversível se, e somente
se, existe B tal que: Propriedades
AB = BA = I. Nesse caso dizemos que B é a inversa de A, e a I. Se A e B são semelhantes, então para todo (∀λ ∈ ℜ), det (A – λI) =
representamos por A–1. det (B – λI).
Dem.: Considerando A e B semelhantes, então existe P inversível tal que
Lema: Seja A uma matriz quadrada de ordem n. Se somarmos os produtos
A = P–1 BP, logo: (∀λ ∈ ℜ), det(A – λI) = det(P–1 BP – λP–1 P) =
dos elementos de uma fila pelos cofatores dos elementos correspondentes
det[P–1 (B – λI) P] = det(B – λI)
em outra fila o resultado será nulo.
Na última igualdade foi usado o Teorema de Binet.
Dem.: Repare que os cofatores de uma fila de uma matriz não dependem
dos elementos dessa fila, uma vez que, no cálculo do cofator, são riscadas
II. Duas matrizes semelhantes sempre têm o mesmo traço.
sua linha e sua coluna.
Dem.: Basta lembrar que tr(XY) = tr(YX), assim se A e B são semelhantes:
Seja então M = [C1 ... Ci ... Cj ... Cn], em que Ci representa a coluna i. Repare
trA = tr(P–1 BP) = tr(PP–1 B) = tr(IB) = trB
que os cofatores da colunai dessa matriz M são os mesmos cofatores da
coluna i da matriz N obtida pela repetição da coluna j no lugar da coluna
III. Se A e B são semelhantes então Ak é semelhante a Bk, para todo k natural.
i, ou seja, N = [C1 ... Cj ... Cj ... Cn]. Nesse caso, como N possui duas
Dem.: Considere A = P–1 BP, então:
colunas iguais, det N = 0, porém, pelo Teorema de LaPlace aplicado em
Ak = A · A ... A = (P–1 BP)(P–1 BP) ... (P–1 BP) = P–1 Bk P
sua coluna i, a soma dos elementos da coluna j de M multiplicado pelos
cofatores da coluna i deve ser nula.
EXERCÍCIOS RESOLVIDOS
Teorema: Seja a matriz adjunta de A, a matriz obtida pela transposição
01 Na área de informática, as operações com matrizes aparecem
da matriz dos cofatores, ou seja, adjA = (cofA)t então: (adjA) · A = A ·
com grande frequência. Um programador, fazendo levantamento dos
(adjA) = (detA) · I
dados de uma pesquisa, utilizou as matrizes:
Dem.: Seja Aij = cof aij, então:
 1 3 2
5 2 1   
 A11 A21 A31 ..... An1   a11 a12 a13 ..... a1n  A=  ; B = 2 1 2 ; C = A × B. O elemento c23 da
     3 1 4   1 1 1
 A12 A22 A32 ..... An 2   a21 a22 a23 ..... a2 n 
adjA ⋅ A =  A13 A23 A33 ..... An 3  ⋅  a31 a32 a33 ..... a3 n  matriz C é igual a:
   
 ..... ..... ..... ..... .....   ...... ..... ..... ..... ..... 
A   ann  (A) 18. (D) 12.
 1n A2 n A3 n ..... Ann   an1 an 2 an 3 .....
(B) 15. (E) 9.
(C) 14.
Repare que na diagonal principal teremos os cofatores da coluna
k, multiplicando os elementos da coluna k, logo a diagonal será o det A
Solução: Letra E.
pelo Teorema de LaPlace. Fora da diagonal temos cofatores da linha i,
Para determinar o elemento da linha 2, coluna 3 do produto AB,
multiplicando cofatores da coluna j, com i ≠ j, donde pelo Lema, todos
basta tomar a 2a linha de A e multiplicar pela 3a coluna de B, assim:
os elementos fora da diagonal principal são nulos.
c23 = 3 · 1 + 1 · 2 + 4 · 1 = 9
É fácil ver que o mesmo ocorre se fizer A· (adjA).
Logo: 02 Indica-se por det A o determinante de uma matriz quadrada A. Seja
 π 
 det A 0 ... 0   sen  ( i + j )  , se i = j
a matriz A = (aij), de ordem 2, em que: aij =   4 
 
0 det A ... 0  sen  x( i − j ) , se i ≠ j
adjA ⋅ A = A ⋅ adjA =   ⋅ = ( det A) ⋅ I
  
     
 
 Quantos números reais x, tais que –2π < x < 2π, satisfazem a
 0 0 ... det A 
sentença det A = 1/4?
Corolário: A é inversível se, e somente se, det A ≠ 0.
De fato, já havíamos visto que se A é inversível então det A ≠ 0 (por (A) 10. (D) 4.
Binet). Para ver a volta, basta dividir a identidade do teorema anterior por (B) 8. (E) 2.
det A. Nesse caso: (C) 6.
1
A−1 =
detA
( adjA) Solução: Letra B.
π
a11 a12 sen sen( − x ) 1 − senx 1
det A = = 2 = = sen2 x =
Outra consequência desse resultado é que se BA = I, então a21 a22 senx 0 4
senx senπ
AB = I. Basta verificar que BA = I, implica det A ≠ 0, portanto A é inversível
e existe C tal que AC = CA = I. Já vimos anteriormente que isso implica 1  π 5π 7π 11π 
Logo, senx = ± , onde x ∈ ± ; ± ;± ;± .
B = C, portanto AB = I. 2  6 6 6 6 

IME-ITA 121
Matemática III – Assunto 5

03 Analise as afirmativas como V ou F, sendo A, B e C matrizes 3 x 3: Solução: Letra C.


Repare que pelo enunciado, podemos dizer que uma matriz é ortogonal,
a. (AB)3 = A3B3 quando sua transposta coincide com sua inversa, sendo assim:
b. A · (B + C) = AB + CA B = P–1AP. Aplicando det dos dois lados e usando o teorema de Binet,
det B = det(P–1 AP) = det P–1 · det A · det P = det A.
Solução: Outro resultado que poderia ser visto na questão é que B é simétrica, de
a. F. Veja que (AB)3 = (AB)(AB)(AB) = ABABAB. Lembre que, no produto fato: Bt = (Pt AP)t = Pt At (Pt)t = Pt AP = B
de matrizes, não podemos trocar a ordem (o produto não é comutativo!). Vejamos o que deve ocorrer para AB ser simétrica:
Portanto, não temos necessariamente AAABBB, que seria A3B3. (AB)t – BtAt = BA, em que AB é simétrica se, e somente se, AB =
b. F. Vale a propriedade distributiva. Portanto, A · (B + C) = AB + AC. BA. Deste modo as letras (a), (b) e (d) são equivalentes. (é fácil dar
Como não necessariamente AC = CA, a equação dada não precisa contraexemplos para AB = BA).
ser verdadeira. Para ver que B não precisa ser ortogonal, basta ver que toda matriz
ortogonal é inversível, e como, det B = det A, a matriz B é inversível
04 se, e somente se, A também o for. Como não foi afirmado nada sobre
a. Seja A uma matriz quadrada. Prove que A + At é uma matriz simétrica. A nesse sentido, não podemos garantir isso.
b. Seja A uma matriz quadrada. Prove que A – At é antissimétrica.
c. Seja A uma matriz quadrada. Prove que A · At é uma matriz simétrica.  0 1
07 Existe matriz X de ordem 2 tal que X 2 =  ?
d. Seja A uma matriz quadrada. Prove que A pode ser escrita como 0 0
soma de uma matriz simétrica mais uma antissimétrica.
 a b 2
 a2 + bc ab + bd 
Solução: Solução: Seja   ⇒ X =  , em que: ac + cd = 0
c d  ac + cd bc + d 2 
a. Seja X = A + At. Para provar que X é simétrica, devemos provar
que Xt = X. Veja que: Xt = (A + At)t, então Xt = At + (At)t = At + A. ⇒ c=0 ou d=– a.
Portanto, Xt = X e X é simétrica. 1O Caso: c = 0. Como a2 + bc = bc + d2 = 0, temos a = d = 0, em
b. Seja X = A – At. Para provar que X é simétrica, devemos provar que ab + bd = 0 = 1. Absurdo!
que Xt = X. Veja que: Xt = (A – At)t, então Xt = At – (At)t = At – A. 2O Caso: d = – a. Nesse caso, ab + bd = 0 = 1. Absurdo!
Portanto, Xt = – X e X é antissimétrica. Logo não existe matriz X que satisfaz o sistema.
c. Seja X = AAt. Para provar que X é simétrica, devemos provar que
Xt = X. Veja que: Xt = (AAt)t, então Xt = (At)tAt = AAt. Portanto, 08 Determine os valores de x (x é um escalar) tais que det(A – xI) = 0,
Xt = X e X é simétrica. 1 −2
t em que A =   e I representa a matriz identidade 2x2.
d. Pelas letras a e b têm-se A + A simétrica e A − A antissimétrica,
t
1 4 
2 2 1 −2  x 0  1 − x −2 
 A + At   A − At  Solução: Veja que A − xI =  − = .
em que: A =  +  é a soma de uma matriz simétrica 1 4   0 x   1 4 − x
 2   2  Com isso, (A – xI) = (1 – x)(4 – x) – 1 · (–2), ou seja, det (A – xI) =
com uma antissimétrica. x2 – 5x + 6, que tem raízes 2 e 3.

05 Sejam m e n números reais tais que m ≠ n e as matrizes a b c


2 1  −1 1 09 Sendo x y z = 1, determine o valor de:
A= e B=  . Qual a relação necessária entre m e n
3 5   0 1 u v w
para que a matriz não seja inversível?
a + 2x b + 2y c + 2z
2 m − n m + n  D = 3u 3v 3w .
Solução: C =  .
 3m 5m + n x y z
Uma matriz não é inversível se, e somente se, seu determinante é nulo.
Logo: a b c 2x 2y 2z
( 2m − n ) ( 5 m + n ) − ( m + n ) · (3 m) = 7 m2 − 6 mn − n2 = 0, fatorando: Solução: Pelo teorema=
das filas, D 3u 3v 3w + 3u 3v 3w . O
6 m2 − 6 mn + m2 − n2 = 0 ⇒ 6 m( m − n) + ( m + n)( m − n) = x y z x y z
= ( m − n)(7 m + n) = 0 2o determinante é nulo, porque tem duas linhas proporcionais. No 1o,
Como m ≠ n, segue que 7m + n = 0. a b c
podemos colocar o fator 3 da 2 linha para fora, e temos: D = 3 ⋅ u v w .
a

06 Sejam A e P matrizes reais quadradas de ordem n tais que A é x y z


simétrica (isto é, A = At) e P é ortogonal (isto é, PPt = I = PtP), P
diferente da matriz identidade. Se B = PtAP, então:
a b c
(A) AB é simétrica. (D) BA = AB Trocando as duas últimas linhas de lugar, temos: D = −3 ⋅ x y z = −3.
(B) BA é simétrica. (e) B é ortogonal. u v w
(C) det A = det B

122 Vol. 3
Matrizes e determinantes

a x x x x  m 2 3
x a x x x 11 Para que valores de m possui inversa a matriz  m 3 4  ?
 
10 Calcule D = x x a x x em função de x e a.  1 2 1
Solução: Sabe-se que uma matriz possui inversa se, e somente se, seu
x x x a x  m 2 3
x x x x a determinante for não nulo. Portanto, precisamos ter  m 3 4  ≠ 0.
 
 1 2 1
Solução: As somas dos elementos de cada linha são todas iguais. Isso
Utilizando a regra de Sarrus, temos – m – 1 ≠ 0, o que nos dá m ≠ –1.
nos dá a ideia de fazer a operação C1 → C1 + C2 + C3 + C4 + C5 (essa
ideia é muito útil, pois faz aparecer na 1a coluna as somas das linhas).
x z y
a + 4x x x x x 12 Calcule o determinante y x z de duas maneiras e obtenha um
a + 4x a x x x resultado de fatoração. z y x
Daí, temos D = a + 4 x x a x x . Colocando o fator comum da 1a Solução: Usando a regra de Sarrus, temos que o determinante é igual
a + 4x x x a x a
a + 4x x x x a x3 + y3 + z3 – 3xyz. Por outro lado, como as somas das linhas são todas
iguais, podemos fazer C1 → C1 + C2 + C3 e temos que o determinante
1 x x x x x+y+z z y
1 a x x x é igual a: x + y + z x z .
coluna em evidência, temos que D= ( a + 4 x ) ⋅ 1 x a x x . x+y+z y x
1 x x a x
Colocando o fator comum da 1a coluna em evidência, temos que o
1 x x x a
1 z y
determinante é ( x + y + z ) 1
⋅ x z.
Agora, usando a regra de Chió, temos:
1 y x

a− x 0 0 0 = (x + y + z) . (x2 + y2 + z2 – xy – yz – xz).
0 a− x 0 0
D= ( a + 4 x ) ⋅ , logo, ∆ = (a + 4x) . (a – x)4. Portanto, x3 + y3 + z3 – 3xyz = (x + y + z) . (x2 + y2 + z2 – xy – yz –
0 0 a− x 0
xz), que é uma conhecida identidade algébrica.
0 0 0 a− x

EXERCÍCIOS NÍVEL 1

3 5 04 Toda matriz de ordem 2 x 2, que é igual à sua transposta, possui:


  4 
01 Considere as matrizes: A =  2 1  , B =   e C =  2 1 3  (A) pelo menos dois elementos iguais.
 0 −1 3  (B) os elementos da diagonal principal iguais a zero.
(C) determinante nulo.
A adição da transposta de A com o produto de B por C é: (D) linhas proporcionais.
(E) todos os elementos iguais a zero.
(A) impossível de se efetuar, pois não existe o produto de B por C.
(B) impossível de se efetuar, pois as matrizes são todas de tipos diferentes.  2 1 1 2 
(C) impossível de se efetuar, uma vez que não existe a soma da transposta A  3 2=
05 Sendo= e B  3 − 3 , determine o valor de 2A – B.
 −4 0  2 1 
de A com o produto de B por C.    

( ) ( ) ( )
(D) possível de se efetuar e o seu resultado é do tipo 2 x 3. 2 1 , B −= 1 2 e C 4 − 1 , então determine a
(E) possível de se efetuar e o seu resultado é do tipo 3 x 2. =
06 Se A =
3 −1 1 0 2 1

02 (AFA-2002) As matrizes A, B e C são do tipo m x 3, n x p e 4 x r, X − A B+ X


matriz X de ordem 2, tal que: = + C.
respectivamente. Se a matriz transposta de (ABC) é do tipo 5 x 4, então: 2 3

(A) m = p. (C) n + p = m + r.
07=
Sejam A (=
1 4) (x y )
1 2 e B 2 − 1 duas matrizes. Se B é a inversa de

(B) mp = nr. (D)


r = n.
A, então determine x + y.
03 Sejam A e B matrizes. Prove que se AB e BA existem, então AB e BA
são quadradas.
08=
Dadas A (=
0 − 2) (3 − 51) e B 131 (75a 10b ), determine os
3 0 , P 2=

valores de a e b, tais que B = PAP–1.

IME-ITA 123
Matemática III – Assunto 5

09 Define-se distância entre duas matrizes A =(aij) e B = (bij) quadradas 2 1 2


e de mesma ordem n pela fórmula: d(A, B) = max|aij – bij|, ∀i,j|1n. Assim,  
(C) 2 2 1
 1 3  −1 −4   1 2 2
=
determine a distância entre as matrizes: A = eB  
4 7 4 2
2 1 2
 
10 Sejam A e B matrizes reais 3×3. Se tr(A) denota a soma dos elementos (D)  1 2 2
da diagonal principal de A, considere as afirmações:  1 2 2

I. tr(At) = tr(A)  1 2 2
II. Se A é inversível, então tr(A) ≠ 0.  
(E) 2 2 1
III. tr(A + λB) = tr(A) + λtr(B), para todo λ ∈ R.
 1 1 2
(A) Todas as afirmações são verdadeiras.
cos θ − senθ 
(B) Todas as afirmações são falsas. 14 (EN-1983) Se cada θ real define a matriz: Tθ =   então,
(C) Apenas a afirmação I é verdadeira. o produto Tα · Tβ é igual a: senθ cosθ 
(D) Apenas a afirmação II é falsa.
(E) Apenas a afirmação III é falsa. Tα + β Tα − β
(A) (D)
2 2
11 O produto das matrizes:
(B) Tα + β (E) Tα – β
 a b c d
A=  eB=  é tal que: (C) T2(α – β)
 b a d c 
 a b
15 (ITA-1983) Seja a matriz A =   , em que a = 2 2 ; b = 2
(1 + log 5) (log 8)
2
 ac bd  c d 
(A) AB =  
 bd ac  c = log 3 81; d = log 3 27. Uma matriz real quadrada B, de ordem 2, tal

que AB é a matriz identidade de ordem 2 é:


 ad bc 
(B) AB =  
 bd ac  log 27 2   2 −3 / 2
(A)  3  (D)
 
 2 log 81  −3 / 2 log2 5 
3
 ac + bd 
(C) BA =  
 bd + ac   −3 / 2 2  log2 5 3 log 3 81
(B)   (E)
 
 3 −5   5 −2log2 81 
 abcd abcd 
(D) BA =  
 abcd abcd   −3 / 2 2 
(E) AB = BA, para quaisquer valores de a, b, c, d. (C)  
 2 −5 / 2

12 (UFRJ) Considere as matrizes:


16 (ITA-1991) Sejam m e n números reais com m ≠ n e as matrizes:
19941994 19941994   1 −1
A=  eB =   . Seja A2 = A · A e B2 = B · B.
19941994 19941995   −1 1   2 1  −1 1 
A= , B =  
Determine a matriz C = A2 – B2 – (A + B) (A – B). 3 5  0 1

0 1 0   a  b Para que a matriz mA + nB seja não inversível é necessário que:


13 Multiplicando-se A = 0 0 1 por X =  b  , obtemos AX =  c  , uma
 1 0 0   c   a  (A) m e n sejam positivos.
(B) m e n sejam negativos.
permutação dos elementos de X. Existem cinco outras matrizes da mesma (C) m e n tenham sinais contrários.
ordem da matriz “A”, com apenas elementos 0 e 1, que, multiplicadas por (D) n2 = 7 m2.
X, formam as outras permutações dos elementos de X. A soma dessas (E) n.d.a.
cinco matrizes é:
17 (ITA-1994) Seja a uma matriz real quadrada de ordem n e B = I – A,
 1 2 2 em que I denota a matriz identidade de ordem n. Supondo que A é inversível
(A) 2 1 2 e idempotente (isto é, A2 = A), considere as afirmações:
2 2 1
I. B é idempotente. IV. A2 + B2 = I
2 1 2 II. AB = BA V. AB é simétrica.
  III. B é inversível.
(B)  1 2 2
2 2 1

124 Vol. 3
Matrizes e determinantes

Com respeito a estas afirmações, temos: 26 Marlos Charada, o matemático espião, concebeu um código para
transformar uma palavra P de três letras em um vetor Y de R3 como descrito
(A) Todas são verdadeiras. a seguir.
(B) Apenas uma é verdadeira. A partir da correspondência:
(C) Apenas duas são verdadeiras. A B C D E F G H I J L M N O P Q R S T U V X Z
(D) Apenas três são verdadeiras.                       
(E) Apenas quatro são verdadeiras. 1 2 3 4 5 6 7 8 9 10 111 12 13 14 15 16 17 18 19 20 21 22 23
A B C D E F G H
I J L M N O P Q3 R S T U V X Z
A palavra P é transformada em um vetor X de R . Em seguida, usando a
                      
 1 m
18 Seja x =   uma matriz quadrada 2 × 2 em que m é um número 1 2 3 4 5 6 7 2 8 92 100 11 12 13 14 15 16 17 18 19 20 21 22 23
0 1  matriz código A = 3 3 1 , o vetor Y é obtido pela equação Y = AX.
inteiro qualquer. Se P = (aij) é uma matriz definida por P = Xn + Xn–1 +  1 0 1
Xn–2 + ... + X em que n é um número inteiro positivo (n ≥ 1), então podemos
afirmar que: Por exemplo, a palavra MAR corresponde ao vetor X = (12, 1, 17) e é
codificada como Y = A X = (26, 59, 29). Usando o processo acima,
n( n + 1) decodifique Y = (64, 107, 29).
(A) um elemento aij da matriz P é igual a m .
2
n( n − 1) 0 x +1 1
(B) um elemento aij da matriz P é igual a m .
2 27 Resolva a equação: 1 −4 x −1 =
0.
m( m − 1) 1 1− 3 x 1
(C) um elemento aij da matriz P é igual a n .
2
(D) P é uma matriz cujos elementos são todos inteiros, se, e somente se,
m é par. 28 Sendo A uma matriz 3×3 tal que det A = 2, calcule o determinante
(E) n.r.a. da matriz 4 A–1.

19 Determine todas as matrizes X de ordem 2 e com elementos reais tais 1 0 −1 2


que X2 = – I. 0 1 2 1
29 Calcule o determinante D = .
3 5 0 2
( )
20 Dada A = i 0 , em que i2 = –1, deduza a fórmula para as potências
0 i 0 1 −1 4

inteiras positivas de A. 1 1 1  1
1 1− x 1  1
21 Sejam M e B matrizes quadradas de ordem n tais que M – M–1 = B. 30 Resolva a equação: 1 1 2 − x  1 =
0.

Sabendo que Mt = M–1 , podemos afirmar que: 1 1 1  n− x
(A) B2 é a matriz nula. (D) B é hemissimétrica.
(B) B2 = –2I. (E) n.r.a. 1 1 1  1
(C) B é simétrica. 1 0 1  1
31 Calcule o determinante 1 1 0  1 .

22 Se A e B são matrizes de ordem n, das afirmativas abaixo, quais são 1 1 1  0 n xn
verdadeiras? Justifique.

(I) (A + B)2 = A2 + 2AB + B2 32 Calcule o determinante:


(II) (A + B)(A – B) = A2 – B2 1 x x2  x n−2 x n −1
(III) Se A . B = 0, então A = 0 ou B = 0 x n −1 1 x  x n −3 x n−2
n−2 n −1 n−4
(IV) Se A = P–1 BP, então A4 = P–1B4P x x 1  x x n −3

x x2 x3  x n −1 1
23 Sejam A, B matrizes n x n tais que A2 = 0, B2 = 0 e (A + B)2 = 0.
Mostre que (AB)3 = 0.
 cos α -sen α 
1 2 3  33 (AFA-2001) Considere T ( α ) =   matriz quadrada
24 Determine a inversa da matriz A =  2 4 5 .  sen α cos α 
3 5 6
  definida para todo α real. Sendo cof (T(α)) e det (T(α)), respectivamente,
a matriz cofatora e o determinante da matriz T(α), é correto afirmar que:
25 São dadas duas matrizes, A e B, quadradas de ordem p. A matriz Ip
e a matriz 0p são, respectivamente, a matriz identidade e a matriz nula, (A) T(– α) = –T(α)
quadradas, de ordem p. Nessas condições: (B) cofT(α) = T(– α)
(C) T(– α) = (T(α))– 1
(A) AB = BA. (D) Det(T(2α)) = 4 det(T(α))
(B) Se AB = 0p então BA = 0p.
(C) Se AB = Ip então BA = Ip.
(D) AB = BA se e só se AB = I.
(E) n.r.a.

IME-ITA 125
Matemática III – Assunto 5

2x 8x 0 1 1 1 1
34 (AFA-2000) O produto das raízes da equação log2 x log2 x 2 0 =0 2 3 5 4
com x ∈ *+ , é 40 Calcule o determinante: D = .
1 2 3 4 9 25 16
8 27 125 64
(A) 1/2. (C) 4/3.
(B) 3/4. (D) 3/2.
41 Determine, usando o método dos cofatores, a inversa da matriz
35 (AFA-2001) Sejam A uma matriz quadrada de ordem 3, det A = d,  2 1 3
det(2A · At) = 4k, em que At é a matriz transposta de A, e d é a ordem da  −2 1 2 
 .
matriz quadrada B. Se det B = 2 e det 3B = 162, então o valor de k + d é:  0 2 3 

(A) 4. (C) 32.


(B) 8. (D) 36. EXERCÍCIOS NÍVEL 2
01 A matriz B, de ordem 4, é tal que B4 = 0. Mostre que a matriz inversa
36 (ITA-1981) Dizemos que uma matriz real quadrada A é singular, se de I – B é I + B + B2 + B3 .
det A = 0, ou seja, se o determinante de A é nulo, e não singular se det A
≠ 0. Mediante essa definição qual das afirmações abaixo é verdadeira? 02 Uma matriz A é dita nilpotente, se existe n ∈ N, tal que An = 0. O menor
n com essa propriedade é dito índice de A.
(A) A soma de duas matrizes, A e B, é uma matriz singular, se det A = det –B.
(B) O produto de duas matrizes é uma matriz singular se, e somente se, (A) Dê um exemplo de matriz 3 x 3 de índice 3.
ambas forem singulares. (B) Se A é matriz nilpotente de índice 2, calcule (I – A)–1 em função de I e
(C) O produto de duas matrizes é uma matriz singular se pelo menos uma A.
delas for singular. (C) Se A é matriz nilpotente de índice n, determine (I – A)–1 em função de
(D) Uma matriz singular possui inversa. I, A, A2,..., An – 1.
(E) A transposta de uma matriz singular é não singular.
03 Seja A uma matriz m x n de elementos reais. Mostre que se tr(AAt) = 0,
37 (ITA-1995) Dizemos que duas matrizes n×n, A e B, são semelhantes então a matriz A é, necessariamente, nula.
se existe uma matriz n×n inversível P tal que B = P–1AP. Se A e B são
matrizes semelhantes quaisquer, então: 04 (UFC) A matriz quadrada M, de ordem n > 1, satisfaz a equação
M2 = M – I, em que I é a matriz identidade de ordem n > 1. Determine,
(A) B é sempre inversível. em termos de M e I, a matriz M2003.
(B) Se A é simétrica, então B também é simétrica.  1 1 1
(C) B2 é semelhante a A.
05 Determine An dado que A =  0 1 1.
(D) Se C é semelhante a A, então BC é semelhante a A2.
 0 0 1
(E) det(λI – B) = det(λI – A), em que λ é um real qualquer.  

38 (ITA-93) Sabendo-se que a soma das raízes da equação 1 1 x


 
 1 −1 0 2  06 Considere a matriz C =  2 1 y  . Determine os valores de x, y e z
  3 4 z 
 
 x 0 x 0  = 0 é –8/3 e que S é o conjunto destas raízes, podemos
0 b x x  para que os elementos da diagonal principal de C–1 sejam todos iguais a
  1.
 b x 2 b 
07
afirmar que: a. Prove que se A é matriz 2 x 2, então A2 – (trA) . A + (det A) . I = 0.
b. Prove que se A é matriz 2 x 2 tal que A3 = 0, então A2 = 0.
(A) S ⊂ [–17, –1] (D) S ⊂ [–10,0]
(B) S ⊂ [1, 5] (E) S ⊂ [0, 3] ax + b
(C) S ⊂ [–1, 3] 08 Seja F o conjunto das funções homográficas=
(f ( x ) , ad − bc ≠ 0).
cx + d
39 (ITA-1992) Seja C = {X ∈ M2×2; X2 + 2X = 0}. Dadas as afirmações: Seja M2 o conjunto das matrizes 2 × 2. Sendo P: F → M2 que leva
ax + b  a b
=
f( x ) , ad − bc ≠ 0 em P( f ) =  :
I. Para todo X ∈ C, (X + 2I) é inversível. cx + d c d
II. Se X ∈ C e det(X + 2I) ≠ 0 então X não é inversível.
III. Se X ∈ C e det X ≠ 0 então det X> 0. a. mostre que P(fog) = P(f) · P(g);
b. conclua que P(fofo...of) = [P(f)]n.
Podemos dizer que: (cos θ) x − sen θ
c. Seja f ( x ) = , calcule fofo...of.
(sen θ) x + cos θ
(A) todas são verdadeiras.
(B) todas são falsas. 09 (EN-1983) Se a, b e c são as medidas dos lados opostos aos ângulos
(C) apenas II e III são verdadeiras. opostos aos ângulos A, B e C do triângulo ABC, então o determinante:
(D) apenas I é verdadeira.
(E) n.d.a.

126 Vol. 3
Matrizes e determinantes

1 1 1 18 Sabendo que 11843, 13273, 26325, 70824 e 92443 são múltiplos


de 13, prove que ∆ também o é.
∆= a b c é nulo:
senA senB senC 1 1 8 4 3
1 3 2 7 3
(A) somente se a = b = c.
(B) somente se a2 = b2 + c2. D =2 6 3 2 5
(C) somente se a > b > c. 7 0 8 2 4
(D) somente se a = b. 9 2 4 4 3
(E) quaisquer que sejam a, b e c.
19 Sejam p < m dois inteiros positivos. Calcule:
 2 2 2 
 
10 Considere a equação: det  G( x ) 2x F( x )  = 0, em que :  m  m  m
 2 2     ...  
0 1  p
2
 G( x ) 4x  F( x ) 

x + x − x + 1 G( x ) = x − 1
4 3 2
 m + 1  m + 1  m + 1
F( x ) = e com x∈ R, x ≠ 0. Sobre as raízes     ...  
x2 x  0   1   p 
reais dessa equação, temos:    

(A) Duas delas são negativas.  m + p  m + p  m + p


    ...  
(B) Uma delas é um número irracional.  0   1   p 
(C) Uma delas é um número par.
(D) Uma delas é positiva e outra negativa. 20 Calcule o determinante:
(E) n.d.a.

11 (ITA) Julgue: Sejam A, B e C matrizes quadradas n × n tais que A e a0 a1 a2  an −1 an


B são inversíveis e ABCA = At, então det C= det(AB)–1. −1 x 0  0 0
0 −1 x  0 0
12 (EFOMM) Sejam A, B e C matrizes de ordem 3 × 3 inversíveis tais 0 0 −1  0 0

 1  0 0 0  x 0
que det A–1 = 3 e det  ( AB ) + I  = 4. Sabendo-se que I é a matriz
−1

 2  0 0 0  −1 x
identidade de ordem 3, tal que I = –3C–1 · (2B–1 + A)t, o determinante de
a 0 0 b
C é igual a: 0 a  b 0
21 Calcule o determinante  .
(A) – 8/3. (D) – 54. 0 b  a 0
b 0  0 a 2 n ×2 n
(B) – 32/3. (E) – 288.
(C) – 9. 22 Sendo ∆n o valor do determinante tridiagonal n × n
2 3
a b c 1 a a
2 2 2 5 2 0  0 0
13 Prove que: a b c = 1 b2 b2
3 5 2  0 0
bc ac ab 1 c2 c3
0 3 5  0 0
, determine:
a a a a      
a b b b 0 0 0  5 2
14 Calcule o determinante: .
a b c c 0 0 0  3 5
a b c d
1+ x 2 3 4 23 Calcule o determinante:
1 2+ x 3 4
15 Resolva a equação: =0
1 2 3+ x 4 1 1 0  0 0
−1 1 1  0 0
1 2 3 4+ x 0 −1 1  0 0
0 0 −1  0 0
a− b−c 2a 2a 
2b b− a−c 2b 0 0 0  1 1
16 Se a + b + c = 2, qual o valor numérico de ?
2c 2c c− a− b 0 0 0  −1 1
1 cos a cos2 a
17 Calcule o determinante: D = 1 cos b cos2 b . 1 2 3  n −1 n
2 3 4  n 1
1 cos c cos2c 24 Calcule o determinante 3 4 5  1 2 .

n 1 2  n − 2 n −1

IME-ITA 127
Matemática III – Assunto 5

0 x x  x
25 (Romênia) Determine a(s) matriz(es) A, sabendo que sua matriz adjunta é: y 0 x  x
07 Calcule o determinante: y y 0  x .
 m2 − 1 1 − m 1 − m  
  y y y  0
adjA =  1 − m m2 − 1 1 − m  , m ≠ 1, −2
 1 − m 1 − m m − 1
2
  08 (IMC) A e B são matrizes de ordem n tais que AB + A + B = 0. Prove
que AB = BA.
EXERCÍCIOS NÍVEL 3
09 (IMC) Sejam A, B e C matrizes quadradas de entradas reais de mesma
ordem, e supondo A inversível. Prove que se (A – B)C = BA–1, então
01 Sejam M e N matrizes do tipo n × n distintas tais que:
C(A – B) = A–1 B.
I. M3 = N3
II. MN2 = NM2 a b c d
−b a d −c
10 Calcule .
É possível que x = M2 + N2 seja inversível? −c −d a b
−d c −b a
02 Sejam A e B matrizes reais n × n invertíveis. Mostre que se vale a
condição (AB)k = AkBk para três valores inteiros consecutivos de k, então
AB = BA.
11 Calcule o determinante:
03 (UFC) Sejam A, B e A + B matrizes n × n (n ≥ 1) invertíveis. Encontre 1 cos λ1 cos2λ1  cos( n − 1)λ1
uma expressão para (A–1 + B–1)–1 em termos de A, (A + B)–1 e B. 1 cos λ2 cos2λ2  cos( n − 1)λ2
1 cos λ3 cos2λ3  cos( n − 1)λ3
04 (Método da variação de parâmetros) Seja A uma matriz de ordem n 
e B a matriz obtida ao somarmos x a cada elemento da matriz A. Mostre 1 cos λn cos2λn  cos( n − 1)λn
que:
det B = det A + x (soma dos cofatores de todos os elementos de A). 12 Determine todas as matrizes A e B n × n tais que AB – BA = I.

05 Calcule o determinante: 13 Sejam A e B matrizes reais quadradas de ordem n satisfazendo:

a1 x x  x tr(AAt + BBt) = tr(AB + At Bt)


x a2 x  x
x x a3  x Prove que A = Bt.

x x x  an 14 Sejam A e B matrizes 3 × 3 com elementos reais tais que:

det A = det B = det(A + B) = det(A – B)


06 Calcule o determinante:
Prove que det(xA + yB) = 0 , para todo x, y reais.
x +1 x x  x
x x+a x  x
2
x x x+a  x

x x x  x + an

RASCUNHO

128 Vol. 3
Circunferência A ssunto
3
Matemática IV

Introdução  D 
2
E
2
D2 + E 2 − 4 F
 x +  + y +  =
Nesta seção, estudaremos as circunferências. Além de estar presente  2  2 4
na natureza (ex.: envoltória do sol e da lua), a circunferência e o círculo
foram a base para duas invenções importantíssimas da história: a roda Obs.: Para que a equação geral represente uma circunferência real, é
(para uso em transportes) e a engrenagem (para criação de máquinas necessário ter D2 + E2 > 4F.
após a revolução industrial).
1.3 Equação parametrizada
Em provas, os problemas de circunferência podem aparecer de forma
Em alguns problemas, é interessante P (x, y)
isolada – normalmente mais simples, envolvendo apenas a relação entre
escrever um ponto P = (x, y) da circunferência
sua equação algébrica e seus elementos geométricos – ou combinados
em função do ângulo θ entre o raio e o eixo x. R
com outras curvas – geralmente retas, que estudamos na seção anterior, θ
ou cônicas, que estudaremos na próxima.
Os seus objetivos nesta seção incluem entender as diferentes  x = a + Rcosθ O (a, b)

equações algébricas do círculo, conseguir encontrar uma reta tangente a  y = b + Rsenθ
uma circunferência dada, relacionar o conceito geométrico de potência
com a interpretação algébrica e resolver problemas envolvendo famílias
de circunferências. Em que (a, b) representa o centro e R o raio da circunferência.

1. Equações da circunferência Demonstração: completando o triângulo retângulo como na figura, temos:

Circunferência é o conjunto dos pontos cuja distância a um ponto fixo x−a y−b
(centro) é constante (raio). Exceto quando indicado em contrário, usaremos cosθ = , senθ =
R R
O = (a, b) para representar o centro e r para o raio da circunferência.

1.1 Equação reduzida EXERCÍCIOS RESOLVIDOS


Dados o centro O = (a, b) e o raio R de uma circunferência, podemos 01 Determine o centro e o raio da circunferência da equação
escrever sua equação como:
x2 + y2 – 8x + 6y + 10 = 0.
2 2 2
( x − a) +( y − b) = R
Solução: Em vez de decorar fórmulas que relacionam os coeficientes
da equação com as coordenadas do centro e o raio, é mais eficaz
Demonstração: usando a fórmula de distância ponto-ponto, a equação entender o processo de “completar os quadrados”. Veja:
acima equivale a dizer que o ponto P = (x, y) satisfaz PO = R.
(x2 – 8x) + (y2 + 6y) = – 10 →
1.2 Equação geral (x2 – 8x + 16) + (y2 + 6y +9) = – 10 + 16 + 9 →
(x – 4)2 + (y + 3)2 = 15.
Expandindo os produtos notáveis na equação reduzida, vemos que
toda circunferência pode ser escrita como:
Portanto, temos que o centro é o ponto (4,– 3) e o raio é 15.

x 2 + y 2 + Dx + Ey + F = 0 02 Para quais valores de k a equação x2 + y2 – 2x – 4y + k = 0


representa uma circunferência?

 D E Solução: Há uma condição de existência desenvolvida na teoria.


Em que o centro da circunferência é O =  − , −  e o raio é
A importância desse desenvolvimento é mostrar que é possível,
 2 2
D2 + E 2 − 4 F . completando os quadrados, verificar se a equação representa de fato
R= uma circunferência.
4
Fazendo isso, temos:

Demonstração: completando quadrados, podemos escrever a equação (x2 – 2x) + (y2 – 4y) = – k →
acima na forma reduzida: (x2 – 2x + 1) + (y2 – 4y + 4) = – k + 1 + 4 →
(x – 1)2 + (y – 2)2 = 5 – k
D2 E 2 D2 E 2
x 2 + Dx + + y 2 + Ey + = + −F Para ser uma circunferência, como vale R2 = 5 – k, devemos ter
4 4 4 4 5 – k > 0, ou seja, k < 5.

IME-ITA 129
Matemática IV – Assunto 3

Reta exterior à circunferência: d(O, t) > R


03 Determine a equação da circunferência que passa pelos pontos Reta tangente à circunferência: d(O, t) = R
(7, 2), (– 5, – 12) e (10, – 3). Reta secante à circunferência: d(O, t) < R
Solução:
Atenção: Existem diversas formas de se encontrar uma reta tangente
1a solução: Escreva a equação da circunferência no formato x2 + y2
a uma circunferência, mas a condição d(O, t) = R é, normalmente,
+ Ax + By + C = 0. Substituindo os pontos na equação, chegamos
a maneira mais eficiente.
7 A + 2 B + C =− 53

ao sistema:  −5 A − 12 B + C =− 169 .
10 A − 3 B + C =− 109 2.3 Posição de uma circunferência em

relação a outra
Resolvendo o sistema, obtemos A = – 2, B = 10 e C = – 59.
Com isso, a equação é x2 + y2 – 2x + 10y – 59 = 0. Para garantir Para descobrir a posição relativa de duas circunferências de centros
que a equação é realmente de uma circunferência (e não um caso C e C’ e raios R e R’, olhamos para a distância d(C, C’) entre os centros:
degenerado), precisamos levá-la à forma reduzida. Completando os
I. Exteriores
quadrados, chegamos a (x – 1)2 + (y + 5)2 = 85, que representa
uma circunferência de centro (1, – 5) e raio 85.
C C’ d(C, C’) > R +R’
2a solução: O centro da circunferência é a interseção das mediatrizes
dos lados do triângulo. Então, precisamos achar as equações dessas
mediatrizes. Sejam A = (7, 2), B = (– 5, – 12) e C = (10, – 3).
II. Tangentes exteriores
6
A equação da mediatriz de AB é y + 5 =− ( x − 1) ;
7

C C’ d(C, C’) = R +R’


1 3 17 
A equação da mediatriz de AC é y + =  x −  .
2 5 2
Fazendo a interseção das retas, obtemos o centro O = (1,– 5).
III. Secantes
Para achar o raio, basta calcular OA, por exemplo. Como OA =
62 + 72 =85 , obtemos a resposta encontrada na 1a solução.
C C’ |R –R’|< d(C, C’) < R + R’
3 solução: A equação da circunferência pode ser encontrada através
a

de um determinante. Esta solução é mais trabalhosa. Veja o penúltimo


exercício deste assunto.
IV. Tangentes Interiores

d(C, C’) =|R – R’|


2. Posições relativas C C’

2.1 Posição de um ponto em relação a uma


circunferência V. Interiores
Para descobrir a posição de P = (x 0, y 0) em relação a uma
circunferência, olhamos para a função potência f(x, y) = (x – a)2 + d(C, C’) <|R – R’|
(y – b)2 – R2 (ou f(x, y) = x2 + y2 + Dx + Ey + F) aplicada a este ponto: C C’

Ponto interior à circunferência: f(a, b) < 0


Ponto sobre a circunferência: f(a, b) = 0 VI. Concêntricos
Ponto exterior à circunferência: f(a, b) > 0

Demonstração: veja que f(a, b) = OP2 – R2, logo f(a, b) > 0 ⇔ OP > r d(C, C’) =0
C = C’
Obs.: É comum nos referirmos indistintamente a circunferência f(x, y) =
0 e a sua função potência f.

2.2 Posição de uma reta em relação a uma Atenção: A condição d(C,C’) = R + R’ (resp. d(C, C’) = |R – R’|) é
necessária e suficiente para caracterizar duas circunferências tangentes
circunferência exteriores (resp. interiores).
Para descobrir a posição de uma reta t: Ax + By + C = 0 em relação
a uma circunferência de centro O(x0, y0) e raio R, olhamos para a distância
A · x0 + B · y0 + C Nota: Duas circunferências secantes são ditas ortogonais quando
d( O, t )= do centro à reta: suas tangentes no ponto de interseção são perpendiculares. Na notação
A2 + B2
2 2 2
usual, esta condição é equivalente a O1O2 = R1 + R2.

130 Vol. 3
Circunferência

EXERCÍCIOS RESOLVIDOS
04 Determine a posição relativa entre as circunferências C1: x2 + y2 – 06 Determine as equações das retas tangentes à circunferência de
6x + 8y = 0 e C2: x2 + y2 – 4x + 6y = 10. equação C: x2 + y2 – 6x + 8y = 0 que passam pelo ponto A = (7, 0).

Solução: Solução: Neste problema, o ponto não está sobre a curva. A melhor
Inicialmente, escrevemos as equações reduzidas das circunferências. estratégia é usar a distância do centro à reta. Essa distância deve ser igual
Essas são: ao raio da circunferência. Há outras abordagens, mas esta, normalmente,
C1: (x – 3)2 + (y + 4)2 = 25 e C2: (x – 2)2 + (y + 3)2 = 23. gera menos esforço .
Reduzindo a equação: C: (x – 3)2 + (y + 4)2 = 25. Portanto, o centro
Temos que R1 = 5, R2 = 23, O1 = (3,– 4), O2 = (2,– 3). é o ponto O = (3,– 4) e o raio é 5.
2 2
Então, d = O1O2 = 1 + 1 =2 . É fácil ver que as tangentes não serão retas ‘verticais’ (faça um desenho).
Portanto, podemos dizer que, por passarem por A, terão o formato
Para sabermos a posição relativa entre as circunferências, devemos y – 0 = m(x – 7), ou seja, mx – y – 7m = 0. Como a distância do
comparar d com |R1 – R2| e R1 + R2. Veja que vale |R1 – R2| < d < 3m − ( − 4) − 7m
centro à reta deve ser igual ao raio, devemos ter = 5.
R1 + R2 (pois 5 − 23 < 2 < 5 + 23 ), então, as circunferências são m2 + 1
secantes (ver teoria).
Essa equação é equivalente a 4 − 4 m= 5 m2 + 1 . Como os dois
05 Determine a equação da reta tangente à circunferência C: x2 + y2 – 4x
+ 6y = 4 que passa pelo ponto P = (1, 1). lados são não negativos, podemos elevar ao quadrado e chegamos
à equação do 2o grau em m: 9m2 + 32m + 9 = 0, que tem raízes
Solução:
Inicialmente, veja que o ponto (1, 1) pertence à curva, pois satisfaz a − 32 ± 10 7
m= .
equação (1 + 1 – 4 + 6 = 4). A equação reduzida da circunferência 18
− 32 ± 10 7
é (x – 2)2 + (y + 3)2 = 17, portanto, o centro é o ponto O = (2, – 3). =
Portanto, as tangentes são y
18
( x − 7) .
Agora, veja que o fato de P estar sobre a circunferência ajudará demais! Obs.: Para verificar se o ponto realmente está fora do círculo, basta
A reta tangente t é perpendicular à reta OP, portanto mt ∙ mOP = – 1. substituir suas coordenadas em C(x, y) = x2 + y2 – 6x + 8y e ver se o
–4 1 resultado será positivo (positivo é para pontos no exterior, negativo para
Como mOP = , segue que mt = . Como a reta t passa por (1, 1),
1 4 pontos no interior e zero para pontos sobre a curva). Neste caso, o ponto
1 x+3
temos que t: y −= 1
4
( x − 1) . Então, a resposta é t: y = 4 . realmente está fora, pois C(7,0) = 72 + 02 – 6 ∙ 7 + 8 ∙ 0 = 7 > 0.
No entanto, isso não é estritamente necessário na solução. Como
encontramos duas tangentes, isso já garante que o ponto está fora.

3. Potência e eixo radical


3.1 Potência de um ponto Podemos escrever:
em relação a uma circunferência A = (a, 0), B = (–a, 0), P = (p, 0), O = (0, –b)
A função potência f: 2 →  de uma circunferência é dada, como PA · PB = (p + a)(p – a) = p2 – a2
visto anteriormente, por f(x, y) = (x – a)2 + (y – b)2 – R2, ou, expandindo
e agrupando, f(x, y) = x2 + y2 + Dx + Ey + F. Dessa forma, temos: OP2 – r2 = OP2 – OA2 = p2 + b2 – a2 – b2 = PA · PB
Pela fórmula da distância ponto-ponto, dado um ponto P(x, y), temos
que f(x, y) = OP2 – R2.Essa quantidade recebe o nome de potência de P Como relacionamos a quantidade PA · PB com uma constante
em relação à circunferência. (OP2 – r2) que não depende dos eixos considerados nem dos pontos A e
B escolhidos, podemos escrever PA · PB = PX · PY = PT2.
Obs.: A função potência sempre tem coeficiente de x2 + y2 igual a 1.
Por exemplo, a função potência de x2 + y2 + 2x = 1 e de 2x2 + 2y2 + 4x = 2 3.2 Eixo radical
é a mesma (f(x, y) = x2 + y2 + 2x – 1), pois ambas equações representam Dadas duas circunferências, o seu eixo radical é o lugar geométrico
a mesma circunferência. dos pontos que têm igual potência em relação às duas circunferências.
Este lugar geométrico é sempre uma reta, e quando as circunferências
Teorema y P são secantes, essa reta é a extensão da corda comum.
x
Se PAB é uma reta secante à A Demonstração: sendo f1(x, y) = x2 + y2 + D1 x + E1 y + F1 e
circunferência, então PA · PB = X f2(x, y) = x2 + y2 + D2 x + E2 y + F2 as respectivas funções potência,
B b r um ponto terá mesma potência em relação as duas se, e somente se,
PotCP0 = constante
O f1(x, y) = f2(x, y).
Demonstração: Dada uma Cancelando os termos quadráticos, obtemos a equação de uma
circunferência e uma secante T reta. Não é difícil ver que se existirem pontos de interseção entre as duas
PAB como na figura, considere Y circunferências, esses pontos estão no LG (pois tem potência zero em
um sistema de eixos com a origem no interior da circunferência, de relação a cada uma das duas).
forma que o eio y passe pelo seu centro e o eixo x sobre a corda AB.

IME-ITA 131
Matemática IV – Assunto 3

EXERCÍCIOS RESOLVIDOS equação não muda se multiplicarmos todos os lados por uma constante
k, isso implica f – f1 = k · r, k ∈ .
07 Determine o comprimento das tangentes traçadas à circunferência
2x2 + 2y2 + 5y = 7 pelo ponto A(1, 1).
4.2 Compartilhando as
Solução: Na notação anteriror, temos: interseções de duas circunferências dadas
5 7 Dadas duas circunferências secantes com funções potências
f ( x , y )= x 2 + y 2 + y −
2 2 f1(x, y) = x2 + y2 + D1x + E1y + F1 e f2(x, y) = x2 + y2 + D2x + E2y + F2,
5 7 uma circunferência pela interseção das duas é dada por:
AT 2 = f (1,1) =12 +12 + · 1 − =1 ⇒ AT =1
2 2
f = f1 + t · (f2 – f1), t ∈ 
08 Determine a equação da reta que passa pelas interseções das
circunferências x2 + y2 + 4x + y – 3 = 0 e x2 + y2 – 2x – y – 11 = 0. Demonstração: como a corda comum r dos pares de circunferência
(f2, f1) e (f1, f) é a mesma, temos f2 – f1 = k · r e f – f1 = k’ · r. Eliminando
Solução: O enunciado já indica que as circunferências são secantes, r e fazendo t = k’/k, temos f – f1 = t · (f2 – f1)
pois se intersectam (caso isso não fosse dito, poderíamos utilizar 2.3.
para descobrir a posição relativa entre as circunferências). EXERCÍCIOS RESOLVIDOS
Como a reta que passa pelas interseções é o eixo radical, basta subtrair
as funções potências para obter a reta f1(x, y) – f2(x, y) = 0, ou seja: 09 Determine a equação do círculo que passa pelo ponto (–10, –2)
6x + 2y + 8 = 0 ⇔ 3x + y + 4 = 0. e pelas interseções do círculo x2 + y2 + 2x – 2y – 32 = 0 com a reta
x – y + 4 = 0.

Solução:
4. Família de circunferências A família de circunferências compartilhando as interseções entre a
circunferência e a reta dadas é:
4.1 Compartilhando as
x2 + y2 + 2x – 2y – 32 +k ⋅ (x – y + 4) = 0
interseções de uma circunferência e uma
reta dada Substituindo x = – 10 e y = – 2, obtemos o valor de k:
Dada uma circunferência com função potência f1 e uma reta r(x, y), (–10)2 + (–2)2 + 2 · (–10) – 2 · (–2) – 32 + k · (–10–(–2) + 4) = 0 ⇒
com r(x, y) = Ax + By + C, as demais circunferências que passam pela k = 14
interseção da reta com a circunferência dada são definidas por:
Logo, a circunferência procurada é:
f = f1 + k · r, k ∈ * x2 + y2 + 2x – 2y – 32 + 14 · (x – y + 4) = 0
x2 + y2 + 16x – 16y + 24 = 0
Demonstração: como r será o eixo radical das duas circunferências, o (x + 8)2 + (y – 8)2 = 104
resultado 3.2. mostra que a equação de r é dada por f – f1. Como uma

EXERCÍCIOS NÍVEL 1

01 Ache a equação da circunferência de raio 13 que passa por (0,0), 04 Determine a equação da circunferência que passa pelos pontos
sabendo que a abscissa do centro é – 12. A(– 1, 15); B(1,3); C(– 1, 2).

02 (AFA) Os pontos A(–5, 2) e B(1, 6) são extremos de um dos diâmetros 05 Ache a equação do diâmetro do círculo x2 + y2 + 4x – 6y – 17 = 0
das circunferências de equação: perpendicular à reta 5x + 2y – 13 = 0.

(A) x2 + y2 – 2y – 25 = 0 06 (EFOMM) A interseção da reta y + x – 1 = 0 com a circunferência


(B) x2 + y2 + 4x – 8y + 7 = 0 x2 + y2 + 2x + 2y + –3 = 0 determina uma corda cujo comprimento é:
(C) x2 + y2 – 4x + 4y – 57 = 0
(D) x2 + y2 + 8x – 14y + 39 = 0 (A) 7. (D) 5 .
(B) 2 . (E) 6.
03 (AFA) De acordo com a figura abaixo, y B
podemos afirmar que a área do triângulo (C) 3 .
isósceles ABC, em unidade de área, é:
07 A equação de um círculo é x2 + y2 = 50. O ponto médio de uma corda
2 0
deste círculo é (– 2, 4). Ache a equação dessa corda.
(A) 2 3 .
(B) 3 3 . 3 08 Determine a equação do círculo cujo centro é o ponto (7, –6) e que
A C x
(C) 4 5 . passa pelo ponto (2, 2).
(D) 5 5 .

132 Vol. 3
Circunferência

09 (EFOMM) Sendo r a equação de uma reta que passa pelo centro da 21 Determine o valor de k sabendo que a reta 2x + 3y + k = 0 é tangente
circunferência x2 + y2 + 10x + 20y + 121 = 0 e é perpendicular à reta à circunferência x2 + y2 + 6x + 4y = 0.
2x + 6y – 5 = 0, sua equação é:
22 Determine a equação cartesiana de uma reta, sabendo que esta passa
(A) – x + y – 5 = 0. (D) – 3x + y – 5 = 0. pelo ponto P(2, 9) e é tangente à figura determinada pelas equações
(B) 2x + 2y + 5 = 0. (E) – 2x – y + 5 = 0.  x =+
2 8senα
(C) – 3x + y + 5 = 0. paramétricas  , α ∈ .
 y =1 + 8cos α
10 Determine as coordenadas dos pontos de interseção da reta 7x – y +
12 = 0 com a circunferência (x – 2)2 + (y – 1)2 = 25. 23 Se dois círculos (x – 1)2 + (y – 3)2 = r2 e x2 + y2 – 8x + 2y + 8 = 0
intersectam-se em dois pontos distintos, então:
11 (AFA) A equação da reta que passa pelo centro da circunferência
2x2 + 2y2 – 8x – 16y – 24 = 0 e é paralela à reta – 8x + 2y – 2 = 0 é: (A) 2 < r < 8.
(B) r < 2.
(A) y = 2x. (C) y = 4x – 8. (C) r = 2.
(B) y = x + 2. (D) y = 4(x – 1). (D) r > 2.

12 (AFA) A intersecção da reta y + x + 1 = 0 com a circunferência 24 Mostre que as circunferências C1: x2 + y2 – 3x – 6y + 10 = 0 e C2:
x2 + y2 + 2x + 2y + 1 = 0 determina uma corda cujo comprimento é: x2 + y2 – 5 = 0 são tangentes. Determine a equação da circunferência
tangente a C1 e a C2 em seu ponto comum que passa pelo ponto (7, 2).
(A) 2 . (C) 2 3.
25 (EFOMM) Dados os pontos A(2, 3), B(–1, 2) e C(0, 3). Determine
(B) 2 2 . (D)
3 2.
suas posições em relação à circunferência (x – 2)2 + (y – 3)2 = 4.

13 Ache as equações dos círculos de raio r = 5, tangentes à reta (A) A, interior; (D) A, exterior;
x – 2y – 1 = 0 no ponto M1(3,1). B ∈ à circunferência; B ∈ à circunferência;
C, exterior. C, interior.
14 Ache sobre o círculo 16x2 +16y2 + 48x – 8y – 43 = 0 um ponto M1
o mais próximo possível da reta 8x – 4y + 73 = 0 e calcule a distância (B) A, interior; (E) A ∈ à circunferência;
d do ponto M1 a essa reta. B, exterior; B, exterior;
C ∈ à circunferência. C, interior.
15 A circunferência da equação x2 + y2 – 8x – 8y + 16 = 0 e centro
C é tangente ao eixo das abscissas no ponto A e é tangente ao eixo das (C) A ∈ à circunferência;
ordenadas no ponto B. A área do triângulo ABC vale: B, interior;
C, exterior.
(A) 4. (C) 12.
(B) 8. (D) 16. 26 Desde o ponto A(–2, –1) é traçada uma reta tangente ao círculo
x2 + y2 – 6x – 4y – 3 = 0. Sendo B o ponto de contato, determine o
16 (AFA) A circunferência, com centro em (2, 1) e tangente à reta comprimento do segmento AB.
x – y + 3 = 0, tem equação:
27 Encontre a distância do ponto à circunferência em cada um dos casos
(A) x2 + y2 – 4x – 2y – 3 = 0. (C) x2 + y2 – 4y – 2x – 7 = 0. a seguir:
(B) x2 + y2 – 4y – 2x – 3 = 0. (D) x2 + y2 – 4x – 2y – 7 = 0.
a. (6, –8); x2 + y2 = 9;
17 Determine a equação da reta tangente ao círculo x +y – 3y = 4
2 2 b. (3, 9); x2 + y2 – 26x + 30y + 313= 0;
passando pelo ponto (2, 3). c. (–7, 2); x2 + y2 – 10x – 14y – 151= 0.

18 Ache as equações das tangentes ao círculo x2 + y2 – 2x + 4y = 0 28 Determine a equação do círculo que passa pelo ponto (–10, –2) e
perpendiculares à reta x – 2y + 9 = 0. pelas interseções do círculo x2 + y2 + 2x – 2y – 32 = 0 com a reta
x – y + 4 = 0.
19 Determine a equação de uma reta tangente ao círculo x2 + y2 – 3y = 4
que passa pelo ponto (5, –1). 29 Determine o comprimento da corda e a equação da secante comuns
aos círculos x2 + y2 – 8y + 6 = 0 e x2 + y2 – 14x – 6y + 38 = 0.
20 (AFA) A circunferência x2 + y2 = 5 possui duas retas tangentes, t1 e
t2, que são paralelas à reta r: y = –2x + 3. As equações gerais das retas 30 (IIT) O número de tangentes comuns às circunferências x2 + y2 = 4 e
t1 e t2, respectivamente, são: x2 + y2 – 6x – 8y = 24 é:

(A) 2x + y – 5 = 0 e 2x + y + 5 = 0 (A) 0.
(B) 2x + y – 15 = 0 e 2x + y + 15 = 0 (B) 1.
(C) 3.
(C) 2 x + y − 5 5 = 0 e 2 x + y +5 5 = 0
(D) 4.
4 5 4 5
(D) 2 x + y − = 0 e 2x + y + =0
5 5

IME-ITA 133
Matemática IV – Assunto 3

EXERCÍCIOS NÍVEL 2 x y
13 A reta − = 1, a > 0 , intercepta os eixos coordenados x e y nos
01 (AFA) Qual das equações abaixo representa a circunferência inscrita a a
no triângulo de vértice A(3, 5), B(7, 5) e C(3, 8)? pontos P e Q, respectivamente. A equação geral da circunferência tangente
ao eixo x no ponto P e tangente ao eixo y no ponto Q é:
(A) x2 + y2 – 12x – 8y + 70 = 0.
(B) x2 + y2 – 8x – 12y + 51 = 0. (A) x2 + y2 – 2ax + 2ay + a2 = 0.
(C) x2 + y2 – 8x – 10y + 68 = 0. (B) x2 + y2 + 2ax – 2ay + a2 = 0.
(D) x2 + y2 – 10x – 14y + 72 = 0. (C) x2 + y2 + 2ax + 2ay + a2 = 0.
(D) x2 + y2 – 2ax – 2ay + a2 = 0.
02 Determine o lugar geométrico dos pontos do plano cuja distância ao 14 Fazem-se passar pelo ponto P(2, – 3) as tangentes ao círculo (x – 1)2 +
ponto A(– 6,– 3) é o dobro de sua distância ao ponto B(3, 0). (y + 5)2 = 4. Ache a equação da corda que passa pelos pontos de tangência.
03 Calcule o comprimento da corda do círculo x2 + y2 – 2x + 2y – 14 = 0 15 Seja P o ponto da circunferência x2 + y2 – 6x – 8y + 24 = 0 mais
que passa pelos pontos (1,– 1) e (0,2). próximo da origem. A soma das coordenadas de P é:

(A) 18/5. (D) 28/5.


04 Sejam A e B dois pontos do plano. Mostre que o lugar geométrico dos
(B) 7/2. (E) 13/2.
PA
pontos P do plano tais que = k é um círculo (Apolônio) (para k ≠ 1) (C) 9/2.
PB
e determine o centro desse círculo. 16 Ache as equações dos círculos tangentes às três retas 4x – 3y – 10 = 0,
3x – 4y – 5 = 0 e 3x – 4y – 15 = 0.
05 Demonstre analiticamente que a mediatriz de uma corda em uma
circunferência é sempre reta suporte de um diâmetro dessa circunferência. 17 (AFA) As equações das retas tangentes à circunferência (x – 2)2 +
(y – 1)2 = 4 e paralelas à reta x + y – 2 = 0 são:
06 (AFA) Dada a circunferência x2 + y2__– 8x – 4y – 5 = 0 e os pontos
D(–1, 2) e E(8, 5), pode-se afirmar que DE: (A) x + y − (3+2 2)= 0 e x + y − (3 − 2 2)= 0 .
(B) x + y + (3+2 2)= 0 e x + y + (3 − 2 2)= 0 .
(A) é um diâmetro de circunferência. (C) x + y + ( −3+2 2)= 0 e x + y + ( −3 − 2 2)= 0 .
(B) não intercepta a circunferência.
(D) x + y − ( −3+2 2)= 0 e x + y − ( −3 − 2 2)= 0 .
(C) intercepta a circunferência em um único ponto.
(D) é uma corda de circunferência, mas não contém o centro.
18 Considere a família de retas dada por a(3x + 4y – 10) + b(3x – y – 5) = 0,
a,b ∈ . Determine as retas dessa família que são tangentes à
07 Determine o ângulo formado na interseção da reta 3x – y – 1 = 0 com a
circunferência x2 + y2 + 2x – 4y = 0.
circunferência (x – 2)2 + y2 = 5. (Este ângulo, por definição, é o ângulo que
esta reta forma com a reta tangente à circunferência no ponto de interseção.)
19 Do ponto P = (– 9, 3) são traçadas as tangentes à circunferência de
equação x2 + y2 – 6x + 4y – 78 = 0. Determine a distância do centro da
08 Considere um círculo C de raio 5 cm com centro O em (0,0) e um ponto
circunferência à corda que une os pontos de contato.
P sobre a circunferência deste círculo. Seja M a projeção do ponto P sobre o
eixo OX . Determine a equação do lugar geométrico do centro de gravidade 20 O ponto C(3, –1) é o centro de uma circunferência que determina
do triângulo OPM, quando P se desloca sobre a circunferência do círculo C. sobre a reta 2x – 5y + 18 = 0 uma corda, de comprimento 6. Determine
a equação desta circunferência.
09 Em um sistema de eixos ortogonais, o vértice A do triângulo ABC está
na origem e o eixo dos x é o suporte do lado AB. O coeficiente angular do 21 Determine o ângulo formado pela interseção das circunferências
lado AC é 1. Sabendo que os pontos B, C e M, este último de coordenadas (x – 3)2 + (y – 1)2 = 8 e (x – 2)2 + (y + 2)2 = 2. (Chama-se ângulo
(0,– 4), estão alinhados e que as distâncias BM e BC são iguais, determine formado por duas circunferências o ângulo compreendido entre suas
a equação da circunferência circunscrita ao referido triângulo. tangentes no ponto de interseção.)
10 (IIT) As tangentes traçadas do ponto P(1, 8) à circunferência x2 + y2 – 22 Encontre a equação do círculo que passa pelo ponto (– 8, 5) e pelas
6x – 4y – 11 = 0 intersectam a circunferência nos pontos A e B. A equação interseções dos círculos x2 + y2 – 16x – 4y + 3 = 0 e x2 + y2 – 18x –
do circuncírculo do triângulo PAB é: 4y + 67 = 0, caso eles sejam secantes.
(A) x2 + y2 + 4x – 6y + 19 = 0. (C) x2 + y2 – 2x + 6y – 29 = 0. 23 Determine a equação do círculo que passa pelas interseções dos dois
(B) x2 + y2 – 4x – 10y + 19 = 0. (D) x2 + y2 – 6x – 4y + 19 = 0. círculos x2 + y2 – 6x + 4 = 0 e x2 + y2 – 2 = 0 e é tangente à reta
x + 3y – 14 = 0.
11 Fazem-se passar pelo ponto A(4, 2) as tangentes ao círculo x2 + y2 = 10.
Calcule o ângulo formado por essas tangentes. 24 Mostre que a equação da reta tangente à circunferência x2 + y2 +
Dx + Ey + F = 0 no ponto (x0, y0) é dada por:
12 Ache as equações dos círculos que, tendo seu centro sobre a reta x + x0 y + y0
4x – y – 3 = 0 são tangentes às retas x – 2y = 5 e 2x + y + 3 = 0. x · x0 + y · y0 +D · +E · +F = 0
2 2
25 Mostre que dadas três circunferências cujos centros não estejam
alinhados, sempre existe um ponto com igual potência em relação às três
circunferências (centro radical).

134 Vol. 3
Circunferência

EXERCÍCIOS NÍVEL 3

01 Dado o feixe de retas a(x – 8y + 30) + b(x + 5y – 22) = 0, determine 07 Determine a equação e o raio do círculo de menor diâmetro que possui
as retas desse feixe que determinam na circunferência de equação com o círculo x2 + y2 – 8x – 25 = 0 eixo radical y – 2x – 5 = 0.
x2 + y2 – 2x + 2y – 14 = 0 cordas de comprimento 2 3 .
08 Determine a condição para que as circunferências (x – a)2 + (y – b)2 =
02 Prove que a corda comum às circunferências que têm por diâmetros r2 e (x – A)2 + (y – B)2 = R2 sejam ortogonais. (Duas circunferências
as medianas BB’ e CC’ de um triângulo é parte da reta que contém a altura são ortogonais quando se cortam formando um ângulo reto.)
AA’ deste triângulo. (Dados: A(0, 2a); B(2b, 0); C(2c, 0) e bc < 0.)
09 Mostre que a equação da circunferência que passa por (x1, y1), (x2, y2)
03 Prove, analiticamente, que dados três círculos, ao tomarmos os eixos e (x3, y3) (pontos não colineares) é dada por:
radicais de cada par destes círculos, as três retas obtidas são concorrentes.
x2 + y2 x12 + y12 x 22 + y 22 x32 + y32
(Esse ponto é chamado de centro radical.)
x x1 x2 x3
= 0.
 1  1  1  1 y y1 y2 y3
04 Mostre que os pontos  a,  ,  b,  ,  c,  ,  d,  pertencem a uma
 a  b  c  d 1 1 1 1
mesma circunferência, então abcd = 1.
10 Sejam ABC e BCD triângulos equiláteros, com A e D situados em
05 Sejam dois pontos fixos A(a,0) e B(0,b) sobre os eixos Ox e Oy. Tomam- semiplanos distintos. Seja r uma reta variável passando pelo ponto D.
-se, respectivamente, os pontos A’ e B’ tais que AA’ = BB’ . Demonstre que Esta reta intersecta a reta AB em X e a reta AC em Y. Seja T a interseção das
quando varia o comprimento dos segmentos iguais AA’ e BB’, a mediatriz retas BY e CX . Determine o lugar geométrico de T, quando a reta r varia.
de A’B’ passa por um ponto fixo.

06 Demonstre que as duas circunferências x2 + y2 – 2mx + 2ny – m2 +


n2 = 0 e x2 + y2 – 2nx – 2my + m2 – n2 = 0 se cortam formando um
ângulo reto.
RASCUNHO

IME-ITA 135
Cônicas A ssunto
4
Matemática IV

Introdução Elipse: e < 1


e = 1/2
Parábola: e = 1 e=1
Iniciaremos agora o estudo das cônicas (elipses, hipérboles e
parábolas), que são as curvas obtidas a partir da interseção de um Hipérbole: e > 1 F
P
plano com um cone circular reto. Na natureza, as cônicas aparecem, por É possível provar que, no caso da
exemplo, na descrição de órbita de planetas (normalmente, elipses), na e=2
elipse e da hipérbole, sempre existirão um
interseção de ondas sonoras com a superfície (normalmente, hipérboles) outro foco F’ e uma outra diretriz d’, tais diretriz d
P’
e em lançamentos oblíquos (parábola).
PF'
Os seus principais objetivos nesta seção são internalizar as definições que = e.
dist( P, d')
geométricas, memorizar as relações entre os elementos de cada tipo de
cônica e resolver problemas utilizando equações algébricas para modelar
cônicas com eixos paralelos aos eixos coordenados. Na próxima seção, 1.3 Definição geométrica
você estudará tópicos mais avançados, como fórmulas para o raio vetor, Os três tipos de cônicas também podem ser definidos a partir de
equações para retas tangentes, propriedades óticas e equações polares. propriedades geométricas bem específicas, como veremos agora:
Elipse: conjunto dos pontos cuja soma das distâncias a dois pontos fixos
1. Definições (focos) é constante: PF + PF’ = 2a. A distância entre os dois pontos é denominada
c
Existem algumas definições possíveis para cônicas, todas elas distância focal 2c e pode-se mostrar que a excentricidade de 1.2. satisfaz e = .
a
equivalentes. Mostraremos aqui três dessas definições, mas o estudo inicial Hipérbole: conjunto dos pontos cujo módulo da diferença das
será feito a partir de 1.3. A equivalência entre as definições apresentadas distâncias a dois pontos fixos (focos) é constante: |PF – PF’|= 2a. A
virá como exercício quando estudarmos geometria espacial (1.1 e 1.3) e distância entre os dois pontos é denominada distância focal 2c e pode-se
na segunda seção sobre cônicas (1.2. e 1.3). c
mostrar que a excentricidade de 1.2. também satisfaz e = na hipérbole.
a
1.1 Definição espacial Parábola: conjunto dos pontos cuja distância a um ponto fixo (foco)
é igual à sua distância a uma reta dada (diretriz): PF = dist (P, diretriz) e
uma reta dada (diretriz) é constante.

EXERCÍCIOS RESOLVIDOS

01 Dadas duas circunferências, C1 e C2, com centros O1 e O2 e raios


Parábola Círculo r1 > r2, determine o lugar geométrico dos centros das circunferências
que são simultaneamente tangentes a C 1 interiormente e a C 2
Hipérbole
lelo

Elipse exteriormente.
a
Par

Solução: Seja P um ponto do lugar


geométrico e R o raio da circunferência
a s s o c i a d a a P . Pe l a t e o r i a d e O1 r1 P
circunferências tangentes:
R
r2 R
PO1 = r1 – R, PO2 = r2 + R
Uma cônica (ou seção cônica) é a interseção de um plano com O2
um cone reto duplo infinito, como ilustrado na figura. Essa cônica será
Somando as equações para eliminar o
definida como:
parâmetro R:
Elipse: se a interseção for limitada (i.e., se o ângulo entre a seção e a
base do cone for menor que o ângulo entre a geratriz e a base do cone). PO1 + PO2 = r1 + r2
No caso em que a seção é paralela à base do cone, a elipse vira um círculo.
Pela definição 1.3, segue que P pertence a uma elipse de focos O1 e
Parábola: se o plano for paralelo à geratriz do cone.
O2. Reciprocamente, podemos ver que todo ponto desta elipse pertence
Hipérbole: nos demais casos (i.e., se o ângulo entre a seção e a base ao lugar geométrico.
do cone for maior que o ângulo entre a geratriz e a base do cone). No caso
em que a seção é perpendicular à base do cone, a hipérbole é dita equilátera.

1.2 Definição astronômica 2. Elipse


Uma cônica é o conjunto dos pontos cuja razão a um ponto fixo
(denominado foco F) e a uma reta fixa (denominada diretriz d) é constante 2.1 Elementos
PF A definição PF + PF’ = 2a implica que a curva tem dois eixos de
(denominada excentricidade e), i.e., = e . Essa cônica poderá simetria: a reta que une os focos e a mediatriz dos focos. Para esses eixos
ser: dist( P, d )
e para seus pontos notáveis, a nomenclatura usual é:

136 Vol. 3
Cônicas

Focos: F, F’; Para a elipse em pé (focos no eixo y), basta trocar x por y e y por –x
Centro: O; B na equação acima.
P
Vértices: A, A’, B, B’.
Eixo maior: AA’ = 2a (pois AA’ EXERCÍCIOS RESOLVIDOS
= AF + A’F = AF + AF’ = 2a). A’ c
F’ O F A x2 y2
Eixo menor: BB’ = 2b. 02 Considere a elipse de equação ε : + = 1 . Determine a área
b a
Distância focal: FF’ = 2c. 9 4
do retângulo, inscrito em ε, de lados paralelos aos eixos, tal que os
Relação fundamental: B’
focos da elipse estão nos lados do retângulo.
a2 = b 2 + c 2
Solução: Veja o desenho abaixo. Os focos da elipse estão sobre o eixo
x, já que o maior denominador está no x. Temos que a2 = 9, b2 = 4;
Demonstração: como B’ está na elipse e B’F = B’F’, temos B’F + portanto, como a2 = b2 + c2, segue que c2 = 5.
B’F’ = 2a ⇒ B’F = a. A relação fundamental, portanto, é o teorema de
Pitágoras aplicado ao triângulo OB’F.

2.2 Equação reduzida


P
A equação de uma elipse com centro na origem e eixos paralelos aos
eixos coordenados é dada por: F

x2 y2
Elipse deitada (focos no eixo x): + =1
2
a b2

y2 x2
Elipse em pé (focos no eixo y): + =1 Portanto, x F = 5 , em que F é o ponto de abscissa positiva. Considere
a2 b2
o ponto P como na figura. Como PF é vertical, temos que x P = 5
Demonstração: para a elipse deitada, tem-se o eixo x sobre os focos,
Como P pertence à elipse, suas coordenadas devem satisfazer a
o eixo y na mediatriz dos focos e as coordenadas indicadas na figura: 2 2 4
equação da curva: x P + y P =
1, o que nos dá y P = (já que P está
9 4 3
B(0, b) no 1o quadrante). Vamos, então, ao cálculo da área do retângulo. A
P(x, y)
base é igual à distância focal 2c = 2 5 . A altura é o dobro de PF,
8
O (0, 0) logo, é igual a 2 y P = . Portanto, a área é igual a 2 5 ⋅ 8 =16 5
3 3 3
A’ (–a, 0) A (a, 0) unidades de área.
F’(–c, 0) F(c, 0)
03 Sejam F1 e F2 os pontos do plano cartesiano de coordenadas

B’(0, –b) ( ) ( )
F1 = − 3,0 e F2 = 3,0 . Determine as coordenadas dos pontos
da reta r de equação x – y = 1 cujas somas das distâncias a F1 e F2
sejam iguais a 4 (isto é: determine as coordenadas dos pontos P sobre
Partindo da definição: a reta r que satisfazem PF1 + PF2 = 4).
PF + PF' = 2 a ⇔
Solução: A 1a ideia é utilizar a fórmula de distância entre pontos para
( x + c)2 + y 2 + ( x − c)2 + y 2 = 2 a ⇔ calcular PF1 e PF2. No entanto, isso nos leva a contas muito grandes e
desnecessárias. A melhor ideia é, inicialmente, caracterizar os pontos P
( x − c)2 + y 2 = 2 a − ( x + c)2 + y 2
tais que PF1 + PF2 = 4. Veja que esses pontos determinam uma elipse
centrada na origem com eixos sobre os eixos x e y tal que 2a = 4
Elevando os dois termos ao quadrado e simplificando, um ponto na (logo, a = 2) e =2c F= 1F2 2 3 (logo, c = 3 ). Usando que a2 =
elipse precisa satisfazer: b + c , obtemos b = 1.
2 2

( x − c)2 + y 2 = 4 a2 − 4 a ( x + c)2 + y 2 +( x + c)2 + y 2 ⇔ x2 y2


Portanto, temos que P pertence à elipse de equação + = 1.
Como queremos que P também esteja na reta x – y = 4 1, devemos
1
4 a ( x + c)2 + y 2 = 4 a2 +4 cx (*) achar a interseção entre essa reta e a elipse. Basta, então, substituir
2 2
y = x – 1 em x + y = 1 . Fazendo isso, temos a equação do 2o grau
Cancelando o 4, elevando (*) ao quadrado e substituindo a2 – c2 = b2: 4 1
( ) x 2 ( x − 1)
2
a2 x 2 +2cx + c2 + y 2 = a4 +2cxa2 + c2 x 2 ⇔ 8
+ = 1 , que tem raízes x = 0 e x = . Como y = x – 1,
4 1 5
x2 y2
( a2 − c2 ) x 2 + a2 y 2 = a2( a2 − c2 ) ⇔ 2
+ =1 8 3
temos os pontos (0, – 1) e  ,  .
a b2
5 5

IME-ITA 137
Matemática IV – Assunto 4

Demonstração: para a hipérbole deitada, tem-se o eixo x sobre os focos,


04 Identifique a direção do eixo principal de cada uma das elipses o eixo y na mediatriz dos focos e as coordenadas indicadas na figura:
abaixo (i.e., determine se elas estão “deitadas” ou “em pé”):

a. 2x2 + y2 = 1 B(b, 0) P(x, y)


b. x2 + 2y2 = 1 F’(–C, 0) F(c, 0)
Solução: Colocando as equações na forma reduzida (3.1.), obtemos A’(–a, 0) A(a, 0)
x2
y 2 B’(–b, 0)
+ y 2 =1 e x 2 + =1.
1 1
2 2

Na primeira equação, o eixo maior está sob y2, o que significa,


comparando com as equações reduzidas, que a elipse está em pé. Na Partindo da definição:
segunda equação, o eixo maior está sob x2, logo, esta elipse está deitada. PF − PF' = 2 a ⇔

( x + c)2 + y 2 − ( x − c)2 + y 2 = 2 a
3. Hipérbole
Fazendo inicialmente a conta para o ramo direito da hipérbole (depois,
3.1 Elementos basta trocar x por –x), o argumento do módulo será positivo e:
A definição |PF – PF’| = 2a também implica que a curva tem como ( x + c)2 + y 2 = 2 a + ( x − c)2 + y 2
eixos de simetria a reta que une os focos e a mediatriz dos focos. Para
esses eixos e para seus pontos notáveis, a nomenclatura usual é:
Focos: F, F’; Centro: O; Vértices: A, A’, B, B’. Elevando os dois termos ao quadrado e simplificando, um ponto na
hipérbole precisa satisfazer:
Eixo principal: AA’ = 2a (pois AA’ = AF’ – A’F’ = AF’ – AF = 2a).
Distância focal: FF’ = 2c. ( x + c)2 + y 2 = 4 a2 − 4 a ( x − c)2 + y 2 +( x − c)2 + y 2 ⇔
Eixo transverso: BB’ = 2b. 4 a ( x − c)2 + y 2 = 4 a2 − 4 cx (*)
Relação fundamental: Cancelando o 4, elevando (*) ao quadrado e substituindo c2 – a2 = b2:

( )
a2 x 2 − 2cx + c2 + y 2 = a4 − 2cxa2 + c2 x 2 ⇔
B
x2 y2
P ( c 2 − a2 ) x 2 − a2 y 2 = a2 ( c 2 − a2 ) ⇔ − =1
a2 b2
F’ a F
A’ b A Para a hipérbole em pé (focos no eixo y), basta trocar x por y e y por
B’ – x na equação acima.

3.3 Assíntotas
As assíntotas de uma hipérbole com centro na origem e eixos paralelos
aos eixos coordenados são:
c2 = a2 + b2 b
Hipérbole deitada (focos no eixo x): y = ± x
a
Explicação: na hipérbole, o b não tem uma interpretação geométrica
tão intuitiva quanto na elipse. Por ora, definiremos b2 = c2 – a2 e, depois, a
veremos que esse b tem relação com as assíntotas da hipérbole. Hipérbole em pé (focos no eixo y): y = ± x
b
3.2 Equação reduzida x2 y2
Demonstração: para a hipérbole deitada, tem-se 2 − 2 =1.
A equação de uma hipérbole com centro na origem e eixos paralelos a b
No 1o quadrante, tem-se:
aos eixos coordenados é dada por:
Dividindo a equação da hipérbole deitada por x e tomando limites:
2

x2 y2 y b
Hipérbole deitada (focos no eixo x): − =1 lim = . Dividindo a equação por x, tomando limite e substituindo
a2 b2 x→∞ x a
 b 
y2 x2 o resultando anterior: lim  y − x  = 0 .
Hipérbole em pé (focos no eixo y): − =1 x→∞  a 
a2 b2
O resultado nos outros quadrantes segue da simetria em relação aos
eixos coordenados. Para a hipérbole em pé, basta trocar x e y.

138 Vol. 3
Cônicas

EXERCÍCIOS RESOLVIDOS
05 Considere os pontos A = (– 1, 0) e B = (1, 0). Determine o lugar 12 3
geométrico dos pontos P tais que o produto dos coeficientes angulares Como M está na reta dada: y M = 2 x M − 3 = 2 · − 3=
7 7
das retas AP e BP seja igual a 2.
Obs.: atente para a ideia de se utilizar as relações de Girard para evitar
y y contas com raízes quadradas em problemas deste tipo.
Solução: Seja P = (x,y). Sabemos que mAP = e mBP =
x +1 x −1
y y 07 Identifique a direção do eixo principal de cada uma das hipérboles abaixo:
. Queremos então que ⋅ = 2 , ou seja, y2 = 2(x2 – 1), que é
x +1 x −1
x2 y2 a. x2 y2
equivalente à hipérbole de equação − = 1. − =1
1 2 4 9
Há um detalhe (muito comum, inclusive, em problemas de lugar 2 2
geométrico). Veja que os pontos A e B pertencem a essa hipérbole. No b. x − y =1
entanto, quando P coincide com A ou B, o problema não faz sentido 9 4
(não faria sentido falar em coeficiente angular da reta AP quando A e P 2 2
c. − x + y =1
x2 y2 4 9
coincidem, por exemplo). Portanto, a resposta é a hipérbole − = 1
, excluindo-se seus ‘vértices’ A e B. 1 2 2 2
d. − x + y =1
9 4
06 A hipérbole x2 – 2y2 = 1 intersecta a reta y = 2x – 3 em dois pontos
distintos A e B. Determine o ponto médio M de AB.
Solução: Comparando com a equação reduzida, vemos que a e b são
deitadas (eixo principal sob o eixo x) e c e d são em pé (eixo principal sob
Solução: Substituindo uma equação na outra:
o eixo y). Na hipérbole deitada, o sinal de “menos” está sempre na frente
x2 – 2 · (2x – 3)2 = 1 ⇔ – 7x2 + 24x – 18 = 0
do termo em y2 na equação reduzida, independentemente do tamanho
As duas raízes x1 e x2 dessa equação nos dão as coordenadas de
dos eixos.
A e B. Usando a fórmula para soma dsa raízes de um trinômio:
x + x2 24 12
xM = 1 =− =
2 2 ⋅ ( −7) 7

4. Parábola
4.1 Elementos 4.2 Equação reduzida
A definição PF = dist(P, diretriz) implica que a parábola tem um eixo A equação de uma parábola com vértice na origem e eixo focal em
de simetria, que é a reta perpendicular à diretriz passando pelo foco. A um dos eixos coordenados é dada por:
nomenclatura usual é:
Parábola deitada para a direita (foco no eixo x): y 2 = 2 px
d
P Parábola em pé para cima (foco no eixo y): x 2 = 2 py
V
F Demonstração: para a parábola deitada com concavidade para a direita,
colocamos os eixos e as coordenadas dos pontos principais como na
figura:

p/2 p/2 d P(x,y)


Partindo da definição:
PF = dist(P, diretriz) ⇔
Foco: F
2
Vértice: V  p 2 p F(p/2,0)
Parâmetro: p (mede a distância do foco à diretriz) x −  +y = +x
 2  2
Eixo focal: VF
Relação fundamental: p/2 p/2
Elevando ao quadrado e simplificando:
p
VF = p2 p2
2 x 2 − px + + y 2 = + px + x 2 ⇔
4 4
Demonstração: por definição, VF + dist (V, diretriz) = p. E, como V y 2 = 2 px
pertence à parábola, tem-se VF = dist (V, diretriz).
Para a parábola deitada com concavidade para a esquerda, basta trocar
p por –p. Para a parábola em pé, o raciocínio é análogo.

IME-ITA 139
Matemática IV – Assunto 4

EXERCÍCIOS RESOLVIDOS origem e de eixo maior igual a 18, sobre Ox, sabendo-se que P(c, b/3)
pertence à elipse, em que c é a abscissa de um dos focos e b é o semieixo
08 Determine o vértice, o foco e o parâmetro da parábola y = 4x2. menor.

1 07 A figura mostra a representação de algumas das ruas de nossas


Solução: Comparando a equação dada x 2 = y com a equação reduzida cidades. Essas ruas possuem calçadas de 1,5 m de largura, separadas
4
por uma pista de 7 m de largura. Vamos admitir que:
x2 = 2py, tem-se que a parábola tem eixo focal no eixo y (está em pé),
1 1 I. os postes de iluminação projetam sobre a rua uma área iluminada na
que 2 p = , p= e que o vértice está na origem.
4 8 forma de uma elipse de excentricidade 0,943;
Como o foco fica a uma distância p/2 do vértice e a parábola está em II. o centro dessa elipse encontra-se verticalmente abaixo da lâmpada,
 p  1  no meio da rua;
pé, tem-se F =  0,  =  0,  . III. o eixo menor da elipse, perpendicular à calçada, tem exatamente a
 2   16 
largura da rua (calçadas e pista).

09 Determine o vértice, o foco e o parâmetro da parábola y – 2 = 4(x – 1)2. Se desejarmos que as elipses de luz se tangenciem nas extremidades dos
eixos maiores, a distância, em metros, entre dois postes consecutivos
Solução: Note que essa parábola é a mesma do exercício anterior, deverá ser de, aproximadamente:
exceto por uma translação. Nesse caso seu vértice se encontra no
(Dado: 0,9432 ≈ 0,889 e 0,111 ≈ 0,333 .)
ponto (1, 2) e seu foco no ponto (1, 33/16).

EXERCÍCIOS NÍVEL 1
01 (AFA) A equação da elipse que, em um sistema de eixos ortogonais,
5 
tem focos F1 (–3, 0) e F2 (3, 0) e passa pelo ponto P  , 2 3 , é:
2 
x2 y2 x2 y2
(A) + =1 (C) + =1
36 25 25 36
(A) 35. (D) 20.
2 2
(B) 30. (E) 15.
x2 y2
(B) x + y =1 (D) (C) 25.
+ =1
16 25 25 16 x2 y2
08 (AFA) Na figura abaixo, F1 e F2 são focos da elipse + =1.
25 9
02 (AFA) A distância focal da elipse x2 + 16y2 =4 é:  3
O ponto C, de coordenadas  0,  , per tence ao segmento MN .
__ __  2
(A) 1. Os segmentos AC , CB , MN são, respectivamente, paralelos aos
(B) 3. segmentos
(C) 15 .
F1P , PF 2 e F1 F2 . Área da figura sombreada, em unidades de área, é:
(D) 20 .
y
03 (AFA) Se A(10, 0) e B(–5, y) são pontos de uma elipse cujos focos são P
F1(–8, 0) e F2(8, 0), o perímetro do triângulo BF1F2 é:
M C N
(A) 24. (C) 40.
(B) 36. (D) 60.
(A) 3.
F1 A B F2 x
04 (AFA) A excentricidade da elipse que tem centro na origem, focos em
um dos eixos coordenados e que passa pelos pontos A(3,2) e B(1,4) é: (B) 6.
(C) 9.
(D) 12.
2 2
(A) (C)
3 2 y2 9
09 A elipse x 2 + =e a reta y = 2x + 1, do plano cartesiano, se
2 4
3 (D) 3 interceptam nos pontos A e B. Determine o ponto médio do segmento AB .
(B)
3 2
10 O cometa Halley tem uma órbita elíptica com eixo maior e eixo menor
iguais a 540 × 107 km e 140 × 107 km, respectivamente. Sabendo que
05 Uma cônica tem equação 252x + 9y = 28. Determine a área do
2 2
d
quadrilátero convexo com dois vértices sobre os focos e os outros dois o Sol está em um dos focos da elipse, calcule o valor , em que d
sobre as extremidades do menor dos eixos da cônica. 107
é a menor distância entre o Sol e o cometa, medida em quilômetros.
06 Determine a excentricidade e a equação de uma elipse de centro na Desconsidere a parte fracionária de seu resultado, caso exista.

140 Vol. 3
Cônicas

11 Uma elipse cuja distância focal mede 1 cm está inscrita em um 20 (AFA) O parâmetro da parábola que passa pelo ponto P(6, 2) e cujo
retângulo (de lados paralelos aos eixos principais da elipse) de área igual vértice V(3, 0) é o seu ponto de tangência com o eixo das abcissas é:
a 2 cm². Determine as medidas dos lados do retângulo.
(A) 9/5. (C) 3.
12 Assinale V (verdadeiro) ou F (falso). Em um sistema de eixos cartesianos (B) 9/4. (D) 9/2.
ortogonais, considere os pontos A(5; 0), B(0; 3), C(– 5; 0) e D(0; – 3).
EXERCÍCIOS NÍVEL 2
( ) A equação da reta que contém os pontos A e B é 3x + 5y + 15 = 0.
( ) A área do quadrilátero ABCD, em unidades de área do sistema, 01 (AFA) Sobre o triângulo PF1F2 em que P(2, 2) e F1 e F2 são focos da
é igual a 60.
( ) A equação da circunferência inscrita no quadrilátero ABCD é x2 y2
elipse + =1, é correto afirmar que:
225 9 25
x2 + y2 = .
34 (A) é isósceles.
( ) A equação da elipse que contém os pontos A, B, C e D é 9x + 2 (B) é obtusângulo.
25y2 = 225. (C) tem área igual a 16.
( ) O ponto P(3;2) é interior à elipse que contém os pontos A, B, C e D, (D) tem perímetro igual a 2 2 + 8.
e é exterior ao quadrilátero ABCD.
02 (AFA) O lugar geométrico dos pontos do plano cartesiano que, juntamente
13 (AFA) A equação reduzida da hipérbole, cujos focos são os extremos do com os pontos A(–3, 5) e B(3, 5), determina triângulos com perímetro
eixo menor da elipse de equação 16x2 + 25y2 = 625, e cuja excentricidade 2p = 16 cm é uma:
é igual ao inverso da excentricidade da elipse dada, é:
(A) elipse. (C) hipérbole.
(A) 16y2 – 9x2 = 144 (C) 9x2 – 16y2 = 144 (B) parábola. (D) circunferência.
(B) 9y2 – 16x2 = 144 (D) 16x2 – 9y2 = 144
03 Dada uma elipse de semieixos a e b, calcule, em termos destes
14 Determine a equação de uma hipérbole que tem vértices em (0, 3) e parâmetros, a área do quadrado nela inscrito, com lados paralelos aos
(0, –3) e focos em (0, 5) e (0, –5). eixos da elipse.

04 Uma porta colonial é formada por um retângulo de 100 cm × 200 cm


x2 y2
15 (IIT) Se a excentricidade da hipérbole − =1 é o recíproco da e uma semielipse.
a2 b 2 Observe as figuras:
excentricidade da elipse x2 + 4y2 = 4 e se a hiérbole passa por um dos
focos da elipse, então:

(A) a2 = 3, b2 =2.
(B) o ponto (2, 0) é um dos focos da hipérbole.
5
(C) a excentricidade da hipérbole é dada por .
3
(D) a equação da hipérbole é x2 – 3y2 = 3.

16 Um ponto se move de modo que o produto dos coeficientes angulares


das retas que o ligam aos pontos (– 4, – 2) e (4, 2) dá sempre – 4. Na semielipse, o eixo maior mede 100 cm e o semieixo menor, 30 cm.
Determine seu lugar geométrico. Calcule a medida da corda PQ, paralela ao eixo maior, que representa a
largura da porta a 224 cm de altura.
17 Dada a hipérbole 4x2 – y2 = 32, determine uma reta paralela ao eixo 05 Seja b um número real. Encontre os valores de b, tais que no plano
dos y tal que seus pontos de interseção com a hipérbole formem com o
x2
foco F (de abscissa positiva) um triângulo retângulo em F. cartesiano xy, a reta y = x + b intercepta a elipse + y2 = 1 em um
único ponto. A soma dos valores de b é: 4
18 O vértice, o foco e a reta diretriz da parábola de equação y = x2 são
dados por: (A) 0. (D) 5.
(B) 2. (E) – 2 5.
(A) Vértice: (0, 0); Foco: (0, 1/4); Reta diretriz y = – 1/4. (C) 2 5.
(B) Vértice: (0, 0); Foco: (0, 1/2); Reta diretriz y = – 1/2.
(C) Vértice: (0, 0); Foco: (0, 1); Reta diretriz y = – 1. 06 (AFA) Dada a equação ax2 + by2 = c, em que a, b e c são reais não
(D) Vértice: (0, 0); Foco: (0, – 1); Reta diretriz y = 1. nulos, é correto afirmar que, necessariamente, sua representação gráfica
(E) Vértice: (0, 0); Foco: (0, 2); Reta diretriz y = – 2. é uma:

19 A equação da circunferência de centro C = (–3, –1), que contém o (A) circunferência, se a= b.


vértice da parábola y + 2x2 + 4x = 0 é: (B) hipérbole, se a = –b e c = b.
(C) elipse de centro na origem, se a ≠ b e c = 1.
(A) (x + 3)2 + (y + 1)2 = 5 (C) (x – 3)2 + (y – 1)2 = 5 (D) circunferência, se a = b e c > 0.
(B) (x + 3)2 + (y + 1)2 = 13 (D) (x – 3)2 + (y – 1)2 = 13

IME-ITA 141
Matemática IV – Assunto 4

07 Determine as equações das retas do plano que passam pela origem y


do sistema de coordenadas e que não interceptam a curva do plano dada
x2 y2
pela equação − =1.
4 9
08 Considere o círculo x2 + y2 = r2 de raio r e a hipérbole x2 – y2 = 1.
Nesse caso, pode-se afirmar que: O x
(A) se r < 1, então as curvas se intersectam em quatro pontos. Calcule o raio da maior circunferência, nas condições acima, que tem um
(B) se r = 1, então as curvas têm quatro pontos em comum. único ponto de interseção com a parábola.
(C) se r = 1, as curvas se intersectam em (0, 1) e (0, –1).
(D) se r = 17, então as curvas se intersectam apenas nos pontos 13 Sejam os pontos A(2, 0) e A’(– 2, 0). Por A’, traça-se uma reta
(3, 2 2) e (3, – 2 2). variável que intersecta o eixo das ordenadas em B. Por A, traça-se uma
(E) se r > 17, então as curvas se intersectam em quatro pontos. perpendicular à reta AB que intersecta a reta A’B em M. Determine o lugar
geométrico do ponto M.
09 (AFA) Considere as afirmativas abaixo:
14 (IME) Considere uma elipse e uma hipérbole centradas na origem,
x y  x = 2t + 1 O, de um sistema cartesiano, com eixo focal coincidente com o eixo
I. As retas r : + =1 e s:  são perpendiculares. OX. Os focos da elipse são vértices da hipérbole e os focos da hipérbole
2 −3  y = 3t
20
II. A equação 4x = y2 representa uma parábola com eixo de simetria são vértices da elipse. Dados os eixos da elipse como 10 cm e cm,
3
horizontal. determine as equações das parábolas, que passam pelas interseções da
2 2
III. − x − y =1 representa uma hipérbole. elipse e da hipérbole e são tangentes ao eixo OY na origem.
3 9
15 É dada uma circunferência (C) de centro na mesma origem e raio R. Nesta
É(são) correta(s) a(s) afirmativa(s): circunferência, é traçada uma corda variável AB, paralela ao eixo das abscissas.
Pelo ponto A, traça-se a reta (r), paralela à bissetriz dos quadrantes ímpares
(A) I, II e III. (C) III, somente. e pelo ponto B, a reta (s), perpendicular à reta 2y + x + 5 = 0. Determine e
(B) I, II. (D) II, somente. identifique o lugar geométrico das interseções das retas (r) e (s).

10 (IIT) Dada a família de hipérboles x2 y2 EXERCÍCIOS NÍVEL 3


2
− 2
=1 , qual das
cos α sen α
alternativas abaixo é constante quando o ângulo α varia? 01 Considere o conjunto das cordas de uma elipse formando um ângulo
θ com o eixo maior. Determine o lugar geométrico dos pontos médios
(A) Abscissa dos vértices. (C) Excentricidade. dessas cordas.
(B) Abscissa dos focos. (D) Diretriz.
02 Uma hipérbole tem seu centro na origem e seu eixo conjugado
11 (IME) Calcule as coordenadas dos pontos de interseção da elipse com coincidente com o eixo X. O comprimento de cada latus rectum é 2/3 e a
a hipérbole, representadas na figura abaixo, sabendo-se que:  3
hipérbole passa pelo ponto  −2 3,  . Determine sua equação.
 3 
a. os pontos C e C’ são os focos da elipse e os pontos A e A’ são os
focos da hipérbole; x2 y2
03 (IIT) Seja P um ponto variável na elipse +
=1, 0<b<a.
b. BB’ é o eixo conjugado da hipérbole; a2 b 2
c. OB = OB’ = 3 m e OC = OC’ = 4 m. Considere que a reta paralela ao eixo y passando por P intersecta a
circunferência x2 + y2 = a2 em um ponto Q, tal que P e Q têm ordenada
y de mesmo sinal. Determine o lugar geométrico dos pontos R pertencentes
B PR m
D’ D ao segmento PQ tais que = .
RQ n

A’ C’ C A 04 Determine o lugar geométrico dos focos de uma elipse da qual se


O x conhecem um ponto M(a, b) e o círculo principal x2 + y2 = a2.

05 Um segmento de reta OB encontra o círculo x2 + y2 = ax no ponto B;


B’ B a partir deste ponto é traçada uma perpendicular BC ao eixo Ox. Traça-se
B’ então uma perpendicular CM a OB (C ∈ Ox, M ∈ OB). Determine a equação
do lugar geométrico do ponto M.

12 A figura a seguir mostra, no plano cartesiano, o gráfico da parábola de x2 y2


x2 06 Considere a elipse + = 1 , com a > b. Seja F e F’ seus focos,
equação y = e uma circunferência com centro no eixo y e tangente ao a2 b 2
4 sendo F o de abscissa positiva. Para um ponto M qualquer sobre a elipse,
eixo x no ponto O. determine MF e MF’ em função da abscissa de M.

142 Vol. 3
Conceitos básicos de geometria espacial A ssunto
11
Matemática V

1. Introdução Dois planos


No estudo de geometria euclidiana, começamos vendo a parte associada à
geometria plana, em que toda a estrutura era contida no plano, um dos conceitos
primitivos. Agora, na continuação do estudo, veremos a estrutura da geometria r
no espaço, que também é um conceito primitivo. Para tanto, temos que assumir
alguns axiomas relativos a ele e estabelecer algumas novas definições.
Acrescentem-se aos axiomas da geometria plana mais alguns:
Ax. 1) Três pontos não colineares determinam um plano que os contém. // = =r
(paralelos) (secantes)
Ax. 2) Se dois pontos estão num plano, a reta determinada por eles r : aresta dos planos
está contida nesse plano. [Ax. de Inclusão]
Ax. 3) Existem infinitos pontos dentro e fora de um plano.
Reta e plano
Além disso, é importante adicionar uma modificação à definição de r
retas paralelas: dizemos que duas retas são paralelas se, e somente se,
não se intersectam e são coplanares, ou seja, deve existir um plano que
contenha as duas retas. r
P
Por meio dos axiomas anteriores, podemos concluir quatro maneiras
de determinar um plano:
r r ∩ = {P}
Det. 1) Três pontos não colineares determinam um plano que os contém. (contido no plano) (secante)
Det. 2) Uma reta e um ponto fora dela determinam um plano que os contém. P : traço de r em
r
Det. 3) Duas retas concorrentes determinam um plano que as contém.
Det. 4) O par de retas paralelas determina um plano que as contém.

A P
r// r∩ =
r (paralela)
B C
= pl(ABC) = pl(r, P)
3. Paralelismo
r r//s Como vimos na geometria plana, retas paralelas têm particularidades
s muito interessantes na resolução de problemas. O mesmo vale para
P
problemas em geometria espacial: além do aspecto métrico de manutenção
s de razões e proporções [teorema de Tales, semelhança de triângulos,
= pl(r, s) = pl(r, s) paralelogramo, etc], retas paralelas servirão ainda como argumento para
transporte de ângulos.
Além das maneiras tradicionais de obter paralelas, vistas em geometria
2. Geometria posicional plana, temos mais três formas de fazê-lo no espaço:
Estudemos as possíveis interseções entre os conjuntos primitivos.
Reta // Plano
Duas retas r Como obter: uma reta é paralela a um plano se existir nele uma outra
r
P r r reta paralela à primeira.
P s Como usar: a seção gerada num plano por outro que contém uma reta
s paralela ao plano é paralela à reta.
s r//s r, s coplanares
r s = {P} r//s r r, ss coplanares
= r//α
s r
r s = {P}
(concorrentes) r(paralelas)
s=
(concorrentes) (paralelas)
t
s
s α

s//r α
r e s não coplanares
r e s(reversas)
não coplanares
r (reversas) Obtém-se r//α
r Obtém-se r//t

IME-ITA 143
Matemática V – Assunto 11

Plano // Plano 6. Ângulos entre reta e plano /


Como obter: dois planos são paralelos se duas retas concorrentes de
um são paralelas ao outro.
projeção ortogonal
Como usar: as seções de dois planos paralelos geradas por um plano Primeiro, define-se que uma reta é perpendicular a um plano quando ela
transversal são paralelas entre si. é perpendicular ou ortogonal a toda reta desse plano. Para provar que uma
reta é perpendicular a um plano, basta identificar duas retas concorrentes
do plano às quais a reta original é perpendicular ou ortogonal. Dizemos
também que, nesse caso, o ângulo entre a reta e o plano é de 90°.
r// r

r
s//
t

// Se r então r t, para toda t

Obtém-se // s r
t
Obtém-se r//t

Transitividade do paralelismo Se r s, r t,
Se r//s e s//t, então r//t. com s e t concorrentes, s
então r pl(s, t)

4. Ângulos entre retas reversas t

O ângulo entre duas retas reversas é igual ao ângulo entre duas retas
paralelas a elas, concorrentes entre si.
Dessa maneira, conseguimos projetar perpendicularmente um ponto
r num plano: a projeção de um ponto P num plano α é o ponto P’, tal que
PP’ é perpendicular a α. Assim, prova-se que a projeção de uma reta
r’//r não perpendicular a α é uma outra reta.
r^s = r’^s’ = x O ângulo entre um plano α uma reta não perpendicular a ele é definido
x
como o ângulo entre a reta e a sua projeção em α.
s’//s
r
s
Sendo r’ a projeção de r
no plano α ,
Dizemos que duas retas são ortogonais quando são reversas e o r’ = projαr r^a = r^r’ = x
ângulo entre elas é de 90°.
x

5. Ângulos diedros α
Definimos o ângulo entre dois planos concorrentes como o ângulo
entre duas retas perpendiculares à aresta dos planos. Diedro é a região
espacial interna compreendida entre os planos. Teorema das três perpendiculares
Seja r uma reta contida em um plano α, e P um ponto externo ao plano.
Se P’ é a projeção de P em α, e Q é a projeção de P’sobre r, então PQ é
perpendicular a r, ou seja, Q é a projeção de P sobre a reta r.
r P

Na figura,
x r α
s s r P’
logo, α^β = r^s = x
Q

Ilustrando o teorema das três perpendiculares


Dizemos que dois planos são perpendiculares quando o diedro entre Pode-se entender da seguinte maneira: a projeção de P em r coincide
eles mede 90°. com projetar P no plano α, e então projetar na reta r.

144 Vol. 3
Conceitos básicos de geometria espacial

7. Distância entre retas reversas Plano bissetor


Dado um diedro formado por dois planos, o plano bissetor é o plano
A distância entre duas retas reversas é igual à medida do segmento
que divide o diedro em dois diedros congruentes. O par de planos bissetores
perpendicular comum aos dois.
perpendiculares entre si é o lugar geométrico dos pontos que equidistam
r dos planos dados.

Na figura, d é a distância
d entre as retas reversas r e s .

P
s
Caso necessário, sendo r e s as retas reversas, seja α um plano
paralelo a r contendo s. A distância d(r,s) é igual à distância d(r, α).
d(P, α) = d(P, )
r

9. Outros teoremas importantes


d
d r’ = projαr Teorema da projetividade do ângulo reto
A condição necessária e suficiente para que a projeção ortogonal de
α//r dist(r, s) = d = dist(r, α) um ângulo reto seja outro ângulo reto é que um de seus lados seja paralelo
ao plano de projeção, e que o outro lado não seja perpendicular ao plano.
s

8. Eixo / plano mediador / plano


bissetor r//α

Eixo s’
Dados três pontos não colineares A, B e C, chama-se de eixo a reta r’
perpendicular ao plano (ABC) passando pelo circuncentro do triângulo α
ABC. O eixo de ABC é o lugar geométrico do ponto P que equidista de A,
Teorema da projeção de áreas
B e C, ou seja, PA=PB=PC.
P A área da projeção ortogonal de uma figura plana é igual ao produto
da área da figura pelo cosseno do ângulo entre os planos da figura e de
projeção.
Se P’ = proj P, então
C PA = PB = PC P’ é
circuncentro do ABC
A
P’
B

Plano mediador S
Dados dois pontos A e B, chamamos de plano mediador do segmento S*
cos(α^β)=
AB o plano perpendicular a AB pelo ponto médio de AB. O plano mediador S
do segmento AB é o lugar geométrico dos pontos P que equidistam de A
e B, ou seja, PA = PB.

P
S*

A M B

IME-ITA 145
Matemática V – Assunto 11

EXERCÍCIOS RESOLVIDOS
01 (Lema importante) Se dois planos paralelos são cortados por um
II. AB // α e A’, B’, A, B coplanares ⇒ AB // A ' B '
plano transversal, então as interseções são retas paralelas.
Juntando I e II, temos que AB ^ AA ' (*).
Solução: Seja β o plano definido pelas paralelas AA' e CC'. Por (*) e BÂC = 90°, veja
Sejam α e β os planos paralelos e γ o plano transversal. Além disso, que AB ⊥ β , pois é perpendicular a duas retas que passam por A. Como
defina r = α ∩ g e s = b ∩ g. AB // A ' B ' , segue que β. Com isso, A ' B ' é perpendicular a qualquer reta
Por um lado, veja que r e s não se intersectam, pois estão em planos
paralelos (α e β). de β que passe por A’. Em particular, A ' B ' ^ A'C', ou seja, B’Â’C’ = 90°.
Por outro lado, veja que r e s são coplanares, pois estão sobre o plano g. 2 a
parte (ida): Vamos usar a mesma notação da 1a parte. Inicialmente,
Portanto, r//s. suponha que A’ = A.
Suponha agora que B’ÂC = 90° = BÂC.
02 (Projeção do ângulo reto) Prove que um ângulo reto se projeta como Suponha, por absurdo, que B e C não pertençam a a. Então, existem pontos

reto se, e somente se, um dos lados é paralelo ao plano de projeção e X e Y sobre as semirretas AB e AC , respectivamente, tais que XY // α .
o outro lado não é perpendicular ao plano. Sejam X’ e Y’ as projeções de X e Y em α, respectivamente. Com um
argumento análogo ao dado em II, temos que XY // X ' Y '. Então, o
Solução: É uma proposição com “se e somente se”. Nesse caso, vamos quadrilátero XX’Y’Y é um retângulo. Logo, XY = X’Y’= x.
dividir em duas partes: ida e volta. Definamos AX = b, AX’ = b’, AY = c e AY’ = c’. Os triângulos AXY e
1a parte (volta): Vamos começar pela volta, que é mais fácil. Seja BÂC = 90° AX’Y’ são triângulos em A, portanto, pelo teorema de Pitágoras, temos
o ângulo reto. Sejam A’, B’ e C’ as projeções ortogonais de A, B, C, que b2 + c2 = b’2 + c’2 = x2(**).
respectivamente, sobre um plano a. Agora, suponha que AB // α e que No entanto, como b’ < b e c’ < c, (**) é um absurdo. Portanto, um
dos pontos B ou C deve pertencer ao plano α.
AC não é perpendicular a a (isso garante que os pontos A’, B’ e C’ são
No caso geral em que A’ ≠ A, basta traçar um plano paralelo a α que
distintos).
passe por A e utilizar o argumento acima. Com isso, provamos que B ou
Acompanhe os seguintes argumentos:
C está nesse plano paralelo, o que garante que AB ou AC é paralelo a α.
I. AA ' ⊥ α ⇒ AA ' ⊥ A 'B' ;

EXERCÍCIOS NÍVEL 1
01 No cubo ABCD-EFGH, determine quantos são os pares de arestas 06 Duas retas não coplanares r e s são separadas por um plano p que lhes
reversas entre si. é paralelo. A reta r dista 12 cm de p e a reta s dista 30 cm do mesmo plano.
Ache a distância entre r e s.
02 Num plano α, é dado o quadrilátero ABCD, e fora do plano toma-se
um ponto P. Sendo E a interseção de AB e CD, F a interseção de AD e BC, 07 Pelo centro O de um triângulo equilátero de lado x, traçou-se um
e G a interseção de AC e BD, determine a interseção dos pares de planos: segmento OP perpendicular ao plano do triângulo com medida x. Calcule:

a. PAB e PCD; a. a distância de P aos vértices do triângulo;


b. PAD e PBC; b. a distância de P aos lados do triângulo;
c. PAC e PBD. c. os ângulos que as oblíquas de P aos vértices do triângulo formam
03 Sendo M, N, P e Q os pontos médios dos lados do quadrilátero reverso com o plano do triângulo.
ABCD, prove que MNPQ é um paralelogramo.
08 Calcule a distância de um ponto do espaço ao plano de um triângulo
04 ABCD-EFGH é um cubo. Determine os ângulos entre os seguintes equilátero de 6 cm de lado, sabendo que o ponto equidista de 4 cm dos
pares de retas reversas: vértices do triângulo.

a. AE e BG; 09 Em um plano p está traçado um triângulo, cujos lados medem 6 cm,


b. EF e BC; 8 cm e 10 cm, respectivamente. O ponto A, exterior ao plano, é equidistante
c. AC e FH; dos três vértices do triângulo e a distância comum é igual ao diâmetro do
d. FH e BG. círculo circunscrito ao triângulo. Calcule a distância do ponto A ao plano p.

05 O segmento AB é diâmetro de uma circunferência sobre a qual 10 Calcule a distância de um ponto M a um círculo de raio 1 e que está
marca-se o ponto C. A reta VA é perpendicular ao plano da circunferência. situado num plano p, sabendo que a distância de M a p é igual a 3 e que
O número de faces do tetraedro VABC que são triângulos retângulos é: a distância de M ao centro do círculo é igual a 5.

(A) 0. 11 Pelo circuncentro de um triângulo equilátero ABC de 12 m de perímetro


(B) 1. traça-se a reta perpendicular ao plano do triângulo, sobre a qual se aplica
(C) 2. o segmento OJ = 4 m. Calcular JA, JB, JC e o valor do ângulo que JA
(D) 3. forma com o plano do triângulo.
(E) 4.
12 São dados um plano a, uma reta r pertencente a a e um ponto A
exterior a a. Sabendo que o ponto A dista 5 cm de a e 13 cm de r, calcule
a distância de r à projeção ortogonal de A sobre a.

146 Vol. 3
Conceitos básicos de geometria espacial

EXERCÍCIOS NÍVEL 2
01 Um plano paralelo às diagonais AC e BD do quadrilátero reverso 08 Sendo dados um círculo, um ponto qualquer S sobre seu eixo e um
ABCD intersecta os lados AB, BC, CD e DA nos pontos P, Q, R e S, quadrilátero convexo ABCD circunscrito ao círculo, demonstre que no
respectivamente. Prove que PQRS é um paralelogramo. ângulo tetraédrico SABCD a soma das áreas das duas faces opostas é
igual à soma das áreas das duas outras.
02 São dadas duas retas concorrentes a e b, e os planos α e β tais que
a^ α e b^β. Prove que a interseção αβ é perpendicular ao plano (a,b). 09 Os vér tices A e B de um triângulo ABC distam 3 m e 5 m,
respectivamente, de um plano p. Calcule a distância do vértice C a esse
03 Calcule a distância de um ponto do espaço ao plano de um triângulo plano, sabendo que p contém o baricentro do triângulo.
retângulo, cuja hipotenusa é a, sabendo que as oblíquas traçadas do ponto
aos vértices do triângulo formam ângulos de 60° com o mesmo plano. 10 AB é a perpendicular comum às retas ortogonais r e s. Sabendo que
AM = 4 m, BN = 3 m e AB = 5 m, calcule MN.
04 ABCD é um quadrado de lado a e M é o ponto da perpendicular ao plano
desse quadrado traçada pelo vértice A, tal que AM = a. Qual a medida do A M
diedro que tem por aresta MC e cujas faces são MBC e MDC?

05 ABCD é um quadrado cujo lado é a. Pelo vértice A levanta-se a


perpendicular ao plano do quadrado e sobre essa perpendicular toma-se N
o segmento AS = a. Calcule: B

a. as distâncias do ponto S aos vértices B, C e D do quadrado;


b. as distâncias do ponto A aos planos SBC, SCD, SBD; 11 Calcule o valor de um diedro, sabendo que um ponto A, a ele interior,
c. a distância do ponto A à reta SC. dista 4 m de sua aresta, 2 m de uma face e 2 2 m da outra face.

06 AB = 15 cm, AC = 13 cm, BC = 4 cm são os lados de um triângulo 12 As retas r e s são reversas. Tomam-se os pontos A e M sobre r, B e N
ABC e O é o ponto exterior ao plano ABC. Sabendo que as distâncias do sobre s, tais que AB é perpendicular comum a r e s. Prove que MN forma
ponto O aos vértices do triângulo ABC são iguais ao diâmetro do círculo ângulos iguais com r e s se, e somente se, AM = BN.
circunscrito a esse triângulo, calcule a distância de O ao plano ABC.
13 Os triângulos ABC e DEF são tais que se situam em planos distintos.
07 Pelo vértice A do triângulo equilátero ABC, traça-se o segmento AP, Sabe-se que as retas AB e DE se cortam em M, AC e DF se cortam em
perpendicular ao plano (ABC), de medida igual a BC. Calcule o ângulo N e BC e EF se cortam em P. Prove que M, N e P são pontos colineares.
entre os planos (PAB) e (PBC).

EXERCÍCIOS NÍVEL 3
01 Três semirretas de mesma origem são perpendiculares entre si. 03 V-ABC é um tetraedro em que o triedro de vértice V é trirretângulo.
Prove que a reta traçada da origem perpendicularmente a um plano Se VA = a, VB = b, VC = c, calcule a distância de V ao plano ABC.
simultaneamente transversal a essas três retas e que não contém a
origem intersecta este plano no ortocentro do triângulo determinado pelas 04 (OMERJ) ABCD é um quadrilátero tal que:
interseções do plano com as semirretas.
 = BCD
ABC  = CDA
 = DÂB = 90° .
02 Num paralelepípedo retângulo ABCD-EFGH, existe uma seção que é
um hexágono regular. Prove que ABCD-EFGH é um cubo. Prove que os vértices de tal quadrilátero são coplanares.

RASCUNHO

IME-ITA 147
Triedros e poliedros A ssunto
12
Matemática V

1. Triedros F = F3 + F4 + F5 + F6 + ...
V = V3 + V4 + V5 + V6 + ...
Define-se triedro, ou ângulo triédrico, como a região gerada por três
semirretas, não coplanares, com mesma origem. Chamamos a origem de 2A = 3F3 + 4F4 + 5F5 + 6F6 + ...
vértice, as semirretas de arestas, e os ângulos formados pelas semirretas 2A = 3V3 + 4V4 + 5V5 + 6V6 + ...
de faces do triedro.
Além dessas equações, existe a Relação de Euler: para todo poliedro
A convexo, vale a seguinte relação: V + F = A + 2. Na verdade, essa
relação vale não apenas para os poliedros convexos. Todos os poliedros
que satisfazem a essa relação [que tem a ver com a topologia do sólido]
Triedro V-ABC
são chamados de poliedros eulerianos.
 vértice V
Os poliedros que possuem todas as faces de mesmo gênero e todos os
^ C faces a, b, c, vértices de mesma ordem são chamados de poliedros de Platão. Prova-se
b C
C arestas VA, VB, VC que são apenas 5 os possíveis poliedros de Platão:
a diedros A, B, C
V ^
B
Tetraedro: F = F3 =4, V = V3 = 4, A = 6 (dual de si mesmo)
Hexaedro: F = F4 = 6, V = V3 = 8, A = 12
B
Octaedro: F = F3 = 8, V = V4 = 6, A = 12 (dual do hexaedro)
Dodecaedro: F = F5 = 12, V = V3 = 20, A = 30
Prova-se que vale a desigualdade triangular para as faces de um triedro:
se a, b e c são os valores das faces, então a ≤ b + c, com igualdade se Icosaedro: F = F3 = 20, V = V5 = 12, A = 30 (dual do dodecaedro)
e somente se a aresta “oposta” a c é interna ao ângulo c [as semirretas
seriam coplanares nesse caso]. Além do mais, tem-se que a + b + c A relação de dualidade acima descrita é a seguinte: tomando um
< 360°. ponto sobre cada face de um dos sólidos, e gerando convenientemente
Em cada aresta, está definido um ângulo diedro. Para os ângulos um poliedro, obtém-se o dual. A relação de dualidade é recíproca.
diedros, tem-se que 180° < A + B + C < 540°.
Chamamos de triedro retângulo, birretângulo e trirretângulo o
triedro que possua uma, duas e três faces iguais a um ângulo reto,
respectivamente. Chamamos de triedro isósceles o triedro que possua
duas faces iguais.

2. Poliedros
Define-se como poliedro uma superfície poliédrica fechada, que é uma Tetraedro regular Hexaedro regular Octaedro regular
superfície composta por um número finito de polígonos não coplanares
unidos pelos lados em comum. Chamamos de vértices do poliedro os
vértices dos polígonos da superfície, de faces os polígonos que formam
a superfície do poliedro, de arestas os segmentos de interseção entre dois
polígonos consecutivos.
Vértices de ordem n são os vértices dos quais partem n arestas no poliedro.

Dodecaedro regular Isocaedro regular

No poliedro, Chama-se diagonal do poliedro a um segmento que une vértices que


F3 = 2, F4 = 2, F5 = 4, F = 8, não estejam numa face do poliedro. O número de diagonais de um poliedro
V3 = 10, V4 = 1, V = 11, A = 17 V (V − 1)
pode ser obtido através de D = − A − d , em que d é o número de
2 n( n − 3)
diagonais das faces do poliedro. Se dn = é o número de diagonais
2
numa face de gênero n, então tem-se d = d4F4 + d5F5 + d6F6 + ...
Existem algumas importantes equações de contagem desses objetos, Para calcular a soma de todos os ângulos nas faces do poliedro, use
a saber [indica-se por Fn o número de faces de gênero n, e Vn o número a seguinte fórmula: S = 360°(V – 2).
de vértices de ordem n.

148 Vol. 3
Triedros e poliedros

EXERCÍCIOS RESOLVIDOS 05 Quantas diagonais possui o dodecaedro convexo que tem 4 faces
quadrangulares e todas as demais triangulares.
01 É possível formar um poliedro regular de 6 faces, sendo todas
triângulos? 06 Calcular o número de diagonais do poliedro convexo de 7 faces e
7 vértices, sabendo que ele é constituído apenas por faces triangulares
Solução: Suponha que F = F6 = 6. Usando 3F3 + 4F4 + ... = 2A, e trapezoidais.
temos que 3F = 2A. Como F = 6, segue que A = 9. Pela relação de
Euler, temos que V – 9 + 6 = 2; logo V = 5. 07 Calcule a soma dos ângulos internos de todas as faces do dodecaedro
Como o poliedro é regular, todos os vértices são do mesmo tipo. Portanto, convexo regular.
de 3V3 + 4V4 + 5V5 + ... = 2A, devemos ter k ⋅ Vk = 18. Como V =
5, deveríamos ter k ⋅ 5 = 18, que não nos daria um k inteiro. Então, não 08 A soma dos ângulos de um poliedro é 7200o. Calcular o número de
é possível. faces, sabendo-se que é 2/3 do número de arestas.

Comentário: Caso aceitássemos poliedros não regulares, seria 09 Um poliedro de 18 arestas só possui faces triangulares e hexagonais.
possível. Basta “colar” dois tetraedros regulares ABCD e ABCE. Apesar Determine quantas faces de cada tipo existem, sabendo que a soma dos
de ter todas as arestas de mesma medida, veja que os vértices A, B, ângulos das faces é 2880o.
C são do tipo 4, enquanto D e E são do tipo 3.
10 Um poliedro convexo é composto de 5 faces triangulares, 5 quadradas,
02 Demonstre que só existem 5 poliedros regulares. Além disso, 5 trapezoidais e uma pentagonal. Calcule o número de arestas e de vértices
determine as quantidades de arestas, faces e vértices de cada um. do poliedro.

Solução: Todas as faces devem ser do mesmo tipo, e o mesmo deve 11 Diga se existe ou não cada um dos triedros a seguir, dadas as suas
acontecer para os vértices. Sejam, então, n e k os tipos das faces e faces:
dos vértices, respectivamente.
2A 2A a. 40°, 50° e 90°;
Temos que nF = 2A e kV = 2A, que nos dão F = e V = . Substituindo b. 90°, 90° e 90°;
n k
c. 200°, 100°, 80°;
na relação de Euler, temos que V – A + F = 2 ⇒ 2 A − A + 2 A = 2 . d. 150°, 140°, 130°.
k n
2 nk
Isolando o A, temos A = . Precisamos, então, determinar 12 Num triedro, duas faces medem 100° e 140°. Diga entre que valores
2( n + k ) − nk pode variar a outra face.
as possíveis combinações de n e k que nos dão A inteiro.
Lembremos que n ≥ 3 e k ≥ 3 . Daí, vem a ideia de fazer as substituições
EXERCÍCIOS NÍVEL 2
n = 3 + a e k = 3 + b. Substituindo na expressão de A, temos que
2(3 + a)(3 + b)
A= . Em particular, temos que 3 – a – b – ab > 0. 01 Um poliedro convexo tem 7 faces. A soma dos ângulos de todas as
3 − a − b − ab
faces é igual a 32 retos. Calcule o número de arestas desse poliedro.
É fácil ver que as únicas opções para (a, b) são (0, 0), (1, 0), (0, 1),
(2, 0) e (0, 2). Se há alguma das variáveis maior ou igual a 3, o
02 Calcule o número de diagonais de todo poliedro convexo de 13 faces
denominador fica claramente menor ou igual a zero. Além disso, é
e 20 arestas.
fácil ver que todas as 5 soluções geram valores admissíveis para A,
F, V. Essas 5 soluções dão os 5 poliedros regulares. Veja:
03 Determine todos os poliedros de 10 arestas.
(I) (0, 0) : n = k = 3, A = 6, F = 4, V = 4: tetraedro
(II) (1, 0) : n = 4, k = 3, A = 12, F = 6, V = 8: cubo
04 Um poliedro convexo possui, apenas, faces triangulares, quadrangulares
(III) (0, 1) : n = 3, k = 4, A = 12, F = 8, V = 6: octaedro
e pentagonais. O número de faces triangulares excede o de faces
(IV) (2, 0) : n = 5, k = 3, A = 30, F = 12, V = 20: dodecaedro
pentagonais de duas unidades. Calcular os números de faces de cada
(V) (0, 2) : n = 3, k = 5, A = 30, F = 20, V = 12: icosaedro
tipo, sabendo que o poliedro tem 7 vértices.

EXERCÍCIOS NÍVEL 1 05 Calcular o número de diagonais do poliedro convexo, cujas faces


são todas pentagonais, sabendo que todos os seus ângulos sólidos são
triedros.
01 Achar o número de vértices de um poliedro convexo que possui
12 faces triangulares. 06 Um poliedro de 11 vértices tem o mesmo número de faces triangulares e
quadrangulares e uma face pentagonal. Calcule o número de faces desse poliedro.
02 Achar o número de faces de um poliedro convexo que possui
16 ângulos triedros 07 Em um poliedro convexo de 18 arestas, o número de faces é igual ao
número de vértices, sendo triangulares 4 dessas faces. Calcule o número
03 Determinar o número de vértices de um poliedro que tem 3 faces de lados de cada uma das faces restantes, sabendo que são polígonos de
triangulares, 1 face quadrangular, 1 pentagonal e 2 hexagonais. mesmo número de lados.
04 O “cubo-octaedro” possui seis faces quadradas e oito triangulares.
Determinar o número de faces, arestas e vértices desse sólido que é
euleriano.

IME-ITA 149
Matemática V – Assunto 12

08 Um poliedro convexo de 24 arestas é formado apenas por faces 10 Prove que, em qualquer poliedro euleriano, F3 + V3 ≥ 8.
triangulares e quadrangulares. Seccionado por um plano convenientemente
escolhido, dele se pode destacar um novo poliedro convexo, sem faces 11 Quantas arestas no máximo pode ter um ângulo poliédrico convexo
triangulares, com uma face quadrangular a mais e um vértice a menos cujas faces medem todas 70°?
que o poliedro primitivo. Calcule o número de faces do poliedro primitivo.
12 É possível existir um triedro cujos diedros meçam 40°, 50° e 60°?
09 Prove que, em todo poliedro euleriano, valem as seguintes relações:
a) A + 6 ≤ 3F ≤ 2A 13 Dois diedros de um triedro medem 60° e 110°. Dê o intervalo de
b) A + 6 ≤ 3V ≤ 2A variação do terceiro diedro desse triedro.

RASCUNHO

150 Vol. 3
Prismas e pirâmides A ssunto
13
Matemática V

1. Volumes Já que as seções são paralelas, prova-se que os polígonos gerados


assim, que chamamos de bases do prisma, são congruentes. Da
Essencialmente, queremos atribuir uma função medida aos sólidos mesma maneira, os polígonos gerados na superfície prismática são
do espaço. Define-se volume como sendo essa função medida, com as necessariamente paralelogramos, que formam o que chamamos de
seguintes propriedades: superfície lateral do prisma. Sendo a altura do prisma a distância entre os
Vol. 1. Se dois sólidos são congruentes, então possuem volumes iguais. planos das bases, tem-se a fórmula do volume: V = Sbase × h, o produto
Vol. 2. O volume da união disjunta de dois ou mais sólidos é a soma da área da base pela altura do prisma.
dos volumes dos sólidos. Chamamos de seção reta de um prisma a seção de um plano
Vol. 3. O volume do cubo de aresta unitária é 1. perpendicular às arestas laterais do prisma. Deduz-se a relação da área
lateral, como: SLAT = alat × (2p)seção reta, ou seja, o produto da aresta lateral
Vol. 4. Se dois sólidos são semelhantes numa razão K, então a razão pelo perímetro da seção reta. Também deduz-se a fórmula do volume
de seus volumes é K³. em função da seção reta: V = alat × Sseção reta, ou seja, o produto da aresta
Dizemos que dois sólidos são equivalentes quando possuem volumes lateral pela área da seção reta.
iguais.
Além dessas propriedades, existe o princípio de Cavalieri, que diz que
se seções paralelas de dois sólidos geram áreas iguais, então os sólidos
possuem o mesmo volume. Precisamente, se existe uma reta tal que os
planos perpendiculares a ela geram seções de mesma área nos sólidos,
então seus volumes são iguais. Esse princípio é uma definição, dada de
forma que o volume funcione como a integral da área ao longo dessa reta.

r
A B

SA SB Podemos classificar um prisma segundo os seguintes critérios:


x
Prisma reto: as arestas laterais são perpendiculares às bases. Dessa
maneira, as faces laterais são todas retangulares.
Prisma oblíquo: é o prisma que não é reto; logo, as arestas laterais
Se SA(x) = SB(x) para todo x e r, então VA = VB são oblíquas aos planos.
Prisma regular: é o prisma reto cujas bases são polígonos regulares.
A partir dessas propriedades, deduzem-se fórmulas e relações Também classificamos como triangular, quadrangular, pentagonal,
de volumes entre os sólidos notáveis que estudaremos a partir de já. etc. de acordo com o polígono que define a base, a saber, triângulo,
É importante saber que volume também é uma maneira eficiente de ganhar quadrilátero, pentágono, etc. Em particular, chamamos de paralelepípedo
relações puramente métricas. o prisma cujas bases são paralelogramos, de romboedro o prisma cujas
faces são losangos, e de ortoedro o prisma cujas faces são retangulares.
2. Prismas Um tronco de prisma é o sólido obtido pelo truncamento (corte) de um
prisma, não necessariamente paralelo à base. Em alguns casos, podemos
Dado um polígono num plano, chama-se de superfície prismática a obter relações interessantes quanto ao volume de um tronco: se o tronco
união de todas as retas paralelas entre si que passam pelos pontos do for triangular ou se a base dele for um paralelogramo, então vale que o
bordo do polígono, não coplanares com ele. O sólido obtido entre duas volume é igual ao produto da área da seção reta pela média das arestas
seções paralelas de uma superfície prismática é chamado de prisma. laterais ou, equivalentemente, o produto da área da base do tronco pela
média das alturas dos outros vértices, relativas à base.
E’ D’ C’

A’ C’
B’ Na figura, o volume do tronco
B’ Prisma ABCDE – A’B’C’D’E’ ABC – A’B’C’ é dado por
h Bases: ABCDE e A’B’C’D’E’ A’ b a b c
Arestas laterais: AA’, BB’, CC’, DD’, EE’ c V SABC
3
E D Altura: h a
C
A
C A
B
B

IME-ITA 151
Matemática V – Assunto 13

3. Pirâmides EXERCÍCIOS RESOLVIDOS


Dado um polígono num plano e um ponto fora desse plano, define-se 01 A figura a seguir mostra uma pirâmide reta de base quadrangular
superfície piramidal como sendo a união de todas as retas traçadas a partir ABCD de lado 1 e altura EF = 1. Sendo G o ponto médio da altura EF
do ponto e dos pontos do bordo do polígono. O sólido obtido através da seção e α a medida do ângulo AGB, então cosα vale:
de uma superfície piramidal por um plano qualquer é chamado de pirâmide,
cuja base é a seção gerada e cujo vértice é o ponto fora do plano da seção. E

Pirâmide V – ABCDE G
vértice V
h base ABCDE α
altura h C
E D D
A C
F
B
A B

A distância do vértice à base de uma pirâmide é a altura da pirâmide. 1


1
1 (A) (D)
Deduz-se a fórmula do volume de uma pirâmide: V = Sbase × h, um terço 2 5
3 1
(B) (E) 1
do produto da área da base pela altura da pirâmide. 3 6
Chama-se de pirâmide regular aquela cuja base é um polígono regular 1
e a projeção ortogonal do vértice sobre a base é o centro da base, ou seja, (C)
4
o vértice está no eixo do polígono da base.

V Pirâmide V – ABCDEF regular Solução: Veja que AF mede metade da diagonal AC , logo, AF = 2 .
2
O: centro da base Usando o teorema de Pitágoras no triângulo AGF, temos que
OM: apótema da base 2
 2   1 2 3 3
VO: altura da pirâmide AG2 =  + , que nos dá AG = . Analogamente, BG = .
VM: apótema da pirâmide (apótema lateral)  2   2  2 2
 
F E
A O Como já sabemos que AB = 1, podemos utilizar a lei dos cossenos no
2 2
D       
B M triângulo ABG: 12 =  3  +  3  − 2  3   3  cosα
C  2   2   2  2 
      
1
Daí, cosα = .
De acordo com o polígono da base, classificamos a pirâmide como 3
triangular, quadrangular, pentagonal, etc., a saber, se a base for um
triângulo, um quadrilátero, um pentágono, etc. Em particular, chama-se de
tetraedro a pirâmide triangular [não necessariamente regular]. Um tetraedro 02 Um triedro trirretângulo é cortado por um plano que intercepta
é dito trirretângulo se existe um vértice cujo triedro é trirretângulo. as três arestas, formando um triângulo com lados medindo 8 m,
Um tronco de pirâmide é o sólido obtido pelo truncamento de uma 10 m e 12 m. Determine o volume, em m³, do sólido formado.
pirâmide por um plano paralelo à base. Dessa maneira, prolongando
as arestas laterais temos duas pirâmides, uma maior e uma menor, Solução: Sejam x, y, z as medidas das arestas que concorrem
semelhantes entre si. perpendicularmente duas a duas.
Utilizando o teorema de Pitágoras, temos o sistema de equações

V  x 2 + y 2 = 64
 2
ABCD – A’B’C’D’ é tronco de pirâmide.  y + z = 100 . Somando todas as equações, temos que
2

V – ABCD ~V – A’B’C’D’  2 2

D’ C’  z + x = 144
x2 + y2 + z2 = 154. Subtraindo esta de cada uma das equações do
A’
B’ sistema, obtemos z = 90, x = 54 e y = 10 .
D C 1 xy xyz
Agora, veja que o volume do sólido é igual a V = ⋅ ⋅ z = (considere
A 3 2 6
B que a base é um triângulo retângulo de catetos x e y, por exemplo).
Logo, o volume é igual a V = 90 54 10 ⇒ V = 15 6 m3 . .
6

152 Vol. 3
Prismas e pirâmides

EXERCÍCIOS NÍVEL 1
03 Uma pirâmide quadrangular regular tem todas as arestas iguais a x.
Quanto vale o volume dessa pirâmide? 01 Um ortoedro tem dimensões a, b e c, diagonal d e área total S.
Prove que (a + b + c)2 = d2 + S.
Solução: Ao traçar a altura da pirâmide, é possível formar um triângulo
02 ABC é um triângulo equilátero de lado 1 sobre um plano a. AA’ = 1,
x 2
retângulo de hipotenusa x e catetos h e . Pelo teorema de BB’ = 2 e CC’ = 3 são perpendiculares a a. Calcule a área do triângulo
2 2 A’B’C’. Calcule também o volume do sólido ABCA’B’C’.
x 2 x 2
Pitágoras, temos que x 2 = h2 +   , que gera h = .
 2  2 03 Dada uma pirâmide quadrangular regular de altura 4 e aresta da base 6,
1 x 2 x3 2 calcule:
Portanto, o volume é V = ⋅ x 2 ⋅ = .
3 2 6
04 No cubo de aresta ‘a’ mostrado na figura adiante, X e Y são pontos a. volume e área total;
médios das arestas AB e GH respectivamente. Considere a pirâmide de b. raio da esfera circunscrita;
vértice F e cuja base é o quadrilátero XCYE. Calcule, em função de a, c. raio da esfera inscrita;
d. raio da esfera tangente às arestas;
H G e. volume de um tronco de altura 1;
Y f. área lateral do tronco.

E 04 Na figura ABC é um triângulo equilátero de 8 m de lado. AM, BN e


F CP são perpendiculares ao plano ABC, sendo BN = CP = 6 m. Sabendo
D que os planos ABC e MNP formam ângulo de 30º, calcule a área total e o
C
volume do tronco de prisma.

A X B N
P
a. o comprimento do segmento XY;
b. a área da base da pirâmide;
c. o volume da pirâmide. M
B
Solução:
a. Basta ver que o quadrilátero AXYH é um paralelogramo, pois AX e YH A C
= AH
são iguais e paralelos. Daí, temos que XY = a 2 (diagonal do
quadrado). 05 Um prisma hexagonal oblíquo de 12 3 m2 de área da base e 5 m de
b. Cuidado para não achar que XCYE é um quadrado, porque não altura tem, por seção reta, um polígono regular de 2 m de lado. Calcular
é. Os triângulos AEX, EHY, CGY e BCX são congruentes (L.A.L.); a área lateral do sólido.
logo, EX = XC = CY = YE, ou seja, o quadrilátero XCYE é um losango.
Então, sua área é igual à metade do produto de suas diagonais. 06 Um prisma hexagonal regular tem altura igual ao diâmetro do círculo
No item a, já calculamos uma delas. A outra é (diagonal do cubo). circunscrito ao polígono da base. As diagonais menores da base medem
a 2 ⋅ a 3 a2 6 15 dm. Calcule a área lateral do prisma, em m².
Então, a área procurada é igual a S = = .
2 2
c. Como queremos o volume da pirâmide e no item anterior 07 (ITA-90) Seja V o vértice de uma pirâmide com base triangular ABC.
calculamos a área de sua base, ficamos tentados a tentar calcular O segmento AV, de comprimento unitário, é perpendicular à base.
sua altura. No entanto, isso não é tão simples. Vamos a uma outra Os ângulos das faces laterais, no vértice V, são todos de 45 graus.
abordagem. Deste modo, o volume da pirâmide será igual a:
Veja, inicialmente, que o plano XCYE divide o cubo em duas partes
congruentes. Para o cálculo do volume pedido, podemos retirar 1
(A) 2 2 −2
de uma dessas metades do cubo as pirâmides YFGC e BCXF, que 6
são iguais. 1
(B) 2− 2
6
a
1 BC ⋅ BX 1
a⋅
a3 (C) 1 2− 2
Temos VBCXF = ⋅ ⋅ BF ⇒ VBCXF = ⋅ 2 ⋅a= . 3
3 2 3 2 12
1
a3 a3 a3 (D) 2 2 −1
Portanto, o volume da pirâmide é igual a V = − 2⋅ ⇒V = . 6
2 12 3
(E) n.d.a.

IME-ITA 153
Matemática V – Assunto 13

08 (ITA-88) As arestas laterais de uma pirâmide regular de 12 faces EXERCÍCIOS NÍVEL 2


laterais tem comprimento l. O raio do círculo circunscrito ao polígono da
2
base desta pirâmide mede l . Então, o volume desta pirâmide vale: 01 Prove que se um poliedro admite uma esfera inscrita, então, seu volume
2 1
é dado por V = Sr, em que S é a área total e r é o raio da esfera.
2 l3 .
(A) 3 2l3 . (D) 3
2 3 02 Prove que se um prisma é circunscrito a uma esfera de raio R, seu
(B) 2 l3. (E) l .
4 1
3 3 volume é dado por V = Sl r , onde Sl é a área lateral.
(C) l. 2
2
03 As áreas de duas faces de um tetraedro são iguais a S e S’, e a é o
09 (CBERJ-89) Na figura, temos uma pirâmide quadrangular regular de comprimento da aresta comum a essas faces. Sendo α o ângulo diedro
altura 8 m e cuja base tem perímetro de 48 m. A distância do vértice “D” entre essas faces, calcule o volume do tetraedro.
à face “AMB” é:
04 ABCD é o paralelogramo contido no plano a. AA’ = a, BB’ = b, CC’ = c
M
e DD’ = d são perpendiculares a a. Que relação existe entre a, b, c e d
para que A’, B’, C’ e D’ sejam coplanares? Considere que A’, B’, C’ e D’
estão num mesmo semiespaço definido pelo plano de A, B, C e D.

C 05 ABCD é um quadrado de lado 2, contido num plano a. AA’ = 2, BB’ = 1,


B CC’ = 4 e DD’ = x são perpendiculares a a e A’, B’, C’ e D’ são coplanares.
Calcule:
O
a. x;
D A b. o volume do tronco de prisma ABCD-A’B’C’D’
c. o ângulo entre os planos (ABCD) e (A’B’C’D’)
(A) 4,8 m.
(B) 9,6 m. 06 Dado um cubo ABCD-RSTU de 4 m de aresta, considere os pontos M
(C) 2,4 m. e N, médios respectivamente das arestas AB e AD, bem como o ponto J,
(D) 5 m. pertencente à aresta BS, e distante 1 m do vértice B. Calcule o perímetro
(E) 1,2 m. e a área da seção produzida no cubo pelo plano MNJ.

10 (EN-92) Em uma pirâmide quadrangular regular a altura é 2 e a aresta da 07 Determine o volume de um tronco de pirâmide de altura h, área da
base é 8. O cosseno do ângulo diedro entre duas faces laterais adjacentes base maior S e área da base menor S’.
vale:
08 Um tronco de pirâmide tem bases de áreas S e S’. Calcule a área da
(A) –1/4. seção média (seção equidistante das bases do tronco).
(B) –1/3.
(C) –1/2. 09 As bases de um tronco de pirâmide são polígonos regulares de bases
(D) –3/4. de áreas a e b (a > b). Calcule o lado da seção paralela às bases que
(E) –4/5. divide o tronco em dois outros de mesmo volume.

11 (EN-04) Em uma pirâmide regular cuja base é um quadrado, os 10 Três esferas iguais de raio r são tangente entre si duas a duas e
números 2, o apótema a da base e a altura h da pirâmide formam, tangentes a um plano a. Calcule o raio da esfera tangente as três esferas
e ao plano a.
nesta ordem, uma progressão aritmética e a soma destes é 9 2. O valor
da área da superfície total desta pirâmide é:
EXERCÍCIOS NÍVEL 3

( ) (
12 3 + 3 17 .
(A) 24 1 + 2 17 . (D) ) 01 Um tetraedro ABCD é tal que as arestas AB e CD são ortogonais e
(B) 48 ( 3 + 34 ) . (E)
24 ( 3 + 34 ) . medem 3 e 6. Calcule a maior área da seção gerada por um plano paralelo
a AB e CD.
(C) 36 ( 2 + 2 34 ) .
02 Considere uma pirâmide quadrangular regular V-ABCD de lado da base
12 (AFA-06) O produto da maior diagonal pela menor diagonal de um 6 e altura da pirâmide 6. Tal pirâmide é seccionada por um plano que passa
prisma hexagonal regular de área lateral igual a 144 cm² e volume igual a pelo ponto médio de VA, e que intercepta o plano BCD segundo uma reta
144 3 cm³ é: externa a ABCD, paralela a BC, distando 6 de BC. Calcule a área da seção
determinada por tal plano.
(A) 10 7.
03 Determine o raio da esfera circunscrita a um tronco de pirâmide
(B) 20 7.
quadrangular regular de altura h sendo os lados das bases 2a e 2b.
(C) 10 21.
(D) 20 21.

154 Vol. 3
Gabarito V olume
3
Matemática I 31 Letra B. 45 Letra B.
32 Letra B. 46 Letra E.
Assunto 8 33 Letra E. 47 Letra B.
34 Letra D. 48 Letra A.
Exercícios Nível 1 35 Letra B. 49 Letra C.
01 36 (– 3, – 6 ) ∪ ( 6 , 3) 50 Letra A.
a. {–3, 5} d. ]– ∞, –2 [ ∪ ] 2, + ∞[ 37 Letra D. 51 1.
b. {0, 2} e. {2} 38 Letra C. 52 {(4, 2)}
 −3 − 5 −3 + 5  39 Letra E.
c. ]–1 / 2, 0[ f.  , ,1
 2 2  53
02 a. 2 b. – 4
a. {2, 3} d. {–1, 1} 54 91.
b. {3} e. {0}
3 Exercícios Nível 2
c. {2} f. 1,  01
03 m = 2; m = 6; m ≤ 3/2.  2
  q p

04 Letra B.    p  p− q  p  p− q 
05 Letra E. a. (1, 1),    ,    
  q   q  
06 Letra A.   
07 Letra C. b. {(1, 1)}
08 Letra C. c. {(1, 1), (1/2, 2)}
09 Letra A. 02 Letra C.
10 Letra D. 03 Letra A.
11 {0} 04 –
12 {–1, 9} 05 – 1
13 {49 – 1} 06
14 ]1/2, 1[ ∪ ]1, +∞[ a. {(1/2, 1/4) ; (1/4, 1/2)} c. {(4, 3)}
15 b. {(2, 1)} d. {(4, 16)}
a. {–3} e. {100} 07
 2 a. S = {5/12}

b. {–1} f.  ,1, 4  b. S = {1; 2}
 2 
 1 + 5   1 ± 46 
c.   g. {0,1; 100} c. S = −1; 
 2   5 
d. {2} {
h. 5± log5 4
} 08 Letra C.
09 –1/2 < x < 1/2.
16 {5 + 4 2 } 10 Letra B.
17 ]0, 1[ 11 Letra C.
18 ∅ 12 Letra E.
19 ]0,3 log 2/32[ 13 Letra C.
20 14 Letra C.
15 Letra B.
a. {log3 5 ± log32 5 + 4 }
b. {log25 6, log6 5} 16 Letra B.
1+ 5 17 –
21 log 3 18
2
2
a. (–∞; 0] ∪ [log2 3; 2)
22 a = b². b. (1; 3/2)
23 logx x1 19 a > 1.
n
24 3. 20 ]1, 3[
25 21 –
a. (– 2 , –1) ∪ (1; 2 ) b. (0,1/ 6 ] ∪ [1,+∞) 22 2001.
26 x = 2 40 Letra A.
Exercícios Nível 3
1 
27 S =  ,10  41 Letra C. 01
 10  x≤1
a. S = [3,+∞) b.
28 {k ∈ ℜ | k ≥ 1} 42 Letra C. 02 x = 10
29 x > 0. 43 Letra C. 03 1.
30 Letra B. 44 Letra C. 04 –

IME-ITA 367
Gabarito

05 – 43 Letra A.
06 210. 44 Letra B.
07 3. 45 p = 0, q = 0 ou p = 5, q = –1.
08 3. 46 1; 2; 3.
09 3. 47 2, 3, –1.
10 6. 48 1, 2, –3 e l = 6.
11 –1/8. 49 (p = –2, q = –1) ou (p = 1, q qualquer).
50 a = 2, b = 3.
Matemática II 51
1 ± 15 i −3 ± 5
e
4 2
Assunto 5 52 Letra D.
53 Letra E.
Exercícios Nível 1 54
01 (x2 + 3)2 + (x2 – 4)2. (A) ( −3 / 2 ) + (1 / 6 ) + ( 5 / 3 )
02 Impossível. x x + 2 ( x − 1) )
03 Impossível.
04 a = 1, b = 2, c = –3
05 a / 3 = b / 2 = –c. (B) ( −1) + ( 2 ) + 1
+
2
06 Letra A. x x − 1 ( x − 1)2 ( x − 1)3
07 Os coeficientes devem ser proporcionais (ad = bc).
(C) 1 + ( )
08 Letra E. −x
2
09 Letra B. x x +4
10 Letra A.
11 Letra B. (D) 2 x + ( − x + 3 )
12 – 135. x 2 + 1 ( x 2 + 1)2
13 – 8x + 14.
14 x2 + 2x + 87.
(E) x−2 2 ( − 3)
15 Letra D. + +
x + x +1 x −1 ( x − 1)
2 2
16 x + 3.
5 − 4x . 55 Letra B.
17
3 Exercícios Nível 2
18 Letra C. 01 P(x) = ax4 –2ax3 + cx2 + (a – c)x + d, a ≠ 0.
19 Letra C. 02 Letra A.
20 – 03 Letra C.
21 Letra B. 04 Letra B.
22 05 Letra D.
a. 9 ;
2
c. 1; 06 x − 2 x 2 + 20
8 07 12 30
b. 51; d. − 13 . a. 2 e 3
3 b. m = 22 e n = 163.
23 m = 1/2. 08 A/C.
24 p = –1, q = –2. 09 Letra B.
25 Letra E. 10 Letra B.
26 p = 6, q = 25. 11 a = 6, b = 7.
10 12 Anulada.
27 x3 – 2x2 + 6x − .
28 1. 3 13 Letra C.
29 –x3 + 1. 14 1, 2, 3, 6.
30 – 15 1, –3 e m = 6.
31 x3 – 6x2 + 21x – 26 = 0. −1 ± 5
16 , –1 ± i.
32 P(x) = 2x2(x + 1)(x2 + 1). 2
33 Letra C. 17 –
34 Letra E. 18 –
35 Letra C. n
36 1, 3, 5. 19 Dica: m é raiz de xn – m = 0.
37 1, 2, 2. 20
38 a = 2. a. S = {–2,1/2}; b. S = {–1/4,3/5,1};
39 k = 6, a = –3, b = 2, c = 1 ou k = –6, a = 3, b = –2, c = –1. 21 Letra D.
40 Letra C. 22 Letra B.
41 Letra D. 23 Letra C.
42 Letra E.

368 Vol. 3
Gabarito

24
a. –;
Matemática III
 −1 ± 3 i −3 ± 5 
b. S =  ; . Assunto 5
 2 2 
Exercícios Nível 1
25 Letra D.
26 Letra C. 01 Letra D.
27 Letra D. 02 Letra A.
28 Letra B. 03 –
3 2 04 Letra A.
 p  q 
29   +   = 0 .  3 0
3 2 05  3 7 
30 0, 1, 2, ..., n – 1  −10 − 1
 
31 2 ± 3 e 7.
32 Não.  28 1 
33 Letra C. 06  
 23 3 
34 Letra D.
35 – 07 0.
36 a = –1, b = ± 1, c = 2, d = 1. 08 24 e –11.
09 7.
1 ± 11i
37 . 10 Letra D.
2
11 Letra E.
38 P(x) = 2t5 + 10t4 + 20t3 + 7t2 – 16t – 7, em que t = 1 – x.  0 1
39 Dica: Analise a diferença entre os polinômios dados. 12  
 −1 0 
 1 1 
40 S = −1, i , − i , − 2, − , 2, ,1 (1 possui multiplicidade 2, e – 1 13 Letra C.
 2 2  14 Letra B.
possui multiplicidade 3). 15 Letra C.
Exercícios Nível 3 16 Letra C.
17 Letra E.
01 P(x) ≡ x. (A ideia é ver que P(x) = x para infinitos valores de x).
18 Letra A.
x3 x2
02 P( x ) = − + cx + d .  0 k  ± − 1− b − c b 
6 2 19   , k ≠ 0 ou   bc < − 1
03 Dica: Por Girard, m + n + p = 0, então 1 é raiz da 2a equação.  −1 / k 0  c  − 1 − bc 
04 Dica: Considere a forma fatorada do polinômio P(x) = f(x) – 12. 20 An = I, A, –I, –A, conforme seja n =4k, 4k + 1, 4k + 2, 4k + 3, em
05 1 0
a. –; que k ∈ Z e I =  
 0 1
q p3 q 2 3 q p3 q 2
b. x = 3 − + + + − − + . 21 Letra D.
2 27 4 2 27 4
22 Só a IV é verdadeira.
06 Dica: Analise a paridade de uma possível raiz inteira de P.
23 –
07
a. Use o teorema de Bolzano;  1 −3 2
b. 3p/4 (desenvolva uma fórmula para tan(a + b + γ)). 24  −3 3 − 1
08 a6 – pa4 – q2 = 0.  2 −1 0 
 
09 Dica: Prove que se P tem coeficientes inteiros, então a – b | P(a) – P(b),
para a e b inteiros. 25 Letra C.
10 Dica: Use a mesma dica do problema anterior. 26 SOL
11 Letra C. 27 3 ou 1.
12 – 28 32.
 p = 4, raízes: − 1, − 1, 1 ± 2 i 29 –48.
 30 1, 2, ... , n – 1.
13  p = − 4, raízes: 1, 1, − 1 ± 2 i 31 (–1)n–1.

 p = 4 i , raízes: i , i , ± 2 − i 32 (1 – xn)n–1

 p = − 4 i , raízes: − i , − i , ± 2 + i 33 Letra C.
14 3. 34 Letra A.
15 1, se n é ímpar; n/(n+2),se n é par. 35 Letra D.
16 Dica: Veja a definição de polinômio recíproco. 36 Letra C.
17 a = 2. 37 Letra E.
18 P(x) = k(x – 1)4. 38 Letra D.
19 ±(x + 1)(x – 4). 39 Letra C.
20 Dica: O polinômio é sempre positivo, portanto não há raiz real e, por 40 –12.
isso, não há fator de grau 1. Escreva-o como produto de dois polinômios
de grau 2 e use o fato de que para x = a e x = b o resultado obtido é 1.

IME-ITA 369
Gabarito

n( n +1)
1 −3 1  1 a n +1 − 1 
06 xa 2
 + n 
41 8 ⋅ −6 −6 10  x a ( a − 1) 
4 4 −4
 x n −1 − y n −1 
Exercícios Nível 2 07 ( −1)n −1 xy  
 x−y 
01 –
02 08 –
 0 1 1 09 –
a.  0 0 1  10 (a2 + b2 + c2 + d2)2.
0 0 0 λi + λk λ − λk
∏ sen
2
  11 2( n −1) sen i
b. I + A. 1≤ i ≤ k ≤ n
2 2
c. I + A2 + A3 + ... + An – 1. 12 Não há (olhe para o traço).
03 – 13 –
04 I – M. 14 –
 n( n + 1) 
1 n 2  Matemática IV

05  0 1 n 
0 0 1  Assunto 3
 
  Exercícios Nível 1
8 6 29 01 (x + 12)2 + (y – 5)2 = 169 ou (x + 12)2 + (y + 5)2 = 169.
06 x = − , y = − , z = − 02 Letra B.
5 5 5
03 Letra D.
07 – 04 (x + 3)2 + (y – 17/2)2 = 185/4.
08 05 2x – 5y +19 = 0.
a. ∆n = 5∆n–1 – 6∆n–2. b. 3n+1 – 2n+1. 06 Letra B.
07 x – 2y + 10 = 0.
09 Letra E.
08 (x – 7)2 + (y + 6)2 = 89.
10 Letra D.
09 Letra D.
11 V
10 (– 1,5) e (– 2,– 2)
12 Letra E.
11 Letra D.
13 –
12 Letra A.
14 a(b – a)(c – b)(d – c).
13 (x – 4)2 + (y + 1)2 = 5 e (x – 2)2 + (y – 3)2 = 5.
15 –10 e 0.
16 8. 14 P(– 7/2, 5/4), 2 5
17 2(cosb – cosa)(cosc – cosa)(cosc – cosb). 15 Letra B.
18 – 16 Letra A.
19 1 17 4x + 3y = 17.
n 18 y = – 2x + 5; y = – 2x – 5.
20 ∑a x
k =0
k
n− k 19 y = – 1 ou 4x + 3y = 17.
20 Letra A.
21 (a2 – b2)n. 21 – 1 ou 25.
22 22 y = 9.
a. ∆n = 5∆n–1 – 6∆n–2. 23 Letra A.
b. 3n+1 – 2n+1. 24 d(c1, c2) = r1 + r2 = 3 5 ↔ c1 e c2 são tangentes exteriores.
23 Fn(n-ésimo termo da sequência de Fibonacci) 2
n( n −1)
 n + 1  n −1 x2 + y2 – 8x – 16y + 35 = 0.
24 ( −1) 2  2 n 25 Letra B.
  26 3 2 .
m 1 1 27
  a. 7.
25 ±  1 m 1 
 1 1 m b. 17.
  c. 2.
Exercícios Nível 3: 28 x2 + y2 + 16x – 16y + 24 = 0.
01 Não. 29 2 2 ; 7x – y – 16 = 0.
02 – 30 Letra B.
03 B · (A + B)–1 · A = A · (A + B–1) · B
04 – Exercícios Nível 2
1 1 1 1  01 Letra B.
05 x( a1 − x )( a2 − x )( an − x )  + + + +  02 Círculo com centro em (6,1) e raio 2 10 .
 x a1 − x a2 − x an − x 
03 8 u.c.

370 Vol. 3
Gabarito

04 – 15 Estão corretas as letras B e D.


05 – 16 Elipse de centro na origem, eixo maior 2 a = 4 17 sobre Oy e eixo
06 Letra D. menor 2 b = 2 17 sobre Ox.
07 45°.
08 9x2 + 36y2 = 100. 17 x = − 2 10 ± 16
3
09 (x – 1)2 + (y – 3)2 = 10.
10 Letra B. 18 Letra A.
11 90°. 19 Letra B.
20 Letra B.
36 36
12 ( x − 1)2 + ( y − 1)2 = ; (13 x − 1)2 + (13 y + 35)2 = .
5 5 Exercícios Nível 2
13 Letra A. 01 Letra B.
02 Letra A.
14 2y + x + 5 = 0.
4 a2 b 2
15 Letra D. 03 2 .
2 2 2 2 a + b2
16  x + 10  +  y + 25  = 1 ou  x − 30  +  y − 5  = 1. 04 60 cm.
 7   7   7   7
05 Letra A.
17 Letra A. 06 Letra B.
18 2x + y – 5 = 0, x – 2y = 0. 07 y = mk, em que m ≥ 3/2 ou m ≤ – 3/2; x = 0
19 7.
20 (x – 3)2 + (y + 1)2 = 38. 08 Letra E.
21 90°. 09 Letra D.
22 Os círculos não são secantes. 10 Letra B.
23 x2 + y2 – 8x + 6 = 0 ou 9x2 + 9y2 + 88x – 106 = 0.
 20 82 9 41 
24 – 11 Os quatro pontos  ± ,± 
25 –  41 41 

Exercícios Nível 3 12 2.
13 x2 – y2 = 4.
01 2x – 3y + 8 = 0,3x + 2y – 14 = 0.
02 – 8 35
14 y 2 =± x
03 Sem gabarito. 63
04 – 15 1x2 : 24xy + 9y2 = 9R2 (elipse).
05 –
06 – Exercícios Nível 3
01 Segmento de reta (limitado pela elipse) passando pelo centro da elipse.
6 5 2 12 26
07 r = ; x + y2 + x− y + 1 = 0. (denominado diâmetro da elipse com respeito à direção q).
5 5 5
( ) ( )
2

08 (A – a)2 + (B – b)2 = R2 + r2.


02 54 + 2919 y 2 − 3 54 + 2919 x 2 = 1
09 –
2
10 Círculo circunscrito ao triângulo ABC. x2 y 2 ( m + n)
03 + =1
Assunto 4 a2 ( am + bn )2
04 Elipse de focos (a, b) e (– a, – b) e círculo principal x2 + y2 = a2.
Exercícios Nível 1 05 (x2 + y2)2 = ax3.
01 Letra D. c c
06 MF = − a − x e3 MF ' = a + x .
02 Letra C. a a
03 Letra B.
04 Letra B. Matemática V
2 3
05 . Assunto 11
3
x2 y2 2 2 Exercícios Nível 1
06 + =1e . 01 24.
81 9 3
02
07 Letra B. a. reta PE.
08 Letra B. b. reta PF.
 1 1 c. reta PG.
09  − ,  .
 3 3 03 –
10 9. 04
a. 45°. c. 90°.
11 1 e 2
b. 90°. d. 60°.
12 F – F – V – V – V.
05 Letra E.
13 Letra A.
2 2
06 42 cm.
14 y − x =1
9 16

IME-ITA 371
Gabarito

07 08 40.
09 F3 = 8, F6 = 2.
a. 2 x 3
3 10 A = 30, V = 16
11
x 39
b. a. Não existe. Trata-se de um triedro degenerado, onde suas semirretas
6 são coplanares.
c. 60°. b. Existe.
08 2 cm. c. Não existe.
09 5 3 . d. Não existe.
10 3 2. 12 40° < f < 120°

11 JA = JB = JC =
8 3
, 60°.
Exercícios Nível 2
3 01 15.
12 12 cm 02 14.
03 (F5 = 1, F3 = 5) e (F4 = 2, F3 = 4)
Exercícios Nível 2 04 F3 = 3, F4 = 2, F5 = 1
01 – 05 100.
02 – 06 11.
a 3
03 07 4.
2 08 13.
04 120° 09 –
05 10 –
a. SB = SD = a 2 11 5.
12 Não, pois a soma dos diedros é menor que 180°.
SC = a 3
13 (10°,130°)
b. d(A, (SBC)) = d(A, (SCD)) = a 2
2 Assunto 13
d(A, (SBD)) = a 3 Exercícios Nível 1
3 01 –
a 6 15
c. 02 [A’B’C’] =
3 4
3
06 65 3 VABC – A’B’C’ =
2
8
03
 7
07 arccos   a. V = 48, S = 96. d. 6 17 − 9 2 .
 7  4
08 – b. 17/4. e. 27.75
09 8 m ou 2 m. c. 3/2. f. 26.25
10 5 2 m. 224 3
04 V = , Ssup = 144 + 16 3
11 75° 3
12 – 05 120 m².
13 – 06 9 m².
Exercícios Nível 3 07 Letra A.
01 – 08 Letra E.
02 – 09 Letra B.
10 Letra E.
1
03 11 Letra E.
1 1 1
+ + 12 Letra D.
a2 b 2 c 2
04 –
Exercícios Nível 2
01 –
Assunto 12 02 –
2S ⋅ S '
Exercícios Nível 1 03 senα
3a
01 8. 04 a+c=b+d
02 10. 05
03 Não existe tal poliedro. a. 5.
04 F = 14, A = 24, V = 12 b. 12.
05 17.
 14 
06 3. c. arccos  
07 6480°.  7 

372 Vol. 3
Gabarito

06 2p = 6 5 + 2 2
10 r/3
S = 7 6
Exercícios Nível 3
h
(
07 V = ⋅ S + SS ' + S '
3
) 01 9/2
2 02 7 53
 S + S'  4
08  
 2
( 2a )
2
  2
+ h 2 − 2 b2
2 03 R = + 2 b2
 a+ b 4 h2
09  3 
 2 

ANOTAÇÕES

____________________________________________________________________________________________________________________

____________________________________________________________________________________________________________________

____________________________________________________________________________________________________________________

____________________________________________________________________________________________________________________

____________________________________________________________________________________________________________________

____________________________________________________________________________________________________________________

____________________________________________________________________________________________________________________

____________________________________________________________________________________________________________________

____________________________________________________________________________________________________________________

____________________________________________________________________________________________________________________

____________________________________________________________________________________________________________________

____________________________________________________________________________________________________________________

____________________________________________________________________________________________________________________

____________________________________________________________________________________________________________________

____________________________________________________________________________________________________________________

____________________________________________________________________________________________________________________

____________________________________________________________________________________________________________________

____________________________________________________________________________________________________________________

____________________________________________________________________________________________________________________

____________________________________________________________________________________________________________________

____________________________________________________________________________________________________________________

____________________________________________________________________________________________________________________

____________________________________________________________________________________________________________________

____________________________________________________________________________________________________________________

____________________________________________________________________________________________________________________

IME-ITA 373

Você também pode gostar